FM Manual Guo

March 21, 2018 | Author: misosoupxd | Category: Present Value, Compound Interest, Money, Factor Income Distribution, Business Economics
Share Embed Donate


Short Description

Actuarial FM Exam MAnual...

Description

Deeper Understanding, Faster Calculation --Exam FM Insights & Shortcuts Part I:

Theories of Interest

12th Edition

by Yufeng Guo Fall 2010

http://actuary88.com

This electronic book is intended for individual buyer use for the sole purpose of preparing for Exam FM. This book may NOT be resold or otherwise redistributed to others. No part of this publication may be reproduced for resale or multiple copy distribution without the express written permission of the author.

© 2010, 2011 By Yufeng Guo

1

Contents Part One: Theories of Interest Chapter 1

Exam-taking and study strategy ............................ 5

A tale of two Exam FM takers, Mr. Busy and Mr. Lazy ................................................ 5 Truths about Exam FM ................................................................................................... 9 How to study hard yet fail the exam miserably ............................................................ 11 Recommended study method........................................................................................ 13 How to build a 3 minute solution script........................................................................ 14 How to eliminate errors ................................................................................................ 30

Chapter 2 Chapter 3

Getting started ............................................................. 48 FM Fundamental.......................................................... 49

Time value of money .................................................................................................... 49 Principal ........................................................................................................................ 49 Interest rate.................................................................................................................... 49 Simple interest rate ....................................................................................................... 50 Compound interest rate ................................................................................................. 50 Force of interest ............................................................................................................ 52 Nominal interest rate..................................................................................................... 57 APR............................................................................................................................... 58 Annual effective interest rate ........................................................................................ 59 Continuous compounding ............................................................................................. 60 Effective annual rate of discount .................................................................................. 60 Simple annual rate of discount...................................................................................... 63 Nominal annual rate of discount ................................................................................... 63 m m Relationship between i , d , , i ( ) , d ( ) ....................................................................... 64 Future value .................................................................................................................. 75 Present value ................................................................................................................. 75 Convert interest rate to discount rate or vice versa....................................................... 77 PV of a stream of cash flows ........................................................................................ 82 Net Present Value ......................................................................................................... 82 Internal rate of return (IRR).......................................................................................... 82 Asset and its price ......................................................................................................... 84 Convert a cash flow from one point of time to another point of time .......................... 85 Collapse multiple cash flows into a single cash flow ................................................... 86 Annuity – collapsing n parallel cash flows into a single cash flow .............................. 89 Avoid the common pitfall ............................................................................................. 90 Perpetuity .................................................................................................................... 104 Annuity – payable m-thly in advance ......................................................................... 107 Annuity – payable m-thly in arrears ........................................................................... 109 Increasing annuity....................................................................................................... 121 Continuously increasing annuity................................................................................. 121 Decreasing annuity...................................................................................................... 122

2

Chapter 4

Calculator tips ............................................................ 137

Best calculators for Exam FM .................................................................................... 137 New features added in BA II Plus Professional.......................................................... 137 How to reset calculators to their best conditions for FM............................................ 137 Compound interest ...................................................................................................... 140 Annuity ....................................................................................................................... 144 Loan/bond amortization .............................................................................................. 153 Compare Cash Flow Worksheet with TVM Worksheet ............................................. 159 Increasing annuity....................................................................................................... 159 Comprehensive calculator exercise............................................................................. 167

Chapter 5 Geometrically increasing annuity ..................... 175 Chapter 6 Real vs. nominal interest rate............................. 204 Chapter 7 Loan repayment and amortization.................... 210 Chapter 8 Sinking fund ................................................................ 240 Chapter 9 Callable and non-callable bonds ......................... 247 Chapter 10 Valuation of stocks................................................... 272 Chapter 11 Price of a bond sold between two coupon payments 275 Chapter 12 Time weighted return and dollar weighted return 288 Chapter 13 Investment year & portfolio method ............... 303 Chapter 14 Short Sales ................................................................... 305 Chapter 15 Term structure of interest rate, spot rate, forward rate, and arbitrage............................................................. 316 Chapter 16 Macaulay duration, modified duration, convexity 341 Chapter 17 Immunization.............................................................. 376 Chapter 18 Cash flow matching.................................................. 394 Value of this PDF study manual...................................................... 401 About the author................................................................................. 402

3

4

http://actuary88.com

Chapter 1 strategy

Exam-taking and study

Read this chapter before opening your textbooks! This chapter requires some knowledge about the time value of money, annuity, and loan amortization. If you don’t know these concepts, don’t worry. Just skip the detailed math calculations and focus on the main ideas in this chapter. Later on, after you understand the time value of money, annuity, and loan amortization, come back to this chapter and go through the math. It’s critical that you understand the essence of this chapter before you rush to read the textbooks.

A tale of two Exam FM takers, Mr. Busy and Mr. Lazy It was the best of the times, it was the worst of times, it was the age of being lazy, it was the age of being busy, it was the epoch of passing Exam FM, it was the epoch of failing Exam FM, it was the hope of getting ASA, it was the despair of going nowhere. Two actuarial students, Mr. Busy and Mr. Lazy, are both preparing for Exam FM. They have the same height and weight. They have the same level of intelligence. As a matter of fact, they are similar about almost everything except that Mr. Busy is very busy and Mr. Lazy is very lazy. Mr. Busy and Mr. Lazy both would have a worry-free life if they don’t need to amortize a loan with geometrically increasing payments. Challenge -- loan amortization with geometrically increasing payments Mr. Busy and Mr. Lazy both love standard annuity problems that require the use of memorized formulas such as an i and an i . They both hate geometrically increasing annuity and loan amortization problems. They would gladly solve one hundred standard annuity problems than amortize a messy loan. Sadly though, SOA loves to test the problems that Mr. Busy and Mr. Lazy hate.

5

Finally the exam day has come. Mr. Busy and Mr. Lazy walk into the exam room. The 1st problem in the exam is about increasing annuity and loan amortization. Problem 1 Date of loan Amount of loan Term of loan Payments Interest rate

1/1/2005 $150,000 25 years Annual payments with first payment due 12/31/2005. Each subsequent payment is 2% larger than the previous payment. 8.5% annual effective

Question: What’s the total interest paid during the first 18 payments? Mr. Busy: Though not fond of this type of problems, Mr. Busy was not a bit worried. When preparing for Exam FM, Mr. Busy bought the solution manual for Broverman’s textbook. He solved all of the practice problems in the solution manual. Some of the problems in the solution manuals are geometric annuity and loan amortization problems. In addition, Mr. Busy bought another manual with tons of practice problems and solved all the practice problems in the manual. Mr. Busy felt ready to tackle this problem. He solved over a thousand practice problems. Surely some of the problems he solved were about loan amortization where payments were geometrically increasing. He started to mentally search for how he solved such problems in the past, hoping to recall a quick solution. To his dismay, Mr. Busy couldn’t remember any quick solutions to loan amortization with geometrically increasing payments. Though he solved many problems before the exam, Mr. Busy was always in a rush to solve the next practice problem. He never had the time to condense his solutions to quickly recallable solutions ready to be used in the exam. He didn’t even have the time to thoroughly understand the basic concept behind loan amortization and behind the present value calculation of a geometrically increasing annuity. He was always in a big hurry to solve more practice problems. Time seemed to go much faster in the exam room. And the pressure was keen. 5 minutes passed. Mr. Busy was going nowhere. Reluctantly, Mr. Busy abandoned this problem and moved to the next one.

6

Mr. Lazy: Unlike Mr. Busy, Mr. Lazy has a lazy approach to loan amortization. Mr. Lazy realized that loan amortization and geometric annuity problems were repeatedly tested in the past. Year after year, SOA asks candidates to amortize a loan. Sometimes the loan to be amortized has level payments; other times the payments are arithmetically or geometrically increasing or decreasing. Loan amortization and geometric annuity problems are so predictable that Mr. Lazy suspected that SOA would test it again this year when he takes the exam. Mr. Lazy starts to strategize: 1. SOA loves to test loan amortization. Such a problem is doomed to occur when I take the exam. 2. Loan amortization is nasty, especially when the payments are geometrically increasing or decreasing. It’s hard for me to figure it out from scratch in the heat of the exam. 3. I’m lazy. I want to pass Exam FM with least effort. Mr. Lazy’s conclusion: 1. Before the exam, I’ll design a standard cookie-cutter solution to loan amortization with geometrically increasing payments. This way, I don’t have to invent a solution from scratch in the exam. 2. I’ll make my solution less than 3 minute long; 3 minutes is pretty much all the time I have per question in the exam. 3. I’ll walk into the exam room with the 3 minute solution script ready in my head. I’ll use this script to solve a nasty loan amortization problem 100% right in 3 minutes under pressure. Result: 1. Mr. Busy got a 5 in the exam. Though he solved hundreds of practice before the exam, he never tried to build any reusable solution scripts to any of the commonly tested problems in Exam FM. As a result, every repeatable problem tested in the exam became a brand new problem, which he must solve from scratch. This, in turn, makes his solution long and prone to errors. Sorry, Mr. Busy. Good luck to your 2nd try for Exam FM. 7

2. Mr. Lazy got a 6 in the exam. He didn’t bother to solve any practice problems in any textbooks. Nor did he buy old SOA problems dug up from the graveyard. He just downloaded the Sample FM Questions and May and November 2005 FM exam from SOA website. Then for every problem tested in the Sample FM Exam and 2005 FM exam, he built a reusable 3 minutes solution script. Then when he was taking FM exam, he solved all of the repeatable problems using his scripts. Of course, SOA threw in some new problems, to which Mr. Lazy simply guessed the answers. Nice job, Mr. Lazy. See you in Exam M. Lessons to be learned from Mr. Busy and Mr. Lazy: Good exam takers solve problems. Great exam takers build 3 minute solution processes (i.e. scripts). For example, good candidates can solve many integration problems using integration-by-parts: x

x 2 e x dx,

x 2 e 2 dx,

x

x

x 2 e 3 dx,

x 2 e 4 dx, ...

However, because the integration-by-parts is a complex and error-prone process, those candidates who use this method often fluster in the heat of the exam. Great exam takers, on the other hand, focus on building a flawless solution process. Prior to the exam, they built the following generic solution: + a

x2

1

e

x/

dx =[(a + )2 +

2

]e

a/

After getting this generic process right, great exam takers simply apply + 1 x/ this generic solution to every integration problem x2 e dx and a

are able to solve similar problems 100% right in a hurry. Why building a process is superior to solving problems With a generic process, if you have solved one problem, you have solved this type of the problems once and for all. In contrast, if you solve 99 individual problems without a building generic process, you’re never sure that you can solve the 100th problem correctly.

8

Building process, not solving problems, is the most efficient way to pass SOA exams, especially when you are short of study time. Follow this study method and rigorously build a flawless solution process for each of the previously tested FM problems. Next, test your solution process in the exam condition and solve all the previously tested FM problems 100% right. Use this study method for Exam M and C. You’ll zip through tough exams with a fraction of study time while other busy folks get stuck in one exam for years. It may take you a little while to get used to this process-oriented study method. Though you may feel insecure when other candidates boast of having solved 1,000 practice problems, please be assured that this approach is far superior. This manual is written to teach you how to build a generic process to solve SOA FM problems. It’s not a book to give you 500 problems for you to solve.

Truths about Exam FM 1. To pass Exam FM, you need to learn how to solve problems in a hurry under pressure. In many professions, the difference between an expert and an amateur is that an expert can solve a routine problem flawlessly in a hurry under pressure, while an amateur can solve a problem right only under no time constraints. For example, an expert car mechanic can change a flat tire flawlessly in less than 15 minutes. In comparison, amateurs like me can change a flat tire only after several hours. Similarly, those who pass FM can generally solve a complex problem in 3 minutes under the exam pressure, while those who fail might be able to solve a complex problem perhaps in twenty minutes. 2. Understand what’s going in the real world. One common mistake in preparing for SOA exams at all levels is to treat business problems as pure math problems. For example, when studying short sales, many candidates simply solve one short sale problem after another without really understanding what’s going on in a short sale. If you don’t understand the business essence in a short sale, solving problems is garbage in, garbage out and you don’t learn much. When learning a business concept such as short sales, pricing of a bond between two coupon dates, immunization, cash flow matching, try to understand what’s going on the real world. Think through the business meaning. This way, you’ll find that difficult formulas begin to make sense. You’ll be able to solve problems must faster.

9

3. Always solve problems systematically. Research indicates that experts become experts because they always use systematic approaches to problem solving. They never solve problem haphazardly. 4. When preparing for Exam FM, focus on building a flawless solution process to all of the previously tested problems, not on aimlessly solving one problem after another with a shaky process. If you solve a great number of problems (including SOA problems) with shaky process, you’ll make the same misstate over and over. In contrast, if you have a correct process, you’ll find the right answer without the need to solve many problems. 5. Simplify fancy jargon and complex formulas into simple ones. While talking fancy and thinking fancy may impress lot of amateurs, talking simple and thinking simple are the key to solving thorny problems 100% right in a hurry under pressure. Common sense concepts and simple solutions are always the easiest solutions to remember and use in the heat of the exam. Complex and unintuitive concepts and formulas are prone to errors. For example, many candidates waste their time memorizing the fancy phrase “annuities payable more (or less) frequently than the interest is convertible” and the related complex formulas. What they should have done is to simplify complex annuities into simple annuities and throw away the fancy phrase and complex formulas once for all. 12 Example. The interest rate is i ( ) = 12% , but the annuity payments of $1 are made quarterly in arrears for 2 years. If you need to calculate the present value of this annuity, use the payment frequency (quarterly) as the interest compounding period and calculate the quarterly effective interest rate:

i (12) 1+ 12

3

1 = 1+

12% 12

3

1 = 3.03%

After using quarterly as the compounding period, the original annuity becomes a standard immediate annuity with 8 quarterly payments. The present value of this annuity is a8 3.03% . This approach is far better than using the following complex formula:

10

a(m) = n i

1 vn m i( )

6. Rely on recalling a pre-built 3 minute solution script to solve repeatable problems in FM. Three minutes is like the blink of an eye in the heat of the exam. In three minutes, most people can, at best, only regurgitate solutions to familiar problems. Most likely, they cannot invent a fresh solution to a previously unseen type of problem. Inventing a solution requires too much thinking and too much time. In fact, if you find yourself having to think too much in the exam, prepare to take Exam FM again.

How to study hard yet fail the exam miserably 1. Walk into the exam room without a mental 3 minute solution script, hoping to invent solutions on the spot. This is by the far the most common mistake. Let’s look at a few common myths: Myth #1 I don’t need a script. I think fast on my feet. #2 I don’t need a script. I’ll outperform myself in the exam.

#3 I don’t need a script. If I solve hundreds of practice problems before the exam, a solution will automatically come to me when I’m taking the exam.

Reality Check Why putting yourself on the spot when you can easily come up with a script ahead of time? Never take any unnecessary risks. There’s no such thing as outperforming yourself in the exam. You always underperform and score less than what your knowledge and ability deserve. This is largely due to the tremendous amount of pressure you inevitably feel in the exam. If you don’t have a script ready for an exam problem, don’t count on solving the problem on the spur of the moment. A solution may come to you automatically, but such a solution is often crude, complex and prone to errors. Even if you have solved a great number of practice problems, you still need to reduce your solutions to a easily repeatable 3 minute process.

2. Walk into the exam room without mastering SOA problems. A Chief Executive Officer was about to retire. He asked for the three most promising candidates to come to his office for a quiz. After the three candidates arrived, the CEO asked the first candidate, “How much is one plus one?” “One plus one is two, sir,” replied the 11

first candidate. The CEO shook his head disappointedly. He turned to the second candidate and asked the same question. The second candidate replied, “One plus one is three, sir.” Once again, the CEO shook his head. Finally, he turned to the third candidate and asked again, “How much is one plus one?” The third candidate replied, “How much do you want it to be, sir?” The CEO smiled and appointed the third candidate as the next CEO. Key points • To pass Exam FM, you need to tell SOA what it wants to hear. SOA does not want you to be creative. SOA wants you to demonstrate understanding of core concepts through a standard and methodical solution in keeping with SOA format. •

If you master SOA problems, you pass FM; if you do not, you fail.



While practice problems in textbooks, study manuals, or seminars are useful, always master SOA problems before mastering any other problems.

3. Solve hundreds of practice problems (even SOA problems) without generating reusable solutions. In every exam sitting, there are always busy candidates who take great pride in solving hundreds of SOA problems administered many years back. While solving problems are necessary for passing Exam FM, solving too many problems adds little value. Here is why: •

Solving too many problems encourages “garbage in, garbage out.” When a candidate is busy solving a great number of practice problems, often the focus is on searching for any solution that magically produces the correct answer provided in the book (often the book merely provides an answer with little explanation). This encourages problem-solving without fully understanding the nuances of the problem. If your goal is to solve 800 practice problems in 3 months, you miss the point of fully understanding core concepts and problems.



Solving too many practice problems exaggerates your ability. When a candidate focuses on solving hundreds of practice problems, he often does not put himself under exam-like conditions. This almost always leads to inefficient solutions, solutions that look good on paper but fall apart in the heat of the exam.

12

Recommended study method 1. Sense before study. Before opening any textbooks, carefully look at Sample FM and get a feel for the exam style. This prevents you from wasting time trying to master the wrong thing. 2. Quickly go over the textbook and study the fundamentals (the core concepts and formulas). Do not attempt to master the complex problems in the textbooks. Solve some basic problems to enhance your understanding of the core concepts. 3. Put yourself under exam conditions and practice the previous Course 2 exams (if the problems are still on the syllabus), the Sample FM Exam, and November 2005 FM problems. (1) Put yourself under the strict exam condition. (2) Practice one exam at a time. (3) After taking a practice exam, take several days to analyze what you did right and what you did wrong (don’t do this in a hurry). (4) For each problem in the practice exam, build a reusable 3 minute solution script. (5) Take the same practice exam the 2nd time, using your 3 minute solution script. This puts your 3 minute solution script to test. Find which script works and which doesn’t. Improve your 3 minute solution scripts. (6) Take the next SOA practice exam, repeating Step (1) to Step (5) listed above. You will have more and more 3 minute solution scripts. You will continue refine your 3 minute solution scripts. 4. Work and rework Sample FM Exam and any released FM exams until you can get them right 100%. Continue refining your 3 minute solution scripts. Mastering Sample FM Exam and newly released FM exams is the foundation for passing FM. 5. Don’t worry about solving the same FM problems over and over. No candidates, however intelligent, can over-solve SOA FM exams. Besides, if you really put yourself under the exam condition, it’s highly unlikely that you’ll memorize the answer to a previously solved problem. 6. Never, never walk into the exam room without being able to solve Sample FM problems and newly released FM exams 100% right. 7. Set up a study schedule and follow it through.

13

8. In the two weeks prior to the exam date, dry run Sample FM Exam and May 2005 FM exam, even though this may be your 4th or 5th dry run. You can never practice SOA FM problems too much.

How to build a 3 minute solution script Building a 3 minute solution script is critical to passing Exam FM. Let’s look at a few examples. They are not necessarily the best, but they really work in the heat of the exam. And feel free to create your own solution scripts. 3 minute solution script example #1 -- loan amortization where payments are level Traditional method (3 minute solution script) A loan is borrowed at time zero. It is repaid by n level payments of X , the 1st payment occurring at t = 1 . In other words, a loan is repaid through an n year annuity immediate with level payments of X . Question – how to split each level payment X into a principal portion and the interest portion? Time t Cash flow Principal Interest

0

1

2

$X $X n Xv Xv n 1 X Xv n X Xv n

… … ... 1

...

k $X Xv n +1 k X Xv n +1



k

n $X Xv X Xv

Please refer to the textbook to understand why the principal portion is indeed Xv n , Xv n 1 , Xv n +1 k , Xv at t = 1, 2, k , n respectively. You should walk into the exam room with this rule memorized in your head. Then if an exam problem asks you to split a level payment into principal and interest, you don’t need to calculate the answer from scratch. You simply apply this memorized script and quickly find the answer. Improved 3 minute solution script --- Imaginary cash flow method The above script has a trouble spot. You have to memorize that the principal is Xv n +1 k at time k . However, memorizing the discount factor v n +1 k is a pain. In the heat of the exam, you might use a wrong discount

14

factor such as v n k or v k . How can you remember the correct discount factor? This leads to the imaginary cash flow method. To find the principle portion of each payment, we add an imaginary cash flow one step after the final payment (i.e. we add a cash flow of $ X at n + 1 ).

Time t

Cash flow

0

1

X

2



X

...

k



X

X v n +1



n

n +1

X

k

$X Imaginary Cash flow

Principal Interest

Xv n Xv n 1 X Xv n X Xv n

Xv n +1 k n +1 ... X Xv ...

1

k

Xv X Xv

To find the principal portion of the payment X occurring at t = k where k is a positive integer and 1 k n , we simply discount our imaginary cash flow $ X at n + 1 to t = k . The discounted cash flow X v n +1 k is the principal portion of the payment. The interest portion of the payment X occurring at t = k is X

X v n +1 k .

Do we need to come up with an intuitive explanation for this script? We don’t have to. If this method generates the correct answer, we’ll use it as our script, even though we may not have an intuitive explanation for it. Now assume that you walk into the exam room with this script in your head. And you see the following problem:

15

Problem 1 Date of loan Term of loan Payments 1st payment Loan interest

1/1/2005 30 years Annual payments of $2,000 at the end of each year 12/31/2005 6% annual effective

Question -- What’s the present value of the interest payments at 1/1/2005 over the life of the loan at a 10% annual effective interest rate? This is NOT a simple problem. If you have to figure out the solution from scratch using the prospective or retrospective method, you may have to spend five to ten minutes on it. What’s more, you are likely to make an error here and there if you solve a problem from scratch. Let’s use our imaginary cash flow script to solve this problem.

Time t

0

Cash flow

1

2

2

2

k





30

2

2v31

31

2 k

2 Imaginary cash flow

Principal Interest

31 k 2v 30 2v 29 ... 2v 2 2v30 2 2v 29 … 2 2v31

2v 2 2v

... k

...

First, to simply our calculation, we’ll use $1,000 as one unit of money. So $2,000 annual payment is 2 units of money. Next, we’ll add an imaginary cash flow of 2 at t = 31 . Then we find the principal portion of each level payment by discounting this imaginary cash flow to t = 1, 2,....,30 . The present value at 1/1/2005 of the interest payments over the life of the loan at a 10% annual effective interest rate:

(

) (

)

(

)

(

)

PV = 2 2v30 V + 2 2v 29 V 2 + 2 2v 28 V 3 + ... + 2 2v V 30

16

In the above expression, v =

(

PV = 2 V + V 2 + V 3 + ... + V 30

1 1 + 6%

) 2 (v

and V =

1 . 1 + 10%

V + v 29V 2 + v 28V 3 ... + vV 30

30

)

V + V 2 + V 3 + ... + V 30 = a30 10% = 9.42691447 v30V V 31 1.06 301.1 1 1.1 31 = = 2.92003944 V 1.1 1 1 1 v 1.06 1 2 ( 2.92003944 ) 13.01375 = $13, 013.75

v30V + v 29V 2 + v 28V 3 ... + vV 30 =

PV = 2 ( 9.42691447 )

Please note that the imaginary cash flow method also works for a loan repaid by an annuity due. I’ll let you prove it.

A loan is borrowed at time zero. It is repaid by n level payments of X , the 1st payment occurring at t = 0 . In other words, a loan is repaid through an n year annuity due with level payments of X . Question – how to split each level payment X into a principal portion and the interest portion? Time t

Cash flow

0

$X

1

$X

k



n 1

$X … $X



$X

2



Xv n

k

n

$X Imaginary cash flow

Principal Interest

Xv n Xv n 1 X Xv n X Xv n

Xv n k n … X Xv …

1

… k



Xv X Xv

17

3 minute solution script example #2 -- loan amortization where payments are NOT level Because SOA can ask us to split non-level payments into principal and interest, we need to build a 3 minute solution script for this. A loan is borrowed at time zero. It is repaid by n payments of X 1 , X 2 , …, X n at t = 1, 2,..., n respectively. Of the total payment X 1 + X 2 + ... + X k made during the first k payments, how much is the principal payment? How much is the interest payment? Time t

0

Cash flow

1

$X 1

2

$X2

k

… …



$Xk

n $Xn

3 minute solution script: Step 1 – Calculate P0 , the outstanding balance of the loan at t = 0 . Step 2 – Calculate Pk , the outstanding balance of the loan at t = k immediately after the k th payment is made. Step 3 – Calculate P0 Pk , the reduction of the outstanding balance between t = 0 and t = k . P0 Pk should be the total principal repaid during the first k payments. Step 4 – Calculate ( X 1 + X 2 + ... + X k )

( P0

Pk ) . This should the total

interest paid during the first k payments The core logic behind this script: X 1 + X 2 + ... + X k = Total payments made during the first k payments

P0

Pk

+

Interest Payment

Principal reduction

18

Problem 2 Date of loan Amount of loan Term of loan Payments Interest rate

1/1/2005 $150,000 25 years Annual payments with first payment due 12/31/2005. Each subsequent payment is 2% larger than the previous payment. 8.5% annual effective

What’s the total interest paid during the first 18 payments? Solution Time t

0

Cash flow

1

X

2

3

1.02X 1.02 2 X





18

19

1.0217 X 1.0218 X





25

1.0224 X

Step 1 – calculate the outstanding balance at t = 0 . P0 = 150, 000 Step 2 – calculate the outstanding balance at t = 18 immediately after the 18th payment is made (we discount future cash flows to t = 18 ): P18 = 1.0218 Xv + 1.0219 Xv 2 + ... + 1.02 24 Xv 7 = X (1.0218 v + 1.0219 v 2 + ... + 1.0224 v 7 )

So we need to calculate X . Because the present value of the loan at t = 0 is 150,000, we have: Xv + 1.02 Xv 2 + 1.022 Xv 3 ... + 1.0224 Xv 25 = 150, 000

X ( v + 1.02v 2 + 1.022 v3 ... + 1.0224 v 25 ) = 150, 000

v + 1.02v 2 + 1.022 v 3 ... + 1.0224 v 25 = X=

v 1.02 25 v 26 1.085 1 1.02 251.085 = 1 1.02v 1 1.02 (1.085 1 )

26

= 12.10103628

150, 000 = 12,395.6326 12.10103628

The outstanding balance at t = 18 immediately after the 18th payment is: P18 = X (1.0218 v + 1.0219 v 2 + ... + 1.02 24 v 7 )

19

1.0218 v + 1.0219 v 2 + ... + 1.02 24 v 7 =

1.0218 v 1.0225 v8 1.02181.085 1 1.02251.085 = 1 1.02v 1 1.02 (1.085 1 )

8

= 7.714226

P18 = 12,395.6326 ( 7.714226 ) = 95,622.7113 Step 3 – Calculate the reduction of the outstanding balance between t = 0 and t = 18 .

P0

P18 = 150, 000 95, 622.7113 = 54,377.2887

This is the total principal payment during the first 18 payments. Step 4 - Calculate ( X 1 + X 2 + ... + X 18 )

( P0

P18 ) . This is the total interest

paid during the first 18 payments. X 1 + X 2 + ... + X 18 = X (1 + 1.02 + 1.022 + ... + 1.0217 ) = Xs18 2%

= 12,395.6326s18 2% = 265, 419.1574

( X 1 + X 2 + ... + X 18 ) ( P0

P18 ) = 265, 419.1574 54,377.2887 = 211, 041.8687

Extend our script What if we want to calculate the total principal and interest during the 3rd, 4th , and 5th payments? In other words, how can we split X 3 + X 4 + X 5 into principal and interest? Step 1 – Calculate P2 , the outstanding balance of the loan at t = 2 immediately after the 2nd payment is made. Step 2 – Calculate P5 , the outstanding balance of the loan at t = 5 immediately after the 5th payment is made. Step 3 – Calculate P2 P5 , the reduction of the outstanding balance between t = 2 and t = 5 . This is the total principal repaid during the 3rd, 4th, and 5th payments. Step 4 – Calculate ( X 3 + X 4 + X 5 )

( P2

P5 ) . This should the total interest

paid during the 3rd, 4th, and 5th payments. The core logic behind this script: X3 + X4 + X5 = P2 P5 Total payments made during 3rd, 4th, and 5th payments

+

Interest payment

Principal reduction

20

Similarly, if you are asked to split the 3rd payment into principal and interest, you can create a script using the following logic: =

X3 3rd payment

P2

P3

+

Interest payment

Principal reduction

3 minute solution script example #3 – present value of a geometrically increasing annuity Let’s look at our solution to Problem 2, which involves a geometrically increasing annuity. In our solution, we calculated the present value of several geometrically increasing annuities from scratch. Each time, we use the sum rule of a power series:

a + aq + aq + ... + aq 2

n 1

a aq n where q 1 = 1 q

We ask ourselves, “Why not create a script for a geometrically increasing annuity to avoid calculating its present value from scratch?” Luckily, we find a script: n payments

(1 + k )

1

1 a 1+ k n

(1 + k )

2

......

(1 + k )

n 1

i k 1+ k

j=

(1) an j = i

k 1+ k

For a geometrically increasing annuity where (1) n geometrically increasing payments are made at a regular interval; (2) the 1st payment is $1; (3) the next payment is always (1 + k ) times the previous payment. Then

21

(1) The present value one step before the 1st payment is

1 a 1+ k n

i k 1+ k

j=

. This

1 because the geometric payment pattern at one 1+ k 1 interval prior to the 1st payment is . This value also has an annuity 1+ k factor of a i k , where j is the adjusted interest rate.

value has a factor of

n j= 1+ k

(2) The present value at the 1st payment time is (1) a

n j=i k 1+ k

. The present

value has a factor of 1 because the 1st payment is 1 . This value has an annuity factor of a i k , where j is the adjusted interest rate. n j= 1+ k

To find the proof of this script, see the chapter on geometrically increasing annuity. For now, let’s focus on how to use this script. From this script, we see that the present value of a geometrically increasing annuity is always the product of a payment factor and an annuity factor: PV of geometric annuity = Payment Factor × Annuity Factor @

j =

i k 1+ k

The above script is simple yet powerful. Let’s redo Problem 2 using the geometric annuity script. Problem 2 Date of loan Amount of loan Term of loan Payments Interest rate

1/1/2005 $150,000 25 years Annual payments with first payment due 12/31/2005. Each subsequent payment is 2% larger than the previous payment. 8.5% annual effective

Q: What’s the total interest paid during the first 18 payments? Solution

22

Time t

0

1

X

Cash flow

2

3



1.02X 1.02 2 X

18

19



1.0217 X 1.0218 X





25

1.0224 X

Step 1 – calculate the outstanding balance at t = 0 . P0 = 150, 000 Step 2 – calculate the outstanding balance at t = 18 immediately after the 18th payment is made.

(

P18 = ( payment factor @ t = 18 ) × annuity factor @ j = 1i +kk

)

payment factor @ t = 18 is 1.0217 X ;

annuity factor @ j = 1i +kk is: a

25 18 j = i k 1+ k

P18 = 1.0217 Xa7

=a

7 j=

8.5% 2% 1.02

= a7 j =6.372549%

j =6.372549%

Next, we’ll calculate X . Because the present value of the loan at t = 0 is 150,000, we have:

(

150, 000 = PV of geometric annuity= ( payment factor @ t = 0 ) × annuity factor @ j = 1i +kk payment factor @ t = 0 is

)

X 1.02

if we extend the payment pattern to time zero, we’ll have a payment of X at time zero 1.02 annuity factor @ j = 1i +kk is: a

25 j = i k 1+ k

150, 000 =

X a 1.02 25

= a25

j =6.372549%

j =6.372549%

Using BA II Plus/BA II Plus Professional, we quickly find that

a25

j =6.372549%

= 12.34305703

23

X=

150, 000 (1.02 )

a25

= 12,395.6326

j =6.372549%

P18 = 1.0217 Xa7

j =6.372549%

= 1.0217 (12,395.6326 )( 5.50921141) = 95, 622.7113

Step 3 – Calculate the reduction of the outstanding balance between t = 0 and t = 18 . P0

P18 = 150, 000 95, 622.7113 = 54,377.2887

This is the total principal payment during the first 18 payments. Step 4 - Calculate ( X 1 + X 2 + ... + X 18 )

( P0

P18 ) . This is the total interest

paid during the first 18 payments.

X 1 + X 2 + ... + X 18 = X (1 + 1.02 + 1.022 + ... + 1.0217 ) = Xs18 2% = 12,395.6326s18 2% = 265, 419.1574

( X 1 + X 2 + ... + X 18 ) ( P0

P18 ) = 265, 419.1574 54,377.2887 = 211, 041.8687

You see that our calculation is much faster if we use the geometric annuity script. 3 minute solution script example #4 – present value of an arithmetically increasing/decreasing annuity Among all the annuities, an arithmetically increasing or decreasing annuity is most prone to errors. Errors associated with an arithmetically increasing or decreasing annuity often come from two sources:



Incorrectly identify the cash flow pattern. In an increasing or decreasing annuity, we can easily find the precise cash flow at each time point. However, to calculate the present value of an increasing or decreasing annuity, knowing the precise cash flow at each time is not enough. The present value formula forces us to find the pattern by which the cash flows change over time. Identifying the cash flow pattern is much harder than identifying cash flows.

24



Incorrectly apply the present value formula. Even if we find the cash flow pattern, we still have to use the complex and awkward formula to calculate the present value. Any mistake in using the formula will destroy our previous work.

If calculating the present value of an increasing or decreasing annuity sounds complex, imagine solving such a problem in the exam condition where your heat beats fast and your hands tremble. How can we find a magic script that will enable us to calculate the present value of an arithmetically increasing or decreasing annuity 100% in a hurry under pressure? The bad news is that we can’t always find a script. The good news is that we can find a script that works most of time. 3 minute solution script -- PV of increasing/decreasing annuity If the # of cash flows is NOT enormous (fewer than 50 for example), simply enter the cash flows into BA II Plus/BA II Plus Professional Cash Flow Worksheet. Problem 3 22 end-of-the year deposits to a fund

Year 1 Year 2 to Year 11 Year 11 to Year 15 Year 15 to Year 22

Interest rate

$50,000 Annually increasing by $5,000. Level Annually decreasing by $6,000

8% annual effective

Calculate the present value of the fund. Solution We don’t want to spend lot of time analyzing the cash flow pattern or using the complex formula. We just identify all of the cash flows: Time t Cash flow

0 $0

1 $50

2 $55

3 $60

4 5 6 7 8 9 10 11 $65 $70 $75 $80 $85 $90 $95 $100

Time t 11 12 13 14 15 16 17 18 19 20 21 22 Cash flow $100 $100 $100 $100 $100 $94 $88 $82 $76 $70 $64 $58

25

In the above table, we use $1,000 as one unit of money to simplify our calculation. Next, we enter the above cash flows into Cash Flow Worksheet: Cash flow Frequency

CF0 C01 C02 C03 C04 C05 C06 C07 C08 C09 C10 C11 $0 $50 $55 $60 $65 $70 $75 $80 $85 $90 $95 $100 F01 F02 F03 F04 F05 F06 F07 F08 F09 F10 F11 1 1 1 1 1 1 1 1 1 1 6 C12 C13 C14 C15 C16 C17 C18 $94 $88 $82 $76 $70 $64 $58 F12 F13 F14 F15 F16 F17 F18 1 1 1 1 1 1 1

Cash flow Frequency

Set I=8 (i.e. the interest rate is 8%). You should get: NPV = 797.80666333 (units)= $797,806.66333 Problem 4 Annuity #1

Annuity #2

interest rate

An amount payable at the end of each quarter beginning with a $2,000 payment on 3/31/2005. Each subsequent payment is 1.5% larger than the previous payment. The annuity pays for 15 years. An amount payable at the end of each year beginning with an X payment on 12/31/2005. Each subsequent payment is $30 less than the previous payment. The annuity pays for 20 years. 9% annual effective

On 1/1/2007, the present value of the remaining Annuity #1 equals the present value of the remaining annuity #2. Calculate X . Solution This problem is conceptually simple. However, tracking down the timings and amounts of the cash flows of each annuity is a nightmare. Unless you use a systematic approach, you’ll make errors here and there.

26

First, we draw a cash flow diagram: Annuity #1 Time t (Year)

0

1

1 4

2 4

3 4

1

2005

2

1 4

2 4

3 4

3

1 4

1

2006

2 4

3 4

...

15

1

2007

15 2 = 13 years remaining 13 4 =52 payments remaining Payment factor =2,000(1.015)7 Annuity factor = a52 j 1

(1 + 9% ) 4 j=

1

1.5%

1 + 1.5%

PV of remaining payments =2,000(1.015)7 a52

j

Annuity #2 Time t (Year)

0

1

2005

2

2006

X

3

2007

X

30

4



20

2008

X

2 ( 30 )

X

3 ( 30 ) ... X 19 ( 30 )

Annuity #2

20 2 = 18 years remaining 18 payments remaining

27

We’ll break down the remaining payments of Annuity #2 into two streams: Time t (Year)

0

1

2005

2

2006

3

2007

4



20

2008

X

X

3

4

X

...

Xa18 9% Time t (Year)

0

1

2005

2

2006

2007



20

2008

2 ( 30 )

3 ( 30 )

...

19 ( 30 )

PV of an increasing annuity PV of the remaining payments = Xa18 9% - PV of a increasing annuity To pin down the PV of the increasing annuity, we’ll use BA II Plus/BA II Plus Professional Cash Flow Worksheet. Enter the following cash flows:

Cash flow Frequency

Cash flow Frequency

CF0 C01 C02 C03 C04 C05 C06 C07 C08 C09 $0 $2 $3 $4 $5 $6 $7 $8 $9 $10 F01 F02 F03 F04 F05 F06 F07 F08 F09 1 1 1 1 1 1 1 1 1 C10 C11 C12 C13 C14 C15 C16 C17 C18 $11 $12 $13 $14 $15 $16 $17 $18 $19 F10 F11 F12 F13 F14 F15 F16 F17 F18 1 1 1 1 1 1 1 1 1

In the above cash flow table, we used $30 as one unit of money to simplify our data entry.

28

Next, in Cash Flow Worksheet, set the interest rate to 9%. You should get: NPV=72.39722563. Because one unit of money represents $30, so the real NPV is (30) 72.39722563=2,191.916769 You might groan at such a solution as ugly. However, this quick and dirty solution has its beauty: it provides a mechanic solution to a complex problem. It really works in the heat of the exam where you are stressed. If you really want to use the PV formula, you can add a cash flow of $30 at t = 0 . This gives you a clean and nice increasing annuity. Time t (Year)

0

2005

1

2006

2007

PV = 30 ( Ia )19



3 ( 30 )

...

18

2008

2 ( 30 )

30 30

2

19 ( 30 )

30

So the present value of the decreasing annuity at time zero is: 30 ( Ia )19

a19 19v19

30 = 30

d

1 = 2,171.916769 @ i = 9%

Finally, we are ready to solve for X :

2,000(1.015)7 a52 j = Xa18 9% 2,171.916769 1

where j =

(1 + 9% ) 4

1

1.5%

1 + 1.5%

Solving the equation, we have:

X = 11,354.44396

29

Key points to remember about 3 minute solution scripts: •

Scripts prevent us from reinventing a wheel again and again. Using scripts, we invent a wheel once; next time, we simply reuse the wheel invented before.



Scripts cut to the chase and quickly get to the core of the problem at hand, enabling us to solve a complex problem 100% right in a hurry under pressure.



Don’t blindly copy solutions from SOA exams, textbooks, study manuals, or seminars. Customize those solutions so they are simpler, faster, and less prone to errors. The scripts created by you are the best scripts



Keep refining your scripts.

How to eliminate errors Of all the SOA exams, Exam FM requires us to have the least amount of creativity. The textbooks have given us a myriad of formulas for virtually every concept from level annuity, to varying annuity, to loan amortization, to bond evaluation, to duration, to convexity. We need only to memorize these formulas and apply them to specific situations. Then why can’t everyone pass Exam FM? Why didn’t everyone pass Course 2? Answer: errors. Even if we have memorized all the formulas, mastered all the concepts, and created powerful 3 minute solution scripts, we may still not be able to solve every problem right. After all, we are fallible. We are especially fallible when we have to solve a problem in a hurry and under stress. To convince yourself that you are fallible, put yourself under the exam condition and take a Course 2 exam administered in the past. See how many incorrect answers are caused by your silly mistakes. Truth #1: To pass Exam FM, you need to do two tasks: (1) learn new things and expand your knowledge, (2) reduce errors in areas you already know and defend your knowledge.

30

Truth #2: Most candidates spend too much time learning new things; they spend too little time reducing errors. Truth #3: If you are a borderline student knowing the minimum amount knowledge on FM, your chance to pass Exam FM solely depends on whether you can eliminate errors. Say you solved only 70% of the exam leaving the remaining problems blank. If you made zero error in your solutions, you surely passed FM if the passing standard was solving 66% of the problems right. How can we reduce errors? Let’s look at an everyday situation. Situation Grocery shopping Error Forgetting to buy certain items Error elimination strategy Don’t trust your memory. Rigorously use a shopping list. Check off an item from your list once you have bought it. Before leaving the store, make sure that you have checked off all the items in your list. How to eliminate calculation errors – Use a systematic approach 1. Don’t think that “Others will make mistakes but I won’t” or “If I just solve hundreds of practice problems, I will never make mistakes again.” Remember that mistakes will surely creep in like a thief unless you use a systematic approach to problem solving. 2. Trust that if you understand the concept and if you use a systematic approach, the right solution will surely emerge. 3. Under a systematic approach, always move from general principles to specific answers. For example, when solving a cash flow problem, you first set up the fundamental equation such as the present value of the cash inflows is equal to the present value of the cash outflows. Then you calculate the missing item (the interest, a specific cash flow, or the number of payments, etc) by solving this equation. 4. Solve a problem step by step; don’t jump steps. For example, if you have a choice of solving a problem in one giant step or solving the problem in three small steps, use the three-step approach to avoid errors.

31

5. Unless a problem is overly simple, always draw a cash flow diagram. This avoids the common off-by-one error (forgetting one payment or adding a payment that does not exist). 6. When taking the exam, use calculators to their full power and delegate calculations to your calculators as much as you can. When solving practice problems, however, solve them in two ways. First, don’t use a calculator’s built in functionalities (such as BA II Plus Professional’s TVM or modified duration) and solve the problem manually; this sharpens your conceptual thinking. Next, maximize the use of your calculator and minimize your work; this quickens your solution. 7. Don’t do mental math; use your calculator for even the simplest calculations. In the heat of the exam, people tend to make silly mistakes for even the simplest calculations. For example, if you need to calculate 11 - 9.4 (eleven minus nine point four) in the exam. Is the answer 1.6 or 2.6? If you do mental math, you are likely to miscalculate. So don’t be creative. Just let your calculator do the math for you. 8. When the payment frequency differs from the interest compounding frequency, use the payment frequency as the interest compounding frequency. This greatly reduces the number of formulas you have to memorize. 9. Never transfer intermediate values between the scrap paper and your calculator. Always store the intermediate values in your calculator’s memories. Use symbols to keep track which number is in which memory. Example of using a systematic approach Problem 1 John buys the following bond: Face amount $1,000 Term to maturity 20 years Coupons 8% payable semi-annually Yield to maturity 10% With 5 years to maturity and immediately after receiving the 30th coupon, John sells the bond at a price yielding 9% annual effective to the buyer. Calculate John’s annual effective return on his investment in the bond.

32

Solution This problem seems difficult at first glance. However, let’s solve it step by step using a systematic approach. First, we draw a cash flow diagram to help us visualize the information given to us. Because the cash flows occur every 6 months, we will use 6 months as the interest compounding period. We’ll convert an annual effective interest rate to a 6-month effective rate.

Unit time = 6 months Time t

0

Cash flow

1

2

3

$40

$40

$40

...

30

31

32



39

40

$40

$40

$40



$40

$40 $1,000

John owns these 30 cash flows

John sold these 10 cash flows @9%

John bought the bond @10% John got out of the bond

We are still not sure how to calculate John’s return for investing in the bond; we don’t have a formula ready. What should we do? In a systematic approach, we move from general principles to specifics. How do we calculate an investment return in general? General principle We deposit $ X ( 0 ) at time zero in a fund. At time t the fund grows to

$ X ( t ) . Our return for investing in the fund during the time horizon [ 0, t ] is r . We can solve for r as follows:

X ( 0 )(1 + r ) = X ( t ) t

X (t ) r= X ( 0)

1 t

1

33

Let’s apply this general principle to John’s investment. To find John’s return, we need three data:

X ( 0 ) - John’s initial out-of-pocket expense to buy the investment t - John’s investment horizon X ( t ) - John’s total wealth at the end of the investment horizon John’s cost for buying the bond is the bond’s market price. We know that a bond’s fair market price is the present value of the bond’s cash flows discounted at YTM (yield to maturity). X ( 0 ) = 40a40 + 1,000v 40

1

@ i = (1 + 10% ) 2 1

How long did John investment his money in the bond? He bought the bond at time zero. He got out of the bond at time 30. So his investment horizon is t = 30 . Please note that the time unit is still 6 months. Finally, we need to find X ( 30 ) , John’s total wealth at time 30. If John counts his money at time 30, how much does he have? John’s total wealth at time 30 comes from two sources: (1) reinvesting 30 coupons, (2) selling 10 cash flows at 9%.

X ( 30 ) = 40s30

1

@ i = (1 + 10% ) 2 1 + 40a10 + 1, 000v10

The accumulated value at t =30 of John reinvesting 30 coupons @10% per year

1

@ j = (1 + 9% ) 2 1

John's price for selling 10 cash flows at t =30 @9% per year

In the above equation, we use 10% as John’s annual effective reinvest rate. Though the reinvestment rate can differ from the YTM, this problem doesn’t specifically give us John’s reinvestment rate. As a result, we assume that John reinvests his 30 coupons at YTM of 10%. Finally, we are ready to calculate John’s return r . However, we need to proceed cautiously. We’ll perform many messy calculations to get r . If we make a silly mistake in one calculation, all the good work we have done so far is ruined. To flawless perform messy calculations, we will follow 3 rules:

34



Rule #1 In the exam, delegate work to our calculator as much as we can. Our calculator is our obedient servant, who always executes our orders precisely and flawlessly. As long as our orders are clear and unambiguous, we can rest assured that our calculator will get the job done 100% right.



Rule #2 In doing practice problems, solve the problems twice. First time we solve the problem manually using formulas. Second time we let the calculator do most of the work (just as we do in the exam).



Rule #3 Don’t transfer intermediate values between our calculator and the scrap paper. Store all the intermediate values in our calculator’s memories and systematically track which memory stores which value.

Let’s apply these rules. We’ll solve the problem with two methods. We’ll systematically track down our intermediate values in each method. The calculation procedure for Method #1 - use BA II Plus/BA II Plus Professional TVM Worksheet Step

Formula

#1 Convert annual 10% into 6-month rate

i = (1 + 10% ) 2 1 i = 0.04880885 100i = 4.88088482

1

Store in memory

Track down values stored in memory

M0

100i = M 0

There’s a slight difference between i and 100i . BA II Plus/BA II Plus Professional displays a number up to 8 decimal places. If a number has more than 8 decimal places, the calculator’s rounds the number to 8 decimal places in the display. However, internally BA II Plus/BA II Plus Professional stores a number in 13 decimal places, even though it displays up to 8 decimal places. Please note that the display does NOT affect how a number is stored internally in the calculator.

35

#2 Calculate X ( 0 ) using TVM.

PMT=40, N=30, FV=1,000 I/Y=M0 (to recall M0, press “RCL 0”) X ( 0 ) = 40a40 + 1, 000v 40

M1

X ( 0) = M1

M2

100 j = M 2

@i

X ( 0 ) = 846.3502152

#3 Convert annual 9% into 6month rate #4 Calculate 40 s30 i

1

j = (1 + 9% ) 2 1 = 4.40306509% 100 j = 4.40306509

PMT=40, N=30, I/Y=M0 (to recall M0, press “RCL 0”)

X ( 30 )

40s

30 i

= M3

= 2, 603.829665

40s

30 i using TVM #5 PMT=40, I/Y=M2, N=10, FV=1,000 Calculate 40a10 + 1, 000v10 @ 40a10 + 1, 000v10 @ j = 967.9540424

using TVM #6 Calculate

M3

)

(

40s30 i + 40a10 + 1, 000v10 @ j

M4

40a10 + 1,000v10 @ j = M4

M5

X ( 30 ) = M 5

M6

r = M6

M7

R = M7

= M 3 + M 4 = 3,571.783707

(Press “RCL 3” and “RCL 4” to recall M3 and M4 respectively) #7 Calculate r , John’s return in a 6-month period

r=

X ( 30 ) X (0)

M5 r= M1

1 30

1 30

1

1 = 4.91667086%

(Press “RCL 5” and “RCL 1” to recall M5 and M1 respectively) #8 Convert r into an annual rate R

R = (1 + r )

2

1 = (1 + M 6 )

2

1

R = 10.07507825%

(Press “RCL 6” to recall M6)

So John’s annual effective return for investing in the bond is 10.08%.

36

The calculation procedure for Method #2 (formula driven approach) - use general functions of BA II Plus/BA II Plus Professional Step

Formula

#1 Convert annual 10% into 6-month rate #2 Calculate a40 i

i = (1 + 10% ) 2 1 i = 4.88088482%

1

a40 i =

1 v 40 i

(1 + M 0 )

1

a40 i =

Store in memory

Track down values stored in memory

M0

i = M0

M1

a40 i = M 1

M2

X ( 0) = M 2

40

M0

a40 i = 17.44266468

#3 Calculate X ( 0 )

X ( 0 ) = 40a40 + 1, 000v 40

@i

X ( 0 ) = 40M 1 + 1, 000 (1 + M 0 )

40

X ( 0 ) = 846.3502152

#4 Convert annual 9% into 6-month rate #5 Calculate s30 i

1

j = (1 + 9% ) 2 1 = 4.40306509%

(1 + i )

30

s30 i =

s30 i

#6 Calculate a10

j

a10 j =

1

(1 + M 0 )

30

=

j = M3

M3

1

M4 s

30 i

i M0 = 65.09574162

(1 + M 3)

1 v10 1 = j

10

= M4

a10 j = M 5

M5

M3

a10 j = 7.95056640

#7 Calculate 40a10 + 1, 000v10 @ j

40a10 + 1, 000v10

@j

= 40M 5 + 1,000 (1 + M 3)

40a10 + 1, 000v10 @ j = M 6

M7

X ( 30 ) = M 7

M8

r = M8

M9

R = M9

10

= 967.9540424

#8 Calculate X ( 30 )

M6

)

(

40s30 i + 40a10 + 1, 000v10 @ j = 40M 4 + M 6 = 3,571.783707

#9 Calculate r , John’s return in a 6-month period

r=

X ( 30 ) X (0)

M7 r= M2

#10 Convert r into an annual rate R

R = (1 + r )

1 30

2

1 30

1

1 = 4.91667086% 1 = (1 + M 8 )

2

1

R = 10.07507825%

37

In the two methods above, we painstakingly track down which number is stored in which memory. This tracking system has the following advantages: •

Eliminate the need to transfer numbers back and forth between a calculator and a scrap paper.



Eliminate the errors caused by transferring numbers between a calculators and a scrap paper.



Eliminate the loss of precisions caused by transferring a fraction numbers between a calculators and a scrap paper. For example, if we have to transfer i = 4.88088482% back and forth between a calculator and a scrap paper, we feel compelled to round i to i = 4.88% . However, if we store i = 4.88088482% in a calculator’s memory and recall it whenever we use it, BA II Plus/BA II Plus Professional will store i in 13 decimal places in its internal calculations, yielding results with good precision.



Leave an audit trail, isolating good calculations from bad. For example, after arriving the final answer of R , we realized that our Step #1 calculation was wrong. To fix the error, we simply redo Step 1 calculation and reload the newly calculated i to M 0 . Next, we redo all the calculations that use M 0 , leaving the calculations that do not involve M 0 intact. In contrast, if we don’t have such a good tracking system in place, one single error will blow up all the calculations, forcing us to recalculate everything from scratch.

Problem 2 (EA-1 #1 2003) Over a 3-year period, a series of deposits are made to a savings account. All deposits within a given year are equal in size ad are made at the beginning of each relevant period. Deposits for each year are total $1,200. The following chart shows the frequency of deposits and the interest rate credited for each year. Year

Frequency of deposits

Interest rate credited during the year

1 2 3

Semi-annually Quarterly Every 2 months

d (12) = 6% i ( 3) = 8% = 7%

Calculate the value of the account at the end of the 3rd year.

38

Solution This problem is tricky. To solve it right, we need to systematically track down the timing and the amount of each deposit. To simplify our calculation, we’ll set 1 unit of money = $100. So each year, the total amount of deposits made each year is 12. Time (year)

0

Mode

Semiannual $6 6 3 (12) d = 6%

Deposit Interest rate

0.5 1

1. 1.5 25

1.7 5

2

2

Quarterly 3

3 3 i = 8% ( 3)

2 4 2 12 12

2

6 12

2

8 12

2

10 12

3

Once every 2 months 2

2

2

2

2 2 = 7%

Getting this table right is half the battle. If you can set up a table like this, you are on the right track. Next, we need to accumulate deposits year by year; the interest rates earned are different year by year. Find FV @ t = 1 of the 1st year deposits: d( ) ; the monthly 12 12

During Year 1, the monthly discounting factor is 1

d( ) 1 12 12

accumulating factor is just the reverse of the discount factor:

1

.

In other words, if you have $1 at the beginning of a month, then this $1 d( ) will accumulate to 1 12 12

1

at the end of the month. You should learn

this technique and quickly convert a nominal discount rate d ( n ) into an accumulating factor.

39

FV @ t = 1 of the 1st year deposits: d( ) 6 1 12

12

12

$6 @ t =0 accumulate for 12 months

12

6% =6 1 12

d( ) + 6 1 12 12

6

$6 @ t =0.5 accumulate for 6 months

6% +6 1 12

6

= 6 ( 0.995

12

+ 0.995

6

) = 12.55517

Next, we need to accumulate this amount through Year 2 and Year 3. The FV @ t = 3 of Year 1 deposits: i ( 3) 1+ 3

12.55517

3

accumulate 4 quarters in Year 2

e accumulate 1 year in Year 3

8% = 12.55517 1 + 3

3

e7% = 14.57175

Find FV @ t = 2 of the 2nd year deposits: Compounding period = 0.25 (quarterly) # of compounding periods = 4 The quarterly interest rate j . To find j , let’s accumulate $1 at the beginning of Year 2 to the end of Year 2. If we use the quarterly interest rate j , the accumulated value @ the end of Year 2 is (1 + j ) . On the other hand, if we accumulate $1 4

using i (3) , the accumulate value is:

i ( 3) 1+ 3

(1 + j )

4

3

8% = 1+ 3

8% = 1+ 3

3

,

3

8% j = 1+ 3

3 4

1 = 1.993406%

FV @ t = 2 of the 2nd year deposits: 3s4 1.993406% = 12.610062 Next, we accumulate this amount through Year 3

40

The FV @ t = 3 of Year 2 deposits: 12.610062

= 12.82162 e7% = 13.52439

e accumulate 1 year in Year 3

Find FV @ t = 3 of the 3rd year deposits: Compounding period = two months # of compounding periods = 6 Interest rate per period: e

2 12

1= e

7%

2 12

1 = 1.1734988%

FV @ t = 3 of the 3rd year deposits: 2s6 1.1734988% = 12.50262 Finally, we sum up the FV @ t = 3 of each year’s deposits: 14.57175 + 13.52439 + 12.50262 = 40.59876

Remember that early on we set one unit of money=$100. So the account value @ the end of Year 3 is $4,059.88. Problem 3 Bond face amount Term to maturity Coupons Redemption Yield to maturity

$100 30 years 10% semiannual Par 12.36% annual effective

You are also given the following formula to calculate a bond’s Macaulay duration: n

DMAC =

t =1 n

t CF ( t ) vt

t =1

CF ( t ) vt

In the above formula: •

CF ( t ) = the cash flow at time t (measured in # of years)



v=

1 , where i is the annual effective yield of the bond. 1+ i

41

Calculate DMAC using a systematic approach. Solution First, we draw a cash flow diagram. Because the coupons are paid semiannually but the interest given is an annual rate, we’ll use the coupon payment frequency as the unit time. Unit time = 6 months Time t

Cash flow

0

1

2



59

$5

$5

$5



$5

60

$5 $100

The interest rate per unit time is: i = 1 + 12.36% 1 = 6% ,

v=

1 1 = 1 + i 1.06 n

Next, we expand, step by step, the formula DMAC =

t =1 n

t CF ( t ) vt

t =1 60

DMAC =

t =1

1 t CF ( t ) vt 2 60 t =1

CF ( t ) vt

We add a factor of 60 t =1

60

=

1 2

t =1 60

CF ( t ) v

. t

t CF ( t ) vt

t =1

CF ( t ) vt

1 1 in the right hand side because 6 months = year. 2 2

t CF ( t ) vt = 1( 5 ) v + 2 ( 5 ) v 2 + 3 ( 5 ) v 3 + ... + 60 ( 5 ) v 60 + 60 (100 ) v 60

= 5 ( Ia )60 i + 60 (100 ) v 60 60 t =1

CF ( t ) vt = 5a60 + 100v 60

DMAC

60 1 5 ( Ia )60 i + 60 (100 ) v = 2 5a60 + 100v 60

We’ll use BA II Plus/BA II Plus Professional to solve this problem.

42

To avoid transferring intermediate values back and forth between a calculator and the scrap paper, we’ll track down what value is stored in which memory. v=

1 1 = = M 0 (so we store v in M0 ) 1 + i 1.06

( Ia )60 i = a60

a60

6%

1 v n 1 v 60 1 ( M 0 ) = = = d 1 v 1 M0

( Ia )60 i = a 60

60v 60

a60

60v 60 6%

1 v n 1 v 60 1 = = = i 6%

DMAC

=

60

= M1

M 1 60 ( M 060 )

( M 0) 6%

6%

= M2

60

= M3

60 60 1 5 ( Ia )60 i + 60 (100 ) v 1 5 ( M 2 ) + 60 (100 )( M 0 ) = = = M4 60 2 5a60 + 100v 60 2 5 ( M 3) + 100 ( M 0 )

After doing the calculation, we get DMAC = 8.69472773 Problem 4 Bond face amount Term to maturity Coupons Redemption Yield to maturity

$100 30 years 10% semiannual Par 12.36% annual effective

Calculate the bond’s duration. Solution When taking the exam, we want to delegate calculations to our calculators as much as we can. The solution to Problem 2 is complex. We want to avoid this hard core calculation in the exam if we can.

43

Before taking Exam FM, we have thoroughly researched SOA approved calculators. We know that BA II Plus Professional has a bond worksheet, which can calculate the modified duration of a bond. So we’ll let BA II Plus Professional Bond Worksheet find the modified duration of the bond. Then we convert the modified duration into Macaulay duration.

Please note that Bond Worksheet in BA II Plus does NOT have functionality for directly calculating a bond’s modified duration. To calculate the modified duration of the bond, we enter the following in Bond Worksheet. Key strokes in BA II Plus Professional (duration functionality not available in BA II Plus Bond Worksheet): 2nd Bond Enter SDT=1.0100

This activates Bond Worksheet. SDT = settlement date (i.e. purchase date of the bond)

This sets SDT=101-2000

We arbitrarily set the purchase date of the bond is to 1/1/2000. You can set the purchase date to another date such as 6/1/1988. However, 1/1/2000 is an easy number. Set coupon is 10% of par. RDT = redemption date (or bond’s maturity)

Enter CPN=10 Enter RDT=1.0130 This sets RDT=101-2030 Enter RV=100

We set RDT=1-01-2030 (the bond has a 30 year maturity).

Redemption value. Because the bond is redeemed at par and the par=100, we set RV=100. Day counting Use 360 counting method (i.e. assume every year has method 360 days and every month has 30 days). Don’t use the actual counting method. Coupon frequency 2/Y (i.e.. twice a year) Enter Set bond’s yield to maturity to 12%. YLD=2(6)=12 We need to be careful here. The yield to maturity in Bond Worksheet must be a nominal interest rate compounding at the same frequency as the coupons are paid. Because we set coupon frequency as semiannual (remember we set by 2/Y as coupon frequency), we need to enter a nominal yield

44

CPT PRI (compute price of the bond) AI=0

compounding semiannually. So we double the 6% semiannual effective interest rate to get the nominal rate compounding semiannually. Don’t enter the 6month effective interest rate of 6% as the yield to maturity .You need to remember this in order to use Bond Worksheet. We get PRI=83.83857229. This is the PV of the bond cash flows discounted at 12.36% annual effective (or 6% per 6 months). Accumulated Interest ( AI ) is zero if bond is sold exactly at a coupon date. When a bond is sold between two coupon payments, the bond buyer must pay the original bond holder a portion of the coupon. AI is calculated as follows: AI=Coupon ×

DUR

Days between settlement and last coupon payment Total days between two coupon payments

This is a minor function of Bond Worksheet. Don’t worry about this. DUR=8.20257333 Remember this is the modified duration calculated using the Wall Street method. This is different from the textbook definition.

DMODWall Street = 8.20257333 Please note that the bond worksheet in BA II Plus/BA II Plus Professional uses the Wall Street convention in quoting a bond. In Wall Street, a bond’s yield to maturity is quoted as a nominal yield compounding as frequently as coupons are payable per year. In contrast, in Exam FM, the yield to maturity of a bond is almost always quoted as an annual effective interest rate. As a result, if you use Bond Worksheet to do any calculations, you’ll need to enter the nominal yield to maturity, not the effective yield. This is an annoying detail to remember. So I recommend that you use Bond Worksheet only to calculate the bond’s duration. Don’t use it to calculate the bond price; you can calculate the bond price using TVM Worksheet.

45

Please also note that BA II Plus Professional uses the Wall Street convention to calculate the bond’s modified duration:

DMODWall Street =

DMAC

YLD ( 1+ m

m)

(using nominal yield)

In the above equation, m is the coupon frequency and YLD( ) is the nominal yield to maturity compounding at the coupon frequency. m

In contrast, the textbook by Broverman use the following method:

DMODTextbook =

DMAC 1+ r

(where r is the annual effective yield)

Please note that r is really the opportunity cost of capital (i.e.. the interest rate earned by investors if they invest in other securities). As a result, the yield to maturity is really the prevailing market interest rate; the investors can earn the prevailing market interest rate if they invest in other securities. Finally, we’ll convert the Wall Street modified duration into Macaulay duration.

DMAC = DMOD 1 +

YLD( m) 12% = 8.20257333 1 + = 8.69472773 m 2

If you don’t understand the Macaulay duration and the modified duration, that’s OK; I will explain them later. For now, you just need to know that you can calculate Macaulay duration using BA II Plus Professional Bond Worksheet. Because BA II Plus does not have the modified duration functionality, you might want to buy a BA II Plus Professional calculator. Please note that the modified duration function in BA II Plus Professional Bond Worksheet works only for a bond. A bond has a neat cash flow patterns. If an exam problem gives you a stream of random cash flows (such as $100 at t=1, $104 at t=2, $200 at t=3,…), then you can’t use Bond Worksheet to calculate the duration. Later in this book, I’ll explain how to calculate the duration of a stream of random cash flows.

46

Lesson learned from Problem 5 Calculators can save you time. However, you need to know how to properly use a calculator. In this problem, if you don’t know that you need enter a nominal yield, if you don’t know that the duration generated by Bond Worksheet is a Wall Street version of the modified, you’ll get a wrong result. There is extra complexity in using BA II Plus Professional Bond Worksheet to calculate the duration of bond when a bond is not redeemed at par. Don’t worry about it now. We’ll pick up this topic later when we study, in more depths, the concept of duration and convexity.

47

http://actuary88.com

Chapter 2

Getting started

To best use this study manual, please have the following items ready: 1. Textbooks recommended by SOA. Only one textbook of the four books is required. • Mathematics of Investment and Credit • Mathematical interest theory • Theory of interest • Financial Mathematics – A Practical Guide for Actuaries and other Business Professionals 2. Have a BA II Plus or BA II Plus Professional calculator. BA II Plus Professional is preferable because it has several new features (such as calculating modified duration) not found in BA II Plus 3. Download Sample FM Questions, Sample questions for Derivatives Markets, May 2005, and November 2005 FM Exam, all from the SOA website. 4. Review online discussion forums about FM. There are two major discussion forums: www.actuarialoutpost.com and http://www.actuary.com. Here are some discussion threads: http://www.actuarialoutpost.com/actuarial_discussion_forum/sh owthread.php?t=153162 http://www.actuarialoutpost.com/actuarial_discussion_forum/sh owthread.php?t=140223 http://www.actuary.com/actuarial-discussionforum/showthread.php?t=10394 http://www.actuary.com/actuarial-discussionforum/showthread.php?t=8575

48

http://actuary88.com

Chapter 3

FM Fundamental

Time value of money youtube video: http://www.youtube.com/watch?v=BXm5mZqMp6Y http://www.youtube.com/watch?v=ks33lMoxst0 http://www.youtube.com/watch?v=4LSktB7Pk http://www.youtube.com/watch?v=3SgVUlEcOBU http://www.youtube.com/watch?v=6WCfVjUTTEY •

The value of money depends on not only the amount, but also when we receive it.



We all intuitively understand the time value of money. If someone owes us money, we want the money now; if we have to pay bills, we want to put them off as late as we can.



$100 today is worthy more than $100 tomorrow. If we have $100 today, we can spend it for pleasure today. Alternatively, we can lend $100 out today and earn interest on it.

Principal •

The principal is the initial amount borrowed and yet to be repaid.

Interest rate http://www.youtube.com/watch?v=GtaoP0skPWc http://www.youtube.com/watch?v=t4zfiBw0hwM •

An interest rate is the rental price of money. When you borrow someone’s money, you pay a rent for using the money for a period of time.



An interest rate is the rent paid by the borrower to the lender per unit of money per unit of time. An annual interest rate of 5% means that for every $1 borrowed for a year, the borrower pays a $0.05 annual rent to the lender.

49

http://actuary88.com

Simple interest rate http://www.youtube.com/watch?v=B3IdfBcXrLA&feature=related http://www.youtube.com/watch?v=QeRe0WCOujw •

Interest is paid only on the original amount borrowed (called principal) for the length of time the borrower has the use of the money.



You deposit $100 into a bank account and earn 5% simple interest per year. You are actually lending $100 to your bank. The bank pays an annual rent equal to 5% of the principal ($100).



At the end of Year 1, your account grows to 100(1+5%)=$105. $100 is your original principal and $5 is the interest.



At the end of Year 2, your accounts grows to 100(1+5%+5%)=$110. $100 is the original principal. You have earned $5 interest in Year 1 and $5 interest in Year 2. Notice that the $5 interest you have earned in Year 1 does not earn any additional interest in Year 2 (sitting idle in the bank for 1 year).



At the end of Year 3, your accounts grows to 100(1+5%+5%+5%) =$115. $100 is the original principal. You have earned $5 interest in Year 1, $5 interest in Year 2, and $5 interest in Year 3. Notice that the $5 interest you have earned in Year 1 doesn’t earn any interest in Year 2 or Year 3 (sitting idle for 2 years). The $5 interest you have earned during Year 2 does not earn any interest in Year 3 (sitting idle for 1 year).



At the end of Year n , your account grows to 100 (1 + n5% ) . $100 is your original principal. You have earned $5 interest per year for n years. All the interest you have earned year after year is sitting idle.

Compound interest rate http://www.youtube.com/watch?v=qEB6y4DklNY http://www.youtube.com/watch?v=dzMvqJMLy9c



Both the principal and the accrued interest earn interest.



Interest is paid on the original principal plus all interest accrued to that point in time.

50

http://actuary88.com



You deposit $100 into a bank account and earn 5% compound interest per year. This means that for each dollar in your account in the beginning of the year, you will earn $0.05 at the end of the year.



At the end of Year 1, your account grows to 100(1+5%)=$105. At 2 the end of Year 2, your accounts grows to 100 (1 + 5% ) = 110.25 . At the end of Year n , your account grows to 100 (1 + 5% ) . At the end n

of each year, you earn 5% interest on your account balance at the beginning of the year. Comparison of simple interest rate and compound interest rate Time T

t=0

Simple interest rate 5%

Compound interest rate 5%

t =1

100 100 (1 + 5% ) = 105

100 100 (1 + 5% ) = 105

t=2

100 (1 + 5% + 5% ) = 110

105 (1 + 5% ) = 100 (1 + 5% ) = 110.25

t =3

100 (1 + 5% + 5% + 5% ) = 115

110.25 (1 + 5% ) = 100 (1 + 5% ) = 115.7625

t=4

100 (1 + 5% + 5% + 5% + 5% ) = 120 115.7625 (1 + 5% ) = 100 (1 + 5% ) = 121.550625

t=n

100 (1 + n5% )

2

3

4

100 (1 + 5% )

n

If n =1, simple interest rate and compounding interest rate give you the same amount of wealth; if n >1, you accumulate more money under a compounding interest rate; if n 5

Calculate (1) Your account value after 10 years. (2) The equivalent simple interest you have earned during the 10 years. (3) The equivalent compound interest you have earned during the 10 years. Solution The account value after 10 years:

A (10 ) = A ( 0 ) exp

10

5

( t ) dt

= 100 exp

0

= 100 exp

( 2% ) t

10

( 2% ) dt + ( 5% + 0.3%t ) dt

0

1 + 5%t + 0.3%t 2 2

5 0

5 10

= 100e46.25% = $158.8039 5

Find the equivalent simple interest rate:

(

A (10 ) = A ( 0 ) 1 + 10i simple

)

(

)

158.8039 = 100 1 + 10i simple , i simple = 5.88%

Find the equivalent compound interest rate:

(

A (10 ) = A ( 0 ) 1 + i compound

)

10

(

158.8039 = 100 1 + i compound

)

10

, i compound = 4.73%

53

http://actuary88.com Example 2 (May 2004 SOA EA-1 #2) On 1/1/2004, Smith purchases an annuity certain that has three semiannual payments of $500 each, with the first payment to be made 7/1/2009. The force of interest at time t is given by: 1 where t 0 ; t is measured in years from 1/1/2004 (t ) = 50 + 2t In what range is the present value of the annuity on 1/1/2004? [A] Less than $1,350 [B] $1,350 but less than $1,355 [C] $1,355 but less than $1,360 [D] $1,360 but less than $1,365 [E] $1,365 or more Solution The correct answer is A 1/1/2004 Time

7/1/2009

0

1/1/2010

5.5

Amount

7/1/2010

6

$500

6.5

$500

$500

( t ) , the present value of a

Generally, for a given the force of the interest cash flow X occurring at time T is PV = X exp Time

t

!

( s ) ds

0

0

"

.

T

PV = X exp

t

!

( s ) ds

0

X

"

Applying this general rule, we find that the present value of the annuity is: 5.5

500 exp

!

0

( t ) dt

6

"

+ exp

!

0

( t ) dt

6.5

"

+ exp

!

0

( t ) dt

"

54

http://actuary88.com 5.5

= 500 exp

1 dt + exp 50 + 2t " !

6

1 dt + exp 50 + 2t " 0 !

! 0 The tricky part is doing the integration. x

Let’s find

1 dt . Set 50 + 2t = y, 50 + 2t 0

x

1 dt = + 50 2 t 0

50+ 2 x

x

exp

!

50

11 1 dy = ln y y2 2

( t ) dt

0

"

= exp

t=

50 + 2 x

= 50

1 dt = exp ! 0 50 + 2t " 1 2

0

1 dt 50 + 2t "

y 1 25 and dt = dy 2 2

1 50 + 2 x 1 x ln = ln 1 + 2 50 2 ! 25 "

x

x = exp ln 1 + ! 25 "

6.5

1 x ln 1 + 2 ! 25 "

x = 1+ ! 25 "

1 2

The present value is: 5.5 500 1 + 25 " !

1 2

6 + 1+ ! 25 "

1 2

6.5 + 1+ 25 " !

1 2

= 1, 347.14274

Example 3 (SOA May 2005 EA-1 #25) Loan amount: $10,000 Payment Terms: Two payments: End of Year 1: X End of Year 2: 1.1X Force of interest:

0.06 + 0.01 t , for t

2

In what range is X ? (A) Less than $5,210 (B) $5,210 but less than $5,280 (C) $5,280 but less than $5,380 (D) $5,380 but less than $5,480 (E) 45,420 or more

55

http://actuary88.com Solution Generally, for a given the force of the interest cash flow X occurring at time T is PV = X exp Time

( t ) , the present value of a t

!

( s ) ds

0

0

"

.

T

PV = X exp

t

!

( s ) ds

0

X

"

Then it follows: 1

X exp

2

( 0.06 + 0.01 t ) dt

0

0.065

= 10, 000

0

PV of X at t =1

Xe

( 0.06 + 0.01 t ) dt

+ 1.1X exp

+ 1.1Xe

0.14

PV of 1.1X at t =2

= 10, 000

X=

e

10, 000 + 1.1e

0.065

0.14

= 5, 281.61

So the answer is C. Example 4 You are given the following force of interest:

(t ) =

0.09 0.005 t 0.08

for t 6 for t > 6

(1) Calculate the accumulation at t = 10 of $100 invested at t = 0 (2) Calculate the present value at t = 0 of a continuous payment stream at the rate of e0.2t from t = 10 to t = 15 . Solution (1) The accumulation value is: 10

100 exp

( t ) dt

0

= 100 exp

6

= 100 exp

( 0.09

10

0.005 t ) dt + 0.08dt

0

0.09t

1 × 0.005 t 2 2

6

6

+ [ 0.08t ] 6

10

= 100 exp ( 0.45 + 0.32 ) = 215.98

0

56

http://actuary88.com (2) If we have $1 at t where t>6, then its PV at time zero is t

e

( s )ds

0

6

=e

( 0.09

t

0.005 s ) ds + 0.08 ds

0

6

.

If we have e0.2 t at t where t>6, then its PV at time zero is 6

e0.2 t e

( 0.09

t

0.005 s ) ds + 0.08 ds 6

0

If we have a continuous cash flow of e0.2 t from t=10 to t=15, then the PV of the cash flow stream is t

15

e0.2 t e

( s )ds

0

6

15

dt = e0.2 t e

10

( 0.09

t

0.005 s ) ds + 0.08 ds

0

6

10 15

= e0.03 e0.12 t dt = 10

15

dt = e0.2 t e

( 0.45+ 0.08t

0.48 )

10

0.03

e e0.12t 0.12

Example 5 The force of interest

15 10

=

15

dt = e0.2 t e

0.08t + 0.03

dt

10

0.03

e e1.8 e1.2 ) = 23.44 ( 0.12

( t ) = a + bt 2

where a and b are constants. An

amount of $100 invested at t = 0 accumulates to $135 at t = 6 and $200 at t = 9 . Calculate a and b . Solution 6

1 135 = 100 exp ( a + bt ) dt = 100 exp at + bt 3 3 0

6

2

9

1 200 = 100 exp ( a + bt ) dt = 100 exp at + bt 3 3 0

0

9

2

ln1.35 = 6a + 72b ,

= 100e6 a +72b

= 100e9 a + 243b

0

ln 2 = 9a + 243b

a = 0.0284 , b = 0.0018

Nominal interest rate http://www.youtube.com/watch?v=wzvpD5eaunk Many banks calculate and pay interest on the quarterly, monthly, or even daily basis. When the interest is calculated more frequently than annually, the interest rate, however, is often still quoted on the annual basis.

57

http://actuary88.com For example, you deposit $100 in a bank with a nominal interest rate of 6% compounded monthly. Here the compounding frequently is monthly, but the interest rate is still quoted as an annual rate. To calculate you account value, you’ll need to convert the annual interest rate to the monthly interest rate. The monthly interest rate is 6%/12=0.5%. Your interest is 100(0.5%)=$0.5 at the end of Month 1. At the end of Month 1, your bank account grows to $100.5. At the end of Month 2, your bank account grows to 2 100 (1 + 0.5% ) = $101.0025 . At the end of Month 3, your bank account grows to 3 100 (1 + 0.5% ) = $101.5075125 . If the interest rate is quoted on the annual basis but is actually applied to shorter intervals (such as quarterly, monthly, daily), then it is called a ( p) nominal interest rate. A nominal interest rate is often expressed as i , meaning that interest rate is compounded p -thly. If you deposit $100 into a bank account with a nominal interest rate

2 i ( ) = 6% , then the interest you have earned is calculated twice a year,

using a 6 month interest rate of 6%/2=3%. Your bank account at the end of Month 6 is $100 (1 + 3% ) = $103 ; your bank account at the end of Month 12 is $100 (1 + 3% ) = $106.09 . 2

If you deposit $100 into a bank account with a nominal interest rate

4 i ( ) = 6% , then the interest you have earned is calculated four times a

year, using a quarterly interest rate of 6%/4=1.5%. Your bank account at the end of Month 3 is $100 (1 + 1.5% ) = $101.5 ; at the end of Month 6 it is $100 (1 + 1.5% ) = $103.0225 ; at the end of Month 9 it is $100 (1 + 1.5% ) ; at the 2

3

end of Month 12 it is $100 (1 + 1.5% ) . 4

APR APR stands for annual percentage rate. APR calculation considers the fact that a borrower often pays various fees in getting a loan and pays a higher interest rate than the stated interest rate.

58

http://actuary88.com For example, you borrow $150,000 from a bank to finance your new home. The interest rate is nominal 5% compounded monthly. You plan to pay off your mortgage in 20 years. The followings are the fees your bank charges you when you get your mortgage: Stated Principal Fees charged: Loan origination fees (1% of the principal) Underwriting fees Credit report Processing, documentation, and miscellaneous fees Total fees Net amount borrowed

$150,000 1%(150,000)=$1,500 $400 $50 $500 1,500+400+50+500 = $2,450 150,000-2,450= $147,550

To calculate the APR, we need to first find the monthly payment using the stated principal of $150,000. In other words, when you sign the mortgage contract with your bank, your bank calculates your monthly payment using the stated principal of $150,000. So the question is this: You borrow $150,000 at a nominal interest rate of 5%; you are paying off the loan in 20 years. How much do you need to pay per month (assuming that you pay your monthly mortgage at the end of each month)? Right now, you probably haven’t learned how to calculate the monthly payment. That’s OK. Let me give you the answer -- $989.93. I calculated this number using BA II Plus TVM. Next, you ask the question, “I’m paying $989.93 per month for 20 years. However, the net amount I borrowed is really $147,550. What’s the monthly interest rate am I paying?” We do some math and find the monthly interest rate is 0.433%. Then, the nominal interest rate compounded monthly is 0.433%(12)= 5.20%. The APR is 5.20%. The stated interest rate is 5%; the APR is 5.2%. The APR is higher than the stated interest rate.

Annual effective interest rate If you deposit $100 into a bank account with a nominal interest rate

2 i ( ) = 6% , then the interest you earned is calculated twice a year, using a

6 month interest rate of 6%/2=3%. Your bank account at the end of 2 Month 12 is $100 (1 + 3% ) = $106.09 . As a result, the actual interest rate on

59

http://actuary88.com the annual basis is (1 + 3% )

2

1 = 6.09% . 6.09% is the effective annual

interest rate. If the nominal interest rate is i

1+

p i( )

( p)

, then the annual effective rate is

p

1.

p

Continuous compounding You deposit $1 into a bank account. The interest is calculated instantly at a nominal rate of i

1+

+% i( )

( +%)

= . At the end of the year, $1 will become

+%

p

= lim 1 + =e p" p +% !

+%

At the end of t years (t can be fractional), $1 will become p

lim p % +

!

1+

p"

t

p

= lim p +%

!

1+

p"

t

=e t

Example. The annual effective interest rate is 10%. What’s the continuously compounded annual interest rate? Solution If we invest $1 at time zero, then we’ll have $1.1 at time one.

e t = 1.1 where t = 1

= ln (1 + i ) = ln1.1 = 9.531%

Effective annual rate of discount •

Interest payable in advance for borrowing $1 for a unit time

Example 1. 5% effective annual rate of discount means that for every dollar you borrow for one year, you need to pay $0.05 interest in advance (i.e. at the beginning of the year). Effective annual rate of discount is just another way to calculate the interest a borrower needs to pay to the lender. 60

http://actuary88.com

Example 2. If you borrow $100 from a bank one year. At the end of Year 1, you need to return the principal to the bank. The bank charges you a 5% annual effective rate of discount. Identify your cash flows and the bank’s cash flows. Calculate the actual interest rate the bank actually charges you. Solution Time t (year) Your cash flow

0 100 100 ( 5% ) = $95

1 $100

100 + 100 ( 5% ) = $95

Bank’s cash flow

$100

At the time of borrowing ( t = 0 ), the bank gives $100. Simultaneously, the bank charges you 100(5%)=$5 interest. So you really get $95 net at t = 0 . At t = 1 , you pay back $100 to the bank. To calculate the interest rate your loan is charged, note that you actually borrow $95 at t = 0 and pays back $100 at t = 1 . So the borrowing rate is 100 95 = 5.263% 95

We can also calculate the effective interest rate from the bank’s point of view. The bank lends you $95 at t = 0 but gets $100 at t = 1 . So the bank’s earning rate is: 100 95 = 5.263% 95

Example 3. If you borrow $1 from a bank for one year. At the end of Year 1, you return the principal to the bank. The annual effective rate of discount is d . Identify your cash flows. Convert the effective annual discount rate to the effective annual interest rate. Solution Time t (year) Your cash flow Bank’s cash flow

0 $ (1 d )

$ (1 d )

1 $1 $1

61

http://actuary88.com The bank invests (1 d ) at t = 0 but gets back $1 at t = 1 . So the equivalent annual effective interest rate is:

i=

ending fund value - beginning fund value 1 (1 d ) 1 = = 1 beginning fund value 1 d 1 d 1+ i =

1 1 d

If we set v =

, 1 d=

1 1+ i

1 , then 1 d = v 1+ i

Example 4. Derive the following equation and explain the meaning of the equation. d=

i 1+ i

Solution 1 i = = iv 1+ i 1+ i The above equation says that if we borrow $1 for one unit of time, then the advance interest d paid at the time of the borrowing (i.e. t = 0 ) is simply the present value of the interest i due at t = 1 . This makes intuitive sense. d =1

Example 5. Derive the following equation and explain the meaning of the equation. i=

d

1 d

= d (1 d )

1

Solution 1+ i =

1 1 d

&

i=

1 1 d

1=

d

1 d

= d (1 d )

1

To understand the meaning of i = d (1 d ) , please note that (1 d ) 1

1

is the

accumulating factor for a unit time. If we invest $1 at t = 0 , then this $1 will grow into (1 d )

1

at t = 1 .

62

http://actuary88.com i = d (1 d )

1

means that if we borrow $1 at t = 0 , then the interest due at

t = 1 is simply the accumulated value of the advance interest d paid at t = 0 . This makes lot of sense.

Simple annual rate of discount Just as interest rate can be a simple rate or an annual effective rate, a discount rate can be a simple rate or an annual effective rate. It’s hard to think intuitively what a simple discount rate really means. So don’t worry about interpreting the simple discount rate. Just need to remember the following key formula: If a simple discount rate is d , then $1 at time t is worth 1 dt at t = 0 ; $1 at time t = 0 is worth (1 dt )

1

at time t .

Nominal annual rate of discount This is similar to nominal annual rate of interest rate. The discount rate is quoted on the annual basis but is actually applied to shorter periods (such as monthly or quarterly). d ( m) The formula is: 1 d = 1 m

m

63

http://actuary88.com

Relationship between

i , d,

Example. If d ( 2 ) = 10% . Calculate d , i ,

, i (m ) , d ( m)

, i (12) .

Strategy: Don’t just memorize a host of formulas. Understand the meaning of each symbol. Understand how money travels over time. You should do fine.

Let’s look at each symbol. First, i . This symbol means the effective interest rate per unit of time (typically per year). You are most familiar with this symbol. If i =10%, then if you borrow $1 at t = 0 , you’ll need to pay 10%*$1=$0.1 at the end of year. Next, memorize the following diagram:

t=0

t =1

$1

$ (1 + i )

$v =

1 1+ i

$1

$1 at t = 0 is worth $ (1 + i ) at t = 1 . $1 at t = 1 is worth $ v = Wealth Ratio =

1 at t = 0 . 1+ i

Wealth at t = 1 = 1+ i Wealth at t = 0

64

http://actuary88.com d is the interest paid in advance for every $1 you borrow. If you borrow $1 at time zero. Then immediately you are charged the interest d for each dollar you borrow. If d = 0.09 and you borrow $1 at time zero, the lender immediately takes away 0.09*$1=$0.09. You walk away with $0.91 in your pocket. Generally, if you borrow $1 at t = 0 , you walk away with (1 d ) in your pocket; you need to pay $1 at t = 1 to pay off your loan.

Next, memorize the following diagram:

t=0

t =1

$ (1 d )

$1

$1

$

$ (1 d ) at t = 0 is worth $1 at t = 1 . $ Wealth Ratio =

1 1 d

1 1 d

at t = 1 is worth $1 at t = 0 .

Wealth at t = 1 1 = Wealth at t = 0 1 d

65

http://actuary88.com is the force of the interest, the interest rate you earn during a tiny interval such as one second. If you deposit $1 into a bank account that earns a constant force of interest , then for a tiny interval of time [t , t + dt ] , you’ll earn dt interest. If you deposit $1 at 1

t = 0 , then this $1 will grow into e

ds 0

= e at t = 1 (here we assume a constant

).

Next, memorize the following diagram:

t=0

t =1 1

ds

$1

$e

$e

$1

$1 at t = 0 is worth $ e at t = 1 . $1 at t = 1 is worth $ e Wealth Ratio =

0

=e

at t = 0 .

Wealth at t = 1 =e Wealth at t = 0

66

http://actuary88.com

i ( 2 ) means that instead of paying the interest once a year at t = 1 , you are paying the interest twice a year, 1st time at t = 0.5 (end of 6-th month) and the 2nd time at t = 1 (end 2 i( ) th of 12 month). Your effective interest rate per 6 month is . Similarly, i (12) means that 2 instead of paying the interest once a year at t = 1 , you are paying the interest monthly at 1 2 11 12 t= (end of Month 1), (end of Month 1), …, (end of Month 11), and (end of 12 12 12 12 2 i( ) (m) Month 12). Your effective interest rate per month is . i means that interest is paid 12 1 2 3 m 1 m m -thly per year at t = , , ,..., , . m m m m m Next, memorize the following diagram:

1 m

t=0

t=

t=

$1

i( ) 1+ m m

( m 1)

i( ) 1+ m m

t=

… 2

i( m) 1+ m

m

i( m) 1+ m

2 m

i( m) 1+ m



i( ) 1+ m m

( m 2)

t= m 1

i( m) ... 1 + m

i( ) $1 at t = 0 is worth 1 + m

1 i( ) at t = , 1 + m m

i( m) $1 at t = 1 is worth 1 + m

m

m

m 1 m

m

2

m =1 m

i( m) 1+ m

m

1

$1

2 i( ) at t = , …, and 1 + m m m

m

at t = 1 .

at t = 0 .

Wealth at t = 1 i( m) Wealth Ratio = = 1+ Wealth at t = 0 m

m

67

http://actuary88.com Imagine that $1 at t = 0 travels through the time going through m consecutive doors. i( ) When going through each door, $1 expands to 1 + . After going through m doors, it m m

i( m) lands at t = 1 becoming 1 + m

m

.

Finally, let’s look at the most difficult symbol d ( m ) . You walk into a bank to borrow $1. The bank charges you, d ( 2 ) , a nominal discount rate convertible semiannually. d ( 2 ) means that cash is deducted twice from your declining principal before it is lent to you. Imagine a loan officer puts $1 worth of coins into a jar. This $1 is the amount you want to borrow. You really want to take away this $1 and go home, but the loan officer says, “Wait a minute. Let me deduct your interest payment twice. Then you can take away the 2 d( ) st portion of coins out of the jar. Because rest.” The 1 time, the loan officer takes out 2 2 2 2 d( ) d( ) d( ) the initial principal is just $1, portion of $1 is ×1 = . This is your 1st interest 2 2 2 2 d( ) payment in advance. After this payment, your principal shrinks from $1 to 1 . So 2 2 2 d( ) d( ) now the jar has only 1 worth of coins. You really want to take 1 and go 2 2 home, the loan officer says, “Wait a minute. I need to deduct your interest payment the 2 2 d( ) d( ) d( ) portion what’s left in the jar. So 1 2 2 2 2

2nd time.” So he takes out

d ( 2) coins is taken out from the jar, leaving only 1 2 jar. Your 2nd interest payment in advance is d( ) d( ) from 1 to 1 2 2 2

2

worth of

d ( 2) d ( 2) d ( 2) 1 = 1 2 2 2

2

in the

d( ) d( ) . Now your principle shrinks 1 2 2 2

2

d ( 2) 1 2

2

2

.

Your total interest payment in advance is:

d ( 2 ) d ( 2) d ( 2) 2 + 1 = d( ) 2 2 2

d ( 2) =1 4

68

http://actuary88.com The result should make intuitive sense. Originally, the jar has $1 worth of coins. After d ( 2) two deductions, the jar has only 1 2

2

worth of coins left. So the total coins deducted

(i.e. your total interest payment in advance) is simply 1 d ( 2) with 1 2

d ( 2) 1 2

2

. So you walk away

2

in your pocket.

At t = 1 , you pay $1 back to the bank, paying off the loan. Now you owe your bank nothing. Let’s draw a diagram to describe your loan:

t=0

t =1

d ( 2) 1 2

d ( 2) 1 2

d ( 2) 1 2

2

$1

2

at t = 0 is worth $1 at t = 1 .

2

at t = 0 is equivalent to $1 t = 1 . Then it follows that $1 at t = 0 is equivalent

d( ) to 1 2 2

2

at t = 1 . Now we can change the diagram into:

69

http://actuary88.com Cash flow diagram under d ( 2 )

t=0

d ( 2) 1 2

t =1 2

$1

d ( 2) 1 2

$1

1

d ( 2) 1 2

2

at t = 0 is worth $1 at t = 1 .

1 ( 2)

1

1

2

2

at t = 1 is worth $1 at t = 0 .

d 2

Wealth Ratio =

Wealth at t = 1 = Wealth at t = 0

1 d( ) 1 2 2

1

2

We can easily extend the above reasoning to d ( m ) . If you want to borrow $1 from a bank under a nominal discount rate d ( m ) , then your loan officer immediately deducts interest m d( ) payments in advance m times from your initial principal of $1. In the 1st deduction, m ( m) ( m) d d portion of your initial $1 is deducted, leaving 1 left. In the 2nd deduction, m m d( ) d( ) portion of the remaining 1 is deducted, leaving 1 m m m

m

2

left. After m

70

http://actuary88.com d (m) deductions, only 1 m

m

d (m) is left. You walk away with 1 m

m

in your pocket.

Then one year later at t = 1 , you pay $1 back to the bank. After this payment, you owe the bank nothing. m Cash flow diagram under d ( ) :

t=0

t =1

d ( m) 1 m

m

$1

d ( m) 1 m

$1

d (m) 1 m d ( m) 1 m

m

m

at t = 0 is worth $1 at t = 1 .

m

at t = 1 is worth $1 at t = 0 .

Wealth at t = 1 d ( m) = 1 Wealth Ratio = Wealth at t = 0 m

m

71

http://actuary88.com , i ( m ) , d ( m ) . You should

Now, we are ready to derive the relationship between i , d , memorize the following diagram:

t=0

t =1

$1

$ (1 + i )

$1

$

1 1 d

$1

$e

$1

i( m) 1+ m

$1

d ( m) 1 m

i( m) Immediately, we see that 1 + i = = e = 1+ m 1 d

1

m

m

d ( m) = 1 m

m

m

From here you can derive all sorts of formulas. For example, 1 1 =1 d , d =1 =1 v 1+ i 1+ i 1+ i = e , = ln (1 + i ) i( m) 1+ i = 1+ m

m

d ( m) 1+ i = 1 m d ( m) = 1 m 1 d

1

i( m) i = 1+ m

,

m

m

d (m) i= 1 m

,

m

,

1

m

d ( m) 1 d= 1 m

1

m

72

http://actuary88.com You can also derive the relationships using the following diagram:

t=0

t =1

1 =v 1+ i

1

1 d

1

e

1

i( m) 1+ m d ( m) 1 m

m

1

m

1

Now you should clearly see that : 1 = v =1 d = e 1+ i

i( m) = 1+ m

m

d ( m) = 1 m

m

From here, you can derive all sorts of relationship formulas. Yet, there’s the third approach – to use the wealth ratio. The wealth ratio should the same under different measurements of interest. Hence: Wealth at t = 1 1 i( m) Wealth Ratio = = 1+ i = = e = 1+ Wealth at t = 0 1 d m

m

d ( m) = 1 m

m

73

http://actuary88.com Finally, let’s solve the problem: If d ( 2 ) = 10% . Calculate d , i , 2

d ( 2) 1 d= 1 2

= 1

10% 2

2

, d =9.75%

1 = 1 d = 1- 9.75%, i =10.8% e = 1 + i , 1+ i

i (12 ) 1+ 12

, i (12) .

= ln (1 + i ) = ln1.108 = 0.10259

12

= 1 + i = 1.108 , i (12) =10.3%

Example 2. You invest $150 at time zero in an account that earns the following interest: During Year 1, i ( 4 ) = 10% . During Year 2, d (12) = 10% . During Year 3, = 10% During Year 4, ( t ) = 2 + 0.1t Calculate your account value at the end of Year 4. Solution

Let A ( t ) represent your account value at time t . A ( 0 ) = 100

10% A (1) = A ( 0 ) 1 + 4 " !

A ( 3) = A ( 2 ) e10% 4

4

A ( 4 ) = A ( 3) exp

( 2 + 0.1t ) dt = ( 2t + 0.05t 2 )4 = 2 ( 4 3

A ( 2 ) = A (1) 1 ! 4

!

( 2 + 0.1t ) dt

3

10% 12 "

12

"

3) + 0.05 ( 42 32 ) = 2.35

A ( 4 ) = A ( 3) e 2.35

3

4

10% 10% A ( 4 ) = 100 1 + 1 4 " ! 12 " ! 4

10% 10% = 100 1 + 1 4 " ! 12 " !

12

4

e10% exp

!3

( 2 + 0.1t ) dt

"

12

e10% e 2.35 = 1, 414.26

74

http://actuary88.com

Future value http://www.youtube.com/watch?v=3ZmiOGsL8G4 Future value is tomorrow’s value of today’s deposit. You deposit $100 into a bank account and your money grows at a compound interest rate of 6%. Then at the end of Year 1, your original deposit of $100 will grow into $100(1+6%)=$106. $106 is the future (1 year from now) value of your original $100 deposit. Similarly, at the end of Year 2, your original deposit of $100 will become $100(1+6%)2=$112.36. So the future (2 years from now) value of your original deposit is $112.36. Generally, if you deposit A ( 0 ) at t = 0 and your deposit grows at a compound interest rate of i , then the future value of your original t deposit t years from now is A ( t ) = A ( 0 )(1 + i ) .

Present value http://www.youtube.com/watch?v=zGdu2DHu6yA Present value is today’s value of tomorrow’s money. It is the deposit you must make today in order to receive some money in the future. For example, one year from now you will receive $100. Assume the interest rate is 6%. How much money do you need to deposit into a bank account in order for you to receive $100 one year from now? Let A ( 0 ) represent the amount of money you must deposit into a bank account now. Then A ( 0 )(1 + 6% ) = $100

A (0) =

$100 ' $94.34 1 + 6%

So to receive $100 one year from now, you must deposit $94.34 today and have it grow at 6%. $94.34 is the present value of receiving $100 one year from now. You are receiving A ( t ) amount of money in t years. The interest rate is i . What’s the present value of A ( t ) amount of money t years from now? A ( t ) = A ( 0 )(1 + i )

t

A ( 0) =

A (t )

(1 + i )

t

= A ( t )(1 + i )

t

75

http://actuary88.com So the present value of receiving A ( t ) amount of money in t years is A ( t )(1 + i )

t

.

Example 1 Calculate the present value of $100 at t = 10 at the following rates: (1) a rate of interest rate of 6% per year convertible monthly (2) a rate of discount of 6% convertible monthly (3) a force of interest rate of 6% Solution 6% (1) 100 1 + 12 " !

(3) 100e

12×10

= 100e

10

12×10

6% (2) 100 1 12 " !

= 54.96 10×6%

= 54.80

= 54.88

Example 2 (SOA May 2000 EA-1 #2) A loan of $1,800 is to be repaid by a single payment of $2,420.8 two years after the date of the loan. The terms of the loan are quoted using a nominal annual interest rate of 15%. What’s the frequency of compounding? (A) monthly (B) every two months (C) quarterly (D) semiannually (E) annually Solution First, let’s calculate the effective annual interest rate. 1,800 (1 + i ) = 2, 420.8 , 2

i = 15.9693%

We are given that the nominal interest rate is 15%. One way to find the frequency of compounding is to test different frequencies. The general formula: Given i ( p ) , the nominal interest rate compounding p -thly a year, the

i( p ) equivalent annual effective rate is: i = 1 + p

p

1

76

http://actuary88.com First, let try an interest rate compounding monthly. The equivalent annual effective rate of 15% compounding monthly is ( p = 12 ): 12

!

1+

15% 12 "

1 = 16.075%

( 15.9693%

So the monthly compounding is not good. Next, let’s try an interest rate compounding every two months. The equivalent annual effective rate of 15% compounding every two months is ( p = 6 ) : 15% 1+ 6 " !

6

1 = 15.9693

' 15.9693%

This is good. So the answer is B.

Convert interest rate to discount rate or vice versa Problem 1 (EA-1 #1 2001) Selected values: 1, 000d ( m ) = 85.256 1, 000d ( 2 m ) = 85.715

Calculate 1, 000i ( 3m ) Solution Instead of memorizing complex formulas, let’s use the “discount $1” method. If we have $1 at time one, what’s its value at time zero? 1 thly from time zero to time 1. m The total # of discounting periods is m . Per period discount factor is

Discounting Method 1

discount

d( ) . 1 m m

d( ) m m

PV = 1

m

85.256 1, 000 = 1 m

m

= 1

0.085256 m

m

77

http://actuary88.com

Discounting Method 2

Discount

1 thly from time zero to time 2m

d( ) . 2m 2m

1. The total # of periods is 2m . Per period discount factor is 1

d( ) PV = 1 2m 2m

2m

85.715 1, 000 = 1 2m

2m

0.085715 = 1 2m

2m

The PV should be the same: 0.085256 1 m

m

0.085715 = 1 2m

2m

0.085256 1 m

0.085715 = 1 2m

2

1 m 2 1 0.085256 x = (1 0.0428575 x ) = 0.04285752 x 2 2 ( 0.0428575 ) x + 1

Let x =

0.085256 x = 0.04285752 x 2 2 ( 0.0428575 ) x 0.085256 = 0.04285752 x 2 ( 0.0428575 ) x = 0.25

m=

1 =4 x

Discounting Method 3

Discount

1 thly from time one to time 3m

i ( 3m ) zero. The total # of periods is 3m . Per period discount factor is 1 + 3m i(3m) PV = 1 + 3m

1

.

3m

Once again, we should get the same PV: i ( 3m ) 1+ 3m

3m

i ( 3m ) 1+ 3× 4

3

0.085256 = 1 m

0.085256 =1 4

m

i ( 3m ) 1+ 3m

3

= 1

0.085256 m

i ( 3m ) 0.085256 1+ = 1 3× 4 ! 4 "

1 3

78

http://actuary88.com

i ( 3 m ) = 0.08648788 ,

1, 000i ( 3m ) ' 86.49

Problem 2 12 4 If the annual effective rate i = 8% , calculate i ( ) , d ( ) , and the force of interest . Solution We’ll use “the accumulate $1 method.” Assume we have $1 at t = 0 . What’s the FV of this $1? If we accumulate $1 using i = 8% , then FV = 1 + i . (12)

i (12) , then FV = 1 + 12

If we accumulate $1 using d

( 4)

d ( 4) , then FV = 1 4

If we accumulate $1 using

, then FV = e

If we accumulate $1 using i

12

4

We should accumulate the same amount of wealth. i (12 ) 1+ i = 1+ 12

12

1 i (12) , 1+ = (1 + i )12 12

1

1

i (12 ) = 12 (1 + i )12 1 = 12 1.0812 1 = 7.7208% d ( 4) 1 4

1

4

d ( 4) = 1 + i, 1 = (1 + i ) 4

1 d ( 4) = (1 + i ) 4 , d ( 4 ) = 4 1 4

e = 1 + i,

1 4

(1 + i )

1 4

= 4 1 1.08

1 4

= 7.6225%

= ln (1 + i ) = ln1.08 = 7.696%

79

http://actuary88.com Problem 3 Prove the following equation 1 d

=

( m)

1 1 + (m) m i

Solution Assume we have $1 at t = 0 and we want to accumulate this $1 to t = 1 Accumulate $1 from t = 0 to t = using d ( m ) : m

Accumulate $1 from t = 0 to t = d( ) 1 m

1

m

i( ) d( ) = , 1 m m m

= 1+

m

1 using i ( m ) : m

d (m) FV= 1 m

1 . m

1

i( ) m m

FV= 1 +

1 m = m ( m) i m + i( ) 1+ m

d ( m) m i ( m) =1 = m m + i (m) m + i( m) m m + i( m) m = ( m) = ( m) + 1 ( m) d i i

1 d

( m)

=

1 1 + (m) m i

Problem 4 Prove lim d ( m ) = lim i ( m ) = m

%

m

%

Solution Based on Problem 3, we have:

lim

m

%

1 d

( m)

= lim m

%

1 1 1 1 1 + lim ( m ) = lim ( m ) = = ( m) % % m m m i i lim i m

%

lim d ( m ) = lim i ( m ) =

m

%

m

%

80

http://actuary88.com Problem 5 List the relationship among

, i, v, d .

Solution = i=

e

1

v=

e

d=

1 e

i ln (1 + i )

v ln v

i

1 1 v v

1 d 1 d

1 v

d

1 1+ i i 1+ i

d ln (1 d )

d

No need to memorize the above table. However, make sure you can derive the table.

81

http://actuary88.com

PV of a stream of cash flows Time t

Cash flow

0

CF ( 0 )

1

CF (1)

PV = CF ( 0 ) +

……

k

……

CF ( k )

n

CF ( n )

CF (1) CF ( 2 ) CF ( n ) + + ... + 2 n 1+ i (1 + i ) (1 + i )

Net Present Value http://www.youtube.com/watch?v=IH1Uh2_XFbM http://www.youtube.com/watch?v=PCrBvhTJiAw NPV is just the PV of future cash flows minus the initial cost at time zero. To make money in the future, often we have to spend money at time zero (to buy machines or build a factory for example). Then to calculate our total wealth at time zero, we need to subtract, from the PV of future cash flows, our initial cost at time zero. The result is called NPV. NPV = - Cost + PV Example. If you invest $10 today, you’ll get $6 at the end of Year 1 and $8 at the end of Year 2. The interest rate is 12%. What’s your NPV? PV = 10 +

6 8 + = 1.7347 1.12 1.122

Internal rate of return (IRR) http://www.youtube.com/watch?v=B89vwItBFfk IRR is the interest rate such that the net present value of future cash flows is equal to zero. To find the IRR, we need to solve the following equation:

82

http://actuary88.com 0 = CF ( 0 ) +

CF (1) CF ( 2 ) CF ( n ) + + ... + 2 n 1+ i (1 + i ) (1 + i )

You can use BA II Plus/BA II Plus Professional Cash Flow Worksheet to calculate IRR. Example. Calculate IRR of the following cash flows: Time t (year) Cash flow

0 -100

1 24

2 35

3 20

4 16

5 13

Solution Enter the following into BA II Plus/BA II Plus Professional Cash Flow Worksheet: Cash flow Frequency

CF0 -$100

C01 24 F01 1

C02 35 F02 1

C03 20 F03 1

C04 16 F04 1

C05 13 F05 1

Press “IRR” “CPT.” You should get: IRR=3.01%. Multiple IRRs. The number of positive roots in 0 = CF ( 0 ) +

CF (1) CF ( 2 ) CF ( n ) + + ... + 2 n 1+ i (1 + i ) (1 + i )

is equal to the number of sign changes. For example, if CF ( 0 ) is negative and CF (1) , CF ( 2 ) ,…, CF ( n ) are all positive, then there’s only one sign change. As a result, there is only one positive root in polynomial equation; there is a unique IRR. If CF ( 0 ) is negative, CF (1) is positive, CF ( 2 ) is negative, and all the remaining cash flows are positive, then we have two sign changes. As a result, there are two IRRs. Please note that BA II Plus/BA II Plus Professional Cash Flow Worksheet generates only one IRR value.

83

http://actuary88.com Example. Calculate IRR of the following cash flows: Time t (year)

0

1

2

Cash flow

- $100

230

-132

Solution We need to solve the following equation: 230 132 0 = 100 + 1 + r (1 + r ) 2

1.32

(1 + r )

2

2.3 +1 = 0 1+ r

1 1 1 1 =0 ! 1 + r 1.1 "! 1 + r 1.2 "

r1 = 10%, r2 = 20%

Multiple IRR’s are hard to explain. This is one of the drawbacks of using IRR to determine profitability.

Asset and its price Asset = a stream of cash flows The price of an asset = PV of future cash flows Time t

0

1

Asset

CF ( 0 )

CF (1)

PV = CF ( 0 ) +

……

k

……

CF ( k )

n

CF ( n )

CF (1) CF ( 2 ) CF ( n ) + + ... + 2 n 1+ i (1 + i ) (1 + i )

The price of an asset = PV of future cash flows This is a critical concept for Exam FM. This concept is used in pricing bonds and stocks.

84

http://actuary88.com

Convert a cash flow from one point of time to another point of time Time t

0

1

t1

……

……

$A

Cash flow

t2 ???

Assume that the interest rate is i . The discount rate is d = 1 v = 1

(1 + i )

1

.

We have a cash flow of $A at t1 and we want to convert it to a cash flow at t2 , where t2 t1 . $ A (1 + i ) 2 t

$ A @ t1

t1

= A (1 d )

(t2 t1 )

(accumulating)

@ t2

Alternatively, we have a cash flow $B at t2 and we want to convert it to a cash flow at t1 where t2 t1 . Time t

0

1

……

Cash flow

t1

……

$B

???

$ B @ t2

$B v

t2 t1

t2

= B (1 d ) 2 t

t1

(discounting)

@ t1

Example. Assume the discount rate d = 10% . Time t

Cash flow

0

1

$X

2

3

4

5

6

$Y

$10

$10 @ t = 3

$10 (1 d )

$10 @ t = 3

$10 (1 d )

3 1

7

= 10 (1 10% ) = 8.1 @ t = 1

( 7 3)

2

= 10 (1 10% )

4

= 15.24157903 @ t = 7

85

http://actuary88.com Alternative calculation: v = 1 d = 1 10% = 0.9 , 1 + i = v 1 = 0.9

1

(

)

$10 @ t = 3

$10 v3 1 = 10 v 2 = 10 0.92 = 8.1 @ t = 1

$10 @ t = 3

$10 (1 + i )

7 3

= 10 ( 0.9

1

)

4

= 15.24157903 @ t = 7

Collapse multiple cash flows into a single cash flow Many times we need to collapse (i.e. consolidate) multiple cash flows occurring at different times into a single cash flow occurring at a common point of time. To collapse a stream of cash flows into a single cash flow, we can NOT simply add up multiple cash flows (unless the interest rate is zero). Procedures to collapse multiple cash flows into a single cash flow: •

Choose a common point of time.



Convert, either by discounting or by accumulating, each cash flow to an equivalent cash flow occurring at this common point of time.



Add up the converted values of all cash flows. This is the single cash flow into which multiple cash flows collapse.

Example. Time t

0

Cash flow

1

2

3

4

5

$1

6

7

$1

Assume the interest rate is 10%. We have 2 cash flows: $1 at t=2 and $1 at t=6. We want to collapse these 2 cash flows into one cash flow, perhaps because we want to calculate the total value of these 2 cash flows. (1) If we choose t=0 as the common time. $1 @ t = 2 $1 @ t = 6

$1.1 2 = 0.82644628 @ t = 0 $1.1 6 = 0.56447393 @ t = 0

The total value of the 2 cash flows @ t = 0 : $0.82644628 + $0.56447393 = $1.39092021

86

http://actuary88.com

(2) If we choose t=2 as the common time. $1 @ t = 2

$1 @ t = 2

$1 @ t = 6

$1.1 4 = 0.68301346 @ t = 2

The total value of the 2 cash flows @ t = 2 : $1 + $0.68301346 = $1.68301346 (3) If we choose t=4 as the common time. $1 @ t = 2 $1.12 = 1.21 @ t = 0 $1 @ t = 6 $1.1 2 = 0.82644628 @ t = 4 The total value of the 2 cash flows @ t = 4 : $1.21 + $0.82644628 = $2.03644628 (4) If we choose t=6 as the common time. $1 @ t = 2 $1 @ t = 6

$1.14 = 1.4641 @ t = 6 $1 @ t = 6

The total value of the 2 cash flows @ t = 6 : $1.4641 + $1 = $2.4641 (5) If we choose t=7 as the common time. $1 @ t = 2 $1 @ t = 6

$1.15 = 1.61050 @ t = 7 $1.1 @ t = 7

The total value of the 2 cash flows @ t = 7 : $1.61050 + $1.1 = $2.71051 So far, we have collapsed the two cash flows into five single cash flows occurring at t=0, 2, 4, 6, and 7 respectively. Since these five single cash flows each represent the total value of the identical cash flows of $1 @ t=2 and $1 @ t=6, they should convert to each other following the standard conversion rule.

87

http://actuary88.com

Time t

0

1

2

3

4

5

6

7

Cash flow

$1.39092021 $1.68301346 $2.03644628 $2.4641 $2.71051 For example: $1.39092021 @ t = 0

$1.39092021(1.12 ) = 1.68301345 @ t = 2

We did not get $1.68301346 @ t = 2 due to rounding. $2.03644628 @ t = 4

$2.03644628 (1.13 ) = 2.71051 @ t = 7 (OK)

88

http://actuary88.com

Annuity – collapsing n parallel cash flows into a single cash flow •

n parallel evenly-spaced cash flows of $1 each $1

$1

$1

i

$1

i

1

2



$1

i 3

$1

i 4

n 1

n

n cash flows

an i

an i

sn i

sn i



The total value of n parallel cash flows of $1 each at one step to the left of the 1st 1 vn cash flow is an i = . In other words, we can collapse n parallel cash flows i of $1 each into a single cash flow an i at one step to the left of the 1st cash flow.



The total value of n parallel cash flows of $1 each at the 1st cash flow time is 1 vn an i = . We can collapse n parallel cash flows of $1 each into a single cash d flow an i at the 1st cash flow time.



The total value of n parallel cash flows of $1 each at the final cash flow time is

(1 + i ) =

n

1

. We can collapse n parallel cash flows of $1 each into a single i cash flow sn i at the final cash flow time. sn



i

The total value of n parallel cash flows of $1 each at one step to right of the final cash flow is sn

i

(1 + i ) =

d each into a single cash flow sn

n

i

1

. We can collapse n parallel cash flows of $1

at one step to the right of the final cash flow.

89

http://actuary88.com Please note that i is the effective interest rate between two consecutive cash flows. For example, if two consecutive cash flows are 3 months apart, then i is the effective interest rate for the 3 month period (i.e.. 3 months = 1 unit time); if two consecutive cash flows are 3 years apart, then i is the effective interest rate for the 3 year period (i.e.. 3 years = 1 unit time). Since the four single cash flows an i , an i , sn i , and sn i each represent the total value of the identical n parallel cash flows, they should convert to each other following the standard conversion rule. So we have:

an i = v an i , an i = an i (1 + i ) sn i = v sn i , sn i = sn i (1 + i )

an i = v n sn i , sn i = an i (1 + i )

n

an i = v n sn i , sn i = an i (1 + i )

n

an i = v n+1 sn i , sn i = an i (1 + i )

n +1

an i = v n 1 sn i , sn i = an i (1 + i )

n 1

Avoid the common pitfall Question: “What’s the difference between annuity due and annuity immediate?” Answer: In annuity due, the 1st payment is at t = 0 ; in annuity immediate, the 1st payment is at t = 1 . If you agree with this answer, you are wrong! This is a common mistake made by many. This common mistake originates from the fact that when textbooks derive the formula for annuity due and annuity immediate, they always draw the following diagram (making the annuity due have the 1st payment at t = 0 and the annuity immediate have the 1st payment at t = 1 ):

90

http://actuary88.com Common diagram for annuity due—potentially misleading Payment $1 $1 $1 … $1

Time

t=0

t =1

t=2

..

a n i = 1 + v + v 2 + ... + v n 1 =

…. t = n 1

t=n

1 vn 1 vn = 1 v d

The truth, however, is that in an annuity due, the 1st payment can start at any time. For example, the 1st payment can be at t = 2 : Payment

Time

$1

t=0

t =1

t=2 ..

$1



$1

t =3



t=n

a n i = 1 + v + v + ... + v 2

n 1

$1

t = n +1 t = n + 2

1 vn 1 vn = = 1 v d

To avoid the faulty thinking that in an annuity due the 1st payment is at t = 0 , you should use the following diagram(i.e. don’t include the time in your diagram): Payment $1 $1 $1 … $1

1st ..

2nd

a n i = 1 + v + v + ... + v 2

3rd n 1



n-th payment

1 vn 1 vn = = 1 v d

Rule to remember: If you discount n evenly spaced cash flows back to the 1st cash flow time, you’ll have an annuity due, regardless of when the 1st payment is made.

91

http://actuary88.com Common diagram for annuity immediate—potentially misleading Payment

Time

$1

t=0

t =1

$1



t=2

….

an i = v + v 2 + ... + v n 1 + v n =

$1

$1

t=n 1

t=n

v v n +1 1 v n 1 v n 1 v n = = = 1 v 1 1 v i 1 v v

The truth, however, is that in an annuity immediate, the 1st payment can start at any time. For example, the 1st payment can be at t = 2 : Payment $1 $1 … $1 $1

Time

t=0

t =1

t=2

t =3

an i = v + v 2 + ... + v n 1 + v n =



t=n

t = n +1

v v n +1 1 v n 1 v n 1 v n = = = 1 v 1 i 1 v 1 v v

To avoid the faulty thinking that in an annuity immediate the 1st payment is at t = 1 , you should use the following diagram(i.e. don’t include the time in your diagram): Payment $1 $1 $1 … $1

Time

1st

2nd

an i = v + v 2 + ... + v n 1 + v n =

3rd



n-th payment

v v n +1 1 v n 1 v n 1 v n = = = 1 v 1 1 v i 1 v v

Rule to remember: If you discount n evenly spaced cash flows back to one interval prior to 1st cash flow time, you’ll have an annuity immediate, regardless of when the 1st payment is made.

92

http://actuary88.com ..

Similarly, when draw the diagram for sn i and s n i , don’t include the timeline: Payment

$1

$1

1st

2nd

$1

3rd



$1



n-th ..

sn i sn i =

(1 + i )

n

(1 + i ) =

n

1

d

1

i

If you accumulate n evenly spaced cash flows the final cash flow time, use sn i , regardless of when the final payment is made. If you accumulate n evenly spaced ..

cash flows the final cash flow time plus one interval, use s n i , regardless of when the final payment is made. Summary: Just memorize the following diagram and you’ll do fine. Payment $1 $1 $1 … $1

1st

3rd

2nd



n-th

..

an i

..

an i

sn i

sn i

One step before the first cash flow time, use an i ; ..

at the 1st cash flow time, use a n i ; at the final cash flow time, use sn i ; ..

one step after the final cash flow time, use s n i

93

http://actuary88.com Example 1. You are given the following cash flows: Time t

0

1

Cash flow

2

3

4

5

6

7

8

$5

$5

$5

$5

$5

$5

$5

The interest rate is 10%. Calculate, at t=4, the total value of these cash flows. Solution We need to collapse the cash flows into a single cash flow @ t=4. There are many ways to do so. Method 1 First, we split the original cash flows into two streams of cash flows. The first stream consists of cash flows at t=2, 3, and 4. The second stream consists of cash flows at t=5, 6, 7, and 8. If we add up these two streams, we should get the original cash flows. Next, we collapse each of the two streams into a single cash flow at t=4. For the 1st stream of cash flows, we need to collapse 3 parallel cash flows into a single cash flow at the final payment time. This gives us an equivalent single cash flow at t=4 of 5 s3 10% . For the 2nd stream of cash flows, we need to collapse 4 parallel cash flows into a single cash flow at one step to the left of the 1st cash flow. As the result, the equivalent single cash flow at t=4 is 5a4 10% .

Time t

Cash flow

0

1

2

3

4

$5

$5

$5

5 s3 10%

94

http://actuary88.com Time t

4

Cash flow

5a4

5

6

7

8

$5

$5

$5

$5

10%

Consequently, the total value of the original cash flows @ t=4 is: 5 s3

+ 5a4 10%

10%

=5

1.13 1 1 1.1 4 +5 ' 32.40 0.1 0.1

Method 2 We split the original cash flows into two streams of cash flows. The 1st stream consists of cash flows at t=2 and 3. The 2nd stream consists of cash flows at t=4, 5, 6, 7, and 8. Next, we collapse each of the two streams of cash flows into a single cash flow at t=4. For the 1st stream, we need to collapse 2 parallel cash flows to a single cash flow one step to the right of the final cash flow. This gives us an equivalent cash flow of 5 s2 10% at t=4. For the 2nd stream, we need to collapse five parallel cash flows at the first cash flow time. This gives us an equivalent cash flow of 5 a5 10 at t=4. Time t

0

1

Cash flow

2

3

$5

$5

4

5 s2

10%

Time t

4

5

6

7

8

Cash flow

$5

$5

$5

$5

$5

5 a5

10

Finally, we add up these two equivalent cash flows at t=4. 5 s2

10%

+ 5 a5

10

=5

1.12 1 1 1.1 5 + 5 ' 32.40 1 1.1 1 1 1.1 1

95

http://actuary88.com Method 3 This time, we don’t split the original cash flows. First, we collapse the original cash flows to t=1. Since 7 parallel cash flows are collapsed into a single cash flow one step to the left of the 1st cash flow, the equivalent single cash flow is 5a7 10% . Time t

0

1

Cash flow

5a7

2

3

4

5

6

7

8

$5

$5

$5

$5

$5

$5

$5

10%

Next, we’ll convert the single cash flow of 5a7

10%

@ t=1 to a cash flow @

t=4.

$5a7

@ t =1

10%

$5a7

(1.1 ) = 5a 4 1

10%

7 10%

(1.1 )

@ t=4

3

Finally, the total value of the original cash flow @ t = 4 :

5a7

(1.1 ) = 5 1 3

10%

1.1 0.1

7

(1.1 ) ' 32.40 3

Method 4 First, we collapse the original cash flows to t=2. Since 7 parallel cash flows are collapsed into a single cash flow at the 1st cash flow time, the equivalent single cash flow is 5 a7 10 . Time t

0

1

Cash flow

2

3

4

5

6

7

8

$5

$5

$5

$5

$5

$5

$5

5 a7

10

Next, we’ll convert the single cash flow of 5 a7

10

@ t=2 to a cash flow @

t=4.

$5 a7

10

@ t=2

$5 a7

(1.1 ) = 5 a (1.1 ) 4 2

10

2

7 10

@ t=4

96

http://actuary88.com Finally, the total value of the original cash flow @ t = 4 : 5 a7

(1.1 ) = 5 11 2

10

1.1 7 1.12 ) ' 32.40 1 ( 1.1

Method 5 First, we collapse the original cash flows to t=8. Since 7 parallel cash flows are collapsed into a single cash flow at the final cash flow time, the equivalent single cash flow is 5 s7 10% .

Time t

0

1

Cash flow

2

3

4

5

6

7

8

$5

$5

$5

$5

$5

$5

$5

5 s7 Next, we’ll convert the single cash flow of 5 s7 t=4.

$5 s7

@ t =8

10%

$5 s7

(1.1 ) = 5 s

10%

4 8

10%

7 10%

10%

@ t=8 to a cash flow @

(1.1 )

@ t=4

4

Finally, the total value of the original cash flow @ t = 4 : 7

5 s7

(1.1 ) = 5 1.10.1 1 (1.1 4 ) ' 32.40 4

10%

Method 6 First, we collapse the original cash flows to t=9. Since 7 parallel cash flows are collapsed into a single cash flow at one step after the final cash flow time, the equivalent single cash flow is 5 s7 10% .

Time t

Cash flow

0

1

2

3

4

5

6

7

8

$5

$5

$5

$5

$5

$5

$5

9

5 s7 10% Next, we’ll convert the single cash flow of 5 s7 10% @ t=9 to a cash flow @ t=4.

97

http://actuary88.com

$5 s7 10%

$5 s7 10% (1.14

@ t =9

9

)=5s

(1.1 ) 5

7 10%

@ t=4

Finally, the total value of the original cash flow @ t = 4 :

5 s7 10%

1.17 1 (1.1 ) = 5 1 1.1 1 (1.1 5 ) ' 32.40 5

The above 6 methods may seem an overkill, but they are good exercises on how to collapse an identical cash flow stream in various ways. Example 2

( )(a ) = a

Explain why a4n i = an integer and j = (1 + i )

n

i

4 j

n i

(1 + vn + v n + v n ) , where n is a positive 2

3

1.

Solution We can collapse 4n parallel cash flows of $1 into a4n i :









4n cash flows

a4n

i

98

http://actuary88.com Next, we split the 4n parallel cash flows into 4 sets of n parallel cash flows. We collapse each set of n parallel cash flows into a single cash flow of an i , arriving at 4 sets of parallel cash flows of an i each. Finally, we collapse the 4 parallel cash flows of an i each into a single cash flow of

(a )(a ) . n i

4 j









n cash flows n cash flows n cash flows n cash flows

an

an

i

an

i

an

i

i

(a )(a ) n i

4 j

We use an annuity factor of a4 j . j represents the interest rate between two consecutive cash flows of an i . Among the four parallel cash flows of

an i , two consecutive cash flows are n time apart. Consequently, j = (1 + i )

n

1.

Because 4n parallel cash flows can be collapsed into two single cash flows a4n

( ) ( a ) that occur at the same time, it follows that

and an

i

( )(a ) .

a4n i = an

i

Because a4

i

4 j

4 j

= 1 + (1 + j ) + (1 + j ) + (1 + j ) = 1 + v n + v 2 n + v3 n , we have: 1

j

( )(a ) = a

a4n i = an

i

4 k

2

3

(1 + vn + v n + v n ) 2

n i

3

Make sure you understand the above logic. This sharpens your skill to quickly collapse a complex stream of cash flows into a single cash flow.

99

http://actuary88.com

( ) ( a ) using the standard annuity formula.

We can also prove a4n i = an

a4n i = a4 j =

1 v4n , d

1 1

(1 + j ) 1 (1 + j )

4

(a ) (a ) n i

an i =

4 j

=

1 1

i

4 j

1 vn d

(1 + i ) n (1 + i )

4n

=

1 v4n n 2n 3n n = 1+ v + v + v 1 v

1 vn 1 v 4n 1 v 4n = = d 1 vn d

( )(a ) = a

a4n i = an

i

4 j

n i

(1 + vn + v n + v n ) 2

3

Generally, for any positive integer k :

( )(a ) = a

ak n i = an

i

n i

k j

( )(a ) = a

ak n i = a n

i

k j

( )(s ) = s

sk n i = s n

i

i

k j

where j = (1 + i )

n

( k 1) n

1 + v n + v 2 n + ... + v

n i

1 + (1 + i ) + (1 + i )

n i

1 + (1 + i ) + (1 + i )

n

k j

( )(s ) = s

sk n i = s n

n i

1 + v n + v 2 n + ... + v

n

2n

2n

( k 1) n k 1 n + ... + (1 + i )( )

k 1 n + ... + (1 + i )( )

1

You don’t need to memorize the above formula. Just make sure that you can derive the above formulas by collapsing cash flows.

100

http://actuary88.com Example 3 (SOA May 2002 EA-1 #11) S1 = The accumulated value as of 12/31/2002 of $500 invested at the end of each month during 2002 at a nominal interest rate of 8% per year, convertible quarterly. A1 = The present value as of 1/1/2002 of S1, at a nominal discount rate of 6% per year, convertible semiannually. S2 = The accumulated value as of 12/31/2002 of $1,500 invested at the end of each quarter during 2002 at a nominal discount rate of 6% per year, convertible monthly. A2 = The present value as of 1/1/2002 of S2, at a nominal interest rate of P% per year, convertible once every two years. In what range is P% such that A1 = A2? [A] Less than 4.60% [B] 4.60% but less than 4.70% [C] 4.70% but less than 4.80% [D] 4.80% but less than 4.90% [E] 4.90% or more Solution To simplify the calculation, let’s set $500 as one unit of money. Date 1/1/2002 Time t (months) 0 Payments

1 $1

2 1



11 1

12/31/2002 12 1 S1 = s12 i

The quarterly effective interest rate is i ( ) / 4 =8%/4=2% Remember 1 year = 4 quarters 4

The monthly effective interest i is calculated as follows:

(1 + i )

3

=1+2% (remember that 1 quarter = 3 months) 1

i = (1 + 2% ) 3 1 = 0.66227096%

S1 = s12 i = 12.44689341

101

http://actuary88.com Because the nominal discount rate is 6% per year convertible semiannually, the discount rate during a 6-month period is 2 d( ) 6% =1 = 1 3% = 0.97 1 2 2 2

d( ) 1 d= 1 2 2

We can find the annual discount rate d : d ( m) Here we use the formula: 1 d = 1 m

= 0.97 2

m

Let v represent the annual discounting factor for A1. Then A1 = ( S1) v . However, v = 1 d . So we have: 6% A1 = ( S1) v = ( S1)(1 d ) = 12.44689341 1 2

2

= 11.7112820

Next, we calculate S 2 .

Date 1/1/2002 Time t (quarters) 0 Payments

1 $3

2 3

3 3

12/31/2002 4 3

3 units of money=$1,500 S2 = 3s4

j

The nominal discount rate is 6% per year convertible monthly. We can find the annual rate of discount d and the annual discounting factor v .

1 d (12 ) =1 d = 1 v= 1+ i 12

12

d ( m) Once again the formula is 1 d = 1 m

m

Once we have the annual effective rate i , we can find the quarterly effective interest rate j .

(1 + j )

4

= 1+ i = v

1

(1 year=4 quarters)

102

http://actuary88.com

1 + j = (1 + i )

1/ 4

=v

1/ 4

= (1 d )

1/ 4

(12)

= !

d (12 ) j= 1 12

3

6% 1= 1 12

d 1 12

1/ 4

12

d( ) = 1 12 12

"

3

3

1 = 1.51512594%

S 2 = 3s4 j = 12.27548783 To calculate A2 , we need to know how to deal with a bizarre nominal rate: “a nominal interest rate of P% per year, convertible once every two years.” Let’s start with something simple. If we have a nominal interest rate of P % per year convertible once every 6 months, then the 6-monthP% period interest rate is . Here the denominator 2 is the # of 6-month2 periods per year (so one year = two 6-months). Similarly, if we have a nominal interest rate of P% per year convertible monthly, then the P% . Here the denominator 12 is the # of months monthly interest rate is 12 per year (so one year = 12 months). Now let’s apply this logic to a nominal interest rate convertible once every two years. One year is 0.5 of 2 years. Then the interest rate for a 2-year P% period is = 2 P % . Then the discounting factor for a 2-year period is 0.5 1 1 = (1 + 2 P% ) . Then the discounting factor for one year is 1 + 2 P%

(1 + 2 P% )

1

1 2

1

= (1 + 2 P% ) 2 .

A2 = The present value as of 1/1/2002 of S2, at a nominal interest rate of P% per year, convertible once every two years. Then A2 = S2 (1 + 2 P% )

A1 = A2

1 2

= 12.27548783 (1 + 2 P% )

11.7112820 = 12.27548783 (1 + 2 P% )

1 2

1 2

P% = 4.93367417% . So the answer is E.

103

http://actuary88.com

Perpetuity collapsing +% parallel cash flows into a single cash flow •

+% parallel evenly-spaced cash flows of $1 each $1

$1

i

$1

i

1

2

$1

i 3



$1

$1

i 4

+%

+% cash flows

a+% i =

1 i

a+% i =

1 d

We can collapse +% parallel cash flows of $1 each into a single cash 1 flow at one step to the left of the 1st cash flow. i We can collapse +% parallel cash flows of $1 each into a single cash 1 1 flow = 1 + at the 1st cash flow time. d i

104

http://actuary88.com

Example 1 (SOA May 2000 EA-1 #1) Purchase date of a perpetuity-due: 1/1/2000 Level payment amount: $100 Frequency of payments: Annual Cost of perpetuity: $1,100 Interest rate for perpetuity: i % , compounded annually Immediately following the payment on 1/1/2014, the remaining future payments are sold at a yield rate of i % . The proceeds are used to purchase an annuity certain as follows: Term of annuity: 10 years st 1/1/2018 1 payment of annuity: Frequency of annuity payment: semi-annual on January 1 and July 1 i Interest rate for annuity: % compounded annually 2 In what range is the semi-annual annuity payment? (A) Less than $75 (B) $75 but less than $77 (C) $77 but less than $79 (D) $79 but less than $81 (E) $81 or more Solution Perpetual annuity due: Time t (years) 0 1 Payment $100 $100

PV =

100 = 1,100 , d

2 $100

d =1

3 $100

… $100

1 1 = , 1 + i % 11

Perpetual annuity due sold on 1/1/2014: # of payments 1 2 3 4 … 1/1/2014 7/1/2014 1/1/2015 7/1/2015 Time t … Payment $100 $100 $100

selling price =

% $100

i % = 10%

… …

% % $100

100 100 = = 1, 000 i % 10%

105

http://actuary88.com The sales proceeds are used to purchase a 10 year Time 1/1/2018 7/1/2018 1/1/2019 7/1/2019 # of payments 1 2 3 4 Payment X X X X PV = X a20

annuity certain. … 1/1/2027 7/1/2027 … 19 20 X X X

j

Where j is the semi-annual effective interest rate. i% j = 1+ 2 " !

0.5

10% 1 = 1+ 2 " !

0.5

1 = 2.47%

At 1/1/2018, the accumulated value of the sales proceeds should be equal to the PV of the 10 year annuity certain. 4

i 1, 000 1 + % = X a20 j , ! 2 "

1, 000 (1.05 ) = X a20 2.47% , X = 75.87 4

So the answer is C.

106

http://actuary88.com

Annuity – payable m-thly in advance Time t

0

0 m

$

1 m

2 m

...

1 1 1 $ $ m m m



1

……

m 1 m m m

……

$

1 m

……

1 unit of time

n

n 1

0 m

$

1 m

2 m

...

1 1 1 $ $ m m m



m 1 m m m

$

1 m

n-th unit of time

n units of time

d 1 vn d d( ) a n i = ( m) = ( m ) = ( m) an i , where 1 d m d d d ( m)

m

1 vn

m

=1 d = v

1 each m that occur in each unit of time into an equivalent single cash flow. We should have n equivalent single cash flows (because we have a total of n units of time). Next, we collapse these n single cash flows into one

To derive the above formula, we first collapse m cash flows of $

single cash flow of

d a . m ni d( )

107

http://actuary88.com

$

1 1 1 $ $ m m m



$

1 m

m payments

$

1 a m m

1

j

where j = (1 + i ) m 1

1 1 1 (1 + j ) $ am j = m m 1 (1 + j )

Time t

0

0 m

$

1 m

2 m

...

1 1 1 $ $ m m m



m 1

1 1 (1 + i ) = m 1 (1 + i )

1

……

m 1 m m m

……

$

1 m

d m d( )

1 m

d

=

m 1 d !

0 m

……

$

=

1 m

d m d( )

"

n

n 1

1 unit of time

$

1

1 m

2 m

...

1 1 1 $ $ m m m



m 1 m m m

$

1 m

n-th unit of time

$

d m d( )



$

d m d( )

n cash flows

d a m ni d( )

108

http://actuary88.com

Annuity – payable m-thly in arrears Time t

0

0 m

1 m

$

2 m

...

1 1 1 $ $ m m m

1

……

m 1 m m m

……



$

1 m

n

n 1

0 m

……

1 unit of time

1 m

$

2 m

...

1 1 1 $ $ m m m

m 1 m m m



$

1 m

n-th unit of time

n units of time

( m)

an i

( ) i 1 vn i i = ( m) = ( m ) = ( m) an i , where 1 + i m i i i

a(n i) = m

m

1 vn

m

= 1+ i

1 vn

n d i ( m) 1 v = a and a = = ( m) an i are the only two formulas n i ( m) (m) n i ( m) d d i i

you need to memorize for annuities where multiple cash flows occur in a m m one unit time. If a problem asks you to find sn( i) and sn( i) , you can calculate this way: n m m sn( i) = an( i) (1 + i ) ,

n m m sn( i) = a (n i) (1 + i )

(m)

(m)

( m)

( m)

If the symbols and the related formulas of an i , a n i , sn i , and sn

i

look

too ugly and complex, you can always use the cash flow frequency

1 as m

the unit time, thus forcing the cash flow frequency and the interest compounding frequency to be identical. This greatly simplifies the number of concepts and formulas you need to memorize. If the unit time is of

1 long, then the effective interest rate during the m 1

unit time is j = (1 + i ) m 1 .

109

http://actuary88.com Under this simplifying technique, the PV of an annuity payable m-thly in advance in each unit time is:

Time t

0

0 m

$

1 m

2 m

...

1 1 1 $ $ m m m



1

……

m 1 m m m

……

$

1 m

n

n 1

0 m

……

$

1 m

2 m

...

1 1 1 $ $ m m m



m 1 m m m

$

1 m

m n cash flows

1 a m mn

Time t

1 s m mn

j

0

0 m

1 m

$

2 m

...

1 1 1 $ $ m m m

1

……

m 1 m m m

……



$

1 m

n

n 1

0 m

……

j

1 m

$

2 m

...

1 1 1 $ $ m m m

m 1 m m m



$

1 m

m n cash flows

1 a m mn

j

1 s m mn

j

110

http://actuary88.com Example 1 A loan of $100,000 borrowed at 6% annual effective is repaid by level monthly payments in advance over the next 30 year. After 10 years, the outstanding balance of the loan is refinanced at 4% annual effective and is paid by level monthly payments in advance over 20 years. Calculate: • The monthly payment of the original loan. • The principal portion and the interest portion of the 37th payment. • The monthly payment of the refinanced loan. • The accumulated value of the reduction in monthly payments invested at 4% annual effective. Solution Method 1 - use a year as the compounding period Find the monthly payment of the original loan Time t (Year)

0

1

1 12

2 12

3 12

4 12

5 12

6 12

7 12

X 12

X 12

X X 12 12

X 12

X X 12 12

8 12

9 12

X X 12 12

10 12

11 12

X 12

……

30

12 12

X 12

1 Year's payments

30 years' payments

X a 30(12)6% Let

X represent the monthly payment of the original loan. 12 (12)

X a30

6%

= 100, 000 ,

(12)

a 30 6%

1 v 30 = (12 ) , d

d (12) 1 d= 1 12

12

111

http://actuary88.com

d (12) = 12 1

a(3012)6% = X=

(1

1

1

d )12 = 12 1 v 12 = 12 1

1

(1.06 )12

= 5.812767%

1 v30 1 1.06 30 = = 14.2082053 12 5.812767% d( )

100, 000 (12 )

a 30 6%

=

100, 000 = 7, 038.186588 14.2082053

X 7, 038.186588 = = 586.52 12 12

Find the principal portion and the interest portion of the 37th payment The 37th payment is the 1st payment in the 4th year. Time t (Year)

0

1

2

3

4

……

27

28

29

30

27 years' payments

Xa

(12) 27 6%

The # of compounding period remaining immediately after the 37th payment is 30 – 3 =27. X a(27 )6% = X 12

1 v 27 1 1.06 27 7, 038.186588 = = 95,973.09 ( ) 12 5.812767% d( )

The interest portion of the 37th payment is:

( 95, 973.09 )

d( ) 5.812767% = ( 95,973.09 ) = 464.89 12 12 12

The principal portion is: 586.52 464.89 = 121.63

112

http://actuary88.com Calculate the monthly payment of the refinanced loan. Time t (Year)

0

1

2

……

10

……

27

28

29

30

20 years' payments

X a(2012)6% The outstanding balance of the original loan at t =10: X a(20 )6% = X 12

1 v 20 12 d( )

1 v 20

(12)

a 20 6% =

=

1 12

12 1 v ! " Let

i =6%

1 1.06

20 1 12

= 11.83937535

12 1 1.06 ! "

i =6%

A represent the monthly payment of the refinanced loan. 12

A a (2012 )4% = X a (2012 )6%

1 v 20

a (2012)4% =

1 12

12 1 v ! "

A= X

=

a (2012)6% (12 )

a 20 4%

i = 4%

1 1.04

20 1 12

= 13.88301906

12 1 1.04 ! "

= ( 7,038.186588 )

11.83937535 = 6, 002.1334 13.88301906

So the monthly payment in advance of the refinanced loan is: 6, 002.1334 = 500.1777833 12

113

http://actuary88.com

Find the accumulated value of the reduction in monthly payments invested at 4% annual effective. Time t (Year)

0

1

2

……

10

……

27

28

29

30

20 years' payments

(X

(12 ) A) s 20 4%

Reduction of the monthly payment due to refinancing: 1 (X 12

A) =

1 1, 036.0532 = 86.3377667 ( 7, 038.1866 6, 002.1334 ) = 12 12

The accumulated value of the reductions at 4%: ( X (12 )

s 20

4%

(X

(1 + i ) =

20

1

d(

12 )

(1 + i ) =

20

1 1 12

12 1 v ! "

=

1.04 20 1 1 12

(12 ) A) s 20 4%

= 30.4194036

12 1 1.04 ! "

(12) A) s 20 = 1, 036.0532 ( 30.4194036 ) = 31, 516.12044 4%

Method 2 – use a month as the compounding period Find the monthly payment of the original loan Time t (Year) 8

0

1

2

……

357

358

359

Y

Y

Y

……

Y

Y

Y

360

360 monthly payments

Y a360

j

Let Y represent the monthly payment in advance.

114

http://actuary88.com

The number of compounding periods: 30 (12 ) = 360 1

The interest rate per period: j = 1.0612 1 = 0.48675506% Y a360 j = 100, 000

a360 j = Y=

1 v360 1 1.0048675506 360 = = 170.4984712 d 1 1.0048675506 1

100, 000

a360

= 586.5155230 ' 586.52

j

Find the principal portion and the interest portion of the 37th payment Time t (Month)

0

1

2

……

36

……

357

358

Y

Y

Y

……

Y

……

Y

Y

359

360

Y

324 monthly payments

Y a324

j

The # of compounding period remaining immediately before the 37th payment is 27 (12 ) = 324 . The outstanding loan is: Y a324 j = Y

1 v 324 1 1.0048675506 324 = ( 586.5155230 ) = 95,973.09 d 1 1.0048675506 1

The interest portion of the 37th payment is:

( 95,973.09 ) d , where The interest is:

d = 1 v = 1 1.0048675506

95,973.09 (1 1.0048675506

The principal portion is:

1

1

) = 464.8909

586.52 464.89 = 121.63

115

http://actuary88.com Calculate the monthly payment of the refinanced loan. Time t (Month)

0

1

2

……

120

……

357

358

359

Y

Y

Y

……

Y

……

Y

Y

Y

360

240 monthly payments

Y a240

j

The outstanding balance of the original loan at t =120 is: Y a240 j = Y

1 v 240 1 1.0048675506 240 = ( 586.5155230 ) = 83, 327.73367 1 1.0048675506 1 ! d " j =0.48675506%

Let B represent the monthly payment of the refinanced loan. Ba240 k = Ya240

a240 k = B=

j

where k = 1.04

1 12

1 = 0.32737398%

1 v 240 1 1.0032737398 240 = = 166.5962287 1 ! d "k 1 1.0032737398

Ya240

a240 k

j

=

83,327.73367 = 500.177791 166.5962287

Find the accumulated value of the reduction in monthly payments invested at 4% annual effective. Time t (Month)

0

1

2

……

120

……

357

358

359

Y

Y

Y

……

Y

……

Y

Y

Y

360

240 monthly payments

(Y

B ) s240 k

Reduction of the monthly payment due to refinancing: 116

http://actuary88.com

B = 586.5155230 500.177791 = 86.337732

Y

The accumulated value of the reductions at 4%:

(Y

B ) s240 k

(1 + k ) 1 = (1 + 0.32737398% ) 1 = 365.0329421 = 1 1 1 (1 + k ) 1 (1 + 0.32737398% ) 240

s240 k

240

The accumulated value is:

(Y

B ) s240 k = 86.337732 ( 365.0329421) = 31,516.11632

Example 2 (2002 May EA-1 #3)

Given values:

s b2n g = 180.24943 m

d(m) = 0.08

bmg

In what range is s 4n ? [A] [B] [C] [D] [E]

Less than 2,930 2,930 but less than 2,970 2,970 but less than 3,010 3,010 but less than 3,050 3,050 or more

Solution ( m)

s 2n

(1 + i ) = d

(1 + i ) (m )

s4n

B

2n

2n

1

( m)

= 180.24943 . Using d(m) = 0.08, we find that

= 180.24943 ( 0.08 ) + 1 ' 15.42

(1 + i ) = d(

4n

m)

1

(1 + i ) = d(

4n

m)

1 15.422 1 = = 2,959.71 0.08

117

http://actuary88.com Example 3 (2002 May EA-1 #4) A 20-year immediate annuity certain is payable monthly. Immediately after the 43rd payment has been made, the present value of the remaining annuity payments is calculated to be X . N is the number of the payment after which the present value of the X remaining annuity payments is less than for the first time. 2 d ( 4 ) = 0.08

What [A] [B] [C] [D] [E]

is N ? 67 68 171 172 173

Solution First, let’s calculate the monthly interest rate i . We are given d ( 4 ) = 0.08 . 4 d( ) 3 3 v = (1 + i ) = 1 , v = 0.99328839 (monthly discount factor) 4 Let P represent the monthly payment.

Time t (months) Payments

0

1

2



43

P a240

We are asked to find N such that P a240 that P a240

P a240

N i

N i

P a197 i

=

N i

43 i

<

45 P

46 P

… P

240 P

= P a197 i = X X . First, let’s find N such 2

X . 2

X 1 = 2 = X 2

118

http://actuary88.com

P a240

N i

P a197 i

1 v 240 N 1 v 240 N 1 0.99328839240 N 1 i = = = = . 1 v197 1 v197 1 0.99328839197 2 i

We need to solve the equation: 0.99328839240

( 240

N

=1

1 1 0.99328839197 ) = 0.63268311 ( 2

N ) ln 0.99328839 = ln 0.63268311

240 N =

ln 0.63268311 = 67.9788 ln 0.99328839

N = 240 67.9788 ' 172.02

As N increases, the present value of the remaining annuity payments decreases. As the extreme, if N = 240 , then there’s no payments left and present value of the remaining annuity payments is zero. So N = 173 is the 1st time that remaining annuity payments is less than X . The answer is E. 2

119

http://actuary88.com Example 4 (2004 May EA-1 #8) The present value of a 15-year monthly annuity-immediate is $20,600. Payments are as follows: Years 1 – 7: X per month Years 7 – 15: X + $300 per month Interest rate: 8%, compounded annually. Calculate X Solution Time t (year) Time t (month) payment

0

7

0

1

2

….

84

X

X

X

X

15 85

86



180

X + 300 X + 300 X + 300 X + 300

We’ll break down the original cash flows into the following two streams: Stream #1 Time t (month)

0

payment

1

2

….

84

85

86



180

X

X

X

X

X

X

X

X

… 300

180 300

PV = X a180 i 1

Where i = 1.0812 1 = 0.6430301%

(monthly effective interest rate)

Stream #2 Time t (year) Time t (month)

0

7

0

1

2

(

)

….

84

85 300

Payment

PV = 300 a96 i 1.08

7

300 a180

84 i

86 300

= 300 a96 i

So the total PV of the payments is:

(

)

X a180 i + 300 a96 i 1.08 a180 i = 106.4275863 ,

7

= 20, 600

a96 i = 71.45305627

Solving the equation, we get: X = 76.03619

120

http://actuary88.com

Increasing annuity $1

$2

$3

$4

… $ (n

1)

$n

n cash flows

( Ia )n i

( Ia )n i

( Is )n i

Just remember one increasing annuity formula:

( Ia )n i =

( Is )n i nv n

an i

Then, calculate the remaining increasing annuity factors as follows:

( Ia )n i = (1 + i ) ( Ia )n i , ( Is )n i = (1 + i ) ( Ia )n i , ( Is )n i = (1 + i )n+1 ( Ia )n i n

Continuously increasing annuity

( Ia )n

n

i

an = t e t dt =

nv n

0

121

http://actuary88.com

Decreasing annuity $ (n

$n

1) …

$4

$3

$2

$1

n cash flows

( Da )n i

( Da )n i

( Da )n i =

( Ds )n i =

( Ds )n i

n an

i

i

n (1 + i )

n

i

( Da )n i =

,

sn

i

,

( Ds )n i =

n an

( Ds )n i

i

d

n (1 + i )

n

sn

i

d

Just remember one decreasing annuity formula:

( Da )n i =

n an

i

i

Then, calculate the remaining increasing annuity factors as follows: n +1 ( Da )n i = (1 + i ) ( Da )n i , ( Ds )n i = (1 + i ) ( Da )n i , ( Ds )n i = (1 + i ) ( Da )n i

n

122

http://actuary88.com Example 1 A company is participating in a project. The cash flows of the project are as follows: • The company will invest $10 million per year for the 1st three years of the project. The investment will be made continuously. •

The company will receive a cash flow at the end of each year starting from Year 4.



At the end of Year 4, the company will receive the 1st cash flow of $9 million. This amount will be reduced by $0.5 million for each subsequent year, until the company receives $5 million in a year.



Starting from that year, the cash flow received by the company will be reduced by $1 million each year, until the company receives zero cash flow.

Calculate the NPV of the project if the discount rate is 12%. Solution Time t

0

1

2

3

cash

4

5

6

7

8

9

10

11

12

13

14

15

16

$9.0 $8.5 $8.0 $7.5 $7.0 $6.5 $6.0 $5.5 $5.0 $4.0 $3.0 $2.0 $1.0

( )3

10 a

12%

The initial investment at t=0 is: 1 v3 1 1.12 3 10 ( a )3 12% = 10 = 10 = 25.43219763 ln1.12 ! "i =12% Next, let’s calculate the PV of cash flows from t=4 to t=12.

Time t cash

0

1

2

3

4

5

6

7

8

9

10

11

12

13

14

15

16

$9.0 $8.5 $8.0 $7.5 $7.0 $6.5 $6.0 $5.5 $5.0

9a9 12% 0.5 ( Ia )8 5%

123

http://actuary88.com

9a9 12% = 9

1 1.12 1 1.12

0.5 ( Ia )8 12% = 0.5

= 0.5

9 1

= 53.7087579

a8 12% 8 (1.12

8

)

0.12

1 1.12 = 0.5 1 1.12

5.56375654 8 (1.12 0.12

8

8 1

8 (1.12

8

)

0.12

) = 9.71954465

9a9 12% 0.5 ( Ia )8 5% = 53.7087579 9.71954465 = 43.98921325

PV of cash flows from t=4 to t=12:

(1.12 4 ) 43.98921325 = 27.95594028 Next, let’s calculate the PV of cash flows from t=13 to t=16:

Time t

0

1

2

3

4

5

6

7

8

9

10

11

cash

12

13

14

15

16

$4.0 $3.0 $2.0 $1.0

( Da )4 12%

( Da )4 12%

=

n an

PV is: (1.12

i 12

=

4 a4 12% 12%

=

4 3.03734935 = 8.02208878 12%

) 8.02208878 = 2.05907038

So the PV of all cash inflows is: = 27.95594028 + 2.05907038 = 30.01501066

Finally, the NPV of the project is: 30.01501066 25.43219763 = 4.58281303

124

http://actuary88.com Example 2 (May 2004 SOA EA-1 #3) Type of Annuity Annual Payments

Interest rate

Annuity immediate, with 19 annual payments. First payment is $1, increasing each year by $1 until payment reaches $10, then decreasing by $1 each year to the final payment of $1 5% annual effective

In what range is the present value of this annuity at the date of the purchase? [A] Less than $57 [B] $57 but less than $60 [C] $60 but less than $63 [D] $63 but less than $66 [E] $66 or more Solution C The fastest solution is to use the cash flow worksheet in BA II Plus/BA II Plus Professional. Enter the following into Cash Flow Worksheet: Time 0 1 2 3 4 5 6 7 8 9 10 11 12 13 14 15 16 17 18 19

Cash flow CF0 C01 C02 C03 C04 C05 C06 C07 C08 C09 C10 C11 C12 C13 C14 C15 C16 C17 C18 C19

Amt

Frequency

Frequency

$1 $2 $3 $4 $5 $6 $7 $8 $9 $10 $9 $8 $7 $6 $5 $4 $3 $2 $1

F01 F02 F03 F04 F05 F06 F07 F08 F09 F10 F11 F12 F13 F14 F15 F16 F17 F18 F19

1 1 1 1 1 1 1 1 1 1 1 1 1 1 1 1 1 1 1

125

http://actuary88.com Though the above table lists the cash flow frequencies from F01 to F19, you really don’t need to enter any cash flow frequency. If a user doesn’t enter the # of cash flows, BA II Plus/BA II Plus Profession automatically sets the # of a cash flow to one. This should be fine because all our cash flow frequencies are one. Next, set the interest rate to 5%. You should get: NPV = 62.60644983

Alternative method (a little slower, but not too bad) We break down the original cash flows into two streams: Stream #1: Increasing annuity from t=1 to t=9 Stream #2: Decreasing annuity from t=10 to t=19 We then separately calculate the present value of each stream and find the total present value.

( Ia )9 5%

The present value of Stream #1:

To find the present value of Stream #2, we first calculate the present value of this stream at t=9; the PV should be ( Da )10 5% . Then, we discount this PV to t=0. v9 ( Da )10 5%

The present value of Stream #2: The PV of the original cash flows is:

( Ia )9 5%

=

a9 5% 9 (1.05

9

)

5%

v9 ( Da )10 5% = 1.05

9

=

(

7.46321276 9 1.05

10 a10 5% 5%

( Ia )9 5%

5%

= 1.05

9

9

+ v9 ( Da )10 5%

) = 33.23465027

10 7.72173493 = 29.37179956 5%

33.23465027 + 29.37179956 = 62.60644983

126

http://actuary88.com Example 3 (May 2001 SOA EA-1 #7) Repayment schedule for a loan: End of each odd numbered year 1 3 5 … … X … … 25 Interest rate:

Amount of repayment $100 $300 $500 … … $ 100X … … $2,500

6% per year, compounded annually

A is the total of the payments to be made after the 15th year. B is the present value of the remaining payments as of the beginning of the 16th year.

In what range is A B ? (A) Less than $3,120 (B) $3,120 but less than $3,150 (C) $3,150 but less than $3,180 (D) $3,180 but less than $3,210 (E) $3,210 or more Solution C Let’s set $100 as one unit of money. The interest rate per 2 years is: i = 1.06 2 1 = 12.36% Time t (year) Reset time t (2 years) Repayment

15

16

0

17 1 $17

18

19 2 $19

20

21 3 $21

22

23 4 $23

24

25 5 $25

A = 17+19+21+23+25=105

Next, we’ll calculate B . Please note that B is the PV as of the beginning of the 16the year (i.e. end of the 15th year). So B is the PV at t = 15 , not at t = 16 .

127

http://actuary88.com You can use the increasing annuity formula to calculate B . However, that calculation is overly complex. A simple approach is this: B = 17v + 19v 2 + 21v 3 + 23v 4 + 25v5

=

17 19 21 23 25 + + + + = 73.37424439 3 4 2 1.1236 1.1236 1.1236 1.1236 1.12365

A B = 105 73.37 = 31.63 = $3,163

The fastest solution is to use BA II Plus Cash Flow Worksheet. Enter the following into Cash Flow Worksheet: CF0 $0

C01 $17

C02 $19

C03 $21

C04 $23

C05 $25

Then set I=12.36 (so the interest rate is 12.36%). You should get: NPV=73.37424439.

B = 73.37424439 .

To calculate B , simply set I=0 (so the interest rate is zero). You should get: NPV=105. A = 105

A B = 105 73.37 = 31.63 = $3,163

Moral of this problem: Having an increasing annuity doesn’t mean you have to use the increasing annuity formula. Example 4 (May 2004 SOA EA-1 #25) Smith buys a 10-year decreasing annuity-immediate with annual payments of 10, 9 , 8, …, 2, 1. On the same date, Smith buys a perpetuity-immediate with annual payments. For the first 11 years, payments are 1, 2, 3, …, 11. After year 11, payments remain constant at 11. At an annual effective interest rate of i , both annuities have a present value of X . Calculate X .

128

http://actuary88.com Solution As usual, we draw a cash flow diagram: Time t Annuity #1 Annuity #2

0

1 2 $10 $9 $1 $2

3 $8 $3

4 $7 $4

5 $6 $5

6 $5 $6

7 $4 $7

8 $3 $8

9 10 11 $2 $1 $9 $10 $11



%

$11

$11

Since we don’t know the interest rate, we can’t use BA II Plus Cash Flow Worksheet. We have to use the formulas for the increasing and decreasing annuity.

( Da )10 i =

10 a10

i

i

To calculate the PV of annuity 2, we’ll break it down into 2 streams: Stream #1 Time t 0 1 2 3 4 5 6 7 8 9 10 11 … % Annuity #2

$11

11 10 v i

$11

$11

11 i

At t = 10 , the PV of this stream is

11 . At t = 0 , the PV of this stream is i

11 10 v . i

Stream #2 Time t Annuity #2

0

1 $1

( Ia )10 i =

2 $2

3 $3

4 $4

5 $5

6 $6

7 $7

8 $8

9 10 $9 $10

a10 10v10 i

10 11 10 a10 10v So at t = 0 , the PV of Annuity #2 is v + . i i We are told that:

10 a10 i

i

10 11 10 a10 10v = v + =X i i

10 a10 i = 11v10 + a10

10v10

129

http://actuary88.com

a10 + a10 i + v10 10 = 0 To quickly solve this equation, we’ll convert the equation into cash flows: Convert a10 into a stream of cash flows: Time t

0 $1

1 $1

2 $1

3 $1

4 $1

5 $1

6 $1

7 $1

8 $1

9 $1

10

8 $1

9 $1

10 $1

8

9

10 $1

a10 Convert a10 Time t

into a stream of cash flows:

i

0

a10

1 $1

2 $1

3 $1

4 $1

5 $1

6 $1

7 $1

i

Convert v10 10 into a stream of cash flows: Time t

0

1

2

3

4

5

6

7

v10

10 Sum up the above 3 streams of cash flows: Time t

0 - $9

1 $2

2 $2

3 $2

4 $2

5 $2

6 $2

7 $2

8 $2

9 $2

10 $2

PV = a10 + a10 i + v10 10 = 0 To find the interest rate i , we’ll use Cash Flow Worksheet. Enter the following into Cash Flow Worksheet: CF0= - 9, C01= 1, F01 = 10 Press “IRR” “CPT.” You should get: IRR=17.96301385. So i = 17.96301385% X=

10 a10 i

i

=

10 a10

17.96301385%

17.96301385%

= 30.62092178

130

http://actuary88.com

Example 5 (May 2000 SOA EA-1 #10 modified) Term of a 20-year annuity-certain: Initial payment: $300 due 1/1/200 Payment patterns: • All payments are made January 1 •

Payments increase by $300 each year beginning 1/1/2001 through 1/1/2009



Payments decrease by $200 each year beginning 1/1/2001 through 1/1/2019

Interest rate: 7% per year, compounded annually for the 1st 9 years. 6% per year, compounded annually thereafter. Calculate the present value of the annuity. Solution First, let’s list all of the cash flows.

Time t

Date

Payment (Use $100 as one unit of money)

0

1/1/2000

$3

1

1/1/2001

$6

2

1/1/2002

$9

3

1/1/2003

$12

4

1/1/2004

$15

5

1/1/2005

$18

6

1/1/2006

$21

7

1/1/2007

$24

8

1/1/2008

$27

9

1/1/2009

$30

10

1/1/2010

$28

131

http://actuary88.com

11

1/1/2011

$26

12

1/1/2012

$24

13

1/1/2013

$22

14

1/1/2014

$20

15

1/1/2015

$18

16

1/1/2016

$16

17

1/1/2017

$14

18

1/1/2018

$12

19

1/1/2019

$10

Here we have two interest rates: 7% from t = 0 to t = 9 and 6% from t = 9 to t = 19 . As a result, we have to break down the cash flows into two streams. For each stream, we’ll directly enter the cash flows into BA II Plus Cash Flow Worksheet – this is the fastest way. Stream #1 (Enter the following into Cash Flow Worksheet) Payment (Use $100 as one unit of money)

Time t

Date

9

1/1/2009

10

1/1/2010

$28

C01

11

1/1/2011

$26

C02

12

1/1/2012

$24

C03

13

1/1/2013

$22

C04

14

1/1/2014

$20

C05

15

1/1/2015

$18

C06

16

1/1/2016

$16

C07

17

1/1/2017

$14

C08

18

1/1/2018

$12

C09

19

1/1/2019

$10

C10

CF0

132

http://actuary88.com Using the interest rate of 6%, the PV at t = 9 of Stream #1 is: 146.8777947 Stream #2 At t = 9 , we add the PV of Stream #1 to the cash flow of #30 Time t

Date

0

1/1/2000

$3

CF0

1

1/1/2001

$6

C01

2

1/1/2002

$9

C02

3

1/1/2003

$12

C03

4

1/1/2004

$15

C04

5

1/1/2005

$18

C05

6

1/1/2006

$21

C06

7

1/1/2007

$24

C07

8

1/1/2008

$27

C08

9

1/1/2009

$30+146.8777947=176.8777947

C09

Using the interest rate of 7%, you should get: NPV=191.4044063 = $19,140.44 (one unit=$100)

133

http://actuary88.com If you prefer the formula-driven approach, this is how. We still use $100 as one unit of money. We break down the cash flows into two streams. One stream consists of increasing annuity payments from t = 0 to t = 9 ; the other consists of decreasing annuity payments from t = 10 to t = 19 At t = 9 , the PV of the decreasing annuity Time t 9 10 11 12 Cash flow 30-2(1) 30-2(2) 30-2(3) 30 a10 6%

… …

18 19 30-2(9) 30-2(10)

2 ( Ia )10 6% = 146.8777947

At t = 0 , the PV of the increasing annuity Time t 0 1 2 3 Cash flow 3(1) 3(2) 30(3) 3(4)



8 3(9)

9 3(10)

3 ( Ia )10 7% = 111.51261773

The total PV at t = 0 is:

3 ( Ia )10 7% + 30 a10 6% 2 ( Ia )10 6% 1.07

9

= 111.51261773 + 146.8777947 (1.07

9

)

= 191.4044063 = $19,140.44

Example 6 (May 2005 SOA EA-1 #8) n

a2 n i = 8.00407

n 1

a2 n +1 i = 8.63279

i =the annual effective interest, compounded annually.

Calculate i . Solution n

a2 n i and

n 1

a2 n +1 i are symbols for deferred annuities. In a deferred

annuity, all the cash flows are shifted rightwards. For example, this is the cash flow diagram for a2 n i :

134

http://actuary88.com

Time t Cash flow

0

1 $1

2 $1

3

2n 1 2n



$1

$1

a2 n i To draw the diagram for

n

a2 n i , an n -year deferred annuity, we simply

shift all the above cash flows rightwards by n units of time (so the 1st cash flow starts at n + 1 , instead of t = 1 ): Time t

0

1

2



n +1 n + 2

n

Cash flow

2 n 1+ n 2 n + n



$1

n

$1

$1

( )

a2 n i = a2 n i v n

Similarly, we draw the diagram for

n 1

a2 n +1 i (an n 1 year deferred

annuity) by shifting the cash flows in a2 n +1 i to the right by n 1 units of time (so the 1st cash flow starts at n , instead of t = 1 ): Time t

0

Cash flow

1

2

$1

$1

1

2

3



2n 1

2n $1

$1

2 n +1 $1

a2 n +1 i Time t

0



n $1

Cash flow

n 1

(

)

a2 n +1 i = a2 n +1 i v n

n +1 n + 2 $1

0

1

2



n

Cash flow

Time t

n

a2 n i and

n +1 n + 2



$1

n

( 2 n + 1) + ( n 1) $1

1

Come back to the problem. Let’s compare Time t

$1

… $1

n 1

a2 n +1 i :

2 n 1+ n 2 n + n $1

$1

a2 n i 0

1

2



n $1

Cash flow

n 1

n +1 n + 2 $1

$1

… $1

( 2 n + 1) + ( n 1) $1

a2 n +1 i

135

http://actuary88.com

n 1

a2 n +1 i has 2 n + 1 cash flows starting from t = n and ending at

t = ( 2 n + 1) + ( n 1) = 3n . n

a2 n i has 2 n cash flows starting from t = n + 1 and ending at t = 2n + n = 3n .

So

n 1

a2 n +1 i has all the cash flows in

n

a2 n i , except

n 1

a2 n +1 i has one

additional cash flow at t = n . So we have: n 1

a2 n +1 i = n a2 n i + v n vn =

n

( )

n 1

a2 n i = a2 n i v n =

a2 n +1 i

n

a2 n i = 8.63279 8.00407 = 0.62872

1 v2n n v , i

1 0.628722 8.00407 = ( 0.62872 ) , i = 4.75% i

136

http://actuary88.com

Chapter 4

Calculator tips

Best calculators for Exam FM SOA/CAS approved calculators: BA-35, BA II Plus, BA II Plus Professional, TI-30X, TI-30Xa, TI-30X II (IIS solar or IIB battery). Best calculators for Exam P: BA II Plus, BA II Plus Professional, TI-30X IIS. You should bring two calculators to the exam room -- BA II Plus Professional and TI-30 IIS. BA II Plus Professional is good for general calculations and the time-value-of money calculations. TI-30 IIS is good for general calculations. Even if you have BA II Plus, you might want to buy a BA II Plus Professional.

New features added in BA II Plus Professional. We are only concerned with features relevant to Exam FM: BA II Plus Professional Net Future Value (nice feature) Modified Duration In TVM, the default values are P/Y =1 and C/Y=1 (nice improvement over BA II Plus)

BA II Plus Doesn’t have this. Doesn’t have this. In TVM, the default values are P/Y =12 and C/Y=12 (This is a pain)

How to reset calculators to their best conditions for FM According to exam rules, when you use BA II Plus, BA II Plus Professional, and TI-30X IIS for an SOA or CAS exam, exam proctors on site will need to clear the memories of your BA II Plus, BA II Plus Professional, and TI-30X IIS. Typically, a proctor will clear your calculator’s memories by resetting the calculator to its default setting. This is done by pressing “2nd” “Reset” “Enter” for BA II Plus and by simultaneously pressing “On” and “Clear” for TI-30X IIS and TI-30X IIB. You will need to know how to adjust the settings of BA II Plus and TI-30X IIS to your best advantage for the exam.

137

http://actuary88.com Best settings for BA II Plus Default setting

Optimal setting

Display 2 decimal places. Enter 0.12345×2, you’ll get 0.25. Use the chain method. The calculator calculates numbers in the order that you enter them. If you enter 2+3×10, you’ll get 50. The calculator first calculates 2+3=5. Then it calculates 5×10=50.

Display 8 decimal places. Enter 0.12345×2, you’ll get 0.24690000. Use AOS (Algebraic operating system). The calculator follows the standard rules of algebraic hierarchy in its calculation. If you enter 2+3×10, you’ll get 32. The calculator first calculates 3×10. Then it calculates 30+2=32.

Set P/Y=12 and C/Y=12 in TVM (time value of money) Worksheet. P stands for payment. C stands for compounding. Y stands for year.

Set P/Y=1 and C/Y=1.

C/Y=12 means 12 compounding periods per year. If you enter I/Y=6 (i.e.. set 6% annual interest rate), the calculator interprets this as a nominal rate compounding monthly and uses an interest rate of 6%/12=0.5% per month in its calculation.

If you enter 30 payments, you’ll get 30 payments, not 360 payments.

P/Y=12 means 12 payment in a year. If you enter 30 2nd xP/Y, you’ll get 360. This means that you are paying off a loan through 360 monthly payments.

Keystrokes to change the default setting to the optimal setting 2nd Format 8 Enter 2nd [FORMAT], keep pressing multiple times until you see “Chn.” Press 2nd [ENTER] (if you see “AOS”, your calculator is already in AOS, in which case press [CLR Work] ) 2nd P/Y 1 Enter

If you enter 6% per period (per year, per month, per day, etc), you’ll get 6% per period (per year, per month, per day, etc).

2nd C/Y 1 Enter

Always use the setting P/Y=1 and C/Y=1. Under no circumstance should you change this setting. What if you are paying off a loan through monthly payments? Simply enter the monthly interest rate and the # of monthly payments into BA II Plus TVM.

The setting of P/Y=12 and C/Y=12 is useful occasionally and harmful in majority of the times. In the heat of the exam, you can easily forget to switch settings.

BA II Plus Professional – the default setting is 2 decimal display, the chain method, and P/Y=1 and C/Y=1. As a result, you just need to set BA II Plus Professional to display 8 decimal places and the AOS. You don’t need to set P/Y=1 and C/Y=1 because this is the default setting.

138

http://actuary88.com AOS is more powerful than the chain method. For example, if you want to find 1 + 2e 3 + 4 5 , under AOS, you need to enter 1 + 2× 3 2nd e x + 4× 5 x (the result is about 50.1153) Under the chain method, to find 1 + 2e 3 + 4 5 , you have to enter: 1+(2×3 2nd e x )+(4×5 x ) AOS is better because the calculation sequence under AOS is the same as the calculation sequence in the formula. In contrast, the calculation sequence in the chain method is cumbersome. TI-30X IIS --- You need change only one item on your once the proctor resets it. In its default settings, TI-30X IIS displays two decimal places. You should set it to display 8 decimal places. Press 2nd Fix. The choose “8.” The power of the TI-30X IIS lies in its ability to display the data and formula entered by the user. This “what you type is what you see feature” allows you to double check the accuracy of your data entry and of your formula. It also allows you to redo calculations with modified data or modified formulas. For example, if you want to calculate 2e the calculator, you will see the display:

2e ( 2.5)

2.5

1 , as you enter the data in

1

If you want to find out the final result, press the “Enter” key and you will see: 2e ( 2.5)

1

-0.835830003

So 2e ( 2.5)

1 = -0.835830003

After getting the result of -0.835830003 , you realize that you made an error in your data entry. Instead of calculating 2e ( 2.5) 1 , you really wanted to calculate 2e ( 3.5) 1 . To correct the data entry error, you simply change “–2.5” to “–3.5” on your TI-30X IIS. Now you should see:

139

http://actuary88.com 2e ( 3.5) 1 -0.939605233

With the online display feature, you can also reuse formulas. For example, a problem requires you to calculate y = 2e x 1 for x1 = 5, x 2 = 6, and x 3 = 7. There is no need to do three separate calculations from scratch. You enter 2e (5) 1 into the calculator to calculate y when x = 5 . Then you modify the formula to calculate y when x = 6 and x = 7 .

Compound interest Problem 1 Today Mary deposits $23.71 into a bank account and earns 6% annual effective. Calculate the balance of her bank account 2 years from today. Solution The formula for simple interest rate is: A ( n ) = A ( 0 )(1 + i )

n

We are given: A ( 0 ) = 23.71, i = 6%, n = 2 . A ( 2 ) = 23.71(1 + 6% )

2

Method 1 BA II Plus/BA II Plus Professional Procedure Assume you already set the calculator to display 8 decimal place and to use AOS. Calculate 23.71(1 + 6% )2

Keystrokes Omitted

Display Omitted

23.71× ( 6% +1 )

23.71000000 0.06000000 1 1.06000000 1.12360000

x2

=

26.64055600

140

http://actuary88.com •

Watch out the % operator. In BA II Plus/BA II Plus Professional, a ± b% is calculated as a (1 ± b% ) . For example, if you enter 100+5%, you’ll get 105, not 100.05. If you enter 100-5%, you’ll get 95, not 99.95%. This is not a problem if you want to calculate (1 + 6% ) . However, if you want to calculate ( 2 + 6% ) and you enter (2+6%), you’ll get 2.12. To calculate 2+6%, you can enter 6%+2, which gives you 2.06. So to avoid any mistakes, always enter b% + a if you want to calculate a + b% .



Please note that if you are to calculate 23.71(1.06)5.6, you can enter 23.71×1.06 y x 5.6=. You should get 32.85824772.



In BA II Plus/BA II Plus Professional, “ ) ” is the same as “=”. You can solve the problem using the following calculator key strokes: Procedure Calculate 23.71(1 + 6% )

Keystrokes

Display

23.71× (1+ 6% )

23.71000000 1.00000000 0.06000000 1.06000000 1.12360000

2

x2

)

26.64055600

Method 2 --- TI-30 IIS • • •

Set to display 8 decimal places Enter 23.71(1+6%)2. Press Enter. You should get 26.64055600. Double check your data entry. Press . Double check that you indeed entered 23.71(1+6%)2. Yes, you did. So the result 26.64055600 is correct.

Please note that if you are to calculate 23.71(1+6%)5.6, you can enter 23.71(1+6%)^5.6. You should get 32.85824772. Please also note that TI30 IIS calculates a+b% as a+b%. For example, if you enter 2+6%, you’ll get 2.06.

141

http://actuary88.com Method 3 --- Use

% Worksheet of BA II Plus/BA II Plus Professional

Procedure Choose % Worksheet

Keystroke

Display OLD= (old content)

2nd [ % ]

Clear worksheet

OLD=0.00000000 2nd [CLR Work]

Enter principal.

OLD=23.71000000 23.71 Enter

Enter the # of compounding period (can de a integer or fraction)

#PD=1.00000000 (The default period is one) #PD=2.00000000 2 Enter

Enter the interest rate

% CH=0.00000000 (The default is zero) % CH=6.00000000 6 Enter (Enter 6, not 6%)

Calculate the balance

NEW=0.00000000 (The default is zero) NEW=26.64055600 CPT

Note – The # of compounding periods can be a fraction. If you are to calculate 23.71(1+6%)5.6, you simply enter the following: OLD=23.71, % CH=6, # PD=5.6. You should get NEW=32.85824772

142

http://actuary88.com Method 4 --- Use TVM Worksheet of BA II Plus/BA II Plus Professional Procedure Set C/Y=1 and P/Y=1. We always set C/Y=1 and P/Y=1 before using TVM. Clear TVM Worksheet

Keystroke

Display

0.00000000 2nd [CLR TVM]

Enter principal.

PV=23.71000000 23.71 PV Enter

Enter interest rate.

I/Y=6.00000000 6 I/Y Enter

Enter the # of compounding period (can de a integer or fraction) Calculate the accumulated value.

N=2.00000000 2 N Enter FV= - 26.64055600 CPT FV

Note: (1) The negative sign in FV= - 26.64055600 means that you are paying off a loan of $26.64055600. As a general rule, a positive sign in TVM means money coming from someone else’s pocket and going to your pocket; a negative sign in TVM means money leaving your pocket and going to someone else’s pocket. Together PV=23.71, I/Y=6%,N=2, and FV= -26.64055600 mean this: •

at time zero $23.71 flows from someone’s pocket to your pocket; in other words, you borrow $23.71 from someone.



you use the money for 2 years and are charged with 6% interest rate.



at the end of Year 2, $26.64055600 comes from your pocket to someone else’s pocket. In other words, you pay the lender $26.64055600 at the end of Year 2. $26.64055600 is greater than $23.71 because it includes interest payment.

(2) You can also set PV= - 23.71, I/Y=6%,N=2. This will give you FV= 26.64055600. You lend $23.71 for 2 years with 6% interest. At the end of Year 2, you receive $26.64055600. (3) Because there are no regular payments in this problem, it does not matter whether you select end-of-period payments by setting 2nd END or you select beginning-of-period payments by setting 2nd BGN.

143

http://actuary88.com (4) The # of compounding periods can be a fraction. If you are to calculate 23.71(1+6%)5.6, you simply enter the following: PV=23.71, I/Y=6%,N=5.6 You should get FV= - 32.85824772

Annuity Problem 1 Calculate a10 6% Method 1 – Use TVM of BA II Plus/BA II Plus Professional Procedure Set to display 8 decimal places. Make sure that you set C/Y=1 and P/Y=1. This is the golden rule. Never break this rule under any circumstances. Clear TVM Worksheet

Keystroke

Display

2nd [CLR TVM] If your calculator screen displays

0.00000000

0.00000000

Use the immediate annuity function (rather than annuity due). If last time when TVM was used and TVM was in the annuity due mode, you need to set it to the immediate annuity. Forgetting to do so gives you a wrong result without you knowing it.

Then it’s already in the immediate annuity mode. You do nothing. If your calculator screen displays BGN

0.00000000 Then it’s in the annuity due mode. You need to set it to the annuity due mode by: 2nd BGN 2nd Set

END

CE/C (This enables the calculator to the standard calculation mode)

0.00000000

Enter the level payment.

PMT=1.00000000 1 Payment Enter (We enter a positive 1 to indicate that we want to receive a payment of $1.)

Enter # of payments 10 N Enter

N=10.00000000

144

http://actuary88.com Calculate PV when the interest rate is zero. This is an extra step to double check whether we entered the right cash flow amounts and the correct # of payments.

PV= - 10.00000000 (The result is correct. The PV of 10 level payments of $1 @ i=0 is 10. )

CPT PV

The negative sign indicates cash outgo. In other words, if we spend $10 now to buy a 10 year annuity immediate @ i=0, we’ll receive $1 per year. Enter interest rate.

I/Y=6.00000000 6 I/Y Enter

Enter the # of compounding period (can de a integer or fraction). Calculate the present value.

N=10.00000000 10 N Enter PV= - 7.36008705 CPT PV

So a10 6% = 7.36008705

Method 2 – Use Cash Flow Worksheet of BA II Plus/BA II Plus Professional Before using CF Worksheet, let’s identify all of the cash flows: Time t

0

Cash flow $0

1

2

3

4

5

6

7

8

9

10

$1

$1

$1

$1

$1

$1

$1

$1

$1

$1

a10 6% = ?

Procedure Set to display 8 decimal places. Use Cash Flow Worksheet

Keystroke

Display

CF

CF0=(old content)

2nd [CLR WORK]

CF0=0.00000000 CF0=0.00000000

Clear Worksheet

t=0. Because the cash flow is zero, we don’t need to enter anything. Just press the down arrow. Enter the cash flow at

145

http://actuary88.com Enter the dollar amount of the level payments.

C01 0.00000000

1 Enter Enter the # of level payments. We have 10 level payments from t=1 to

F0= 1.00000000 (The default # is 1.)

t=10.

Use NVP portion of Cash Flow Worksheet. Calculate NPV when the interest rate is zero. This is an extra step to double check whether we entered the right cash flow amounts and the correct # of payments.

C01=1.00000000

10 Enter

F0= 10.00000000

NPV

I=0.00000000 (The default interest rate is zero.) NPV= 0.00000000 (The default # is 0.)

CPT

NPV= 10.00000000 (The result is correct. The PV of 10 level payments of $1 @ i=0 is 10.)

Enter the interest rate. I=0.00000000 (The default interest rate is zero.)

6 Enter Calculate NPV when the interest rate is zero. This is an extra step to double check whether we entered the right cash flow amounts and the correct # of payments.

I=6.00000000

NPV= 10.00000000 (This is the previous NPV we calculated last time @ i=0.) CPT

NPV= 7.36008705

So a10 6% = 7.36008705

Problem 2 Calculate a10 6% Solution

146

http://actuary88.com Method 1 – Use TVM Worksheet. The calculation procedures are identical to the procedure for calculating a10 6% , except that we need to use the annuity due mode. You should get: a10 6% = 7.80169227

Method 2 – Use CF Worksheet. Time t

0

Cash flow $1

1

2

3

4

5

6

7

8

9

$1

$1

$1

$1

$1

$1

$1

$1

$1

a10 6% = ? The calculation procedures are identical to the procedure for calculating a10 6% , except that we need to enter the following cash flows: CF0=1; C01=1,F01=9. You should get: a10 6% = 7.80169227 Problem 3 Calculate s10 6% Solution Method 1 – Use TVM • • • • • •

Display 8 decimal places. Set C/Y=1, P/Y=1. Use the immediate annuity mode. Set PMT=1, N=10. CPT FV. You should get FV= - 10 @ i =0. The result is correct. Set I/Y=6. Press: CPT, FV.

You should get FV= - 13.18079494. So s10 6% = 13.18079494

147

http://actuary88.com Method 2 – use Cash Flow Worksheet. Time t

0

Cash flow $0

1

2

3

4

5

6

7

8

9

10

$1

$1

$1

$1

$1

$1

$1

$1

$1

$1

s10 6% = ? Use BA II Plus Professional Keystrokes Procedure Set to display 8 decimal places. Use Cash Flow Worksheet

Keystroke

Display

CF

CF0=(old content)

2nd [CLR WORK]

CF0=0.00000000 CF0=0.00000000

Clear Worksheet Enter the cash flow at

t=0.

Enter the dollar amount of the level payments.

C01 0.00000000

1 Enter Enter the # of level payments. We have 10 level payments from t=1 to

C01=1.00000000

F01= 1.00000000 (The default # is 1.)

t=10. 10 Enter

F01= 10.00000000

NPV

I=0.00000000 (The default interest rate is zero.)

Use FVP portion of Cash Flow Worksheet.

NPV= 0.00000000 (The default # is 0.) NFV= 0.00000000 (The default net future value is zero.)

Calculate NPV when the interest rate is zero. This is an extra step to double check whether we entered the right cash flow amounts and the correct # of payments. Enter the interest rate.

CPT

NPV= 10.00000000 (The result is correct. The NFV of 10 level payments of $1 @ i=0 is 10.)

148

http://actuary88.com NPV

I=0.00000000 (The default interest rate is zero.)

6 Enter

I=6.00000000

Calculate NPV when the interest rate is zero. This is an extra step to double check whether we entered the right cash flow amounts and the correct # of payments.

NPV= 10.00000000 (NPV is 10 @ i=0. Even though we entered i=6%, we didn’t press CPT. If CPT is not pressed, BA II Plus Professional does not update the NPV using the latest interest rate. It merely displays the old NPV value.) NFV= 10.00000000 (NFV is 10 @ i=0. Even though we entered i=6%, we didn’t press CPT. If CPT is not pressed, BA II Plus Professional does not update the NFV using the latest interest rate. It merely displays the old NPV value.) CPT

NFV= 13.18079494 (BA II Plus Professional calculates NFV using the latest interest rate.)

So s10 6% = 13.18079494

Use BA II Plus BA II Plus does not have NFV. However, we can still calculate s10 6% using the following relationship:

sn i = an i (1 + i ) We have a10 6% = 7.36008705 n

( Is )10 6%

= 7.36008705 (1.0610 ) = 13.18079494

Problem 4 Calculate s10 6%

149

http://actuary88.com Solution Method 1 – Use TVM • • • • • •

Display 8 decimal places. Set C/Y=1, P/Y=1. Use the annuity due mode. Set PMT=1, N=10. CPT FV. You should get FV= - 10 @ i =0. The result is correct. Set I/Y=6. Press: CPT, FV.

You should get FV= - 13.97164264. So s10 6% = 13.97164264

Method 2 – use Cash Flow Worksheet. BA II Plus Professional Time t

0

Cash flow $1

1

2

3

4

5

6

7

8

9

10

$1

$1

$1

$1

$1

$1

$1

$1

$1

$0

s10 6% = ?

Please note BA II Plus Professional Cash Flow Worksheet always accumulates cash flows to the final payment time. In order to calculate s10 6% (which accumulates value to t=10), we needed to add an additional cash flow of zero at t=10. This tells BA II Plus Professional Cash Flow Worksheet to use t=10 as the ending time to accumulate cash flows. If we don’t add this additional cash flow of zero to t=10, BA II Plus will calculate the accumulated value to t=9. Keystrokes – BA II Plus Professional only (not for BA II Plus):

Procedure Set to display 8 decimal places. Use Cash Flow Worksheet

Keystroke

Display

CF

CF0=(old content)

2nd [CLR WORK]

CF0=0.00000000

Clear Worksheet

150

http://actuary88.com Enter the cash flow at

CF0=1.00000000

t=0. 1 Enter

Enter the dollar amount of the payments.

C01 0.00000000

1 Enter Enter the # of level payments. We have 9 level payments from t=1 to

C01=1.00000000

F0= 1.00000000 (The default # is 1.)

t=9. 9 Enter Enter zero cash flow at t=10, increasing the # of time periods by one.

F0= 9.00000000

C02 0.00000000 (The default # is zero. At this point, BA II Plus Professional assumes that cash flows end at t=9 and there’s no cash flow at t=10. You can verify this by pressing . If you press , you’ll see that F02 is zero. In other words, the # of cash flow at t=10 is zero.) Enter

C02 =0.00000000

If we omit this step, BA II Plus Professional will accumulate values to t=9 and gives us the final result of NPV=13.18079494. You can verify this yourself.

Once “Enter” is pressed, the display “ C02 0.00000000” becomes “C02 =0.00000000.” By pressing “Enter,” you tell BA II Plus Professional to increase the time period by one. You can verify this by pressing . If you press , you’ll see that F02=1. In other words, the # of cash flow at t=10 is one.

NPV Use FVP portion of Cash Flow Worksheet.

I=0.00000000 (The default interest rate is zero.) NPV= 0.00000000 (The default # is 0.) NFV= 0.00000000 (The default net future value is zero.)

151

http://actuary88.com Calculate NPV when the interest rate is zero. This is an extra step to double check whether we entered the right cash flow amounts and the correct # of payments. Enter the interest rate.

CPT

NPV= 10.00000000 (The result is correct. The NFV of 10 level payments of $1 @ i=0 is 10.)

NPV

I=0.00000000 (The default interest rate is zero.)

6 Enter

I=6.00000000

Calculate NPV when the interest rate is zero. This is an extra step to double check whether we entered the right cash flow amounts and the correct # of payments.

NPV= 10.00000000 (NPV is 10 @ i=0. Even though we entered i=6%, we didn’t press CPT. If CPT is not pressed, BA II Plus Professional does not update the NPV using the latest interest rate. It merely displays the old NPV value.) NFV= 10.00000000 (NFV is 10 @ i=0. Even though we entered i=6%, we didn’t press CPT. If CPT is not pressed, BA II Plus Professional does not update the NFV using the latest interest rate. It merely displays the old NPV value.) NFV= 13.97164264 (BA II Plus Professional calculates NFV using the latest interest rate.)

CPT

So s10 6% = 13.97164264 If we omit the step of entering the cash flow of zero at t=10, BA II Plus Professional will give us NFV=13.18079494. We can verify that 13.18079494 = 13.97164264 (1.06

1

)

You see that 13.18079494 is the accumulated value at t=9, while 13.97164264 is the accumulated value at t=10.

152

http://actuary88.com How can we calculate the NFV if we need to accumulate the same cash flows to t=11? Now the cash flows diagram is:

Time t

0

Cash flow $1

1

2

3

4

5

6

7

8

9

10

11

$1

$1

$1

$1

$1

$1

$1

$1

$1

$0

$0

NFV=?

We simply set C02=0, F02=2. This will move the ending time from to t=9 to t=11. We’ll get the result that NPF = 14.80994120. We can verify that 14.80994120 = 13.97164264 (1.06 ) .

Use BA II Plus BA II Plus does not have NFV. However, we can still calculate s10 6% using the following relationship: s n i = an i (1 + i )

n

We have a10 6% = 7.80169227 s10 6% = a10 6% (1.06 ) = 7.80169227 (1.06 ) = 13.97164263 10

10

Loan/bond amortization In BA II Plus/BA II Plus Professional, TVM Worksheet is automatically tied to Amortization Worksheet. For a given loan or bond, if you find PV (which is the principal of a loan), PMT (regular payment), N (# of payments) and I/Y (the interest rate) through TVM Worksheet, Amortization Worksheet can generate an amortization schedule for you, splitting each payment into principal and interest.

153

http://actuary88.com Problem 1 (#10, Sample FM) A 10,000 par value 10-year bond with 8% annual coupons is bought at premium to yield an annual effective rate of 6%. Calculate the interest portion of the 7th payment. (A) 632

(B) 642

(C) 651

(D) 660

(E) 667

Solution One key thing to remember is that a bond is a loan. When you buy this $10,000 par 10 year bond with 8% annual coupons yielding 6%, you lend your money to whoever issued the bond. The money your lend (principal) is the present value of all the future cash flows (10 coupons of $800 each from t=1 to t=10 plus a final cash flow of $10,000 at t=10) discounted at an annual effective 6%. And the borrower (bond issuer) pays back your loan through 10 coupons of $800 each from t=1 to t=10 plus a final cash flow of $10,000 at t=10. We first draw a cash flow diagram: Time t

0

Cash flow

1

2

3

4

5

6

7

8

9

$800

$800

$800

$800

$800

$800

$800

$800

$800

10 $800 $10,000

For BA II Plus/BA II Plus Professional Amortization Worksheet to generate an amortization schedule, we first need to calculate PV. Let’s calculate PV of the bond. Procedures for using TVM: • Display 8 decimal places. •

Set C/Y=1, P/Y=1.



Use the annuity immediate mode.



Set PMT=800, N=10, FV=10,000. By making PMT and FV positive, we are getting 10 level payments of $800 each year and a final payment of $10,000 at t=10. So we bought the bond and should receive cash flows in the future. This will generate a negative PV,

154

http://actuary88.com which is our purchase price of the bond; we pay now to get cash flows in the future. •

Alternatively, we can make PMT= - 800 and FV= - 10,000; we sold the bond. We’ll get a positive PV, which is our selling price of the bond; we get cash now but pay cash flows in the future. Either way is fine as long as we make PMT and FV have the same signs.



CPT PV. This calculates PV @ i =0. We should get PV= - 18,000. The result is correct. If the interest rate is zero, PV is just the sum of all cash flows. PV=800(10)+10,000=18,000.



Set I/Y=6. Press: CPT, PV.

We should get PV= - 11,472.01741. Now Amortization Worksheet is ready to generate an amortization schedule for us, splitting each payment into principal and interest. Procedure Use Amortization Worksheet.

Keystroke

Display

2nd Amort

P1=old content P1=7.00000000

7 Enter So the payment to be split begins at t =7.

Tell the calculator that we are interested in the 7th payment.

P2= old content. P2=7.00000000 7 Enter So the payment to be split ends at t =7. BAL= - 10,534.60239 Find the outstanding balance after the 7th payment

BA II Plus/BA II Plus Professional always calculates the outstanding balance after the P2 level payment is made. So the outstanding balance AFTER the 7th payment and immediately before the 8th payment is $10,534.60239. Since PV is negative, the outstanding balance should also be negative.

155

http://actuary88.com Find the principal portion of the 7th level payment.

PRN= 158.4187327 BA II Plus/BA II Plus Professional splits the total payments starting from P1 and ending with P2 into the principal repayment and interest payment. Because we set P1=P2=7, the calculator splits only the 7th payment into principal and interest. Of the $800 payment at t=7, $158.42 is the repayment of the principal. A positive $158.42 means that we receive $158.42. This makes sense. Our PV is negative (we lent our money at t=0). As a result, we will receive repayment of our principal.

Find the interest portion of the 7th payment.

INT= 641.5812674 Of the $800 payment at t=7, $641.58 is the interest payment. A positive $641.58 means that we receive $641.58.

So the interest portion of the 7th payment is about $642. We can verify that the results are correct: Time t Cash flow

0

1

2

3

4

5

6

7

8

9

$800

$800

$800

$800

$800

$800

$800

$800

$800

The outstanding principal immediate after the 6th payment

The interest accrued from t=7 to t=8 The principal portion of the 7th payment

800a4 6% + 10, 000 (1.06

4

10 $800 $10,000

) = 10, 693.02112

To verify, in the amortization worksheet, If you set P1=P2=6, you should get BAL= - 10,693.02112. 10, 693.02112 ( 6% ) = 641.58 800 641.58 = 158.42

Please note that Amortization Worksheet uses data you entered or calculated in TVM Worksheet. Whenever you update TVM Worksheet,

156

http://actuary88.com Amortization Worksheet is automatically updated. As a result, you don’t need to use 2nd CLR Work to clear Amortization Worksheet. Amortization Worksheet is always in sync with TMV Worksheet. As long as the data in TVM Worksheet is correct, Amortization Worksheet will generate the correct amortization schedule. Additional calculations on this problem: •

What’s the outstanding balance immediately BEFORE the 7th payment? Remember that BA II Plus/BA II Plus Professional Amortization Worksheet always calculates the outstanding balance after the P2 level payment is made. So first we find the outstanding balance immediately after the 7th payment is made. So we set P2=7. What about P1? We can set P1=1,2,3,4,5, 6, or 7. In other words, P1 needs to be a positive integer equal to or smaller than P2. We should get BAL= - 10.534.60239. Immediately after the 7th payment is made, the outstanding balance is 10.534.60239. So the outstanding balance immediately before the 7th payment is made is 10.534.60239+800=11,334.60239. (Here’s another approach. We already know that the outstanding loan balance immediately after the 6th payment is 10,693.02112. Accumulating this amount with interest for one year is 10, 693.02112 (1.06 ) = 11,334.60239 . So the outstanding loan balance immediately before the 7th payment is 11,334.6023.)



What’s the outstanding balance after the final coupon is paid at t=10? Without using any calculators, we know that the outstanding balance must be $10,000 after the 10th coupon of $800 is paid. Will Amortization Worksheet produce this result? Let’s check. Set P2=10; P1 can be any positive integer no greater than 10. We get BAL= - 10,000.



What’s the total principal and interest paid by the bond issuer during the life of the bond? Remember BA II Plus/BA II Plus Professional always splits the payments starting from P1 and ending with P2 into principal and interest. Let’s set P1=1 and P2=10. We’ll get PRN=1,472.017410 and INT=6,527.982590. Let’s check: The total repayment of the loan (simple sum of all the future cash flows) The principal of the loan (the initial price of the bond)

10(800)+10,000=18,000 PV=11,472.01741=PRN (OK)

157

http://actuary88.com The total interest paid during the life of the bond •

18,000 -11,470.01741 =6,57.982590 (OK)

How to split into principal and interest the total payments made during the 2nd, 3rd, 4th, and 5th payments? Easy! Set P1=2 and P2=5. We get PRN=517.8656978 and INT=2,682.134302. So out of the total payments made during the 2nd, 3rd, 4th, and 5th payments, the principal portion is 517.8656978; the interest portion is 2,682.134302. Let’s check:

Time t Cash flow

0

1

2

3

4

5

6

7

8

9

$800

$800

$800

$800

$800

$800

$800

$800

$800

The outstanding loan immediately AFTER the 1st payment is made The outstanding loan immediately AFTER the 5th payment is made Reduction of principal due to the 2nd, 3rd, 4th, and 5th payments Total repayment made in the 2nd, 3rd, 4th, and 5th payments Total interest paid in the 2nd, 3rd, 4th, and 5th payments

800a9 6% + 10, 000 (1.06

9

)=

10 $800 $10,000

11,360.33845

To verify, in the amortization worksheet, set P1=1 and P2=1. You should get BAL= - 11,360.33845. 800a5 6% + 10, 000 (1.06 5 ) = 10,842.47276 To verify, in the amortization worksheet, set P1=5 and P2=5. You should get BAL= - 10,842.47276 11,360.33845 10,842.47276 = 517.8656979 (The result matches PRN=517.8656978. The slight difference is due to rounding) 800(4)=3,200 3,200– 517.8656979=2,682.134302 The result matches INT=2,682.134302

158

http://actuary88.com

Compare Cash Flow Worksheet with TVM Worksheet Pros of using Cash Flow Worksheet over TVM: • Avoid inadvertently using C/Y=12 and P/Y=12 • Avoid painful switching between the annuity due mode and the annuity immediate mode • Handle level and non-level payments Cons of using Cash Flow Worksheet over TVM • A candidate can forget that the 1st cash flow in Cash Flow Worksheet is the CF0 (which takes place at t=0), not C01 (which takes place at t=1). • A candidate needs to carefully track down the timing of each cash flow. • TVM is automatically tied to Amortization Worksheet and can generate an amortization schedule; Cash Flow Worksheet is NOT tied to Amortization Worksheet and can NOT generate an amortization schedule. I recommend that you master both methods. For a non-amortization exam problem, you can use both methods for the same problem and double check your calculations. If you are good at using BA II Plus/BA II Plus Professional, each method takes you only about 10 seconds. You should have time to use both methods for the same problem.

Increasing annuity Problem 1 Calculate ( Ia )10 5% Solution Time t

0

Cash flow $0

1

2

3

4

5

6

7

8

9

10

$1

$2

$3

$4

$5

$6

$7

$8

$9

$10

( Ia )10 5% We’ll use BA II Plus/BA II Plus Professional Cash Flow Worksheet. 159

http://actuary88.com

Enter the following: CF0=0; C01=1; C02=2; C03=3; C04=4; C05=5; C06=6; C07=7; C08=8; C09=9; C010=10. You don’t need to enter F01=1, F02=1, …,F10=1. BA II Plus/BA II Plus Professional will automatically set them to one. This is how BA II Plus/BA II Plus Professional sets the # of level cash flows: • • •

If a cash flow is zero, the default # of level cash flows is zero. If a cash flow is none zero, the default # of level cash flows is 1. If, for any cash flow, a user does not specifically enter the # of level cash flows, Cash Flow Worksheet uses the default # of level cash flows.

Next, calculate NPV. You should get 55. This is NPV @ i=0. We can verify this is correct: 1 1 + 2 + 3 + ... + 10 = (10 )(11) = 55 2 We have correctly entered the cash flow amounts and the # of cash flows. Next, set I=6. We get NPV=36.96240842. Let’s check. We’ll use the formula:

( Ia )n

( Ia )10 6%

=

=

nv n

an i

a10 6% 10 (1.06 10 ) 7.80169227 5.58394777 6%

=

6%

= 36.96240842

Why bother using the calculator when a formula works fine? The calculator reduces a complex increasing annuity formula to simple keystrokes. As long as you enter the correct data, the calculator will generate the result 100% right. In contrast, if you use the increasing annuity formula, you might miscalculate. The secret to doing error-free calculations for a complex problem in the heat of the exam is to reduce a complex problem into a simple mechanic solution. We may not always be able to do so. However, if some problems

160

http://actuary88.com have mechanic solutions, we prefer to give our brain a rest and use mechanic solutions to solve the complex problems. Knowing that BA II Plus/BA II Plus Professional can calculate an increasing annuity for us, do we still need to memorize the increasing annuity? Yes. SOA can always set up a problem in such way that a mechanic solution is impossible and some amount of thinking is needed. As a result, we’ll still need to memorize the increasing annuity formula. Problem 2 Calculate ( Ia )10 5% Solution Time t

0

Cash flow $1

1

2

3

4

5

6

7

8

9

$2

$3

$4

$5

$6

$7

$8

$9

$10

( Ia )10 5% We’ll use BA II Plus/BA II Plus Professional Cash Flow Worksheet. Enter the following: CF0=1; C01=2; C02=3; C03=4; C04=5; C05=6; C06=7; C07=8; C08=9; C09=10. Calculate NPV. We get NPV=55 @ i=0 (OK). Set I=6. We get NPV=39.18015293. Let’s check. We’ll use the formula:

( Ia )n

=

( Ia )10 6%

=

nv n

an d

=

i ( Ia )n d

0.06 36.96240842 = 39.18015293 1 1.06 1

161

http://actuary88.com

Problem 3 Calculate ( Is )10 5% Solution Time t

0

Cash flow $0

1

2

3

4

5

6

7

8

9

10

$1

$2

$3

$4

$5

$6

$7

$8

$9

$10

( Is )10 5% BA II Plus Professional Cash Flow Worksheet: Enter the following: CF0=0; C01=1; C02=2; C03=3; C04=4; C05=5; C06=6; C07=7; C08=8; C09=9; C010=10. Next, calculate NFV. You should get NFV=55 @ i=0 (OK). Finally, set I=6. We get NFV=66.19404398. Let’s check. We’ll use the formula: n ( Is )n = ( Ia )n (1 + i )

( Is )10 6%

= 36.96240842 (1.0610 ) = 66.19404398

Use BA II Plus Cash Flow Worksheet (which doesn’t have the FPV function) We use the formula: ( Is )n = ( Ia )n (1 + i )

n

Using Cash Flow Worksheet, we get: NPV = ( Ia )10 6% = 36.96240842

( Is )10 6%

= 36.96240842 (1.0610 ) = 66.19404398

162

http://actuary88.com Problem 4 Calculate ( Is )10 5% Solution Time t

0

Cash flow $1

1

2

3

4

5

6

7

8

9

10

$2

$3

$4

$5

$6

$7

$8

$9

$10

$0

( Is )10 5% Use BA II Plus Professional Cash Flow Worksheet. Enter the following: CF0=1; C01=2; C02=3; C03=4; C04=5; C05=6; C06=7; C07=8; C08=9; C09=10. C10=0;F10=1 (This tells BA II Plus Professional to accumulate value at to t=10.) Calculate NPV. We get NFV=55 @ i=0 (OK). Set I=6. We get NFV=70.16568662. n Let’s check: ( Is )n i = ( Ia )n i (1 + i )

( Is )10 6% = ( Ia )10 6% (1.06 )10 = 39.18015293 (1.06 )10 = 70.16568662 Use BA II Plus Cash Flow Worksheet. First, we calculate ( Ia )10 6% = 39.18015293

( Is )10 6% = ( Ia )10 6% (1.06 )10 = 39.18015293 (1.06 )10 = 70.16568662

163

http://actuary88.com Problem 5 Calculate a1(126%) Solution Let’s first draw a cash flow diagram: Time t 0 (Months) Cash flow $0

1

$

1 12

2

$

3

4

1 1 $ 12 12

$

5

1 12

$

6

7

8

1 1 1 1 $ $ $ 12 12 12 12

$

9

10

1 12

$

1 12

11

$

1 12

12

$

1 12

a1(126%) 1 are made monthly yet the 12 interest rate of 6% is compounding annually. When the payments frequency and the compounding frequency do not match, we always use the payment frequency as the compounding period for our calculations in BA II Plus/BA II Plus Professional. Remember this rule. Never deviate from this rule.

This is a case where level payments of

Let’s convert the annual interest rate to the monthly interest rate:

(1 + i )

12

= 1.06

1

i = 1.0612 1 = 0.48675506%

Though here I explicitly write the monthly interest rate as 0.48675506% , when you solve this problem in the exam, you can calculate the monthly interest rate but store the result in one of the calculator’s memories (we talked about this before). This eliminates the error-prone process of transferring a long decimal number back and forth between your calculator and the scrap paper. Next, we’ll change the complex annuity a1(126%) to a standard annuity:

a1(126%) =

1 a 12 12

i =0.48675506%

Using TVM Worksheet or Cash Flow Worksheet, we can easily calculate:

164

http://actuary88.com

a12

i =0.48675506%

a1(126%) =

= 11.62880032

1 a 12 12

i =0.48675506%

= 0.96906669

You can use the same method and calculate an( mi) , an( mi) , sn( mi) , sn( mi) :

an( mi) =

1 1 1 1 am n j , an( mi) = am n j , sn( mi) = sm n j , sn( mi) = sm n m m m m

j

1

Where j = (1 + i ) m 1

Problem 6 Calculate a1(12 ) , where the interest rate is i (12 ) = 6% Solution Let’s first draw a cash flow diagram: Time t 0 (Months) Cash flow $0

1

$

1 12

2

$

3

1 1 $ 12 12

4

$

1 12

5

$

6

7

8

1 1 1 1 $ $ $ 12 12 12 12

$

9

10

1 12

$

1 12

11

$

1 12

12

$

1 12

a1(12) In this problem, the payment frequency is monthly and the interest rate given is the nominal rate compounding monthly. When the frequency of level payments matches the frequency by which a nominal interest compounds, BA II Plus/BA II Plus Professional TVM Worksheet has a shortcut way to calculate the annuity value. However, never use this shortcut; it causes more troubles than good. Let’s first go through the shortcut in TVM Worksheet:

165

http://actuary88.com Dangerous procedure to calculate a1(12 ) : •

Display 8 decimal places.



Set C/Y=12 and P/Y=12. (C=compound. C/Y=12 means that the interest rate compounds 12 times in a year; P=payment. P/Y=12 means paying 12 times in a year).



Use the annuity immediate mode.



Set PMT=1/12, N=12.



CPT PV. This calculates PV @ i =0. We should get PV= - 1 (OK).



Set I/Y=6 (here we enter the nominal interest rate instead of converting the nominal rate to the monthly interest rate).



CPT PV. We should get: PV= - 0.96824434

Though its result is correct, this procedure changes the safe setting of C/Y=1 and P/Y=1 to a dangerous setting of C/Y=12 and P/Y=12. If you set C/Y=12 and P/Y=12 but forget to reset to the safe setting of C/Y=1 and P/Y=1, when you enter an interest rate in TMV, TVM will treat this interest rate as the nominal interest rate compounding monthly. You might say, “This is OK. I’ll remember to change C/Y=12 and P/Y=12 back to C/Y=1 and P/Y=1.” However, in the heat of the exam, it’s very easy to forget to reset C/Y=1 and P/Y=1. If you forget to reset C/Y=1 and P/Y=1, all your annuity calculations where an annual effective interest rate is entered will be wrong. For this reason, always stick to the safe setting C/Y=1 and P/Y=1. Never set C/Y=12 and P/Y=12. The safe procedure to calculate a1(12 ) • • • • • • •

Display 8 decimal places. Set C/Y=1 and P/Y=1. Use the annuity immediate mode. Set PMT=1/12, N=12. CPT PV. This calculates PV @ i =0. We should get PV= - 1 (OK). Set I/Y=6/12=0.5 (we simply enter the monthly interest rate). CPT PV. We should get: PV= - 0.96824434

166

http://actuary88.com

Comprehensive calculator exercise Problem 1 A loan of $100,000 borrowed at 6% annual effective is repaid by level monthly payments in advance over the next 30 year. After 10 years, the outstanding balance of the loan is refinanced at 4% annual effective and is paid by level monthly payments in advance over 20 years. Calculate: • The monthly payment of the original loan. • The principal portion and the interest portion of the 37th payment. • The monthly payment of the refinanced loan. • The accumulated value of the reduction in monthly payments invested at 4% annual effective. Solution Find the monthly payment of the original loan Time t (Month)

0

1

2

……

357

Y

Y

Y

……

Y

358

Y

359

Y

360

Y

360 Months

Y a360

j

Keystrokes for TVM: •

Display 8 decimal places.



Set C/Y=1 and P/Y=1.



Use the annuity due mode.



Set N=360, PV=100,000.

167

http://actuary88.com •

CPT PMT. This calculates PMT @ i =0. We should get PMT= 277.7777778. Check: 100,000/360277.7777778. OK. 1 12



Set I/Y=100 1.06

1 . Remember to multiple the interest rate

by 100 (ex. enter 6 if the interest rate is 6%).



CPT PMT. We should get: PMT= - 586.5155230

Find the principal portion and the interest portion of the 37th payment The 37th payment is the 1st payment in the 4th year. Time t (Month)

0

1

2

……

357

358

359

360

Y

Y

Y

……

Y

Y

Y

Y

360 Months

Y a360

j

Keystrokes for TVM: If you don’t clear TVM, TVM remembers all of the values you entered last time and the values it calculated last time. So you don’t need to reenter anything. You can simply pick up where you left off with TVM. Keystrokes for additional calculations: Enter 2nd AMORT (this activates the amortization worksheet) Enter P1=37, P2=37 (this tells the calculator to look at the 37th payment) Enter , you’ll see: BAL=95,386.57701 (This is the outstanding loan balance AFTER the 37th payment is made. Because you borrowed the PV of future cash flows, the PV is a cash inflow to you. So it’s positive. ) Enter , you’ll see: PRN= - 121.6245226 (This is the principal portion of the 37th payment. The negative sign indicates cash outflow; this amount is your repayment of the principal in your 37th payment of the loan.)

168

http://actuary88.com Enter , you’ll see: INT= - 464.8910004 (This is the interest portion of the 37th payment. The negative sign indicates cash outflow; this amount is your interest portion of the 37th payment.) Calculate the monthly payment of the refinanced loan. Time t (Month)

0

1

2

……

120

……

357

358

359

360

Y

Y

Y

……

Y

……

Y

Y

Y

Y

240 Months

Y a240

j

Keystrokes for TVM: Last time you used Amortization Worksheet. TVM keeps track of all the values in TVM and Amortization Worksheet. So you don’t need to reenter what you entered. You simply pick up where you left off. Keystrokes for additional calculations: Enter CE/C to leave Amortization Worksheet. Enter N=240. CPT PV. You should get: PV = 83,327.72914 (the outstanding balance of the original loan at t=240). 1

Set I/Y=100 1.0412 1 CPT PMT. You should get: PMT = - 500.1777638 Calculate the reduction of PMT due to refinancing: PMT @6% = 586.5155230 (ignored the negative sign). Assume you store this value in a memory. PMT @4%= 500.1777638 (ignored the negative sign). Assume you store this value in another memory.

169

http://actuary88.com

Reduction = 586.5155230 - 500.1777638 = 86.33775916 Set PMT = 86.33775916. By now, you are probably lost as to what values are currently stored in TVM and what keystrokes to press next. When you are lost, you can always clear the current TVM Worksheet and start a fresh TVM Worksheet. Of course, you need to reenter many values you entered before. Under this method, you start a new TVM Worksheet. Enter N=240, 1 12

I/Y=100 1.04

1 . PMT=86.33775916. Then, Press CPT FV. You should

get FV= - 31,526.11850. This is accumulated value of the reductions of monthly payments at 4% to t=240 (months). As an easier alternatively, you keep using the current TVM, recall each input in the current TVM, and change any input as necessary. For example, to find out the value of N, you simply press “RCL N.” You should get N=240. Next, you recall I/Y. You should get 0.32737398. You 1 12

can check that this is 100 1.04

1 . Or if checking the interest rate is too 1

much pain, you simply reset I/Y= 100 1.0412 1 . So TVM uses the 4% annual interest rate. Then you recall PV and should get PV = 83,327.72914. Finally, you recall FV and should get FV = 0. Summary of your recall: 1

N=240, I/Y=100 1.0412 1 , PV = 83,327.72914, FV=0, PMT =86.33775916 How do you calculate the accumulated value? Simply set PV=0. Press CPT FV. You should get FV= - 31,526.11850. Without setting PV=0, if you press CPT FV, you should get a garbage FV= - 214,097.4342. This garbage FV includes not only the accumulated value of the reduction of payments to t =240, but also the accumulated value of 83,327.72914 to t =240. You can check that: 214, 097.4342 = 83,327.72914 (1.0420 ) + 31,526.11850

170

http://actuary88.com I recommend that you work through this problem multiple times with TVM Worksheet. Try to develop a mental picture of how your keystrokes will change the internal setting TVM. Comprehensive calculator exercise #2 A company is participating in a project. The cash flows of the project are as follows: • The company will invest $10 million per year for the 1st three years of the project. The investment will be made continuously. •

The company will receive a cash flow at the end of each year starting from Year 4.



At the end of Year 4, the company will receive the 1st cash flow of $9 million. This amount will be reduced by $0.5 million for each subsequent year, until the company receives $5 million in a year.



Starting from that year, the cash flow received by the company will be reduced by $1 million each year, until the company receives zero cash flow.

Calculate • the NPV of the project if the discount rate is 12%. • the IRR of the project. Solution Use NPV Worksheet (quick and easy)

Time t

0

1

2

3

cash

4

5

6

7

8

9

10

11

12

13

14

15

16

$9.0 $8.5 $8.0 $7.5 $7.0 $6.5 $6.0 $5.5 $5.0 $4.0 $3.0 $2.0 $1.0

10 ( a )3 12%

The initial investment at t=0 is:

10 ( a )3 12% = 10

1 v3

= 10 i =12%

1 1.12 3 = 25.43219763 ln1.12

171

http://actuary88.com Use NPV Worksheet: CF0= -25.43219763; C01=0, F01=3; C04=9; C05=8.5; C06=8; C07=7.5; C08=7; C09=6.5; C10=6; C11=5.5; C12=5; C13=4; C14=3; C15=2; C16=1. CPT NPV. You should get NPV=47.56780237 @ i=0. Double check: -25.43219763+9+8.5+8+7.5+7+6.5+6+5.5+5+4+3+2+1=47.56780237 (OK) Set I=12 and calculate NPV. You should get NPV=4.58281303 Next, press IRR CPT. You should get: IRR=14.60149476. So the IRR is 14.60149476%. To calculate IRR, we need to solve the following equation: Time t

cash

0

1

2

3

4

5

6

7

8

9

10

11

12

13

14

15

16

$9.0 $8.5 $8.0 $7.5 $7.0 $6.5 $6.0 $5.5 $5.0 $4.0 $3.0 $2.0 $1.0

10 ( a )3 12% = 25.43219763 25.43219763 + 9v 4 + 8.5v5 + 8v 6 + 7.5v 7 + 7v8 + 6.5v9 + 6v10 + 5.5v11 +5v12 + 4v13 + 3v14 + 2v15 + v16 = 0

We can NOT solve this equation manually. We need to use the IRR function of BA II Plus/BA II Plus Professional. We get: IRR = 14.60149476%

172

http://actuary88.com Comprehensive calculator exercise #3 (May 2005 FM, #35) A bank customer takes out a loan of 500 with a 16% nominal interest rate convertible quarterly. The customer makes payments of 20 at the end of each quarter. Calculate the amount of principal in the fourth payment. [A] 0.0 [B] 0.9 [C] 2.7 [D] 5.2 [E] Not enough information Solution We’ll solve this problem with our imaginary cash flow method. To use this method, we need to find n , the # of level payments. Once we find n , we’ll set up an imaginary cash flow of $20 at t = n + 1 . Next, we’ll discount this imaginary cash flow from t = n + 1 to t = 4 to find the principal portion of the payment made at t = 4 : Principal of the 4th payment = 20v ( n +1)

4

So we need to calculate n by solving the following equation:

500 = 20an 4% This equation says that the PV of the all the quarterly payments of $20 @ 4% quarterly interest rate is the $500 (the total principal). The quarterly effective interest rate is 4% because we are given that i ( 4 ) = 16% . We’ll use BA II Plus/BA II Plus Professional TVM to solve 500 = 20an 4% . Set PV= - 500, PMT = 20, I/Y=4. CPT N. We get an error message. Perhaps we’ve entered wrong numbers. Once again, we enter: Set PV= - 500, PMT = 20, I/Y=4. CPT N. We get an error message again. Not knowing where the problem is, let’s throw away the calculator and solve the equation manually.

500 = 20an 4%

an 4% = 25

173

http://actuary88.com 1 1.04 4%

n

= 25, 1 1.04

n

= 1, 1.04

n

= 0, n =

Now we see where the trouble is. The loan is repaid through a perpetual immediate annuity. BA II Plus or BA II Plus Professional TVM can’t handle perpetual annuity (immediate or due). Now we have n =

.

Please note that our imaginary cash flow works even when n =

.

Principal of the 4th payment is:

20v( n +1) 4 = 20v

+1 4

=0

Moral of this problem: SOA can purposely design a problem to make our calculators useless. In studying for FM, we need to learn how to solve a problem with a calculator and how to solve it without a calculator.

174

http://actuary88.com

Chapter 5 annuity

Geometrically increasing

Key points: Understand and memorize the following geometric annuity shortcuts: n payments

(1 + k )

1

1 a 1+ k n

(1 + k )

2

......

(1 + k )

n 1

i k 1+ k

j=

(1) an j = i

k 1+ k

(1 + k )

n 1

s

n j= i

k 1+ k

n (1 + k ) sn j = i

k 1+ k

Interpretation of this diagram: For a geometrically increasing annuity where

n geometrically increasing payments are made at a regular interval; the 1st payment is $1; the next payment is always (1 + k ) times the previous payment. Then (1) The present value at one interval prior to the 1st payment is 1 1 a i k . This value has a factor of because the geometric 1+ k 1 + k n j =1+ k payment pattern one interval prior to the 1st payment is

1 . 1+ k

175

http://actuary88.com (2) The present value immediately after the 1st payment is (1) a

n j=i k 1+ k

. The

present value has a factor of 1 because the 1st payment is 1 . (3) The accumulated value immediately after the final payment is n 1 n 1 (1 + k ) s i k . This value has a factor of (1 + k ) because the final n j=

1+ k

payment is made is (1 + k )

n 1

.

(4) The accumulated value at one interval after the final payment is n n (1 + k ) s i k . This value has a factor of (1 + k ) because the geometric n j=

1+ k

payment pattern one interval after the final payment is (1 + k ) . n

Explanations Present value of geometric annuity due: Amount

$1

$ (1 + k )

Time

0

1

(1) an j = i

k 1+ k

$ (1 + k )

2

2

$ (1 + k )

3

$ (1 + k )



n 1

n 1

3

=a

n j= i k 1+ k

Proof. The present value is: 1 + (1 + k ) v + (1 + k ) v 2 + ... + (1 + k ) 2

n 1

vn 1 .

Let’s set V = (1 + k ) v where V is the new discount factor. 1 1 1 1= 1= 1+ k V (1 + k ) v 1+ i 2 n 1 The present value is 1 + V + V + ... + V = a i

The new interest is j =

1=

1+ i i k 1= 1+ k 1+ k

n j= k 1+ k

176

http://actuary88.com How to memorize this formula: •

Geometric annuity due is still annuity due and its present value at time 0 should have an annuity due factor an j where j is the new interest rate.

• •

The payment at time zero is 1 . We say that the payment factor=1. We then multiply an j with 1 .

Accumulated value of geometric annuity due: Amount

$1

$ (1 + k )

$ (1 + k )

Time

0

1

2

$ (1 + k )

2

3



$ (1 + k )

n 1

n

n 1

3

n (1 + k ) sn j = i

k 1+ k

Proof. The accumulated value is

(1 + i )

n

+ (1 + i )

= (1 + k )

n

n 1

1+ i 1+ k

Let’s set 1 + j =

(1 + k ) + (1 + i ) (1 + k ) n

1+ i + 1+ k

n 1

+ ... + (1 + i )(1 + k )

n 2

2

1+ i + 1+ k

n 2

+ ... +

n 1

1+ i 1+ k

1+ i 1+ i i k 1= where j = is the new interest rate. 1+ k 1+ k 1+ k

Then the accumulated value is:

(1 + k ) (1 + j ) n

n

+ (1 + j )

n 1

+ (1 + j )

n 2

+ ... + (1 + j ) = (1 + k ) s n

n j= i k

1+ k

177

http://actuary88.com How to memorize this formula: •

Geometric annuity due is still annuity due and its accumulated i k value at time n has an annuity factor sn j where j = is the new 1+ k interest rate.



If we extend the geometrically increasing pattern to time n , then the payment at time n will be (1 + k ) . We say that the payment n

factor is (1 + k ) . n



Future value= ( Payment Factor )( Annuity Factor ) = (1 + k ) s n

n j= i k 1+ k

Present value of geometric annuity immediate: Amount

Time

$1

$ (1 + k )

1

2

0

1 a 1+ k n

$ (1 + k )

2

……

$ (1 + k )

n 2

$ (1 + k )

n

n 1

3

n 1

i k 1+ k

j=

Proof. The present value is: v + (1 + k ) v 2 + (1 + k ) v 3 + ... + (1 + k ) 2

n 1

vn =

1 1+ k

{ v (1 + k ) + v (1 + k )

2

+ ... + v (1 + k )

n 1

}

Let’s set V = v (1 + k ) where V is the new discount factor. The new interest rate is j =

1 1 1 1= 1= 1+ k V (1 + k ) v 1+ i

1=

1+ i i k 1= 1+ k 1+ k

The present value of geometric annuity immediate is:

178

http://actuary88.com 1 V + V 2 + ... + V n 1+ k

=

1

1 a 1+ k n

j=

i k 1+ k

How to memorize this formula: •

Geometric annuity immediate is still annuity immediate and its present value at time 0 has an annuity immediate factor an j where i k is the new interest rate. 1+ k

j=



If we extend the geometrically increasing pattern to time zero, then 1 the payment at time zero will be . We say that the payment 1+ k 1 factor is . 1+ k



PV = ( Payment Factor )( Annuity Factor ) =

1 a 1+ k n

j=

i k 1+ k

Accumulated value of geometric annuity immediate: Amount

Time

0

$1

$ (1 + k )

1

2

$ (1 + k )

2

……

$ (1 + k )

n 2

$ (1 + k )

n

n 1

3

n 1

(1 + k )

n 1

s

n j= i

k 1+ k

Proof. The accumulated value is

(1 + i )

n 1

= (1 + k )

+ (1 + i ) n 1

n 2

1+ i 1+ k

(1 + k ) + (1 + i ) (1 + k ) n 3

n 1

1+ i + 1+ k

n 2

1+ i + 1+ k

2

+ ... + (1 + i )(1 + k ) n 3

+ ... +

n 2

+ (1 + k )

n 1

1+ i +1 1+ k

179

http://actuary88.com Let’s set 1 + j =

1+ i 1+ i i k where j = is the new interest rate. 1= 1+ k 1+ k 1+ k

Then the accumulated value is:

(1 + k )

n 1

(1 + j )

n 1

+ (1 + j )

n 2

+ ... + (1 + j ) + 1 = (1 + k )

n 1

s

n j=i

k 1+ k

How to memorize this formula: •

Geometric annuity due is still annuity due and its accumulated i k value at time n has an annuity factor sn j where j = is the new 1+ k interest rate.



The payment at time n is (1 + k ) is (1 + k )



n 1

n 1

. We say that the payment factor

.

FV = ( Payment Factor )( Annuity Factor ) = (1 + k )

n 1

s

n j=i

k 1+ k

Sample Problems and Solutions Problem 1 You are given the following cash flows n payments

1+ k

(1 + k )

2

(1 + k )

3

......

(1 + k )

n

A B C D

180

http://actuary88.com Calculate the value of the annuity at four different points: • one step before the 1st payment (point A) • 1st payment time ( point B) • final payment time (point C), and • one step after the final payment (point D). Solution n payments

(1 + k )

1+ k

2

(1 + k )

3

......

(1 + k )

n

a

n j=i k 1+ k

(1 + k ) an j = i

k 1+ k

(1 + k )

n

s

n j= i

k 1+ k

(1 + k )

n +1

s

n j= i k

1+ k

Value of a geometric annuity = payment factor * annuity factor The payment factor at one step before the 1st payment is $1. The 1st payment is 1 + k . If we extend the geometric payment pattern to one step before the 1st payment time, we get $1. So the value of the annuity one step before the 1st payment time is a i k . n j= 1+ k

The payment factor at the 1st payment is 1 + k . The value of the annuity at the 1st payment time is (1 + k ) a

n j=i k 1+ k

.

The payment factor at the final payment is (1 + k ) . n

The value of the annuity at the 1st payment time is (1 + k ) s n

n j= i

k 1+ k

.

181

http://actuary88.com The payment factor at one step after the final payment is (1 + k )

n +1

. If we

extend the geometric pattern to one step after the final payment time, we n +1 get (1 + k ) . So the value of the annuity at the 1st payment time is

(1 + k )

n +1

s

n j= i k

.

1+ k

Problem 2 You are given the following cash flows n payments

(1 + k )

2

(1 + k )

3

(1 + k )

4

......

(1 + k )

n +1

A B C D Calculate the value of the annuity at four different points: • one step before the 1st payment (point A) • 1st payment time ( point B) • final payment time (point C), and • one step after the final payment (point D).

182

http://actuary88.com Solution n payments

(1 + k )

(1 + k )

2

(1 + k )

3

4

......

(1 + k )

n +1

(1 + k ) an j = i

k 1+ k

2 (1 + k ) an j = i

k 1+ k

(1 + k )

n +1

s

n j= i

k 1+ k

(1 + k )

n+ 2

s

n j= i k

1+ k

Problem 3 An annuity immediate has 15 payments. The 1st payment is $100. Each following payment is 8% larger than the previous payment. The annual effective interest rate is 14%. Calculate the present value of this annuity. Solution 15 payments

$100

$100 (1 + 8% )

$100 (1 + 8% )

2

......

PV=? We are asked to find the present value of this geometric annuity at one interval prior to the 1st payment time.

183

http://actuary88.com From the shortcut, we know

PV = ( Payment Factor )( Annuity Factor ) To find the payment factor, we extend the geometric annuity payment pattern to one interval prior to the 1st payment time. Because the 1st payment is $100 and each payment is 8% larger than the previous payment, the payment that would have been made one interval prior to $100 $100 the 1st payment is: . So ( Payment Factor ) = 1.08 1.08 To find the annuity factor, we simply change the original interest of 14% to the new interest rate: j=

i k 14% 8% = = 5.5556% 1+ k 1 + 8%

( Annuity Factor ) = a 15 5.5556% PV = ( Payment Factor )( Annuity Factor ) =

$100 a 1.08 15

5.5556%

= $925.98

Please note that $925.98 is calculated using BA II Plus or BA II Plus Professional. There is no need for you to manually calculate the annuity using the formula:

a n i=

1 vn i

Please also note that you don’t need to draw the time line or to spell out each of the 15 payments; doing so is time-consuming and error-prone. So don’t draw the following diagram: Amount

Time

$1

0

1

$ (1 + 8% )

2

$ (1 + 8% )

3

2

…… $ (1 + 8% )

14

13

$ (1 + 8% )

14

15

This diagram is good for you to initially prove the geometric annuity formula. Once you have proven the formula, don’t draw this diagram any more. 184

http://actuary88.com

All you need to know that the present value of an annuity immediate is the present value one interval prior to the 1st payment. Alternative method:

15 payments

$100

$100 (1 + 8% )

$100 (1 + 8% )

2

......

PV=? We first calculate the present value of the geometric annuity due. Then we discount this present value one interval prior to the 1st payment.

PV geometric annuity due = ( Payment Factor )( Annuity Factor )

( Payment Factor ) = $100 ( Annuity Factor ) = an j = a 15 5.5556% PV geometric annuity due = $100a 15

5.5556%

Next, we discount this value using the discount factor

1 . 1 + 14%

The present value of the geometric annuity immediate is: 1 $100a 15 5.5556% = $925.98 1 + 14%

Problem 4 An annuity immediate has 15 payments. The 1st payment is $100. Each following payment is 8% larger than the previous payment. The annual effective interest rate is 14%. Immediately after the 1st payment is made, this annuity is sold at a price of X . Calculate X .

185

http://actuary88.com Solution 14 payments

$100

$100 (1 + 8% )

$100 (1 + 8% )

2

......

PV=? Immediately after the 1st payment, there are 14 geometric payments left. We are asked to find the present value of these 14 geometric payments one interval prior to the 1st payment of $100(1+8%). This should be the sales price X . Once again, we use the shortcut:

PV = ( Payment Factor )( Annuity Factor ) Please note that among the remaining 14 geometric payments, the 1st payment is $100(1+8%). As a result, if we extend the geometric payment pattern one interval prior to this 1st payment, we’ll get $100.

( Payment Factor ) = $100 , ( Annuity Factor ) = a 14 5.5556% PV = 100a 14

5.5556%

= 955.62

Problem 5 An annuity immediate has 15 payments. The 1st payment is $100. Each following payment is 8% larger than the previous payment. The annual effective interest rate is 14%. Calculate the accumulated value of this annuity immediately after the 15th payment. Solution 15 payments

$100

$100 (1 + 8% )

$100 (1 + 8% )

2

......$100 (1 + 8% )

14

FV=?

186

http://actuary88.com

FV = ( Payment Factor )( Annuity Factor )

( Payment Factor ) = 100 (1 + 8% )

14

14% 8% = 5.5556% 1 + 8% 14 FV = 100 (1 + 8% ) s 15 5.5556% = 6, 609.64

( Annuity Factor ) = s 15 j ,

j=

Problem 6 In a perpetual annuity immediate, the 1st payment is $100 and each following payment is 8% larger than the previous payment. The annual effective interest rate is 14%. Calculate the present value of this annuity. Solution +

$100

payments

$100 (1 + 8% )

$100 (1 + 8% )

2

......

PV=? PV = ( Payment Factor )( Annuity Factor ) 100 , ( Annuity Factor ) = a + 1.08 14% 8% 6% j= = 1 + 8% 1.08

( Payment Factor ) =

PV =

j

=

1 j

100 1 100 = = 1, 666.67 6% 1.08 6% 1.08

Generally, the present value of a perpetual geometric annuity immediate 1 is . i k

187

http://actuary88.com

Amount

Time

0

1 a 1+ k +

$1

$ (1 + k )

1

2

i k j= 1+ k

=

$ (1 + k )

2

$ (1 + k )

……

n 1

3

n 2

$ (1 + k )

n 1

n

+

1 1 1 = 1+ k i k i k 1+ k

Problem 7 In a perpetual annuity immediate, the 1st payment is $100 and each following payment is 8% larger than the previous payment. The annual effective interest rate is 14%. Calculate the present value of this annuity immediately before the 1st payment is made. Solution +

$100

payments

$100 (1 + 8% )

$100 (1 + 8% )

2

......

PV=? PV = ( Payment Factor )( Annuity Factor )

( Payment Factor ) = 100 ( Annuity Factor ) = a+

i k j= 1+ k

=

1 d

188

http://actuary88.com d =1

1 1 i k =1 = i k 1+ i 1+ j 1+ 1+ k

( Annuity Factor ) = a+ PV = 100

i k j= 1+ k

=

1 1+ i = d i k

1+ i 1 + 14% = 100 = 1,900 i k 14% 8%

Generally, the present value of a perpetual geometric annuity due is 1+ i . i k Amount

$1

$ (1 + k )

Time

0

1

a

+

i k j= 1+ k

=

$ (1 + k )

2

2

$ (1 + k )

3

3



$ (1 + k )

n 1

n 1

n

+

1+ i i k

Problem 8 You plan to pay off your $150,000 mortgage by monthly installments for 15 years. Your 1st payment is one month from now. You plan to increase your monthly payments by 10% each year. You pay a nominal interest rate of 12% compounding quarterly. Calculate your 1st monthly payment. Solution To simplify our calculation, let’s find the equivalent annual payments to replace each year’s 12 monthly payments. Let X =your 1st monthly payment. Let Y =the present value of the annuity immediate of your 1st year payment.

189

http://actuary88.com 12 payments

X

X

X

......

PV = Y = X a12 i , where i is the monthly interest rate. Let’s find the monthly interest rate.

(1 + i )

3

= 1+

Y = X a 12

i

( 4)

4

0.99016%

i = 1+

i

( 4)

1 3

12% 1 = 1+ 4

4

1 3

1 = 0.99016%

= 11.2621X

Because your monthly payments increase by 10% per year, the 12 monthly payments in the 2nd year can be replaced by a single cash flow of 1.1Y at the beginning of the 2nd year. Similarly, the 12 payments in the 2 3rd year can be replaced by a single cash flow of (1.1) Y at the beginning of the 3rd year. As a result, you’ll have a geometric annuity immediate for 15 payments (years). 15 payments

Y

(1.1)

1.1Y

PV = 150, 000 = Y a

15 j =

2

Y

......

r k 1+ k

where r is the annual effective interest rate and k = 10% . The interest rate per 3-month is 12%/4=3%. So r = 1.034 1 = 12.5509% r k 12.5509% 10% = = 2.319% j= 1+ k 1 + 10%

150, 000 = Y a15 2.319% = 12.8388Y , Y = 11, 683.3307 X=

Y 11, 683.3307 = = 1, 037.40 11.2621 11.2621

190

http://actuary88.com Alternative method: We can use a single cash flow Z at the end of Year 1 to replace the 1st year’s 12 monthly payments. Then, the 12 monthly payments in the 2nd year can be replaced by a single cash flow of 1.1Z at the end of Year 2. And the 12 monthly payments in the 3rd year can by replaced by a single 2 cash flow of (1.1) Z at the end of Year 3. And so on. 15

Z

PV =

1 Za 15 1.1

150, 000 =

1.1Z

r k j= 1+ k

1 Za 15 1.1

payments

(1.1)

2

Z ......

1 is the payment factor. 1.1

= 150, 000 , where

r k j= 1+ k

=

1 Z a 15 2.319%, Z = 13,149.6670 1.1

Next, we can calculate the 1st monthly payment X .

12 payments

X

X

X

X

......

FV = Z = X s12 i = X s12 X=

Z s12

0.99016%

=

0.99016%

13,149.6670 = 1, 037.40 s12 0.99016%

191

http://actuary88.com Problem 9 An annuity immediate has 15 payments. The 1st payment is $100. Each following payment is 20% larger than the previous payment. The annual effective interest rate is 14%. Calculate the present value of this annuity. Solution 15 payments

$100

$100 (1 + 20% )

$100 (1 + 20% )

2

......

PV=? To find the payment factor, we extend the geometric annuity payment pattern to one interval prior to the 1st payment time. Because the 1st payment is $100 and each payment is 20% larger than the previous payment, the payment that would have been made one interval prior to $100 $100 the 1st payment is: . So ( Payment Factor ) = 1.2 1.2 To find the annuity factor, we simply change the original interest of 14% to the new interest rate: j=

i k 14% 20% = = 5% (OK if the new interest rate is negative) 1+ k 1 + 20%

( Annuity Factor ) = a 15

5%

PV = ( Payment Factor )( Annuity Factor ) =

1 where a 15

5%

=

1 1 5% 5%

$100 a 1.2 15

5%

= $1, 930.78

15

= 23.1694

192

http://actuary88.com Problem 10 An annuity immediate has 15 payments. The 1st payment is $100. Each following payment is 20% larger than the previous payment. The annual effective interest rate is 14%. Calculate the accumulated value of this annuity immediately after the 15th payment. Solution

15 payments

$100 (1.2 )

$100

$100 (1.2 )

2

......$100 (1.2 )

14

FV=?

FV = ( Payment Factor )( Annuity Factor )

( Payment Factor ) = 100 (1.2 )

14

( Annuity Factor ) = s 15 j , FV = 100 (1.2 ) s 15 14

Where s 15

= 5%

5%

j=

14% 20% = 5% 1 + 20%

= 13, 781.81

0.9515 1 = 10.7342 5%

Problem 11 In a perpetual annuity immediate, the 1st payment is $100. Each following payment is 20% larger than the previous payment. The annual effective interest rate is 14%. Calculate the present value of this annuity.

193

http://actuary88.com Solution +

$100 (1 + 20% )

$100

PV =

1 i k

=

payments

1 14% 20%

$100 (1 + 20% )

2

......

= 20

We got a nonsense number of negative 20. This really means that the present value is not defined. We can easily check that the present value is not defined. Payments increase by 20%, which is faster than the discount rate 14%. As a result, the present value of this perpetual annuity becomes + . i k < 0 , then the present value of the 1+ k perpetual geometric annuity is undefined.

Moral of this problem: If j =

Problem 12 Calculate 1 + 4 + 42 + 43 + 44 + 45 Solution This is a geometric annuity problem with i = 0 and k = 3 . 6 payments

(1 + 3)

1

PV = a6

j

j=

(1 + 3)

2

......

i k 0 3 = = 75% , 1+ k 1+ 3

PV = a6

j = 75%

= 1,365

194

http://actuary88.com We can check that the result is correct: 1 + 4 + 42 + 43 + 44 + 45 =

1 46 = 1,365 1 4

Generally, 1 + q + q 2 + ... + q n = an

j

where j =

1 1 (for q 1 ). q

Alternatively, we can use the following formula: 1 + (1 + r ) + (1 + r ) + (1 + r ) + ... + (1 + r ) 2

3

1 + 4 + 42 + 43 + 44 + 45 = s6 300% =

n 1

= sn

(1 + i ) i

n

r

1

=

46 1 = 1,365 3

If you think it’s a joke to use annuity to calculate the sum of a geometric progression, think again. In the heat of the exam, it can be faster to calculate annuity to calculate the sum of a power series. Problem 13 Starting from the current year, each year John will take out a level percentage of his salary and deposit it into a retirement fund. His goal is to accumulate $250,000 immediately after he retires. Facts: John’s current age Retirement age Current salary Annual growth of salary Interest rate earned by the retirement fund Alternative deposit plan #1

Alternative deposit plan #2

40 65 $50,000 3% 7% annual effective At the end of each year deposit X % of the salary into the retirement fund. At the beginning of each year, deposit Y % of the salary into the retirement fund.

Calculate X Y .

195

http://actuary88.com Solution We’ll use $1,000 as the unit money to simplify our calculation. Age Time t

40

41

42

43



64

65

0

1

2

3



24

25

X % ( 50 )

X % ( 50 )1.03

X % ( 50 )1.032

… X % ( 50 )1.0323 X % ( 50 )1.0324

250 At t=25 • the payment factor = X % ( 50 )1.0324 •

the annuity factor = s25

i

where i =

7% 3% = 3.8835% 1 + 3%

X % ( 50 )1.0324 s25 i = 250 X %=

Age

250 = 6% ( 50 )1.0324 s25 i

40

41

42

43



64

65

0

1

2

3



24

25

Y % ( 50 )

Y % ( 50 )1.03

Y % ( 50 )1.032

Time t

… Y % ( 50 )1.0323 Y % ( 50 )1.0324

250 At t=25 • the payment factor = Y % ( 50 )1.0325



the annuity factor = s25 i where i = Y % ( 50 )1.0325 s25 i = 250

Y %=

7% 3% = 3.8835% 1 + 3%

250 = 5.607% ( 50 )1.0325 s25 i

X Y = 6 5.607 = 0.393

196

http://actuary88.com Problem 12 (SOA May 2001 Date of loan: Amount of loan: Frequency of payments: Date of 1st payment: # of payments:

EA-1 #4) 1/1/2001 $100,000 Quarterly 3/31/2001 120

Amount of each of the 1st 110 repayments:

$3,100

Amount of each of the last 10 repayments: Initial repayment of $ X , then doubling every quarter thereafter Interest rate:

12% per year, compounding quarterly

Calculate the amount of the final payment. Solution Let’s use $1,000 as one unit of money and a quarter as one unit of time. Time

0

Amount

1 $3.1

2 $3.1

3 $3.1

…… ……

110 $3.1

111

112

113

$ 2X

2

$2 X

111

112

113

$X

$ 2X

$ 22 X

$X

……

120

…… $ 29 X

PV = $100

The interest rate per quarter is i =

12% = 3% . 4

Let’s break down the cash flows into two streams: Stream #1 Time

0

Amount

1

2

3

……

110

$3.1

$3.1

$3.1

……

$3.1

2

3

……

110

PV = 3.1a110 i Stream #2 Time

0

1

Amount

PV = X a10 Where j =

……

120

…… $ 29 X

j

i k 3% 1 = = 48.5% (here k = 1 ) 1+ k 1+1

197

http://actuary88.com

The present value of these two streams of cash flows should be $100 at t = 0. 100 = 3.1 a110 3% + 1.03

111

X a10

48.5%

a110 3% = 32.04275602 a10

48.5%

= 808.0213752

X=

100 3.1 a110 3% 1.03

111

a10

=

48.5%

100 3.1 ( 32.04275602 ) = 0.02197516 1.03 111 ( 808.0213752 )

Alternatively, we can calculate the present value of the 2nd stream of the cash flows at t = 110 : Stream #2 Time

0

1

2

3

……

110

Amount

PV =

100 = 3.1 a110 3% + 1.03

a10 j = a10 X=

48.5%

X a 2 10

112

113

$X

$ 2X

$ 22 X

X a 2 10

……

120

…… $ 29 X

j

j

= 1,568.973544

100 3.1 a110 3% 1.03

110

111

110

×

a10 2

j

=

100 3.1 ( 32.04275602 ) = 0.02197516 1,568.973544 110 × 1.03 2

The final repayment (i.e. the 120th repayment) is:

29 X = 29 ( 0.02197516 ) = 11.25127973 Since one unit of money represents $1,000, the final repayment is $11,151.28. So the answer is B.

198

http://actuary88.com Problem 13 (SOA May 2005 EA-1 #17) Smith retires on 1/1/2005 and receives his retirement benefit as monthly annuity payable at the end of each month for a period certain of 20 years. The benefit for the 1st year is $2,000 per month. This monthly benefit is increased at the beginning of each year to be 5% larger than the monthly payment in the prior year. X is the present value on 1/1/2005 of the retirement benefit at a nominal interest rate of 6%, convertible monthly.

In what range is X ? (A) Less than $405,000 (B) $405,000 but less than $410,000 (C) $410,000 but less than $415,000 (D) $415,000 but less than $420,000 (E) $420,000 or more Solution We’ll do two things to simplify our calculation. First, we’ll use $1,000 as one unit of money. Second, we’ll convert the 12 monthly payments at Year 1 into one equivalent cash flow. Convert the 12 monthly payments in Year 1 into one cash flow at t = 0 : Time t (months)

0

Payment

1

2

3

……

11

12

$2

$2

$2

$2

$2

$2

P = 2a12 i = 2 (11.61893207 ) = 23.2379 where i =

6% = 0.5% 12

We’ll draw a cash flow diagram for 20 years’ payments. The next year’s payment is 5% larger than the previous year’s payment. Time t (years)

0

Payment

P

1

1.05P

2

3 2

1.05 P

3

1.05 P

……

19

……

1.0519 P

The present value of 20 years’ payments is: P a20 j , where j =

r k r 5% = 1 + k 1 + 5%

199

http://actuary88.com

Because the unit time is one year, r is the annual effective return. 12

 6%  r = 1 +  − 1 = 6.16778% 12   ⇒

j=

r − k 6.16778% − 5% = = 1.11217% 1+ k 1 + 5%

So the present value is:

= P a20 j 23.2379 = a20 1.11217% 23.2379 = (18.0418) 419.2538 Because one unit of money is really $1,000, so the present value is $419,253.8. The answer is D. Problem 14 (SOA May 2002 EA-1 #2) Annual payments into a fund: $10,000 at the end of year one, increasing by $500 per year in the 2nd through the 10th years. After the 10th year, each payment increases by 3.5% over the prior payment. Interest rate:

7% compounded annually.

Calculate the accumulated value of the fund at the end of year 20. Solution We’ll use $100 as one unit of money to keep our calculation simple. First, let’s use a table to keep track of cash flows:

200

http://actuary88.com

Time t

payment

0 1 2 3 4 5 6 7 8 9 10 11 12 13 14 15 16 17 18 19 20

$100 105 110 115 120 125 130 135 140 145 145*1.035 145*1.0352 145*1.0353 145*1.0354 145*1.0355 145*1.0356 145*1.0357 145*1.0358 145*1.0359 145*1.03510

We are asked to calculate the fund’s value at t = 20 . There are many ways to find the answer. One way is to break the cash flows into two streams:

201

http://actuary88.com Stream #1 Time t 0 1 2 3 4 5 6 7 8 9 10

payment $100 105 110 115 120 125 130 135 140 145

Stream #2 11 145*1.035 12 13 14 15 16 17 18 19 20

145*1.0352 145*1.0353 145*1.0354 145*1.0355 145*1.0356 145*1.0357 145*1.0358 145*1.0359 145*1.03510

A fast way to find the accumulated value of stream #1 is to use BA II Plus Cash Flow Worksheet. Enter the following into Cash Flow Worksheet:

202

http://actuary88.com Time t 0 1 2 3 4 5 6 7 8 9 10

payment $0 100 105 110 115 120 125 130 135 140 145

CF0 C01 C02 C03 C04 C05 C06 C07 C08 C09 C10

Set I=7 (so the interest rate is 7%). You should get NPV=840.9359126. This is the PV of Stream #1 at t = 0 . The accumulated value of Stream #1 at t = 20 is: 840.9359126 × 1.07 20 = 3, 254.156635 The accumulated value of Stream #2 at t = 20 is:

145 (1.03510 ) s10 j , where j =

7% 3.5% = 3.38164251% 1 + 3.5%

Using BA II Plus TVM, you should get:

145 (1.03510 ) s10 j = 2,386.418027 So at t = 20 the accumulated value of the entire fund is: 3, 254.156635 + 2,386.418027 = 5, 640.574662

Because one unit is equal to $100, the accumulated value is $564,000.57.

203

http://actuary88.com

Chapter 6 rate

Real vs. nominal interest

Key points: 1. Nominal interest rate • the growth rate of your money • interest rate quoted in the market 2. Real interest rate • the growth rate of your purchasing power 3. Formulas 1 + real interest rate =

1 + nominal interest rate 1 + inflation rate

real interest rate=

nominal interest rate -- inflation rate 1+ inflation rate

real interest rate

nominal interest rate -- inflation rate

Real Cash Flow =

Nominal Cash Flow 1 + inflation rate

4. Constant dollar vs. real dollar •

Current dollars – money received. If you get $100 on your part time job last week, you get $100 current dollars. Your $100 current dollars have not been adjusted for inflation.



Constant or real dollars -- dollars reported in terms of the value they had on a previous date. The $100 you got last week could buy 90 Big Macs from MacDonald’s. However, several years ago, $50 could buy 90 Big Macs. So your current $100 is worth only $50 in constant or real dollars. Constant or real dollars are current dollars adjusted for inflation.

204

http://actuary88.com 5. Discounting rule •

Discount nominal dollar cash flows by the nominal interest rates



Discount real dollar cash flows by the real interest rates

Sample Problems Problem 1 Starting from the current year, each year John will take out money from his salary and deposit it into a retirement fund. The retirement fund earns the market interest rate. His goal is to accumulate $100,000 constant dollars immediately after he retires. Facts: John’s current age John’s Retirement age Market interest rate Inflation Alternative deposit Plan #1 Alternative deposit Plan #2

40 65 7% annual effective 3% per year Deposit X constant dollars at the end of each year into the retirement fund. Deposit Y nominal dollars at the end of Year 1, each subsequent annual deposit is 5% larger than the previous one.

[1] Use the nominal dollar method, calculate the annual deposit at the end of Year 1 for Plan #1 and Plan #2 [2] Use the constant dollar method, calculate the annual deposit at the end of Year 1 for Plan #1 and Plan #2 Solution [1] Nominal dollar method Plan #1 • Use nominal dollar cash flows • Discount all nominal cash flows at the nominal interest rate The nominal interest rate is 7%. The inflation is 3%.

205

http://actuary88.com John wants to accumulate, from t=0 to t=25, $100,000 in constant dollars. $100,000 constant dollars are equivalent to the following 25 25 nominal dollars: $100, 000 (1 + inflation ) = 100, 000 (1.03 ) = 209,377.79

Age Time t

40

41

42

43



65

0

1

2

3



25

Constant dollars Nominal dollars



X

X

X

1.03X

1.032X

1.033X



X

1.0325X

100, 000 (1.03) = 209,377.79 25

Nominal dollars

We have a geometric annuity. We’ll apply the 3 minute script. X 1.0325 s25

= 100, 000 (1.03) , where j = 25

j

payment annuity factor at t =25 factor

Xs25 j = 100, 000,

7% 3% = 3.8835% 1 + 3%

X = 2, 439.12 (constant dollars)

The actual annual deposit at the end of Year 1 is: 1.03 X = 2,512.29 (nominal dollars)

Constant dollar method Plan #1 • •

Use constant dollar cash flows Discount constant dollar cash flows at the real interest rate

The real interest rate is j =

7% 3% = 3.8835% . 1 + 3%

Age Time t

40

41

42

43



65

0

1

2

3



25

X

X

X



X

Constant dollars

Constant dollars

100, 000

206

http://actuary88.com This time, we have a simple annuity. Xs25 j = 100, 000,

X = 2, 439.12 (constant dollars)

This matches the result in the nominal dollar method. In this problem, the constant dollar method is simpler. This is because the problem gives us two figures, the level annual deposit and the accumulate value of the fund, in constant dollar to begin with. [2] Nominal dollar method Plan #2 The fund’s accumulated value t=25: $100, 000 (1 + inflation ) = 100, 000 (1.03 ) = 209,377.79 (nominal dollars) 25

Age Time t

25

40

41

42

43



65

0

1

2

3



25

1.052 Y



Nominal dollars

Y

1.05Y

1.0524 Y

100, 000 (1.03) = 209,377.79 25

Nominal dollars

Once again, we use the 3 minute script for the geometric annuity:

Y 1.0524 s25 k = 100, 000 (1.03) , where k = 25

payment annuity factor at t =25 factor

7% 5% = 3.8835% 1 + 5%

Y = 2, 051.64 (nominal dollars)

So the actual annual deposit at the end of Year 1 is $2,051.64

207

http://actuary88.com Constant dollar method Plan #2 Age Time t

40

41

42

43



65

0

1

2

3



25

Nominal dollars

Constant dollars

Y 1.03-1 Y

… -2 -3 2 (1.03 )1.05Y (1.03 ) 1.05 Y … 1.05Y

1.052 Y

1.0524 Y (1.03-25) 1.0524 Y

100, 000

Constant dollars

Cash flows increase by the following rate: r=

1.05 1 = 1.9417% 1.03

We need to accumulate the cash flows at the real interest rate: j=

7% 3% = 3.8835% 1 + 3%

Once again, we have a geometric annuity.

Y (1.03 25 )1.0524 s25 payment factor at t =25

R

= 100, 000 , where R =

j r 3.8835% 1.9417% = = 1.9048% 1+ r 1 + 1.9417%

annuity factor

Make sure you are not lost in the above equation. Y = 2, 051.63 (nominal dollars)

Problem 2 You invested in a foreign country and earned an annual effective return of 2% for 5 years. The country experienced deflation (negative inflation) of 3% per year effective during the period. Calculate the real rate of return per year over the 5 year period. Solution

208

http://actuary88.com The equation is: 1 + real interest rate =

1 + nominal interest rate 1 + inflation rate

We are given: The nominal interest rate = 2% The inflation rate = - 3% So the real interest rate is:

real interest rate =

1 + nominal interest rate 1 + 2% 1= 1 = 5.15% 1 + inflation rate 1 + ( 3% )

209

http://actuary88.com

Chapter 7 Loan repayment and amortization •

The borrower borrows a loan at t = 0 and pays back the loan through n installments



Loan principal borrowed = PV of future payments



At any time t where 0 t n and t is an integer before the loan is fully paid off, the outstanding balance of the loan can be calculated using one of the two methods 1. Prospective method. The outstanding balance is equal to the PV of remaining payments to be paid in the future. 2. Retrospective method. The outstanding balance is equal to the original loan principal accumulated to t less than the previous payments accumulated to t .



(A special case you need to remember) A loan is paid off by level payments X payable at the end of each period for n periods. For any time t where 0 t n and t is an integer, then 1. PV of the loan at time zero = X an 2. The outstanding balance immediately after the t -th payment is X an t 3. The principal portion of the 1st, 2nd, 3rd, …, t -th, .. n-th payment is Xv n , Xv n 1 , Xv n 2 ,…, Xv n t +1 ,…, Xv . (As time passes, more and more goes to the principal). Notice that the principal repayments nicely form a geometric progression. (However, the principal repayments nicely form a geometric progression only when the repayments are level.) 4. The interest portion of the 1st, 2nd, 3rd, …, k -th, .. n-th payment is X (1 v n ) , X 1 v n 1 , X 1 v n 2 , …, X 1 v n

(

)

(

)

(

t +1

),

…, X (1 v ) . (As time passes, less and less goes to the principal)

210

http://actuary88.com Example 1 A loan of $100,000 is repaid through monthly level payments over the next 20 years, the 1st payment due one month from today. The monthly payments are calculated using a nominal interest rate of 12% compounding monthly. Calculate • The monthly payments • The principal outstanding immediately after 15th payment • The principal and interest split of the 16th payment Solution Use a month as the compounding period Time t (Month)

0

1

2

……

237

238

239

240

Y

Y

……

Y

Y

Y

Y

237

238

239

240

Y

Y

Y

Y

240 month payments

Y a240 = 100, 000 The monthly interest rate is: i ( ) 12% i= = = 1% 12 12 12

Y a240 1% = 100, 000

Y=

Time t (Month)

1

2

Y

Y

0

100, 000 = 1,101.086 a240 1% …

15

16

Y

……

……

225 month payments

B15P = Ya240

15

= Ya225

211

http://actuary88.com Next, we’ll find the outstanding balance immediately after the 15th payment. Prospective method: B15P = Ya240

15

= Ya225 = 1,101.086a225 1% = 98,372.815

Retrospective method: Time t (Month)

0

1

2

Y

Y



15

Y

16

……

……

237

238

239

240

Y

Y

Y

Y

15 month payments

Ys15

The value of the principal accumulated to t = 15 : 100, 000 (1.0115 ) = 116, 096.896 Previous payments from t = 1 to t = 15 accumulated to t = 15 : Ys15 = 1,101.086 s15 1% = 17, 724.066 So the outstanding balance immediately after the 15th payment is: 116, 096.896 17, 724.066 = 98,372.83 The slight difference between the results generated by the prospective method and the retrospective method is due to rounding. The interest portion of the 16th payment:

98,372.8 (1% ) = 983.73

The principal portion of the 16th payment: 1,101.09 983.73 = 117.36 Alternatively, we can use the memorized rule on how to split a payment into principal and interest. The principal portion: Yv n The interest portion:

t +1

= 1,101.09 (1.01 1 )

Y (1 v n

t +1

240 16 +1

= 117.36

) = 1,101.09 1 (1.01 )

1 240 16 +1

= 983.73

212

http://actuary88.com Example 2 (SOA 2005 May EA-1 #3) Terms of a $1,000 loan issued by Smith: Length of loan: 20 years Payments: Level annual payments at the end of each year Interest: 5% nominal, convertible semi-annually When Smith receives each payment, it is immediately reinvested at 6%, compounded annually. R is the effective annual rate of interest earned by Smith on his combined investments over the 20 year period.

In what range is R ? (A) Less than 5.57% (B) 5.57%, but less than 5.62% (C) 5.62%, but less than 5.67% (D) 5.67%, but less than 5.72% (E) 5.72% or more Solution At t = 0 Smith invests (i.e. giving out) $1,000 cash. In return, he gets 20 annual payments. To find Smith’s annual rate of return, we need to find Smith’s wealth at t = 20 . Then, we can calculate Smith’s return by solving the following equation: 1, 000 (1 + R ) = Smith's wealth @ t = 20 20

Let X represent the annual payment. 1, 000 = Xa20 i , where i = 1 +

5% 2

2

1 = 5.0625%

Using BA II TVM or the annuity immediate formula, we get X = 80.668 . Smith’s wealth @ t = 20 is Xs20 6% = 2,967.4338 , using TVM or a formula. 1, 000 (1 + R ) = 2,967.4338 , R = 5.59% 20

So the answer is B.

213

http://actuary88.com Example 3 (SOA 2002 May EA-1 #1) Loan repayment period: 5 years Beginning loan amount: $75,000 Repayment Plan #1: Level annual payments at the beginning of each year Repayment Plan #2: Level semi-annual payments at the end of each 6 month period A = Annual payment under Repayment Plan #1 B = Total payments in a year under Repayment Plan #2

1, 000d ( 4) = 76.225

In what range is the absolute value of A B ? (A) Less than $1,000 (B) $1,000 but less than $1,025 (C) $1,025 but less than $1,050 (D) $1,050 but less than $1,075 (E) $1,075 or more Solution Let X =the level annual payment under Repayment Plan #1 Y =the level semi-annual payment under Repayment Plan #2 Repayment Plan #1 time t (year) payment

0 X

1 X

2 X

3 X

4 X

75,000 = X a5 i where i is the annual effective rate Repayment Plan #2 time t (6 month) 0 payment

1 Y

75, 000 = Y a10

2 Y

3 Y

4 Y

5 Y

6 Y

7 Y

8 Y

9 Y

10 Y

j

214

http://actuary88.com

Where j is the semi-annual effective rate Next, we need to calculate i and j . 1 d ( 4) v= = 1 1+ i 4

(1 + j )

2

4

0.076225 = 1 4

= 1 + i = 1.08 ,

4

,

i = 8%

j = 3.923%

Then using either BA II Plus TVM or annuity formulas, we can solve for X and Y : 75,000 = X a5 i ,

X = 17,392.81

75, 000 = Y a10 j ,

Y = 9, 211.43

Then A = X , B = 2Y A B = X

2Y = 17,392.81 2 ( 9, 211.43) = 1, 030.06

The answer is C. Example 4 (SOA May 2002 EA-1 #5) Two $10,000 loans have the following repayment characteristics: Loan 1: Level quarterly payments at the end of each quarter for five years. Loan 2: Monthly interest payments on the original loan amount at the end of each month for 48 months plus a balloon repayment of principal at the end of the fourth year. The balloon repayment will be made using the accumulated value of a sinking fund created by level annual deposits at the beginning of each of the four years. Effective annual interest rate on the loan: 8%. Effective annual interest rate on the sinking fund: 9%. A = Sum of repayments under Loan 1. B = Sum of interest payments on Loan 2 plus sum of sinking fund payments. In what range is the absolute value of A B ?

215

http://actuary88.com [A] [B] [C] [D] [E]

Less than $875 $875 but less than $950 $950 but less than $1,025 $1,025 but less than $1,100 $1,100 or more

Solution Loan #1 time t (quarter) payment

0

1 X

10, 000 = X a20 i ,

2 X i = 1.08

3 X 1 4

…… X

19 X

20 X

1 = 1.94265%

X = 608.19 (using BA II Plus TVM or annuity formula)

Loan #2 – interest payment time t (month) 0 1 Interest payment Y

2 Y

…… Y

47 Y

48 Y

Y = 10, 000 (1.081 12 1) = 64.34

Loan #2 – installment in the sinking fund time t (year) installment

0 Z

1 Z

2 Z

3 Z

4

Z s4 9% = 10, 000 Z = 2, 006.13 A = 20 X , B = 48 Y + 4 Z

A B = 20 X

B = 20 ( 608.19 )

48 ( 64.34 ) + 4 ( 2, 006.13) = 1,050.96

So the answer is D.

216

http://actuary88.com Example 5 (SOA May 2002 EA-1 #6) Smith obtains a loan for $10,000 with 40 annual payments at an effective annual interest rate of 7%. The first payment is due one year from now. A = Sum of interest paid in the even-numbered payments. B = Sum of principal paid in the odd-numbered payments. In what range is A + B ? [A] Less than $13,800 [B] $13,800 but less than $14,200 [C] $14,200 but less than $14,600 [D] $14,600 but less than $15,000 [E] $15,000 or more Solution D We’ll use the imaginary cash flow method introduced in Chapter 1 “How to build a 3 minute solution script.” imaginary cash flow

time t (year) Total Payment Principal portion Interest portion A (even #) B (odd #)

0

1 X Xv 40

2 X Xv39

3 X Xv38

4 X Xv37

X (1 v 40 ) X (1 v39 ) X (1 v38 ) X (1 v37 ) X (1 v39 ) Xv 40

40 X Xv

41 X

X (1 v 2 ) X (1 v )

X (1 v37 ) Xv38

10, 000 = X a40 7% ,

… 39 X Xv 2

X (1 v ) Xv 2

X = 750.09

A = X (1 v39 ) + X (1 v37 ) + ... + X (1 v )

= 20 X

X ( v 39 + v 37 + v 35 + ... + v ) = X 20

= 750.09 20

1.07 1 1.07 1 1.07 2

B = X ( v 40 + v38 + ... + v 2 ) = X

41

v v 41 1 v2

= 9,832.75

1.07 2 1.07 v 2 v 42 750.09 = 1 v2 1 1.07 2

42

= 4,830.92

A + B = 9,832.75 + 4,830.92 = 14, 663.37

217

http://actuary88.com Example 6 (SOA May 2002 EA-1 #7) Smith purchases a house for $120,000 and agrees to put 20% down. He takes out a 30-year mortgage, with monthly payments, with the first payment one month after the date of the mortgage. The interest rate is 8% compounded monthly. Immediately following the 180th payment, Smith refinances the outstanding balance with a new 10-year mortgage, also with monthly payments, with the first payment one month after the date of the new mortgage. The new interest rate is 7.5% compounded monthly. A = Amount of interest paid in the 100th payment of the first mortgage. B = Amount of principal paid in the 100th payment of the refinanced mortgage. In what range is [A + B]? [A] Less than $1,300 [B] $1,300 but less than $1,325 [C] $1,325 but less than $1,350 [D] $1,350 but less than $1,375 [E] $1,375 or more Solution time t (month) payment

0

1 X

2 …… 180 181 X X X X 8% 120, 000 ( 80% ) = X a360 i , where i = = 0.66667% 12 X = 704.41 , A = 704.41(1 v360 100 +1 ) = 580 @0.66667%

The outstanding balance immediately before the refinancing time t (month) 0 1 2 …… 180 181 payment X X X X X

…… X

360 X

…… X

360 X

P = X a180 i = 73.710.30 The monthly payment in the refinanced mortgage time t (month) 0 1 2 …… 180 payment

181 Y

…… Y

360 Y

Y a180 j = 73.710.30

218

http://actuary88.com Where j =

7.5% = 0.625% 12

The new monthly payment is B = 875v120

100 +1

= 768

73, 710.3 = 875 a120 0.625%

@0.625%

A + B = 1,348

Example 7 (SOA May 2001 EA-1 #3) Amount of the loan # of originally scheduled level annual repayments Time of 1st repayment Additional payments made with the 5th and 10th scheduled repayments Effective annual interest rate

$100,000 30 1 year from the date of the loan $5,000 each 6%

Subsequent to the two additional payments, the loan continues to be repaid by annual repayments of the original size, plus a small final repayment one year after the last full repayment. In what range is the total amount of interest saved due to the two additional payments? (A) Less than $23,500 (B) $23,500 but less than $23,600 (C) $23,600 but less than $23,700 (D) $27,000 but less than $27,800 (E) $23,800 or more Solution This problem is difficult. To solve it, you’ll need to find out the originally scheduled payments and the actual payments. To simplify our calculation, we’ll use $1,000 as one unit of money. Let X represent the originally scheduled level annual repayment.

219

http://actuary88.com Scheduled repayments: Time t (years) Repayments

0

1

2



5



10

..

..

..

30

X

X

X

X

X

X

X

X

X

X

m X

m +1 Y

100 = X a30 6% ,

X = 7.26489115

Actual repayments: Time t (years) Repayments

0

1

2



5



10

..

X

X

X

X +5

X

X +5

X

last full repayment small final repayment

To find m and Y , we can calculate the outstanding loan balance at t = 10 immediately after borrower pays X + 5 . First, though, we’ll calculate the outstanding loan balance at t = 10 immediately after the borrower pays X assuming no extra repayments at t = 5 or t = 10 : Time t (years) Repayments (scheduled) Extra

0

1

2



5



10

..

..

..

30

X

X

X

X

X

X

X

X

X

X

5

5

Pscheduled = X a20 6% = 83.32772914 To find the actual loan balance at t = 10 , we’ll need to account for the two extra repayments of 5 each at t = 5 or t = 10 :

P actual = Pscheduled

( 5 ×1.06

5

+ 5)

= 71.63660125

accumulating two extra payments to t =10

Next, we are ready to calculate the # of actual full repayments after t = 10 :

P actual = X an 6% ,

71.63660125 = 7.26489115 an 6%

n = 15.37019964 (using BA II Plus TVM) 220

http://actuary88.com

So there are 15 full repayments after t = 10 . Actual repayments: Time t (years) Repayments

0

1

2

X

X



5

X

X +5



10

11

… 25

26

X

X +5

X

X

Y

last full repayment small final repayment

P actual = 71.63660125 At t = 10 , the PV of the 15 full repayments from t = 11 to t = 25 and the PV of Y must add up to P actual :

Y (1.06

(

16

) + Xa

Y = P actual

15 6%

= P actual

)

(

)

Xa15 6% 1.0616 = 71.63660125 7.26489115a15 6% 1.0616 = 2.73892982

The total scheduled repayments: 30 X = 30 ( 7.26489115) = $217.94673447 Total interest paid if scheduled repayments are made: 30 X 100 = 217.94673447-100=117.94673447 The total actual repayments: 25 X + 5 + 5 + Y = 25 ( 7.26489115 ) + 5 + 5 + 2.73892982 = 194.361208544 Total actual interest paid: 194.361208544 100 = 94.361208544 Interest saved: 117.94673447 - 94.361208544 = 23.585525926=$23,585.525926 So the answer is B. The following is the scheduled vs. actual payments (rounded to 2 decimals). In the exam, you don’t really need to write out the above table. Here I wrote the table just to make things clear.

221

http://actuary88.com

time t 0 1 2 3 4 5 6 7 8 9 10 11 12 13 14 15 16 17 18 19 20 21 22 23 24 25 26 27 28 29 30 Total

scheduled repayments

actual repayments

$7.26 $7.26 $7.26 $7.26 $7.26 $7.26 $7.26 $7.26 $7.26 $7.26 $7.26 $7.26 $7.26 $7.26 $7.26 $7.26 $7.26 $7.26 $7.26 $7.26 $7.26 $7.26 $7.26 $7.26 $7.26 $7.26 $7.26 $7.26 $7.26 $7.26 $217.95

$7.26 $7.26 $7.26 $7.26 $12.26 $7.26 $7.26 $7.26 $7.26 $12.26 $7.26 $7.26 $7.26 $7.26 $7.26 $7.26 $7.26 $7.26 $7.26 $7.26 $7.26 $7.26 $7.26 $7.26 $7.26 $2.74

$194.36

The interest saved: $217.95 - $194.36 = 23.59=$23,590.

222

http://actuary88.com Example 8 (SOA May EA-1 2001 #6) Amount of the loan: $25,000 Term of loan: 8 years Loan repayments: Quarterly, at the end of each quarter Interest rate: 8% per year, compounded semiannually The 11th and 12th scheduled payments are not made. The loan is renegotiated immediately after the due date of the 12th (2nd missed) scheduled repayment with the following provisions: 13th (1st renegotiated) scheduled repayment: $ X 14th through 32nd repayments: Each even-numbered repayment is $200 greater than the Immediately proceeding (odd-numbered) repayment. Each odd-numbered repayment is equal to the immediately Preceding even-numbered repayment. The loan is to be completely repaid over the original term. In what range is X ? (A) Less than $250 (B) $250 but less than $255 (C) $255 but less than $260 (D) $260 but less than $265 (E) $265 or more Solution C The difficulty of this problem is to neatly keep track of the complex repayments in the renegotiated loan. One simple approach is to exhaustively list all of the repayments: Time (quarters) 0 1 2 3 4 5 6 7 8

loan repayments in the original plan (borrow 250)

loan repayments in the revised plan

P P P P P P P P

P P P P P P P P

223

http://actuary88.com 9 10 11 12 13 14 15 16 17 18 19 20 21 22 23 24 25 26 27 28 29 30 31 32

P P P P P P P P P P P P P P P P P P P P P P P P

P P 0 (1st missed repayment) 0 (2nd missed repayment)

X X +2 X +2 X +4 X +4 X +6 X +6 X +8 X +8 X + 10 X + 10 X + 12 X + 12 X + 14 X + 14 X + 16 X + 16 X + 18 X + 18 X + 20

In the above table, P is the level quarterly repayment originally scheduled. Please note that I used $100 as one unit of money. This way, the incremental repayment of $200 in the revised loan becomes 2; the loan amount changes from 25,000 to 250. This makes it easier for us to keep track of the loan repayments in the revised repayment plan. 8% 250 = P a32 i , i = 1 + 2

0.5

1 = 1.98%

P = 10.6223

The outstanding loan balance at t = 10 immediately after the 10th repayment is made is: P a22 i = 10.6223 a22 1.98% = 187.9560 .This amount accumulates to 187.9560 (1 + i ) = 199.3454 at t = 13 . This outstanding 3

principal must be paid in the renegotiated loan. The repayments in the renegotiated loan consist of the following two cash flow streams:

224

http://actuary88.com Time (quarters) 13 14 15 16 17 18 19 20 21 22 23 24 25 26 27 28 29 30 31 32

loan repayment stream #1 in the renegotiated loan

loan repayment stream #2 in the renegotiated loan

X X X X X X X X X X X X X X X X X X X X

2 2 4 4 6 6 8 8 10 10 12 12 14 14 16 16 18 18 20

The PV of the 1st stream @ t = 13 : X a20 1.98% = 16.7069 X To calculate the PV of the 2nd stream @ t = 13 , we simply enter the following cash flows into BA II Plus Cash Flow Worksheet: Time (quarters) 13 14 15 16 17 18 19 20 21 22 23 24 25 26 27 28 29

2 2 4 4 6 6 8 8 10 10 12 12 14 14 16 16

CF0=0 C01=2, F01=2 C02=4, F02=2 C03=6, F03=2 C04=8, F04=2 C05=10, F05=2 C06=12, F06=2 C07=14, F07=2 C08=16, F08=2

225

http://actuary88.com 18 18 20

30 31 32

C09=18, F09=2 C10=20, F10=1

Set I=1.98 (the interest rate). The NPV=156.1209. The PV of the two streams should be the outstanding loan balance at t = 13 : 199.3454 = 156.1209+16.7069X , X = 2.5872 = $258.72 Example 9 (SOA May 2001 EA-1 #5) On 1/1/2002, Smith contributes $2,000 into a new saving account that earns 5% interest, compounded annually. On each January 1 thereafter, he makes another deposit that is 97% of the prior deposit. This continues until he has 20 deposits in all. On each January 1 beginning on 1/1/2025, Smith makes annual withdrawals. There is to be a total of 25 withdrawals, with each withdrawal 4% more than the prior withdrawal, and the 25th withdrawal exactly depletes the account. In what range is the sum of the withdrawals made on 1/1/2025 and 1/1/2026? (A) Less than $5,410 (B) $5,410 but less than $5,560 (C) $5,560 but less than $5,710 (D) $5,710 but less than $5,860 (E) $5,860 or more Solution First, let’s organize the info into a table. We’ll use $1,000 as one unit of money. In the following table, the red numbers are deposits; the blue numbers are withdrawals. Let X represent the 1st withdrawal.

Time t (years) Deposits (withdrawals)

1/1/2002 1/1/2003 0 1

2

2 ( 0.97 )

PV1 = 2a20 i

… .. …

1/1/2021 19

… …

2 ( 0.97 )



1/1/2025 1/1/2026 23 24

… …

1/1/2049 47



1.04 24 X

19

X

1.04 X

PV2 = X a25

j

Using the geometric annuity shortcut, we know that at t = 0 , the PV of the deposits is:

226

http://actuary88.com

PV1 = 2a20 i , where i =

5%

(

3% )

= 8.24742268% 1 + ( 3% ) Please note that the 97% ratio is equivalent to -3% increase rate.

PV1 = 2a20 8.24742268% = 20.87005282 At t = 23 , the PV of the total withdrawals is: 5% 4% PV2 = X a25 j , where j = = 0.96153824% 1 + 4% PV2 = X a25 j = 22.34096995 X Because the 25th withdrawal depletes the saving account, we have: PV1 (1.05 2 3 ) accumulate deposits to t = 23

=

PV2 PV of withdrawals at t = 23

20.87005282 (1.05 2 3 ) = 22.34096995X ,

X = 2.8692963269

So the sum of the withdrawals made on 1/1/2025 (1st withdrawal) and 1/1/2026 (2nd withdrawal) is:

X + 1.04 X = 2.04 X = 2.04 ( 2.8692963269 ) = 5.853364507 = $5,853.364507 The sum is $5,853. The answer is D.

227

http://actuary88.com Example 10 (SOA May 2001 EA-1 #8) Date of the loan Date of 1st repayment Frequency of repayments Term of loan Amount of each repayment v=

1/1/2001 12/31/2001 Annual 4 years $1,000

1 1+ i

The sum of the principal repayments in years one and two is equal to 10v 2 times the sum of the interest repayments in years three and four. Calculate v . Solution We’ll use the imaginary cash flow method. In addition, we use $1,000 as one unit of money. Date 1/1/2001 12/31/2002 12/31/2003 12/31/2004 12/31/2005 12/31/2006 time t (year) 0 1 2 3 4 5 repayment $1 $1 $1 $1 $1 5 1 4 5 2 3 5 3 2 5 4 v =v v =v v =v v =v principal portion 4 4 4 1 v 1 v 1 v 1 v4 interest portion

Imaginary cash flow We are told:

v 4 + v 3 = 10v 2 (1 v 2 + 1 v )

20 = 0, 11 v = 0.938 (choose the positive root because v > 0 ) v2 + v

228

http://actuary88.com Example 11 (SOA May 2001 EA-1 #9) Amount of a loan Date of loan Term of loan Date of 1st repayment Frequency of repayments Interest rate

$1,000 1/1/2001 30 years 1/1/2004 Every 3 years 4% per year, compounded annually

Calculate the principal repaid in the 5th repayment. Solution We’ll use $1,000 as one unit of money. Date time t repayment

1/1/2001 1/1/2004 1/1/2007 0 1 2

X

X

… …

… 5

… ..

… 10

X

X X v6

X

X

principal repayment

The interest rate per 3 years is i = 1.043 1 = 12.4864% 1 = X a10 i ,

X = 0.18052244

X v 6 = 0.18052244 (1 + 12.4864% ) = 0.08911095 = $89.11 6

So the principal repaid in the 5th repayment is $89.11.

229

http://actuary88.com Example 12 (SOA May 2001 EA-1 #12) A $200,000, 30-year variable rate mortgage loan is obtained. The 1st monthly payment is due one month from the date of the loan. At the time the loan is obtained, the interest rate is 7%, compounded monthly. On the 2nd anniversary of the loan, the interest rate is increased to 7.5%, compounded monthly. On the 4th anniversary of the loan, the interest rate is increased to 8%, compounded monthly, and remains fixed for the remainder of the mortgage repayment period. Calculate the total interest paid on the loan. Solution We’ll use $1,000 as one unit of money. Step 1 Calculate the monthly payment when the interest rate is 7% Time t (month)

0

payment

1

2



24



48



360

X

X

X

X

X

X

X

X

200 = X a360 i , where i =

7% . 12

In BA II Plus TVM, set PV= - 200, N=360, I/Y=7/12. CPT “PMT.” X = 1.33060499

After finding X , don’t erase the data inputs in TVM. You’ll need to reuse these inputs. Step 2 Calculate the outstanding loan balance when the interest rate changes to 7.5% Time t (month) payment

0

1

2



24



48



360

X

X

X

X

X

X

X

X

P24 = X a360

24 i

= X a336 i

In TVM, simply reset N=336. CPT “PV.” You should get: P24 = 195.7898947

230

http://actuary88.com Step 3 Recalculate the level monthly payment under the new interest rate of 7.5% Time t (month)

0

payment

1

2



24



48



360

X

X

X

X

Y

Y

Y

Y

P24 = Y a360

24 j

= Y a336 j , where j =

7.5% 12

In TVM, simply reset I/Y=7.5/12. CPT “PMT.” You should get: Y = 1.39572270

Step 4 Calculate the outstanding loan balance when the interest rate changes to 8% Time t (month)

0

payment

1

2



24



48



360

X

X

X

X

X

Y

Y

Y

P48 = Y a360

48 j

= Y a312

j

In TVM, simply reset N=312. CPT “PV.” You should get: P48 = 191.3502131 Step 5 Recalculate the level monthly payment under the new interest rate of 8% Time t (month) payment

0

1

2



24



48



360

X

X

X

X

Y

Y

Y

Y

P48 = Z a360

48 k

= Z a312 k , where k =

8% 12

In TVM, simply reset I/Y=8/12. CPT “PMT.” You should get: Z = 1.45923312 The total loan repayments:

24 X + 24Y + 312 Z

The total interest paid on the loan: 24 X + 24Y + 312 Z 200 = 320.7125968 = $320, 712.60

231

http://actuary88.com Example 13 (SOA May 2004 EA-1 #27) A loan is made on 1/1/2004. Loan repayments: 120 level monthly payments of interest and principal with 1st payment at 2/1/2004. 12 Interest is charged on the loan at a rate of i ( ) = 7.5% .

Amount of interest paid in the 54th payment of loan = $100 P =Principal outstanding on the loan after the 90th payment.

Calculate P . Solution

time t (month) payments

0

1 X

2 X

… X

54 X

… X

90 X

… X

120 X

Using the imaginary cash flow method, we know that the interest portion of the 54th payment is: X (1 v121

54

) = X (1

v 67 ) , where X is the level monthly payment

Next, we need to find the monthly effective interest rate: i ( ) 7.5% = = 0.625% 12 12 12

i=

X (1 v 67 ) = X 1 time t (month) payments

0

(1 + 0.625% ) 1 X

67

2 X

= 100 , X = 293.02

… X

54 X

… X

90 X

P = X a120

90 i

… X

120 X

= X a30 i

P = X a30 i = 293.02 a30 i = 7,992.98

232

http://actuary88.com Example 14 (SOA May 2004 EA-1 #7) Details of a loan made on 1/1/2004: # of payments: Amount of each payment: Date of 1st payment: Interest rate:

10 $5,000 12/31/2004 8% compounded annually

Immediately after the 6th payment, an additional $10,000 payment is made. The loan is re-amortized over a longer term to provide for annual payment of $1,000 and a final smaller payment of X one year after the last $1,000 payment. Calculate X . Solution We’ll use $1,000 as one unit of money. The outstanding loan balance immediately after the 6th payment is made: Time t (year) 0 1 2 …. 6 7 … 10 payment 5 5 5 5 a4 8% = 16.5606342 At t = 6 , the borrowers pays additional $10 and immediately re-amortize the remaining loan balance. After that, he pays $1 per year for n years: 16.5606342 10 = an i Enter the following into BA II Plus TVM: PV= - 6.5606342, PMT=1, I/Y=8, FV=0. Press “CPT” “N.” You should get: N = 9.66886977 So after the re-amortization, the borrower pays 9 level payments of $1 each and pays a final payment of X after the 9th payment 16.5606342 10 = a9 i + Xv10

Solving the above equation, we get: X = 0.67735471 = $677.35471

233

http://actuary88.com Example 15 (SOA May 2000 Amount of a loan: Frequency of repayments: # of repayments: Interest rate:

EA-1 #24) $250,000 quarterly, at the end of each quarter 100 8% per year, compounded continuously

In which repayment does the principal component first exceed the interest component? Solution Let X represent the level quarterly repayment. Time t (quarter) Payment

0

1 X

2 X

… X

k X

…. X

100 X

Assume that in the k -th repayment the principal component first exceeds the interest component. The principal component of the k -th repayment is X v101 k ; the interest component is X (1 v101

k

).

We need to solve the following equation: X v101 k > X (1 v101

k

)

v101 k > 1 v101 k ,

2 v101 k > 1 , v101 k > 0.5

We are given the force of interest factor is: v=e v101 k > 0.5

4

=e

8% 4

e

=e

= 8% . So the quarterly discounting

0.02

0.02 (101 k )

> 0.5

0.02 (101 k ) > ln 0.5 , 101 k <

ln 0.5 = 34.657 , k > 66.3426 0.02

So the 67th repayment is the 1st time the principal component exceeds the interest component.

234

http://actuary88.com Example 16 (SOA May 2000 EA-1 #13) Date of a loan: 1/1/1990 Amount of loan: $100,000 Interest rate: 12% per year, compounded monthly Term of loan: 360 level monthly repayments First repayment date: 2/1/1990 Immediately after making the 120th repayment, the borrower decides to add $Q to each monthly repayment so that the loan will be repaid after having made a total of 160 monthly repayments. Calculate $Q . Solution Original repayment schedule where X is the monthly repayment Time t (month) 0 1 2 … 120 121 … Cash flow X X X X X X 100, 000 = X a360 i where i = X = 1.028.612597

360 X

12% = 1% . 12

P = X a240 i = 93, 417.9957 So the outstanding balance immediately after the 120th payment made is 93,417.9957. Actual repayment Time t (month) 0 Cash flow

1 X

2 X

… X

120 X

121 X +Q

… X +Q

160 X +Q

93, 417.9957 = ( X + Q ) a40 i X + Q = 2,845.100921

Q = 1,816.488324

235

http://actuary88.com Example 17 Payments are made at the end of the year for 30 years, with the payment equal to $12 for each of the first 20 payments and $9 for each of the last 10 payments. The interest portion of the 11th payment is twice the interest portion of the 21st payment. Calculate the interest portion of the 21st payment. Solution First define some symbols: •

OBt is the outstanding loan balance immediately after the t th payment is made



K t is the payment made at the t th payment. K t = I t + Pt



I t is the interest portion of the t th payment

• •

Pt is the principal portion of the t th payment i is the effective interest rate per payment period (1 year in this problem)

Then I t = OBt 1 × i

Time t (Yr) Payment

0

and Pt = K t

1 2… $12 $12

I t = OBt OBt +1

10 11 12 … $12 $12 $12 …

20 21 22 … $12 $9 $9 …

30 $9

OB10 = 12a10 + 9v10 a10

OB20 = 9a10

( × i = ( 9a ) i

)

The interest portion of the 11th payment is I11 = OB10 × i = 12a10 + 9v10 a10 i The interest portion of the 21th payment is I 21 = OB20

(12a

10

)

(

10

)

+ 9v10 a10 i = 2 9a10 i

236

http://actuary88.com

12 + 9v10 = 2 ( 9 )

v10 =

6 9

(

)

I 21 = OB20 × i = 9a10 i = 9 ×

1 v10 6 × i = 9 × (1 v10 ) = 9 × 1 =3 i 9

Alternative method to calculate I 21 after we find that v10 =

6 . 9

Prospectively, the payments at t =21,22,…,30 are level. So we can use the imaginary cash flow method to calculate the interest portion of the payments made at t =21,22,…,30. So we add an imaginary cash flow of $9 at t = 31 .

Time t (Yr) Payment

0

1

2…

10

11

12 … …

20 21 22 … $9 $9 …

30 31 $9 $9

The principal portion of the 21st payment is P21 = 9v 31

21

= 9v10 = 9 ×

6 = $6 9

The interest portion of the 21st payment is I 21 = K 21 P21 = 9 6 = $3 Example 18 A loan of $100 at a nominal interest rate of 12% convertible monthly is to be repaid by 6 monthly payments. The 1st monthly payment starts one month from today. The first 3 monthly payments are x each; the last 3 monthly payments are 3x each. A =the principal portion of the 3rd payment B =the interest portion of the 5th payment

Calculate A + B

237

http://actuary88.com Solution Time t (month) Payment

0

1 x

2 x

100 = xa3 + 3 xv 3 a3

x=

3 x

4 3x

5 3x

6 3x

5 3x

6 3x

where i =1%

100 100 100 = = = 8.692 3 3 (1 + 3v ) a3 (1 + 3v ) a3 (1 + 3 ×1.01 3 ) 2.941

Time t (month) Payment

(

0

1

2

3 x

4 3x

OB2

)

OB2 = v x + 3xa3 = (1.01 1 ) ( 8.692 + 3 × 8.692 × 2.941) = 84.536 The interest portion of the 3rd payment: I 3 = OB2 × i = 84.536 × 1% = 0.845 The principal portion of the 3rd payment: P3 = K 3 I 3 = 8.692 0.845 = 7.847 Time t (month) Payment

(

OB4 = 3xa2

0

1

2

3

4

5 3x

6 3x

OB4

) = (3 × 8.692 ×1.9704 ) = 51.38

The interest portion of the 5th payment is: I 5 = OB4 × i = 51.38 × 1% = 0.514

Alternative method to calculate I 5 . Since prospectively the payments are level at t = 5, 6 , we can use the imaginary cash flow method. So we set a fake cash flow at t = 7 . Time t (month) Payment

0

1

2

3

4

5 3x

6 3x

7 3x

P5 = 3 x × v 7 5 = 3 × 8.692 × 1.01 2 = 25.562 I 5 = K5

P5 = 3 x × v 7 5 = 3 × 8.692 25.562 = 0.514

P3 + I 5 = 7.847 + 0.514 = 8.361

238

http://actuary88.com Example 19 A loan is repaid by annual installments of X at the end of each year for 20 years. The annual interest rate is 6%. Let A=the total principal repaid in the first 5 years Let B=the total principal repaid in the last 5 years Calculate A/B. Solution A = X ( v 20 + v19 + v18 + v17 + v16 ) A / B = v15 = 1.06

B = X ( v5 + v 4 + v3 + v 2 + v )

= 0.4173

15

Example 20 Payments are made at the end of the year for 30 years, with payment equal to 120 for each of the first 20 years and 90 for each of the last 10 years. The interest portion of the 11th payment is twice the interest portion of the 21st payment. Calculate the interest portion of the 21st payment. Solution Time t (Yr) Payment

0

1 2… 10 11 12 … 20 21 22 … 30 120 120 120 120 120 120 120 120 90 90 90 90

The outstanding balance immediately after the 10th payment is made:

(

OB10 = 120a10 + v10 9a10

)

The interest portion of the 11th payment is

(

)

I11 = OB10 × i = 120a10 + 90a10 v10 i Similarly, the interest portion of the 21st payment is:

(

)

I 21 = OB20 × i = 90a10 i

(120a

+ 90a10 v10 i = 2 90a10 i

i=v

1=

10

1

)

6 9

(

(

)

120 + 90v10 = 2 ( 90 )

1/10

1 = 4.1380%

a10 =

v10 = 6 / 9

1 v10 1 6/9 = = 8.0555 i 4.1380%

)

I 21 = 90a10 i = 90 × 8.0555 × 4.1380% = 30

239

http://actuary88.com

Chapter 8

Sinking fund

Key points •

At t = 0 , the borrower borrows the principal.



At each year for n years, the borrower pays only the interest accumulated during that year, stopping the outstanding principal from growing or declining. Thus, the outstanding principal immediately after the annul interest payment is always equal to the original principal at t = 0 . (The borrower is only dealing with the interest due on year by year basis. He is not worrying about paying the principal at this stage.)



At the end of the term of the loan, the borrower pays a lump sum equal to the borrowed principal, terminating the loan. (Eventually, the borrower has to pay the principal.).



To make sure he can eventually pay off the principal, the borrower periodically deposits money into a fund. This fund accumulates with interest (can be different from the interest rate used to calculate the annual interest payment). At the end of the term of the loan, this fund accumulates enough money to pay off the principal.

Sample problems Problem 1 (#12 SOA May 2003 EA-1) A ten-year loan of $10,000 at an 8% annual effective rate can be repaid using any of the following methods: I.

Amortization method, with level annual payments at the end of each year.

II.

Repay the principal at the end of the years while paying 8% annual effective interest on the loan at the end of each year. The principal is repaid by making equal annual deposits at the end of each year into a sinking fund earning interest at 6% annual effective so that the sinking fund accumulates to $10,000 at the end of the 10th year.

III.

Same as II, except the sinking fund earns 8% annual effective.

IV.

Same as II, except the sinking fund earns 12% annual effective. 240

http://actuary88.com Rank the annual payment amounts of each method. [A] [B] [C] [D] [E]

I < II < III < IV II < I = III < IV I < IV < III < II IV < I < III < II The correct answer is not given by [A], [B], [C], or [D] above.

Solution Without doing any math, you should know I = III IV < III < II Reason for the 1st equation: The loan and sinking fund have the same interest rate. Consequently, our annual payments under I and III should be same. If we have the same interest rate, it doesn’t matter whether we separately pay the annual interest and the principal (the sinking fund method) or we combine the annual interest and the annual principal into one annual payment (loan amortization). We have the same pie, no matter how we slice it. Reason for the 2nd equation: II, III, and IV differ only in annual sinking fund payments. Annual deposits to the sinking fund under II, III, and IV must each accumulate to $10,000 at the end of Year 10. As a result, the higher the interest rate in the sinking fund, the smaller the annual deposit. Because the interest rates earned in the sinking fund are 6%, 8%, and 12% respectively in II, III, and IV, the annual deposit to the sinking fund is largest in II, smaller in III, and smallest in IV. So the correct answer is (E). Alternatively, you can calculate the annual payment for each of the four plans.

241

http://actuary88.com Plan I Time t

0

1

2

X

$X

Xa10 8% = 10, 000

3

X

4



10

X



X

X = 1490.29 (Total annual payment)

Plan II Time t

Time t

0

0

1

2

3

4



10

$800

$800

$800

$800



$800

1

2

3

4



10

A

A



A

$A

A

As10 6% = 10, 000 A = 758.68

X + A = 1,558.68 (Total annual payment)

Plan III Time t

Time t

0

0

1

2

3

4



10

$800

$800

$800

$800



$800

1

2

3

4



10

$B

B

B

B



B

Bs10 8% = 10, 000 B = 690.29

X + B = 1, 490.29 (Total annual payment)

Plan IV Time t

Time t

0

0

1

2

3

4



10

$800

$800

$800

$800



$800

1

2

3

4



10

$C

C

C

C



C

Cs10 12% = 10, 000 C = 569.84

X + C = 1,369.84 (Total annual payment)

242

http://actuary88.com Problem 2 (#17 SOA May 2004 EA-1) Loan term Loan repayment Sinking fund

$100,000 borrowed on1/1/2004, issued at a 5% annual effective interest rate. No repayments of principal or interest are made on the loan until a sinking fund has accumulated to pay the balance in full. This occurs 12/31/2019. $10,000 annual deposits from 12/31/2004 through 12/31/2011 and $5,000 annual deposits beginning on 12/31/2012. The sinking fund accumulation is i % until 12/31/2011 and k % thereafter.

During calendar year 2011, interest accrued on the loan and interest earned on the sinking fund are the same. In what range is k % ? [A] Less than 6.10% [B] 6.10% but less than 6.60% [C] 6.60% but less than 7.10% [D] 7.10% but less than 7.60% [E] 6.60% or more Solution This is not a typical sinking fund. In a typical sinking fund, the borrower pays the annual interest due. In addition, he sets up a sinking fund to accumulate the principal at the end of the loan term. In this problem, however, the borrower sets up a sinking fund to pay, at the end of the loan term, both the principal and the interest due. You shouldn’t be scared. Just apply the general principal of the time value of money and you should do fine. To neatly track down the timing of each cash flow, we’ll convert 12/31 of a year to 1/1 of the next year. For example, we convert 12/31/2019 to 1/1/2020. This helps prevent the off-by-one error. To simplify our calculation, we’ll use $1,000 as the unit money.

243

http://actuary88.com

Loan

$100

5%

Time t

1/1/04 0

Sinking Fund

1/1/05 1

1/1/06 1/1/11 2 … 7

$10

$10



1/1/12 8

1/1/13 9

1/1/14 10



1/1/20 16

$10

$5

$5



$5

$10

i%

k%

We can set up two equations: •

The sinking fund and the loan should accumulate to the same amount at t=16.

(1 + k % )

8

10s

8 i%

value of sinking fund at t =8 (8 parallel cash flows of $10 each are collasped to the final cash flow time at t =8); the interest rate is i %



+

Accumulate from t =8 to t =16 at k %

= 100 (1.05)

16

5s

8 k%

Accumulate loan from t =0 to t =16 at 5%

8 parallel cash flows of $5 each are collasped to the final cash flow time at t =16; the interest rate is k %

The loan and the sinking fund should generate the same interest amount from t=7 to t=8

100 (1.057 ) 5% = 10s7 Loan balance at t = 7

i%

i%

Sinking fund balance at t = 7

The second equation is easier.

s7 i %

(1 + i ) i% =

7

1

i

i % = (1 + i % )

7

1 = 10 (1.057 ) 5% ,

i = 7.907%

Solving the equation:

10s8

7.907%

(1 + k % )

8

+ 5s8

= 100 (1.05 ) , 106.021(1 + k % ) + 5s8 16

k%

We translate 106.021(1 + k % ) + 5s8 8

k%

8

k%

= 218.287

= 218.287 into the follow cash flow

diagram:

244

http://actuary88.com

Time t 0 cash 106.021

1 5

2 5

3 5

4 5

5 5

6 5

7 5

8 5

FV=218.287 In TVM, set PV=106.021, PMT=5, N=8, FV= - 218.287. Use annuity immediate mode. You should get: I/Y=5.98% So the correct answer is [A]. Problem 3 (#22 SOA May 2000 EA-1) Date of a loan: 1/1/2000 Amount of loan: $ X Date of the 1st repayment: 12/31/2000 Frequency of repayments: Annually # of repayments: 10 Amount of each repayment: $1,000 Method of repayment: One half of the loan is repaid by the amortization method using an interest rate of 7% per annum compounded annually. The other half is repaid by the sinking fund method where the lender receives 7% per annum, compounded annually, on this portion of the loan, and the sinking fund accumulates at 6% per annum, compounded annually. Calculate $ X . Solution One half of the loan is repaid through amortization method. Let A represent the level annual repayment in the amortization method. Time t (year) Payment

0

1 A

2 A

X = A a10 7% , 2

A=

…. A

10 A

X 2 a10 7%

245

http://actuary88.com The other half is repaid by the sinking fund method. The annual repayment in this method consists of two parts: X • Annual interest payment of ( 7% ) 2 •

Annual deposit B into the sinking fund to accumulate to t = 10 . The interest rate is 6%.

Time t (year) Payment

0

1 B

2 B

…. B

X = B s10 6% , 2

X at 2

10 B

B=

X 2 s10 6%

So the total annual repayment is: X 2 a10 7%

+

X X = 1, 000 ( 7% ) + 2 2 s10 6%

1 1 X + ( 7% ) + 2 a10 7% s10 6%

a10 7% = 7.02358154 ,

= 1, 000

s10 6% = 13.18079494

X = $6, 938.53077

246

http://actuary88.com

Chapter 9 bonds

Callable and non-callable

Bond is a standardized loan. The borrower borrows money from a lender. The borrower pays back the borrowed money by paying regular installments called coupons and a final lump sum (called redemption value) in the end.

Key concepts: Par or face value • For calculating the size of the coupon • Not necessarily the price that the investor pays for the bond or the payment the investor receives at the bond maturity date Redemption value • The amount the investor receives at the bond maturity date excluding the coupon • If a problem doesn’t specify the redemption value, assume the redemption value is equal to the par (or face amount) Coupon • the interest rate on the nominal amount of the bond • fixed throughout the life of the bond Time to maturity • the length of time until the bond is redeemed The market price of a coupon bond is calculated by discounting all the future cash flows at the yield to maturity • The yield to maturity is the investor’s opportunity cost of capital (i.e. return earned by investing in other assets). •

Because investors demand to earn the prevailing market interest rate, the yield to maturity is the prevailing market interest rate. Here we assume a constant interest rate that doesn’t change with time (i.e. a flat yield curve).



YTM is the IRR an investor would realize by purchasing the bond, holding it to maturity, and reinvesting coupons at YTM.



If the bond is purchased at a premium, YTM is less than the coupon rate. Coupons will be reinvested a lower rate; the

247

http://actuary88.com investor will realize a capital loss when the bond matures. •

If the bond is purchased at a discount, YTM is greater than the coupon rate. Coupons will be reinvested a higher rate; the investor will realize a capital gain when the bond matures.

Bond prices change in the opposite direction from the change of the prevailing market interest rate • If the market interest rate goes up, the price of a bond goes down; we have to discount the future cash flows at a higher discount rate. •

If the market interest rate goes down, the price of a bond goes up; we have to discount the future cash flows at a lower discount rate.

Selling a bond at par, premium, discount • When the coupon rate is equal to the prevailing market interest rates, the price of the bond is equal to its par value. The bond sells at par value. •

When coupon rate is below the prevailing market interest rates at a certain point of time, the price of the bond will fall below its par value. The bond sells at a discount.



When the coupon rate is above the prevailing market interest rates at a certain point of time, the price of the bond will be greater than its par value. The bond sells at a premium.

Bond cash flow diagram – make sure you know how to draw one Unit time = per coupon period Time t

0

1

Coupon

2

3

Coupon

Coupon

4

Coupon



n

Coupon Redemption



Set per coupon period as the unit time. If a bond pays coupons every 6 months, then the unit time is 6 months.



Cash flows consist of regular coupons plus a final redemption amount.

248

http://actuary88.com



If a bond pays 8% coupons convertible semiannually per $100 face amount, this means that you get $4 once every 6 months, not $8 once every year. Make sure you remember this.

Big ideas: 1. Bond is a loan just like a student loan or home mortgage. If you buy a bond issued by AT&T, you are lending your money to AT&T. The principal amount you lend to AT&T is just the present value of the bond cash flows (regular coupons plus face amount at the end of the loan term) discounted at YTM. In turn, AT&T repays the loan through installments (regular coupons plus face amount at the end of the loan term). Everything you learn about loan (such as amortization) applies to a bond. This is the most important concept you need to know about a bond. Knowing this enables you to cut to the chase of many bond problems. 2. Bond is a standardized loan. Difference

Bond

Borrower Lender

Typical loan Can be anyone. Government and corporations (called bond Individuals, issuers). They borrow money to fund special corporations, projects or business growth. government. Anyone who has idle money and wants to earn interest. When you buy a bond, you become a lender. Typically banks. Bond issuers pay back loans through regular payments called coupons and a final payment. Coupons are typically paid once every 6 months. A final payment is made at the end of borrowing period. In old days before computers, investors actually cut off coupons from a coupon book and mailed it to the issuing firm to claim interest payment. This is the origin of the name coupon. Some bonds don’t pay coupons. When a bond matures, the borrower pays the principal plus the interest accrued thus far. These bonds are called zero-coupons.

The borrower and the lender can negotiate and agree on any Payback method payback methods. Can the lender sell the loan to Hard for a homeowner another investor who has idle Bonds are standardized loan and can be easily to sell his mortgage to cash and wants to earn interest? bought and sold in the open market. other investors.

3. Why standardizing loan (i.e. inventing bond)?

249

http://actuary88.com •

One-off borrowing is time-consuming. Corporations and government always need extra money to expand business or finance special projects (such as building highways). If every time they need money they have to walk to a bank and fill out a loan application, the borrowing process is time-consuming.



One-off borrowing is expensive. Banks often charge extra fees to cover costs associated with loan approvals (such as checking borrower’s credits and negotiating loan terms). As a result, banks often charge a higher interest rate to cover these fees. So government and corporations rather not borrow money from banks. By issuing bonds and borrowing money from the general public, government and corporations lower their borrowing cost.



Standardized loans are easier for the general public to understand than one-off loans.



Standardized loans can be sold and traded in the open market (such as New York Stock Exchange and NASDAQ). If a lender wants to back out, he can quickly sell the bond to someone else (may incur a loss though). This helps the sales of the bonds.



Standardized loans allow multiple investors to split a large amount of loan. If a company wants to borrow $100,000,000 cash, it doesn’t have to spend lot of time looking for one wealthy person or a big bank to lend this amount. Instead, the company can simply issue 100,000 bonds with each bond selling $1,000.

4. Call a bond = buy back the bond prior to the bond’s maturity = refinance the loan at a lower coupon rate •

If you ever refinanced your student loan, or if your family refinanced the house, you’ll understand a callable bond.



Refinancing a student loan. Say you borrowed the U.S. government a $50,000 student loan at 8% interest per year. After you graduated, you started to pay your student loan through monthly payments. You were paying 8% interest per year. Gradually, you reduced your loan balance to $45,000. Then the market interest rate dropped to 4%. You didn’t want to continue paying the U.S. government 8% interest rate -- the market rate was only 4%. What could you do? You walked to a bank, borrowed $45,000 at 4%, and immediately mailed this $45,000 check to the U.S. government. Now you no longer owed the U.S. government anything. You just owed the bank $45,000 but at 4% interest rate.

250

http://actuary88.com

Before refinancing After refinancing

Your debt was $45,000; you were paying 8% interest. Your debt was $45,000; you will pay 4% interest.



Call a bond = Refinance the bond. Once the bond is issued, the cash flows of the bond are set. Bond issuers pay the set coupon rates. However, if the interest rate drops in the future, it doesn’t make sense for the bond issuer to continue paying a high coupon rate. As a result, the bond issuer will refinance the bond by backing back the higher coupon bond and by issuing a new bond with a lower coupon rate.



Say AT&T originally took out a loan from you (i.e. you bought a bond issued from AT&T) and paid you annual coupons of 10%. A few years later, the interest rate dropped to 4%. Would AT&T still happily pay you 10% coupons when the market interest rate is only 4%? No. If the bond is callable, AT&T will refinance the loan. AT&T simply borrows money from someone else by issuing a bond that pays a lower coupon rate (such as 4% or 4.5%). Then AT&T immediately pays back what he still owes you at that time using the proceeds it got from issuing the new lower coupon bond. The net result: AT&T gets rid of the 10% coupon bond and replaces it with 4% coupon bond. AT&T saves lot of money by calling a bond (i.e. refinancing a bond). Before refinancing the bond (i.e. calling the bond) After refinancing the bond

AT&T pays 10% coupons. AT&T pays 4% coupons.



In U.S., most corporate bonds are callable bonds. Call features are attractive to bond issuers because bond issuers can refinance their debt in the event that the interest rate drops.



If the interest rate drops, the issuer of a callable bond can issue a brand new bond in the market at a lower borrowing rate; simultaneously, the bond issuer buys back the old bond using the proceeds generated by the brand new bond.

251

http://actuary88.com



However, if the interest rate stays level or goes up, the bond issuer will not exercise the call option. It doesn’t make sense to refinance a debt at a higher interest rate. So call feature is an option, not a duty. The issuer can call a bond if he wants to according to the contract, but it doesn’t have to call.



However, not every bond can be called. To make a bond callable, the bond issuer must state, in the contract (called bond indenture), that the bond to be issued is callable. Calling a bond is bad for the bond holder. After the bond is called, the bond holder must give up his high-yielding bond and look for another bond (a lower coupon bond) to invest in.



Typically, when the interest rate drops a lot, callable bonds get called. This is like student loan refinancing. When the interest rate drops a lot, the number of student loans refinanced goes up.

5. Determine the highest price or the minimum yield of a callable bond. SOA likes to test this type of problems. •

Pricing approach – assume the bond issuer will choose the redemption date most detrimental to the bond holder (the worse case scenario).



An investor will pay only the bottom price under the worse case scenario. This way, the investor won’t get burned.



How to determine the floor price and the redemption date for a callable bond (1) If the redemption amounts are constant for a range of callable dates •

If the modified coupon rate of the bond > the yield rate per coupon period, call the bond ASAP. This makes intuitive sense. If AT&T pays you 10% coupons semiannually, yet the market rate is only 6% nominal compounding semiannually, AT&T will gladly refinance the original 10% coupon bond by issuing a 6% coupon bond.



If the modified coupon rate of the bond < the yield rate per coupon period, don’t call the bond and let it mature. This makes intuitive sense. If AT&T pays you 6% coupons semiannually, yet the market rate is 10% nominal

252

http://actuary88.com compounding semiannually, it will be foolish for AT&T to refinance the original 6% coupon bond by issuing a 10% coupon bond. AT&T will simply ignore its call option. •

If the modified coupon rate of the bond = the yield rate per coupon period, calling the bond or not calling the bond makes no difference.

(2) If the redemption amounts are not constant for a range of callable dates, • Step #1 For each range of callable dates where the redemption amount is constant, apply Rule (1) and determine the redemption date and bond price. •

Step #2 Among the prices calculated in Step #1, choose the minimum price. This is the price to be paid by the investor. The redemption date associated with this price is the worst-case-scenario redemption date.

I recommend that you work through Broverman’s Example 4.5 and 4.6 on page 246 and 247 (if you use the 3rd edition of his Broverman’s textbook). Sample Problems Problem 1 Bond face amount

$1,000

Maturity Coupons

5 years Zero

Selling price

$725

Calculate the annual rate of return earned by the buyer of the bond. Solution This question is about zero coupon bonds. Key points about zero coupon bonds (also called pure discount bonds): • Pay stated face or par value at maturity • Sold at a discount • Zero coupons are paid in the life of the bond • Like a saving account

253

http://actuary88.com

(Unit time =1 year) Time t

0

Cash flow

1

2

3

4

5

$0

$0

$0

$0

$1,000

$725

A zero coupon bond is like a saving account. To find the annual return i , we solve the following equation: 725 (1 + i ) = 1, 000 5

i = 6.643%

Problem 2 Bond issue date Maturity date Face amount Coupons Redemption amount Yield to maturity at purchase

1/1/2005 1/1/2010 $1,000 8% payable 7/1 and 1/1 Par 10.25% annual effective

Explain whether the bond is a premium bond or discount bond; Calculate the premium or discount. Generate an amortization schedule. Solution First, we draw a cash flow diagram. Because coupons are paid once every 6 months, we’ll use 6 months as unit time to simply our calculations. Please note that the term of the bond is 5 years or 10 units of time. (Unit time =6 months) Time t

0

Cash flow

1

2

3

$40

$40

$40



9 …

10

$40

$40 $1,000 1

PV = 40a10 + 1, 000v10 @ i = (1 + 10.25% ) 2 1 = 5%

PV = 922.78

254

http://actuary88.com Please note that the price of the bond is always equal to PV of the bond; PV of the bond is always equal to the cash flows discounted at YTM (yield to maturity). PV=922.78 < Face amount = 1,000 The bond is sold at a discount Discount = PV - Face = 922.78 1,000 = $77.22 We can also intuitively see why this bond sold at a discount. The coupons of the bond pay the buyer 4% per 6 months, while the market interest rate is only 5% per 6 months. If the bond is still sold at par, the bond issuer (the borrower) will underpay the bond buyer, creating unfairness in the transaction. As a result, the bond issuer charges a price below the par. The amount by which the selling price is below the par amount is the discount. To generate an amortization schedule of the bond, we’ll treat the bond as a loan. In this loan, the borrower (the bond issuer) borrows the present value (or price) of the bond; he repays the loan by paying ten semiannual payments of $40 each plus a final payment of $1,000 at the end of Year 5. By treating a bond as a loan, we can amortize a bond the same way we amortize a loan through the following steps: •

Find the outstanding balance Pt of the bond immediately after

repayment X ( t ) is made. We can calculate Pt using the

retrospective or prospective method. Please note that the repayment is $40 from t = 1 to t = 9 and $1,040 at t = 10 (the unit time is 6 months). •

Multiple Pt with the effective interest rate per unit time i (5% in

this problem). This gives us Interest ( t ) = i Pt , the interest portion of

X (t ) . •

Calculate Principal ( t ) = X ( t ) Interest ( t ) , the principal portion of

X (t ) .

255

http://actuary88.com Following the above procedure, we’ll get the follow amortization schedule: Date Loan Interest Principal Outstanding repayment balance of the bond 1/1/2005 $922.78 7/1/2005 $40 $46.14 (1) -$6.14 (2) 928.92 (3) 1/1/2006 40 46.45 -6.45 935.37 7/1/2006 40 46.77 -6.77 942.14 1/1/2007 40 47.11 -7.11 949.25 7/1/2007 40 47.46 -7.46 956.71 1/1/2008 40 47.84 -7.84 964.55 7/1/2008 40 48.23 -8.23 972.78 1/1/2009 40 48.64 -8.64 981.42 7/1/2009 40 49.07 -9.07 990.49 1/1/2010 1,040 49.52 990.48 0 Total 1,400 477.23 922.77 (1) 46.14 = 922.78(5%) (2) -6.14 = 40 – 46.14 (3) 928.92 =922.78 - (-6.14) From the above table, we see that a discount bond has a negative amortization in all the payments except the final one. The earlier periodic repayments do not even cover the interest due, creating negative principal repayments (i.e. increasing the outstanding balance of the loan). Make sure you can manually create the above table. Please also note that we can use the amortization method suggested by the textbook: K-th payment 0

Outstanding balance F 1 + ( r j ) an j

Payment

Interest portion

1

F 1+ (r

j ) an

1 j

Fr

F j + (r

2

F 1+ (r

j ) an

2 j

Fr

F j + (r

k

F 1+ (r

j ) an

Fr

F j + (r

n-1

F 1+ (r

j ) a1

Fr

F j + (r

n

0

Fr+F

F j + (r

k j j

( j ) (1 j ) (1 j ) (1 j ) (1

j ) 1 v nj

)

)

v nj

1

v nj

k +1

) vj ) v 2j

Principal portion

)

F (r

j ) v nj

F (r

j ) v nj

1

F (r

j ) v nj

k +1

F (r

j ) v 2j

F (r

j)vj

256

http://actuary88.com Under this method, we have F = 1,000, r = 4%, j = 5%, n = 10 . Let’s generate an amortization schedule for the first two payments: t Outstanding balance 0 1, 000 1 + ( 4% 5% ) a 10 5% = 922.78

1 1, 000 1 + ( 4% 5% ) a 9 5% = 928.92

2 1, 000 1 + ( 4% 5% ) a 8 5%

Payment

Interest portion

1, 000 ( 4% )

1, 000 5% + ( 4% 5% ) (1 1.05

= 40

= 46.14

1, 000 ( 4% )

1, 000 5% + ( 4% 5% ) (1 1.05

= 40

= 46.45

= 935.37

Principal portion

10

)

1, 000 ( 4% 5% )1.05 = 6.14

9

)

1, 000 ( 4% 5% )1.05 = 6.45

We can also generate the amortization schedule using BA II Plus/BA II Plus Professional Amortization Worksheet. Refer to Chapter 4 on how to generate the amortization schedule. Problem 3 Bond issue date Maturity date Face amount Coupons Redemption amount Yield to maturity at purchase

1/1/2005 1/1/2010 $1,000 8% payable 7/1 and 1/1 Par 6.09% annual effective

Explain whether the bond is a premium bond or discount bond; Calculate the premium or discount. Generate an amortization schedule. Solution (Unit time =6 months) Time t

0

Cash flow

1

2

3

$40

$40

$40



9 …

10

$40

10 $40 $1,000

1

PV = 40a10 + 1, 000v10 @ i = (1 + 6.09% ) 2 1 = 3% PV = 1,085.30

257

9

http://actuary88.com PV=1,085.30 > Face amount = 1,000 The bond is sold at a premium Premium = PV – Face amount = 1,085.30 – 1,000 = 85.30 We can also intuitively see why this bond sold at a premium. The coupons of the bond pay the buyer 4% per 6 months, while the market interest rate is only 3% per 6 months. If the bond is still sold at par, the bond issuer (the borrower) will overpay the bond buyer, creating unfairness in the transaction. As a result, the bond issuer charges a price above and beyond the par. The amount by which the selling price exceeds the par amount is the premium. Amortization schedule: Date Loan repayment 1/1/2005 7/1/2005 1/1/2006 7/1/2006 1/1/2007 7/1/2007 1/1/2008 7/1/2008 1/1/2009 7/1/2009 1/1/2010 Total

Interest

Principal

$40 $32.56 (4) $7.44 (5) 40 32.34 7.66 40 32.11 7.89 40 31.87 8.13 40 31.63 8.37 40 31.37 8.63 40 31.12 8.88 40 30.85 9.15 40 30.57 9.43 1,040 30.27 1,009.71 $1,400 $314.69 $1,085.29

Outstanding balance of the bond $1,085.30 1,077.86 (6) 1,070.20 1,062.31 1,054.18 1,045.81 1,037.18 1,028.30 1,019.15 1,009.72 0

(4) 32.56 = 1,085.30 (3%) (5) 7.44 = 40 – 32.56 (6) 1,077.86 = 1,085.30 – 7.44

258

http://actuary88.com Let’s generate an amortization schedule for the first two payments using the textbook method. We have F = 1,000, r = 4%, j = 3%, n = 10 . t Outstanding balance 0 1, 000 1 + ( 4% 3% ) a 10

Payment

Interest portion

1, 000 ( 4% )

1, 000 3% + ( 4% 3% ) (1 1.03

= 40

= 32.56

1, 000 ( 4% )

1, 000 3% + ( 4% 3% ) (1 1.03

= 40

= 32.34

Principal portion

3%

= 1, 085.30

1 1, 000 1 + ( 4% 3% ) a 9

3%

= 1, 077.86

2 1, 000 1 + ( 4% 3% ) a 8

3%

= 1, 070.20

10

)

1, 000 ( 4% 3% )1.03

= 7.44 9

)

1, 000 ( 4% 3% )1.03

= 7.66

Make sure you can also generate the amortization schedule using BA II Plus/BA II Plus Professional Amortization Worksheet.

Problem 4 Face amount of a callable bond Coupon Purchase date Call date Redemption amount

10

$1,000 8% annual 7/1/2005 Any time between 7/1/2020 and 7/1/2026 Par

Calculate the maximum price the buyer of the bond will pay to guarantee a yield of at least 7%. Solution The bond issuer pays 8% annual coupons, but the buyer of the bond is content to lock in only 7%. Why can’t the buyer get 8% return? Because the market interest rate is below 8%. Why is the bond buyer happy to get a minimum return of 7%? Because the market interest rate is volatile, going up and down; by locking in the 7% floor rate, the buyer can sleep well at night assured that he’s getting at least 7%. Because the buyer is happy to lock in 7%, the bond issuer will gladly refinance the bond at 7% at the earliest callable date. This way, the bond issuer will pay only a 7% annual interest rate on its refinanced debt, instead of continuing paying 8% coupons. As a result, the bond issuer will recall the bond at the earliest recallable date, which is 7/1/2020.

259

9

http://actuary88.com

(Unit time =1 year) 7/1/2005 Time t

0

Cash flow

7/1/2020 1

2

3

$80

$80

$80

… …

14

15

$80

$80 $1,000

PV = 80a15 + 1, 000v15 = 1, 091.08 @ i = 7%

So the maximum price the buyer will pay is $1,091.08. If the bond sells above this price, the buyer’s return for investing the bond will be lower than 7%.

260

http://actuary88.com Problem 5 Bond A $100 7% payable semiannually 100% par 10 years

Face amount Coupons Redemption Term to maturity

Bond B $100 4% payable semiannually 150% par 10 years

Bond A and Bond B have the same yield to maturity and sells at the same price. Calculate the price of each bond. Solution Unit time =6 months A

Time t

0

Cash flow

B

Time t

0

Cash flow

A-B

Time t Cash flow

0

1

2

3



19

20

$3.5

$3.5

$3.5



$3.5

$3.5 $100

1

2

3



19

20

$2

$2

$2



$2

$2 $150

1

2

3



19

20

$1.5

$1.5

$1.5



$1.5

PV = 1.5a20

$1.5 - $50

50v 20 = 0

i = 5.07615296% Using BA II Plus/Professional TVM

Then the price of Bond A is:

3.5a20 + 100v 20 = 80.48386442

Then the price of Bond B is:

2a20 + 150v 20 = 80.48386442

261

http://actuary88.com Problem 6 (May 2004 EA-1) On 1/1/2005, Smith purchases a 20-year bond with a par value of $1,000. The bond pays semi-annual coupons at an annual rate of 6%. The bond is purchased to yield 5% per annual effective. When each coupon is received, it is immediately reinvested at a rate of interest of 6% per annum convertible quarterly. In what range is Smith’s effective annual rate of return over the term of the bond? [A] Less than 5.20% [B] 5.20% but less than 5.30% [C] 5.30% but less than 5.40% [D] 5.40% but less than 5.50% [F] 5.50% or more Solution Unit time = 6 months Time

0

Amount $0

1

2

3

40

$30

$30

$30

$30 $1,000

20 years Smith’s cost of buying the bond at time zero

30a40 + 1, 000v 40

@ i = 1.05 1 Smith’s total wealth at t=40 = reinvest coupons @ 6% + receiving par

30 s40 + 1, 000 Reinvest Coupons

Receiving Face amount

6% @ j = 1+ 4

2

1

Remember that reinvestment rate is 6% per year convertible quarterly

We can solve for r , Smith’s annual rate of return:

262

http://actuary88.com

(30a

40 i

+ 1, 000v 40

)

(1 + r )

20

=

30s40 j + 1,000 Ending Wealth

Initial Investment

Using BA II Plus/BA II Plus Professional TVM, we have:

30a40 + 1, 000v 40 = 1,133.854761 @ i = 1.05 1 30 s40

(1 + r )

6% = 2, 273.610707 @ j = 1 + 4 20

=

30s40 + 1, 000

30a40 + 1, 000v

40

=

2

1

2, 273.610707 + 1, 000 = 2.88715171 1,133.854761

The correct answer is D. Callable bond Problem 7 Facts about a callable bond: Face Coupon Redemption value Call dates

$100 6% semiannually $100 10th through 15th years at par

Calculate [1] Find the price of the bond to yield 8% convertible semiannually [2] Find the price of the bond to yield 4% convertible semiannually Solution [1] Find the price of the bond to yield 8% convertible semiannually Assume the bond is called immediately after the n -th coupon is paid. Because the call dates must be in the 10th through 15th year, we have: n = 20, 21, 22,..., 30

The price of the bond is: P = 3an + 100v n

@ 4% per coupon period

In the above equation, 3 is the semiannual coupon payment. 263

http://actuary88.com We need to minimize P , the purchase price of this callable bond. This is the maximum price the investor is willing to pay in order to lock in the minimum return of i ( 2 ) = 8% . To minimize P , we need to change the formula: P = 3an + 100v n

1 vn an = i

v n = 1 i an

(

)

(

)

P = 3an + 100 1 i an = 3an 4% + 100 1 4% an 4% = 100 an 4%

an 4% is an increasing function with n . To minimize P , we need to maximize n . We choose n = 30 .

P = 3a30 4% + 100 ×1.04

30

= 82.708

[2] Find the price of the bond to yield 4% convertible semiannually P = 3an + 100v n

@ 2% per coupon period

(

)

(

)

P = 3an + 100 1 i an = 3an 2% + 100 1 2% an 2% = 100 + an 2% To minimize P , we need to minimize n . So we set n = 20 .

P = 3a20 2% + 100 (1.02

20

) = 116.35

264

http://actuary88.com Problem 8 Facts about a callable Face Coupon Maturity Redemption value Call dates

bond: $1,000 4% semiannually 20 years if not called $1,000 2nd through 5th years at $1,050 6th through 10th years at $1,025 11th through 19th years at $1,010 20th year at $1,000

Calculate the maximum price an investor is willing to pay in order to lock in a yield of 6% convertible semiannually. Solution The solution process is similar to the process used for the last problem. The price of the bond is: P = 20an + 1050v n = 1050 + ( 20 1050 × 3% ) an 3% = 1050 11.5an 3%

Where n = 4, 5, 6, 7,8, 9,10 P reaches minimum when n = 10 . min P = 20a10 3% + 1050 (1.03 10 ) = 951.903 P = 20an + 1025v n = 1025 + ( 20 1025 × 3% ) an 3% = 1025 10.05an 3%

Where n = 12,13,14,..., 20 P reaches minimum when n = 20 . min P = 20a20 3% + 1025 (1.03 20 ) = 865.067 P = 20an + 1010v n = 1010 + ( 20 1010 × 3% ) an 3% = 1010 10.3an 3%

Where n = 22, 23, 24,..., 38 P reaches minimum when n = 38 . min P = 20a38 3% + 1010 (1.03 38 ) = 778.328 P = 20an + 1000v n = 1000 + ( 20 1000 × 3% ) an 3% = 1010 10an 3%

265

http://actuary88.com Where n = 39, 40 P reaches minimum when n = 40 . min P = 20a40 3% + 1000 (1.03 40 ) = 768.85

So 768.85 is the maximum price the investor is willing to pay to lock in a yield of 6% convertible semiannually. Problem 9 (May 2004 EA-1 #32) Face amount Coupon rate

Bond A $100 6%, payable semi-annually

Redemption Length of bond

Par 20 years

Bond B $100 5%, payable semiannually $125 20 years

Both bonds have the same purchase price and the same yield rate. Calculate the annual effective yield on these two bonds. Solution Bond A Time t (6 months)

0

Cash flows

1

2

3



40

$3

$3

$3

$3

$103

1

2

3



40

$2.5

$2.5

$2.5

$2.5

$127.5

1

2

3



40

$0.5

$0.5

$0.5

$0.5

- $24.5

Bond B Time t (6 months)

0

Cash flows

Cash flows of A B Time t (6 months) Cash flows

0

Because Bond A and B have the same price, the present value of A B should be zero.

266

http://actuary88.com There are at least two ways to calculate the effective yield. Method 1

use BA II Plus Cash Flow Worksheet

Enter the following into Cash Flow Worksheet: CF0=0, C01=0.5, F01=39, C02= - 24.5. Press “IRR” “CPT.” You should get:

IRR=1.10894155

So the 6-month effective yield is 1.10894155%. The annual effective yield is:

(1 + 1.10894155% ) Method 2

2

1 = 2.23018061%

Use BA II Plus TVM Worksheet

We can rewrite the cash flows of A B as follows: Time t (6 months)

0

Cash flows

Enter:

1

2

3



40

$0.5

$0.5

$0.5

$0.5

$0.5 - $25

PMT=0.5, FV= - 25, N=40, PV=0. Press “CPT” “I/Y.”

You should get:

I/Y= 1.10894155.

So the 6-month effective yield is 1.10894155%. The annual effective yield is:

(1 + 1.10894155% )

2

1 = 2.23018061%

267

http://actuary88.com Problem 10 (May 2004 EA-1 #28) For a given bond: Par value Redemption value Term of bond Coupons Issue price Issue price

$1,000 $1,100 10 years r % per year, payable semiannually P if yield to maturity is 4%, compounded annually P 95.5 if yield to maturity is 5%, compounded annually

Calculate r % . Solution Time t (6 months) Cash flows

0

1 5r

2 5r

3 5r

… 5r

20 1100 + 5 r

Bond price is P if yield to maturity is 4%, compounded annually: P = 5 r a20 i + (1100 )1.04

1

10

, where i = 1.04 2 1 = 1.98039027%

Bond price is P 95.5 if yield to maturity is 5%, compounded annually: P 95.5 = 5 r a20 j + (1100 )1.05 5 r a20 i + (1100 )1.04

(

5 r a20 i

a20

j

10

= 5 r a20 j + (1100 )1.05

) = 95.5

1100 (1.04

10

1

10

, where j = 1.05 2 1 = 2.46950766%

10

+ 95.5

1.05

10

)

a20 i = 16.38241895 , a20 j = 15.63418569 r=

95.5 1100 (1.04

(

5 a20 i

10

a20

1.05

j

)

10

) = 7.4%

268

http://actuary88.com Problem 11 (May 2000 EA-1 #10) Issue date of a bond: 1/1/1994 Term of bond; 15 years Par value of bond: $10,000 Coupons: 8% per year, paid on June 30 and December 31 Amortized value on July 1, 2001: $13,741.11 Amortized value on January 1, 2002: $13,629.67 Calculate the redemption amount to be paid upon maturity. Solution Cash flows

$400

Date

1/1/1994 6/30/1994 2/31/1994

Time t (6months)

0

1

$400

$400 …

2

$400

$400

6/30/2001 12/31/2001 15 16

$400 X + 400 …

1/1/2009 30

13, 741.11 = 400 a15 i + X v15 13,629.67 = 400 a14 i + X v14

Let i represent the 6-month effective interest rate; X represent the redemption amount at maturity. Treat this bond as a loan. Then the coupons are just annual repayments of the loan made by the borrower. The amortized value is just the loan outstanding balance. At t = 15 , the loan balance is $13,741.11. This amount accumulates to 3, 741.11(1 + i ) at t = 16 , at which time a loan repayment of $400 is made. Then the loan balance becomes $13,629.67.

13, 741.11(1 + i ) 400 = 13, 629.67 , 13, 741.11 = 400 a15 i + X v15 ,

i = 2.1%

X = 11,800 (using TVM)

Alternatively, 13, 629.67 = 400 a14 i + X v14 ,

X = 11,800 (using TVM)

269

http://actuary88.com Problem 12 (May 2000 EA-1 #11) Issue date of a bond: 1/1/2001 Coupon dates: 12/31/2002 and every two years thereafter, with the final payment on 12/31/2010. Coupon amount: $60 each. Investor’s yield: 8% year annum Price of the bond at issue: $691.49 Amortized value on 1/1/2005: $A Amortized value on 1/1/2007: $B Calculate A B . Solution Cash flow $60 60 60 60 60 + F Date 1/1/2001 12/31/2002 12/31/2004 12/31/2006 12/31/2008 12/31/2010 Time t (2 years) 0 1 2 3 4 5

$691.49

A

B

Here the unit time is 2 years. The effective interest rate per period is: i = 1.082 1 = 16.64% We’ll treat this bond as a loan. Then the amortized value of the bond is the outstanding balance of the loan. We’ll use the respective method. Under this method, the outstanding balance at t = 2 immediately after coupon payment is the accumulated value of the original loan balance less the accumulated value of the coupon payments:

A = 691.49 (1 + i )

2

60 (1 + i ) + 60 = 691.49 (1.1664 )

2

60 (1.1664 ) + 60

= 810.7805110

Similarly, B = A (1 + i ) 60 = 810.7805110 (1.1664 ) 60 = 885.6943881 A B = 74.91

270

http://actuary88.com Problem 13 (May 2002 EA-1 #9) Face value of a bond: $1,000 Redemption value: $1,050 Time to maturity: 10 years Coupon rate: 9% per year, convertible semi-annually Yield rate: 10.25% per year The bond is not callable. Calculate the increase in the book value of the bond during the 3rd year. Solution Cash flows Time t (6 months)

0

$45

$45

$45

$45

$45

$45

$45

$45+1,050

1

2

3

4

5

6



20

P4 = 45 a16 i + 1, 050 v16 P6 = 45 a14 i + 1, 050 v14

The 6-month effective interest rate is i = (1 + 10.25% )

0.5

1 = 5% .

The book value of the bond immediately after the 4th coupon payment is: P4 = 45 a16 i + 1, 050 v16 = 968.7267

The book value of the bond immediately after the 6th coupon payment is: P6 = 45 a14 i + 1, 050 v14 = 975.7602

The increase in the book value of the bond during the 3rd year is: P6

P2 = 975.7602 968.7267 = 7.043

271

http://actuary88.com

Chapter 10

Valuation of stocks

Key points: •

If you own a stock, you are entitled to receive future dividends. In addition, you can sell the stock in the future.



You can think of a stock as a series of cash flows: Time t

0

1

D1

Cash flow

2



D2



T



DT + PT

P0 •

The price of the stock is equal to the PV of future cash flows. P0 = P1 =

D1 + P1 (assume you buy the stock at t=0 and sell it at t=1) 1+ i

D2 + P2 (assume the next owner buys the stock at t=1 and sells 1+ i it at t=1)

P0 =

P2 =

D1 P D D +P + 1 = 1 + 2 21 1 + i 1 + i 1 + i (1 + i )

D3 + P3 (assume the next owner buy the stock at t=2 and sell it 1+ i at t=3)

We can continue this line of thinking.

P0 =

t =+ Dt D1 D2 DT + + ... + + ... = T t 2 1 + i (1 + i ) t =1 (1 + i ) (1 + i )

Stock price = PV of future dividends.

272

http://actuary88.com •

Constant growth model Time t

0

1

D1

Cash flow

2



T

… T 1

D2 = D1 (1 + g )

DT = D1 (1 + g )

P0 Dividend paid at the end of Year 1 = D1

Dividend paid at the end of Year 2 = D2 = D1 (1 + g ) Dividend paid at the end of Year 3 = D3 = D1 (1 + g )

2

…… T 1

Dividend paid at the end of Year T= DT = D1 (1 + g )

If the interest rate r is greater than the dividend growth rate g T 1

D (1 + g ) D (1 + g ) D P0 = 1 + 1 + ... + 1 2 T 1+ i (1 + i ) (1 + i )

+ ... =

D1 r g

Problems Problem 1 Stock price Dividend at the end of Year 1 Dividends growth rate per year forever

$50 $2 8%

Calculate the return expected by investors. Solution P0 =

D1 r g

r=g+

D1 2 = 8% + = 12% P0 50

So investors are expecting 12% return annually.

273

http://actuary88.com Problem 2 Stock purchase date

3 months before the next dividend is due $2 per share Annual 5% per year in perpetuity

Next dividend Dividend payment Dividend growth rate

Calculate the purchase price of the stock if investors want an 8% return annual effective. Solution Method #1 Time t (year) cash

2 a 1.05

0

0.25

1 $2

2 2*1.05

3 2*1.052

… …

! 2*1.05!-1

2 2 = = 66.66667 (geometric annuity shortcut) j 1.05 j 1.05 ( 2.85714% ) 8% 5% where j = = 2.85714% 1 + 5% =

The purchase price of the bond at t = 0.25 (3 month before the next dividend date) is: 66.66667 (1.080.25 ) = 67.96 Method #2

Time t (year) cash

0

0.25

1 $2

..

2a

j

=

2 = dj 1

2 2*1.05 2 1 1 + 2.85714%

3 2*1.052

… …

! 2*1.05!-1

= 72

Then the price of the bond at t = 0.25 is: 72(1.08 – 0.75)=67.96

274

http://actuary88.com

Chapter 11 Price of a bond sold between two coupon payments Exam FM Sample Questions have one question (#50) on this topic. Procedure to determine the market price sold between coupon dates: • Determine the purchase price of the bond assuming the 2nd owner has 100% ownership of the next coupon •

Determine the accrued interest



Determine the market price of the bond using the following equation

Bond (quoted) market price = Bond purchase price (assuming 100% ownership of the next coupon) - Accrued interest as of the last coupon date ExplanationTo calculate the bond’s market price, we first ignore the fact that the original owner deserves a portion of the next coupon. We pretend that the 2nd owner possesses 100% of the next coupon (plus all the other cash flows). We then calculate the purchase price of the bond assuming the 2nd owner has 100% of the next coupon. The purchase price of a bond (assuming 100% ownership of the next coupon) = PV of the bond’s future cash flows (including the next coupon) discounted at the market interest rate. Remember, whenever we discount a cash flow, we implicitly assume the 100% ownership of this cash flow. If the ownership is not 100%, we can NOT discount this cash flow by its full amount. Next, we consider the fact that the original owner deserves a fraction of the next coupon. If a bond is sold between two coupon dates, the buyer of the bond must compensate the seller for the fraction of the next coupon payment the seller deserves but will not receive. This amount is called accrued interest.

275

http://actuary88.com There are different ways of calculating the accrued interest. Let’s not worry about it now. Finally, we calculate the real value of the bond by subtracting the accrued interest from the purchase price of the bond. The logic here is simple. The purchase price of the bond is overstated; it’s calculated on the assumption that the 2nd owner gets the next coupon 100% and the original owner gets none. Since the 2nd buyer must pay the original owner the accrued interest, we need to deduct the accrued interest from the purchase price of the bond. Now you know that the market price of a bond sold between two coupon dates is the PV of bond’s future cash flows minus the accrued interest. However, the actual calculation is messy because the next coupon occurs at a fractional time. We’ll use the following steps to calculate the market price of the bond: Step #1 k=

Find the fractional time days between settlement date and last coupon date a = b Total days per coupon period b

a

Last coupon date

Settlement date (i.e. Bond purchase date)

Next coupon date

276

http://actuary88.com We need to use one of the two day count methods to calculate k . Day count method 360/360

Actual/Actual

Explanation

Example

When used

Assume every month has 30 days and every year has 360 days. Use the actual # of days in a month.

The # of days between 2/1/2005 and 3/1/2005 is 30 days. The # of days between 2/1/2005 and 3/1/2005 is 28 days.

For municipal and corporate bonds For treasury bonds

In Exam FM, if a problem doesn’t specify which day count method to use, use Actual/Actual method. Step #2

Calculate the PV of the bond’s future cash flows.

Time zero is the last coupon payment date. Time t

0

k

1

CF (1)

Cash flow

PVx =

n t =1

CF ( t ) vt

2

3

CF ( 2 ) k

=

n t =1

CF ( 3) CF ( t )

(1 + i )

t k

t

……

=



n

CF ( t ) … CF ( n )



CF (1)

(1 + i )

1 k

+

CF ( 2 )

(1 + i )

2 k

+ ... +

CF ( n )

(1 + i )

n k

277

http://actuary88.com If we don’t like fractional discounting periods such as 1 k , 2 k , … , we can calculate the PV of the future cash flows at t=1 and then discount the this PV to t= k (see the diagram below): Time zero is the last coupon payment date. Time t

0

k

1

2

CF (1)

Cash flow

3

CF ( 2 )

PV1 = CF (1) +

CF ( 3)

……



t



n

CF ( t ) … CF ( n )

CF ( 2 ) CF ( 3) CF ( n ) + + ... + 2 n 1 1+ i (1 + i ) (1 + i )

PV1 = Bond's book value @ the next coupon date PVk =

PV1

(1 + i )

1 k

= ( Bond's book value @ the next coupon date ) v1

k

Alternatively, we can calculate the PV at t=0 by discounting all future cash flows to t=0. Next, we accumulate this PV at t=0 to t= k (see the diagram below): Time t

0

Cash flow

k

1

CF (1)

2

CF ( 2 )

PVk = PV0 (1 + i )

3

CF ( 3)

……



t



n

CF ( t ) … CF ( n )

k

= ( Bond's book value @ the last coupon date )(1 + i ) PV0 =

k

CF (1) CF ( 2 ) CF ( n ) + + ... + n = Bond's book value @ the last coupon date 2 1+ i (1 + i ) (1 + i )

278

http://actuary88.com Step #3 k=

Calculate the accrued interest. days between settlement date and last coupon date a = Total days per coupon period b b

a

Last coupon date

Settlement date (i.e. Bond purchase date)

Accrued Interest = k × coupon = k ( Fr ) =

Step #4

Next coupon date

a ( Fr ) b

Calculate the (quoted) market price.

Market Price = Purchase price of the bond at settlement – accrued interest. The method outlined in the above four steps is the method most often used in the real world. Broverman described this method in his textbook, except he didn’t mention the day count method (perhaps to keep the concept simple). Kellison, however, described a myriad of ways to calculate the bond market price in Table 7.4 (page 223). Flat price (i.e. the bond purchase price)

Accrued interest

Market price

( Fr ) sk

Bt (1 + i )

Theoretical method Practical method

Bt (1 + i )

Bt (1 + k i )

k ( Fr )

Bt (1 + k i ) k ( Fr )

Semi-theoretical method

Bt (1 + i )

k ( Fr )

Bt (1 + i )

k

k

i

k

k

( Fr ) sk

i

k ( Fr )

Under the theoretical method, sk (where k is a fraction) doesn’t have an intuitive explanation. However, SOA loves the complex concept of sk . As

279

http://actuary88.com a result, you need to memorize the theoretical method of calculating the accrued interest rate. Please note that TVM Worksheet in BA II Plus/BA II Plus Professional allows N (the # of compounding periods) to be a fraction. So you can use TVM to calculate sk even when k is a fraction.

Let’s look at Kellison’s method: Flat price (i.e. the bond purchase price)

Accrued interest

Theoretical method

Compounding interest

Compounding interest

Practical method Semi-theoretical method

Simple interest Compounding interest

Simple interest Simple interest

So under Kellison’s method, both the flat price and the accrued interest can be calculated using either a simple interest rate or a compounding interest rate. There is a total of three methods to calculate the market price – theoretical, practical, and semi-theoretical. Because Kellison’s three methods are in the syllabus, you might want to memorize his three methods. Example 1 Bond face amount Coupon rate Coupon payments Issue date Term to maturity Date when bond is resold Yield to maturity to the 2nd buyer

$100 4% semiannual July 1 and December 31 1/1/2000 12/31/2004 5/1/2002 6% semiannual

What’s the market price of the bond at the settlement date assuming the 2nd buyer has 100% ownership of the next coupon and all the other cash flows? Use a compounding interest rate

280

http://actuary88.com Solution

Unit time = 0.5 year The interest rate per coupon period i = 12/31 2001 Time t

5/1 7/1 2002 2002

x

0

PV0

PVX

6% = 3% 2

12/31 2002

1

2

$2

$2

12/31 2004 ……

6

$2 $100

PV1

We’ll discount future cash flows occurring at t=1,2, …, 6 to t=x at a discount rate of 3% per coupon period. The PV of these cash flows is the purchase price of the bond at t=x if we ignore that fact that the previous owner deserves any portion of the next coupon.

PVx =

2

(1.03)

1 x

+

2

(1.03)

2 x

+ ... +

(

PVx = 1.03x 2a6 3% + 100v 6

102

(1.03)

6 x

= 1.03x

2

(1.03)

1

+

2

(1.03)

2

+ ... +

102

(1.03)

6

)

Next, we need to find x . x=

days between settlement date and last coupon date Total days per coupon period

Method #1 – 360/360 day count method (for municipal and corporate bonds) Under this method: # of days between 12/31/2001 and 5/1/2002 is 121 days. 281

http://actuary88.com # of days between 12/31/2001 and 7/1/2002 is 181 days. How do we actually count the days? Though we can do the math ourselves assuming a month has 30 days, we will let BA II Plus/BA II Plus Professional Date Worksheet count the days for us. Key strokes in BA II Plus/BA II Plus Professional Date Worksheet for calculating the # of days between 12/31/2001 and 5/1/2002: Procedure Select Date Worksheet Clear worksheet Enter 1st date Enter 2nd date Choose 360/360 day count method Compute days between dates

Keystrokes 2nd [Date] 2nd [CLR Work] 12.3101 5.0102 2nd Set

Display DT1= (old content) DT1 = 12-31-1990 DT1 = 12-31-2001 DT2 = 5-01-2002 360 DBD = 121

CPT

So there are 121 days between 12/31/2001 and 5/1/2002 under 360/360 day count method. Similarly, we find that there are 181 days between 12/31/2001 and 7/1/2002 under 360/360 day count method. x=

days between settlement date and last coupon date 121 = Total days per coupon period 181

(

)

121

(

PVx = 1.03x 2a6 3% + 100v 6 = 1.03181 2a6 3% + 100v 6

)

2a6 3% + 100v 6 = 94.58280856 (using TVM)

(

)

121

PVx = 1.03x 2a6 3% + 100v 6 = 1.03181 ( 94.58280856 ) 96.47038172

Other methods to calculate PVx ; Alternative Method A PVx = 1.03x PV0

PV0 = 2a6 3% + 100v 6

(

PVx = 1.03x 2a6 3% + 100v 6

)

96.47038172

282

http://actuary88.com Alternative Method B

PVx = v1 x PV1 PV1 = 2a6 35 + 100v5 PVx = v1 =v

x

x

( 2a

( 2a

6 35

6 3%

)

+ 100v 5 = v

)

x

(

(

v 2a6 35 + 100v 5

+ 100v 6 = 1.03 x 2a6 3% + 100v 6

)

) 96.47038172

Method #2 – Actual/Actual day count method (for Treasury bonds) Since SOA doesn’t allow us to bring a calendar in the exam room, we can NOT look up a calendar and count the actual days. Once again, we’ll let BA II Plus/BA II Plus Professional do the work for us. Key strokes in BA II Plus/BA II Plus Professional Date Worksheet for calculating the # of days between 12/31/2001 and 5/1/2002: Procedure Select Date Worksheet Clear worksheet Enter 1st date Enter 2nd date Choose 360/360 day count method Compute days between dates

Keystrokes 2nd [Date] 2nd [CLR Work] 12.3101 5.0102 2nd Set

CPT

Display DT1= (old content) DT1 = 12-31-1990 DT1 = 12-31-2001 DT2 = 5-01-2002 ACT

DBD = 121

So there are 121 days between 12/31/2001 and 5/1/2002 under Actual/Actual day count method. Similarly, we find that there are 182 days between 12/31/2001 and 7/1/2002 under Actual/Actual day count method. x=

days between settlement date and last coupon date 121 = Total days per coupon period 182

In this problem, there is little different between the two day count methods. However, in some problems, the difference between the two methods can be bigger. Finally, we are ready to calculate the purchase price of the 2nd buyer:

283

http://actuary88.com

PVx

121 182 = 1.03

( 2a

6 3%

+ 100v 6

)

96.45990820

An exam problem may not tell you which date count method to use. In that case, use Actual/Actual method. Example 2 Bond face amount Coupon rate Coupon payments Issue date Term to maturity Date when bond is resold Yield to maturity to the 2nd buyer Day count

$100 4% semiannual July 1 and December 31 1/1/2000 12/31/2004 5/1/2002 6% nominal compounding semiannually Actual/Actual

What’s the market price of the bond at the settlement date assuming the 2nd buyer has 100% ownership of the next coupon and all the other cash flows? Use a simple interest rate. Solution

Unit time = 0.5 year The interest rate per coupon period i = 12/31 2001 Time t

0

PV0

5/1 7/1 2002 2002

x

PVX

6% = 3% 2

12/31 2002

1

2

$2

$2

12/31 2004 ……

6

$2 $100

PV1

284

http://actuary88.com We already calculated that under Actual/Actual x=

days between settlement date and last coupon date 121 = Total days per coupon period 182

To find the bond purchase price, we first we discount the future cash flows to t=0 (the prior coupon date):

B 0 = PV0 = 2a6 3% + 100v 6

Next, we accumulate B 0 to time x using a simple interest rate:

(

Bx = PVx = (1 + 0.03 x ) PV0 = (1 + 0.03 x ) 2a6 3% + 100v 6

121 182 121 Bx = PVx = 1 + 3% × 182 Bx = PVx = 1 + 3% ×

( 2a

6 3%

+ 100v 6

)

)

( 94.58280856 ) = 96.46926787

Example 3 Bond face amount Coupon rate Coupon payments Issue date Term to maturity Date when bond is resold Yield to maturity to the 2nd buyer Day count

$100 4% semiannual July 1 and December 31 1/1/2000 12/31/2004 5/1/2002 6% nominal compounding semiannually Actual/Actual

Calculate the market price of the bond immediately after the bond is sold on 5/1/2002, under the following methods: • Theoretical method • Practical • Semi-theoretical method

285

http://actuary88.com Solution Unit time = 0.5 year The interest rate per coupon period i = 12/31 2001 Time t

5/1 7/1 2002 2002

x

0

6% = 3% 2

12/31 2002

1

2

$2

$2

12/31 2004 ……

6

$2 $100

We already calculated that under Actual/Actual day count, x=

days between settlement date and last coupon date 121 = Total days per coupon period 182

Theoretical 121

(

Bx = PVx = 1.03x PV0 = 1.03182 2a6 3% + 100v 6

AI = ( coupon ) sx = 2 sx = 2

(1 + 3% )

121 182

3%

1

)

96.45990820

= 1.32307317

Alternative method to calculate 2 s x : Enter the following into BA II Plus/BA II Plus Professional TVM: PMT=2, N=

121 , I/Y=3. 182

FV= -1.32307317

Bond market price= purchase price – accrued interest = 96.45990820 1.32307317 = 95.14

286

http://actuary88.com Practical method

(

Bx = PVx = (1 + 0.03 x ) PV0 = (1 + 0.03 x ) 2a6 3% + 100v 6

(

)

)

121 2a6 3% + 100v 6 182 121 = 1 + 3% × ( 94.58280856 ) = 96.46926787 182 121 AI = ( coupon ) x = ( 2 ) = 1.32967033 182 = 1 + 3% ×

Bond market price= purchase price – accrued interest = 96.46926787 1.32967033 = 95.14 Semi-theoretical method 121

(

Bx = PVx = 1.03x PV0 = 1.03182 2a6 3% + 100v 6 AI = ( coupon ) x = ( 2 )

)

96.45990820

121 = 1.32967033 182

Bond market price= purchase price – accrued interest = 96.45990820 1.32967033 = 95.13 The results under the three methods are very close. Reference to SOA problems Exam FM Sample Questions #50 Explanation of SOA’s solution to FM Sample Questions #50. This problem asks for the purchase price, not the (quoted) market price. So there’s no need to subtract the accrued interest. The problem also tells you to use a simple interest between bond coupon dates. First, SOA calculates the bond price P= PV0 =906.32 on the previous coupon date of 4/15/2005 using i ( 2 ) = 7% . Next, using the simple interest rate, we calculate the purchase price of the bond on 6/28 as 74 Bx = PVx = (1 + 0.035 x ) PV0 = 1 + 0.035 × 906.32 = 919.15 . 183

287

http://actuary88.com

Chapter 12 Time weighted return and dollar weighted return Time weighted return •

To calculate the time weighted return of a fund over a time horizon [ 0,t ] , we first break down [ 0,t ] into n sub-periods. These n subperiods can be of equal lengths or unequal lengths.



For example, to calculate the time weighted return of a fund’s performance in a year, we can break down a year into 4 quarters. We can also break down a year into 3 sub-periods: sub-period 1 consists of January, sub-period 2 consists of 4 months from February to May, and sub-period 3 consists of 7 months from June to December. (An SOA problem will tell you the whole period is broken down into how many sub-periods.)



Next, we calculate the return for each sub-period as follows: Rk =Return of k - th sub-period=



We then calculate the time weighted return R for the whole period [ 0, t ] by solving the following equation:

(1 + R ) •

Time t

Ending asset value of k - th sub-period 1 Beginning asset value of k - th sub-period

t

= (1 + R1 )(1 + R2 ) ... (1 + Rn )

The time weighted return is the geometric average of all the subperiods’ returns. t

0

1 + R1 Period 1

1 + R2

….

Period 2

(1 + R )

t

1 + Rn Period n

= (1 + R1 )(1 + R2 ) ... (1 + Rn )

288

http://actuary88.com Dollar weighted return •

To calculate the dollar weighted return of a fund over a time horizon [ 0,t ] , we first find the complete history of all the cash inflows and cash outflows during [ 0,t ] . 1. We need to know how much money the fund has at time zero. We call this CF ( 0 ) . This is the beginning account value. 2. We need to know how much money the fund has at time t . We call this CF ( t ) . This is the ending account value. 3. We need to know what happened in between. If any money is added to or withdrawn from the fund during [ 0, t ] , we need to find out (1) when this happened, (2) how much money is added to or withdrawn from the fund. 4. For example, we record a cash flow of CF ( t1 ) at time t1 where

0 < t1 < t . If CF ( t1 ) is an inflow (i.e. more money is added to the fund), we will make CF ( t1 ) positive; If CF ( t1 ) is an outflow (i.e. money is withdrawn from the fund), we will make CF ( t1 ) negative. •

Next, we translate the fund history during [ 0, t ] into the following cash flow diagram: Time t Cash flow



0

CF ( 0 )



t1 CF ( t1 )



t2 CF ( t2 )



t3 CF ( t3 )



t CF ( t )

Finally, we find the dollar weighted return R by solving the following equation (we assume a simple interest rate R ) CF ( 0 )(1 + tR ) + CF (1) 1 + ( t t1 ) R + CF ( 2 ) 1 + ( t t2 ) R + ... = CF ( t )

289

http://actuary88.com

Problem 1 Date 1/1/2004 3/31/2004 6/30/2004 10/31/2004 12/31/2004

Fun’s balance before deposits and withdrawals $100,000 $120,000 $140,000 $198,000 $220,000

Deposit

Withdrawal

$30,000 $90,000

$50,000 $70,000

Find the fund’s time weighted return and dollar weighted return during 2004. Solution Find the time weighted return

(1 + R )

1

=

120, 000 140, 000 198, 000 220, 000 × × × 100, 000 120, 000 + 30, 000 140, 000 50, 000 198, 000 + 90, 000 70, 000 1/1 3/31

3/31 6/30

6/30 10/31

10/31 12/31

R = 148.66%

We calculate the return from 1/1 to 3/31, the beginning fund value of is $100,000 and the ending value is $120,000. When we calculate the return from 3/31 to 6/30, however, the beginning fund value is $120,000 + $30,000 = $150,000 We add $30,000 because $30,000 fresh money flows into the fund at the end of 3/31, perhaps because the fund has earned 20% return from 1/1 to 3/31. Find the dollar weighted return To calculate the dollar weighted return, we need to know the beginning fund value, the ending fund value, and all the cash flows in-between. However, we don’t need to know the interim fund balances. To simply our calculation, let’s first remove the interim fund balances.

290

http://actuary88.com Date

1/1/2004 3/31/2004 6/30/2004 10/31/2004 12/31/2004

Fun’s balance before deposits and withdrawals $100,000 $120,000 $140,000 $198,000 $220,000

Deposit

Withdrawal

$30,000 $90,000

$50,000 $70,000

In addition, to help us track down the time neatly, we convert a monthend dates to a month-begin date. Specifically, we change 3/31/2004 to 4/1/2004; 6/30 to 7/1; 10/31 to 11/1. We’ll keep 12/31. This way, we know that the distance between 3/31 and 1/1 is the distance between 4/1 and 1/1 (3 months apart). If we don’t convert 3/31 to 4/1, we might incorrectly conclude 3/31 and 1/1 are 2 months apart. Is it OK to convert 3/31 to 4/1? Yes. When we count the fund value on 3/31, we actually count the fund value at the end of 3/31, which is the same as the fund value counted in the beginning of 4/1.

Time t

1/1 1/1

(Year)

0

Cash flow $100,000

3/31 4/1

6/30 7/1

3 12

6 12

10 12

12 =1 12

- $50,000

$90,000 - $70,000

$220,000

$30,000

10/31 11/1

12/31 12/31

Next, we set up the following equation: 100, 000 (1 + i ) + 30, 000 1 + 1

100, 000 (1 + i ) + 30, 000 1 +

i=

3 i 12 9 i 12

50, 000 1 + 1

50, 000 1 +

6 10 i + ( 90,000 70, 000 ) 1 + 1 i = 220, 000 12 12

6 2 i + 20, 000 1 + i = 220, 000 12 12

220, 000 100, 000 30, 000 + 50, 000 20, 000 = 119% 9 6 2 + 20, 000 100, 000 + 30, 000 50, 000 12 12 12

So the dollar weighted return is 119%.

291

http://actuary88.com In the above expression, 220, 000 100, 000 30, 000 + 50, 000 20, 000 = 120, 000 is the 9 12

total interest earned in 2004. 100, 000 + 30, 000

50, 000

6 2 + 20,000 12 12

is the

total principal that generated the $120, 000 interest. The ratio represents the interest earned during 2004. In Exam FM, SOA wants us to calculate the time weighted return using a simple interest rate; we shall do so in the exam. In the real world, however, the standard practice is to calculate the time weighted return using a compound interest rate. Let’s calculate the time weighted return using a compound interest rate. We set up the following equation: 1 3 12

100,000 (1 + i ) + 30,000 (1 + i )

1 6 12

50,000 (1 + i ) 9

1 10 12

+ ( 90,000 70,000 )(1 + i )

6

= 220,000

2

100,000 (1 + i ) + 30,000 (1 + i )12 50,000 (1 + i )12 + 20,000 (1 + i )12 = 220,000 We can’t solve this equation manually; we need to use BA II Plus/BA II Plus Professional Cash Flow Worksheet. Because BA II Plus/BA II Plus Professional Cash Flow Worksheet can not use fractional time (all cash flow times need to be entered as a nonnegative integer), we will use a month as the unit time and calculate the monthly effective interest rate r . Time 0

1

2

3

$100,000

4

5

$30,000

7

8

9

10

- $50,000

100,000 (1 + r ) + 30,000 (1 + r ) 12

100,000 + 30,000 (1 + r )

6

9

3

$20,000

11

12

- $220,000

50,000 (1 + i ) + 20,000 (1 + i ) = 220,000 6

50,000 (1+ i ) + 20,000 (1 + i ) 6

2

10

220,000 (1+ i )

12

=0

To solve the above equation, we enter the following into Cash Flow Worksheet:

292

http://actuary88.com To speed up our calculation, we use $1,000 as the unit money. If a cash flow is $30,000, we enter 30. Try to learn this technique in the exam. Cash flow diagram (set $1,000 as one unit of money): Time 0

1

2

3

$100

4

5

30

Cash Flow Worksheet CF0 = 100 C01=0, F01=2 C02=30

C03=0, F03=2 C04= - 50 C05=0, F05=3 C06=20 C07 =0 C08= - 220

6

7

- 50

8

9

10

20

11

12

- 220

1st cash flow at t=0 is $100. Indicate cash flows at t=1, 2 are zero. If a cash flow is zero, still need to press “Enter.” Indicate cash flow at t=3 is 30. No need to set F02=1; Cash Flow Worksheet automatically use 1 as the default cash flow frequency. Indicate cash flows at t=4, 5 are zero Indicate cash flow at t=6 Indicate cash flows at t=7, 8, 9 are zero Indicate cash flow at t=10 Indicate cash flows at t=11 is zero Indicate cash flow at t=12

Let Cash Flow Worksheet solve for IRR. Press “IRR” “CPT.” IRR=6.6794% Finally, we convert this monthly effective rate into an annual effective rate:

(1 + 6.6794 )

12

1 117.25%

Problem 2 A fund has the following transactions: Date 1/1/2000 12/31/2001 1/1/2002 1/1/2003

Account Value (in millions) 1.35 X

Contribution (in millions)

3.95 5.8

293

http://actuary88.com For the period from 1/1/2000 to 1/1/2003, the dollar weighted return using the compound interest rate and the time weighted return are the same. Calculate X

Solution Dollar weighted return When calculating the dollar weighted return, we ignore the interim account values. So X is not needed.

Time

0

1

1/1/2000

1/1/2001

1.35

2

1/1/2002 3.95

3

1/1/2003 5.8

1.35 (1 + i ) + 3.95 (1 + i ) = 5.8 3

This is a difficult equation to solve. Fortunately, we can use the IRR functionality in BA II Plus or BA II Plus Professional Cash Flow Worksheet. Multiplying both sides 1.35 (1 + i ) + 3.95 (1 + i ) = 5.8 with v3 = (1 + i ) , we have: 3

3

1.35 + 3.95v 5.8v3 = 0

This equation translates to the following cash flow diagram: Time

0

1/1/2000 1.35

1

1/1/2001

2

1/1/2002 3.95

3

1/1/2003 - 5.8

Please note the cash flow at t = 3 is a negative 5.8. 294

http://actuary88.com

Enter the following into Cash Flow Worksheet: CF0 C01 C02 C03 1.35 0 3.95 -5.8 F01 F02 F03 1 1 1 Press “IRR” “CPT.” You should get: IRR=6.06031163 So the dollar weighted return using a compounding interest rate is: i = 6.06%

Time weighted return Come back to the table. Account Value (in millions) 1.35 X

Date 1/1/2000 12/31/2001 1/1/2002 1/1/2003

X 1.35 1/1/2000 to 12/31/2001

Contribution (in millions)

3.95 5.8

×

5.8 X + 3.95

= (1 + j )

3

1/1/2002 to 1/1/2003

We are told that j = i = 6.06%

X 5.8 = 1.063 , X = 1.515 1.35 X + 3.95

Problem 3 (SOA May 2005 EA-1 #1) A fund has the following transactions Date

Account Value Contributions Benefits payments C1 1/1/2005 $1,000 0 3/31/2005 0 $100

295

http://actuary88.com 4/1/2005 12/31/2005 1/1/2006

$1,300

0 C2

$1,700

0 $150 0

0

The time weighted rate of return in 2005 is 6.25%. The dollar weighted rate of return in 2005 is 6.00%. In what (A) (B) (C) (D) (E)

range is C1? Less than $405 $405 but less than $430 $430 but less than $455 $455 but less than $480 $480 or more

Solution This problem is simple conceptually, but it involves messy calculations. Time weighted return We break down Year 2005 into the two segments: [1/1, 3/31], [4/1, 12/31]. Segment Begin acct value

1/1 to 3/31 1, 000 + C1

Ending acct value

1,300+100=1,400(1)

4/1 to 12/31 1,300 1, 700 + 150 C2 = 1,850 C2 (2)

(1) On 4/1, the account value is $1,300. Because $100 is taken out on 3/31, the account value on 3/31 is 1,400. (2) On 1/1/2006, the account value is $1,700. On 12/31/2005, C 2 amount of new money comes in and $150 flows out. So the account value on 12/31/2005 is 1, 700 + 150 C2 = 1,850 C2 . We are given that the time weighted return during 2005 is 6.25%. 1, 400 1,850 C2 × = 1.0625 1, 000 + C1 1,300

(Equation 1)

Dollar weighted return When calculating the dollar weighted return, we need to throw away the intermediate account values. So we don’t need to use the account value

296

http://actuary88.com on 4/1/2005. We only care about the account value on 1/1/2005; the account value on 12/31/2005; and the money that flows in or comes out during Year 2005. The Account value on 1/1/2005 is 1, 000 + C1 . This earns a full year interest. On 3/31/2005, $100 flows out; this earns 9 months negative interest. The account value on 12/31/2005 is 1, 700 + 150 C2 = 1,850 C2 . We are told that the timed weighted return during 2005 is 6%:

(1, 000 + C1 )1.06

100 1 +

9 × 6% = 1,850 C2 12

(Equation 2)

Solving these two equations, we have: C1 = 423.84 , C2 = 445.23 So the correct answer is B. Problem 4 (SOA May 2004 EA #10) Date 1/1/2004 4/1/2004 8/1/2004 12/31/2004

Market value of fund $100,000 $85,000 $100,000 $80,000

Contributions None $30,000 None None

Withdrawals None None $20,000 None

Market value of the fund is prior to contributions and withdrawals. A = Time weighted return B = Dollar weighted return

In what range is the absolute value of A + B ? (A) Less than 45% (B) 45% but less than 48% (C) 48% but less than 51% (D) 51% but less than 54% (E) 54% or more Solution

297

http://actuary88.com To speed up our calculation, we use $1000 as one unit of money. Then the original table becomes:

Date 1/1/2004 4/1/2004 8/1/2004 12/31/2004

Market value of fund $100 $85 $100 $80

Contributions None $30 None None

Withdrawals None None $20 None

Time weighted return

85 100

100 85 + 30

×

1/1/2004 to 4/1/2004

80 100 20

×

4/1/2004 to 8/1/2004

= 1+ i

8/1/2004 to 12/31/2004

i = 26.09%

Dollar weighted return We first delete the interim fund values: Date 1/1/2004 4/1/2004 8/1/2004 12/31/2004

100 (1 + j ) + 30 1 +

Market value of fund $100 $85 $100 $80

9 j 12

20 1 +

Contributions None $30 None None

5 j = 80 , 12

Withdrawals None None $20 None

j = 26.28%

A + B = i + j = 26.09% 26.28% = 52.37%

So the answer is D. Problem 5 (SOA May 2003 EA-1 #5)

298

http://actuary88.com All assets of a pension plan are invested by manager Smith and manager Jones. There are no other plan assets. The following chart shows t market value of the plan’s assets with each manager:

Balance Contribution Balance Transfer Balance

Date 12/31/2001 1/1/2002 6/30/2002 7/1/2002 12/31/2002

Smith $2,500,000 $0 $2,800,000 $1,000,000 $4,180,000

Jones $2,500,000 $1,500,000 $4,500,000 ($1,000,000) $3,500,000

X = one-half of the sum of both manager’s time-weighted percentage returns for 2002. Y = dollar weighted percentage return for 2002 for the entire pension plan. Z =Y

X

In what range is Z ? (A) Less than 0.09% (B) 0.09% but less than 0.18% (C) 0.18% but less than 0.27% (D) 0.27% but less than 0.36% (E) 0.36% or more Solution We’ll use $1,000,000 as one unit of money. Doing so will greatly simplifies our calculation and reduces chances of errors. You should learn this technique. Then the table is simplified as follows: Date Smith Balance 12/31/2001 $2.5 Contribution 1/1/2002 $0 Balance 6/30/2002 $2.8 Transfer 7/1/2002 $1 Balance 12/31/2002 $4.18

Jones $2.5 $1.5 $4.5 ($1) $3.5

Time-weighted return Smith: 299

http://actuary88.com

2.8 2.5

×

12/31/2001 to 6/30/2002

4.18 2.8 + 1

= 1+ i ,

i = 23.2%

= 1+ j ,

i = 12.5%

7/1/2002 to 12/31/2002

Jones: 4.5 2.5 + 1.5

×

12/31/2001 to 6/30/2002

X=

3.5 4.5 1 7/1/2002 to 12/31/2002

1 1 ( i + j ) = ( 23.2% + 12.5% ) = 17.85% 2 2

Dollar weighted return for the entire pension plan We combine the transactions by Smith and by Jones into one: Entire Pension Plan Date Smith Jones (Smith + Jones) Balance 12/31/2001 $2.5 $2.5 $5 Contribution 1/1/2002 $0 $1.5 $1.5 Balance 6/30/2002 $2.8 $4.5 $7.3 Transfer 7/1/2002 $1 ($1) $0 Balance 12/31/2002 $4.18 $3.5 $7.68 The interim balance of $7.3 on 6/30/2002 is not needed for the calculation.

5 (1 + k ) + 1.5 (1 + k ) = 7.68 , Z =Y

k = 18.15% ,

Y = k = 18.15%

X = 18.15% 17.85% = 0.3%

So the answer is D. Problem 6 (SOA May 2000 EA-1 #17) Market value of a pension fund: Date Value 1/1/2000 $50,000 3/31/2000 $60,000 6/30/2000 $45,000 9/30/2000 $40,000 300

http://actuary88.com 12/31/2000

$65,000

Contribution and benefit payments: Date 4/1/2000 7/1/2000 10/1/2000

Contributions $0 $17,000 $55,000

Benefit Payments $P P P

Dollar-weighted rate of return for 2000 using simple interest: 7%. In what range is the time weighted rate of return for 2000? Solution We’ll use $1,000 as one unit of money. First, let’s combine the two tables into one. Date 1/1/2000 3/31/2000 4/1/2000 6/30/2000 7/1/2000 9/30/2000 10/1/2000 12/31/2000

Value $50 $60

Contributions

Benefit Payments

$0

$P

$17

P

$55

P

$45 $40 $65

Dollar weighted return 50 (1 + i ) P 1 +

9 6 3 i + (17 P ) 1 + i + ( 55 P ) 1 + i = 65 12 12 12

We are given: i = 7% . 50 (1.07 ) P 1 +

9 6 3 × 0.07 + (17 P ) 1 + × 0.07 + ( 55 P ) 1 + × 0.07 = 65 12 12 12

Solving the above equation, we get: P = 19.986

20

Time weighted return

301

http://actuary88.com 60 50

45 60 P

×

1/1 to 3/31

4/1 to 6/30

×

40 45 + 17 P 7/1 to 9/30

×

65 40 + 55 P

= 1+ j

10/1 to 12/31

60 45 40 65 × × × 1.11 50 60 20 45 + 17 20 40 + 55 20 j = 11%

So the answer is D. Problem 6 (FM May 2005 # 16) At the beginning of the year, an investment fund was established with an initial deposit of 1000. A new deposit of 1000 was made at the end of 4 months. Withdrawals of 200 and 500 were made at the end of 6 months and 8 months, respectively. The amount in the fund at the end of the year is 1560. Calculate the dollar-weighted (money-weighted) yield rate earned by the fund during the year. (A) 18.57% (B) 20.00% (C) 22.61% (D) 26.00% (E) 28.89% Solution 1000 (1 + i ) + 1000 1 +

8 i 12

200 1 +

6 i 12

500 1 +

4 i = 1560 12

i = 18.571%

So the answer is A.

302

http://actuary88.com

Chapter 13 Investment year & portfolio method SOA problems on this topic tend to be simple and straightforward. Here are supplemental explanations. 1. When insurance companies set their premium rates, they do so after taking into the account that premiums collected are invested somewhere earning interests. If premium dollars can be invested prudently and earn a higher interest rate, premium rates will be lower and agents can sell the insurance products more easily. Conversely, if premiums earn a lower interest rate, then premium rates will be higher and the insurance products will be less competitive in the market place. 2. Insurance companies (especially life insurance companies) often declare two interest rates, the guaranteed interest rate and the current interest rate. The guaranteed interest rate is set permanently and can not be changed. The current interest rate is the actual interest rate credited to the policyholder and is adjusted on the on-going basis. 3. For example, a whole life insurance policy may have a guaranteed interest rate of 3% and the current interest rate of 5.5%. •

The guaranteed 3% means that the policyholder will earn at least 3% per year no matter what.



The guaranteed interest rate is written in the insurance contract; once set, it can not be changed.



This guaranteed rate is just the lowest interest possible. It is written in the contract to protect the insurance company against really bad investment experiences. Unless the investment experience is really bad, the insurance company doesn’t really use this rate.



The current interest rate of 5.5% is the rate actually credited to the policyholder.



Insurance companies change their current rates periodically (ex every 6 months or every year) to be line with the prevailing market interest rate.

303

http://actuary88.com 4. Potential buyers of insurance products often want to know the interest rate earned by their premiums. They often shop around and compare interest rates earned by premium dollars. 5. When setting the on-going current interest rates periodically, the insurance company often uses one of the two methods: the portfolio method and the investment year method. The portfolio method •

Assets are combined for different products (ex. whole life and universal life), or different time periods when premiums come to the insurance company (ex. all the premiums collected last year, whether in the 1st quarter or other quarters).



A single interest rate is used for different products, different periods of time.



Simple for the insurance company’s IT department to manage. For example, the IT department does not need to use complex software to keep track of when premiums come in and premiums are for what product.

The investment year method (or the new money method) •

Assets are segmented for different products, or different time periods during which funds are received. Different interest rates are used.



Disadvantage - IT department needs to use complex software to keep track of when new funds come in and new funds are for what products.



Advantage – The interest rate credited to premiums or other funds depends when premiums or funds come in. If the interest rate is higher when new premiums come to the insurance company, new premiums will automatically get the higher interest rate.

SOA problems on the portfolio method and the investment year method are simple, requiring candidates to look up the correct interest rate from a given interest rate table. Please refer to Sample FM #8 to learn how to solve this type of problems.

304

http://actuary88.com

Chapter 14

Short Sales

Short sales of stocks are covered in two areas: (1) theory of interest textbooks (such as Sam Broverman’s textbook Mathematics of Investment and credit section 8.2.2), and (2) Derivatives Markets (section 1.4). According to the SOA’s syllabus, you just need to short sales as explained in Derivatives Markets. You don’t need to worry about short sales as explained in any of the four recommended textbooks for the theory of interest. As a matter of fact, SOA removed Sample FM #38, #39, and #40. All these problems are short sales of stocks as explained in the theory of interest textbooks. This chapter is not on the syllabus. However, I include this chapter to help you better understand Derivatives Markets section 1.4. If you understand Derivatives Markets section 1.4, that’s great and you can just skip this chapter. Company A’s stocks are currently selling $100 per share. You have a good reason to believe that this stock price is not going to last long and will go down soon. Perhaps you heard the rumor that Company A uses aggressive accounting to exaggerate its sales. Or perhaps you learned from the Wall Street Journal that Company A’s top management is incompetent. Or perhaps you found that one of Company A’s chief competitors has just designed a revolutionary product which will make Company A’s main product line obsolete. For whatever reason, you believe that Company A’s stock price will go down in the very near future, say in 15 days. How can you make money on Company A’s stocks based on your analysis? You can make money this way. You can use an internet brokerage firm to sell short 1,000 shares of Company A stocks. Or if you prefer traditional brokerage firm, you can ask your broker to sell 1,000 shares of Company A stocks short for you. Selling short means that you sell something you don’t have. Currently you don’t own any stocks from Company A, but you want to sell 1,000 shares of Company A stocks. So you are selling short. Essentially, you are borrowing 1,000 shares from a brokerage firm and sell them in the open market. Just as you have predicted, 15 days later, the price of Company A’s stocks drops to $80 per share. You buy 1,000 shares of Company A

305

http://actuary88.com stocks in the open market at $80 per share and return the 1,000 shares to the brokerage firm. Let’s see how much money you have made. You sell 1,000 shares at $100 per share. Your cost is $80 per share. So you have earned $20 per share. Your total profit is ($20)(1,000)=$20,000. Nice job. This is the stripped-down version of short sales; the actual transactions are a little more complex. Let’s analyze short sales. First, in short sales, you sell shares high now and buy shares back later. You hope to buy shares back in the near future at a lower price. However, if the share price goes up, you’ll have to buy shares back at a higher price and incur a loss. For example, 15 days later, Company A’s stocks sell $110 per share. All the rumors you heard about Company turn out to be false. Company A is doing quite well. You wait for a while, but Company A stock prices stay stable at $110 per share. Finally, you have to buy back 1,000 shares at $110 per share in the open market and return them to the brokerage firm which did the short sale for you. You incurred a total loss of 1,000($110-$100)=$10,000. Second, laws require that short sales can take place only if the last recorded change in the price of a stock is positive (i.e. the stock price went up last time). You can’t short sell a stock after the stock price went down. You can short sell a stock after the stock price went up. This rule attempts to discourage wild speculations on stocks. Third, in short sales, your liability to the brokerage firm is the number of shares borrowed, not the value of the borrowed shares at the time when short sales take place. In our example, you borrow 1,000 shares of Company A stocks from the brokerage firm at $100 per share. Your liability to the brokerage firm is 1,000 shares of stocks, not $100,000. In the future, you just need to return 1,000 shares to the brokerage firm. If you can return 1,000 shares of stocks to the brokerage firm at a lower price, you’ll make a profit. If you return 1,000 shares at a higher price, you have a loss. Forth, you can borrow shares for long time before paying them back. Have you wondered where the brokerage firm gets the 1,000 shares of Company A stocks to lend them to you? The brokerage firm simply borrows 1,000 shares of Company A stocks from another investor who holds 1,000 shares of Company A stocks in his account at the brokerage firm. This investor will not even know that the brokerage firm has borrowed 1,000 shares from his account. If this investor wants to sell 1,000 shares of Company A stocks after you have short sold his 1,000 shares of stocks, the brokerage firm simply borrows 1,000 shares of 306

http://actuary88.com Company A stocks from another investor. As a result, you can borrow someone else’s shares for a long time. However, in some situations you short sold somebody else’s 1,000 shares of stocks. Then the original owner demanded 1,000 shares of stocks (because he wanted to sell them), but the brokerage firm couldn’t find another investor who had 1,000 shares of stocks. If this happens, the brokerage firm will ask you to immediately purchase 1,000 shares from the open market and return them to the brokerage firm. Fifth, if the borrowed stocks pay dividends after the short sale and before you return the borrowed stocks to the brokerage firm, you are required to pay the dividend to the original owner of the stocks. When you wanted to short sell 1,000 shares of Company A stocks, your brokerage firm decided borrow 1,000 shares from an investor (we call him Smith) and lent them to you. You short sold these 1,000 shares of stocks at $100 per share to someone (we call him John). One month later, Company A decided to pay a dividend of 5 cents per share. Company A found out that John was the owner of the 1,000 shares. At this time you had not paid back the 1,000 borrowed shares to the brokerage firm. Company A would pay the dividend worth a total of $0.05(1,000)=$50 to the new owner John, not to the original owner Smith. However, Smith still owned 1,000 shares of Company A stocks and should get a total of $50 dividend. Consequently, you needed to pay the brokerage firm $50 dividend and the brokerage firm would give the $50 to the original owner Smith. Does this mean that you paid $50 dividend to Smith out of your own pocket? No. The dividend of 5 cents per share was reflected in the stock price. In other words, when you short sold 1,000 shares of stocks at $100 per share, the $100 price already reflected the possibility that Company would distribute certain amount of dividend to its shareholders. Because Company A stocks could possibly generate a dividend of 50 cents per share, you could short sell Company A stocks at $100 per share. If Company was not expected to distribute any dividend in the near future, the sales price of Company A stocks would have been less than $100 per share. Sixth, you must meet the initial margin requirement before you can short sell. For example, the brokerage firm requires a 50% initial margin. This means that before a short sale can take place, your collateral account in the brokerage firm must hold at least 50% of the then current market value of 1,000 shares of Company A stocks.

307

http://actuary88.com Seventh, you must meet the ongoing maintenance margin requirement. The maintenance margin requirement provides another layer of protection to the brokerage firm. For example, the brokerage firm requires a 40% maintenance margin. This means that from the moment when a short sale takes place to the moment immediately before you finally return 1,000 shares to the brokerage firm, your collateral account in the brokerage firm must hold at least 40% of the then current market value of 1,000 shares of Company A stocks. Mathematically, at any time t (where t is between when a short sale takes place to the moment immediately before you finally return the borrowed shares to the brokerage firm) Your equity ( t )

market value ( t ) of the stocks borrowed

Mainteance margin %

[ Mainteance margin %] Equity ( t ) = Asset ( t ) Liability ( t ) Your equity ( t )

market value ( t ) of the stocks borrowed

Please note that the 40% margin requirement can be met by cash, stocks and bonds. If your account at the brokerage firm has cash, stocks of another company, or bonds worth less than $40,000, you will get a margin call, in which cash you must deposit additional cash to your collateral account. For additional information, refer to http://www.investopedia.com/university/shortselling/ For a detailed description of how a short sale works, refer to http://webcomposer.pace.edu/pviswanath/notes/investments/shortsal. html#logistics Sample problems Problem 1 (SOA Course 6 #3 May 2001) (a) With respect to short sales of a security: (1) Describe the process for executing a short sale (2) Outline an investor’s motivation for executing such a transaction (b) You are given the following: Date January 1, 2001 January 15, 2001

Corporate Z Share Price 60 63

308

http://actuary88.com January 31, 2001

58

• •

Corporation Z paid a dividend of 1 on January 15, 2001 The maximum price of Corporation Z shares during the month of January 2001 was 63

• •

On January 1, 2001, Investor A sold short 100 shares of Corporation Z On January 31, 2001, Investor A covered the short position

• •

The initial margin requirement was 50% The maintenance margin requirement was 40%

• •

There were no other transaction costs No interest was earned on the balance with the broker

(3) (4) (5) (6)

Outline the transaction on January 1, 2001 Outline the transaction on January 31, 2001 Determine whether a margin call was necessary Calculate the rate of return to Investor A for January 2001.

Show all the work. Solution Download the official solution from the SOA website. Please note some of the required concepts in FM are from Course 6. Problem 2 (SOA Course 6 #3 May 2003) You are given the following information: • • • •

margin requirement on short sales: 50% maintenance margin: 30% an investor’s account with a broker currently holds: value of T-bills: 10,000 number of shares of XYZ stocks 500 stock prices: Date June 2, 2003 June 3, 2003 June 4, 2003

ABC Stock Price 103 102 99

XYZ Stock Price 75 76 77 309

http://actuary88.com June June June June

5, 2003 6, 2003 9, 2003 10, 2003

100 101 105 115

75 80 72 65

The investor tells the broker to short 1,000 shares of the ABC stock on June 3, 2003. The broker executes the order on the first day allowed. Shares are traded once per day. (a) Calculate the additional cash (if any) necessary to satisfy the margin requirement. (b) Calculate the amount of the margin calls (if any) between June 3, 2003 and June 10. Show all the work. Solution Download the official solution from the SOA website. Problem 3 You short sell 100 shares of Company XYZ, which currently sell for $50 per share. The initial margin requirement is 40%. 1. What’s your rate of return if one year later Company XYZ’s stocks sell $55, $50, or $45 per share? Assume you earn no interest on your margin account. Also assume that Company XYZ doesn’t pay dividend. 2. If the maintenance margin is 30%, how high can XYZ’s stock price go up before you get a margin call? 3. Redo the above two questions assuming that XYZ pays dividend of $1 per share at the end of year 1. Solution 1. To satisfy the margin requirement, you must deposit the following amount of cash into the margin account at t = 0 : $50(100)(40%)=$2,000 Your rate of return is:

310

http://actuary88.com

r=

100 ( 50 P ) profit , where P is the share price at t = 1 = Initial deposit 2, 000

Your returns are:

100 ( 50 55) = 25% 2,000 100 ( 50 50 ) If P = 50 , r = = 0% 2, 000 100 ( 50 45 ) If P = 45 , r = = 25% 2, 000 If P = 55 , r =

2. Your asset consists of: • proceeds from short sales: 100(50)=$5,000 • initial deposit into the margin account: $2,000 Your total asset is $7,000 Your liability is to return 100 shares of stocks: $100P Your equity is:

7,000 – 100P

You get a margin call if the ratio of your equity to the then market value of the stocks is less than the maintenance margin: 7000 100 P < 30% , 100 P

P>

70 = $53.85 1.3

3. Now you have to pay dividend to your broker. The dividend amount is 100($1 per share)=$100. Now your return is

r=

100 ( 50 P ) 100 profit = Initial deposit 2, 000

Your returns are: If P = 55 , r =

100 ( 50 55 ) 100 = 30% 2, 000

If P = 50 , r =

100 ( 50 50 ) 100 = 5% 2, 000

311

http://actuary88.com

If P = 45 , r =

100 ( 50 45) 100 = 20% 2, 000

Your asset consists of: • proceeds from short sales: • initial deposit into the margin account: Total asset:

100(50)=$5,000 $2,000 $7,000

Your liability: • To return 100 shares of stocks: • To pay the dividend: Total liability:

$100P $100 $100P+100

Your equity is:

7,000 – 100P-100

You get a margin call if the ratio of your equity to the market value of the stocks sold is less than the maintenance margin: 7000 100 P 100 < 30% , 100 P

P>

69 = $53.08 1.3

Problem 4 (#17, Nov 2005 FM) Theo sells a stock short with a current price of 25,000 and buys it back for X at the end of 1 year. Governmental regulations require the short seller to deposit margin of 40% at the time of the short sale. The prevailing interest rate is an 8% annual rate, and Theo earns a 25% yield on the transaction. Calculate X Solution At t = 0 , Theo deposited 25, 000 ( 40% ) = 10, 000 into the margin account. At t = 1 when the short sale is closed out, Theo’s wealth is

FV of the initial deposit + gain from the short sale – dividend paid = 10, 000 (1.08 ) + ( 25, 000 X ) 0 Theo earned 25% yield on the transaction:

312

http://actuary88.com

10, 000 (1.08 ) + 25, 000 X = 10, 000 (1.25) ,

X = 23,300

Problem 5 (#36, May 2003 Course 2) (also Sample FM #38) (SOA removed Sample #38 through Sample #44 from the Sample FM Questions. As a result, you can skip the following questions. I included these questions for completeness.) Eric and Jason each sell a different stock short at the beginning of the year for a price of 800 . The margin requirement for each investor is 50% and each will earn an annual effective interest rate of 8% on his margin account. Each stock pays a dividend of 16 at the end of the year. Immediately thereafter, Eric buys back his stock at a price of 800 2X , and Jason buys back his stock at a price of 800 + X . Eric’s annual effective yield, i, on the short sale is twice Jason’s annual effective yield. Calculate i. Solution At t = 0 , Eric deposits 800*50%=400. At t = 1 , Eric’s wealth is: FV of the initial deposit + gain from the short sale – dividend paid = 400 (1.08 ) + 800 ( 800 2 X ) 16 = 400 (1.08 ) + 2 X 16 Eric’s annual effective yield from the short sale is i . Then 2 X + 16 400 (1 + i ) = 400 (1.08) + 2 X 16 , i= 400 At t = 0 , Jason deposits 800*50%=400. At t = 1 , Jason’s wealth is: FV of the initial deposit + gain from the short sale – dividend paid = 400 (1.08 ) + 800 ( 800 + X ) 16 = 400 (1.08 ) X 16 Jason’s annual effective yield from the short sale is j . Then 16 X 400 (1 + j ) = 400 (1.08 ) X 16 j= 400 We are told that i = 2 j :

313

http://actuary88.com 2 X + 16 16 X , =2 400 400

i=

X =4

2 X + 16 2 ( 4 ) + 16 = = 6% 400 400

Sample FM Problem #39 Jose and Chris each sell a different stock short for the same price. For each investor, the margin requirement is 50% and interest on the margin debt is paid at an annual effective rate of 6%. Each investor buys back his stock one year later at a price of 760. Jose’s stock paid a dividend of 32 at the end of the year while Chris’s stock paid no dividends. During the 1-year period, Chris’s return on the short sale is i , which is twice the return earned by Jose. Calculate i . Solution Let X represent the price of the stock when the short sale takes place (i.e. at t = 0 ). At t = 0 , Jose deposits 0.5X . At t = 1 , Jose’s wealth is: FV of the initial deposit + gain from the short sale – dividend paid = 0.5 X (1.06 ) + ( X 760 ) 32 = 1.53 X 792 Jose’s annual effective yield from the short sale is i . Then 1.53 X 792 0.5 X (1 + j ) = 1.53 X 792 , j = 1 0.5 X At t = 0 , Chris deposits 0.5X . At t = 1 , Chris’s wealth is: FV of the initial deposit + gain from the short sale – dividend paid = 0.5 X (1.06 ) + ( X 760 ) = 1.53 X 760 Chris’s annual effective yield from the short sale is i . Then 1.53 X 760 0.5 X (1 + j ) = 1.53 X 760 , i = 1 0.5 X We are told that i = 2 j : 1.53 X 792 1.53 X 760 2 1 = 1, 0.5 X 0.5 X

X = 800

314

http://actuary88.com

i=

1.53 ( 706.8 ) 792 0.5 ( 706.8 )

1 = 16%

Sample FM Problem #40 Bill and Jane each sell a different stock short for a price of 1000. For both investors, the margin requirement is 50%, and interest on the margin is credited at an annual effective rate of 6%. Bill buys back his stock one year later at a price of P . At the end of the year, the stock paid a dividend of X . Jane also buys back her stock after one year, at a price of ( P – 25). At the end of the year, her stock paid a dividend of 2 X . Both investors earned an annual effective yield of 21% on their short sales. Calculate P . Solution At t = 0 , Bill deposits 1000*50%=500. At t = 1 , Bill’s wealth is: FV of the initial deposit + gain from the short sale – dividend paid = 500 (1.06 ) + (1000 P ) X = 1530 P X Bill’s annual effective yield from the short sale is 21%. Then 500 (1 + 21% ) = 1530 P X At t = 0 , Jane deposits 1000*50%=500. At t = 1 , Jane’s wealth is: FV of the initial deposit + gain from the short sale – dividend paid = 500 (1.06 ) + 1000 ( P 25 ) 2 X = 1555 P 2 X Jane’s annual effective yield from the short sale is 21%. Then 500 (1 + 21% ) = 1555 P 2 X 1530 P X = 1555 P 2 X ,

X = 25

500 (1 + 21% ) = 1530 P 25 ,

P = 900

315

http://actuary88.com

Chapter 15 Term structure of interest rate, spot rate, forward rate, and arbitrage Key points: 1. Law of one price (no arbitrage principle) • Two bonds (or other securities) with identical cash flows should sell for an identical price. •

If they don’t sell for the same price (i.e. if one bond sells at a higher price than another bond with identical cash flows), anyone can make money by buying the lower priced bond, turning around, and selling it at a higher price.



The strategy of exploiting loopholes to make money is called arbitrage.



Characteristics of arbitrage: (1) Profit is made with 100% certainty (2) Profit is made with zero cost (3) Profit is made with zero risks taken



No arbitrage principle assumes there are no transaction costs such as tax and commissions.

2. Term structure of interest rates •

Term structure of interest refers to the phenomenon that a bond’s yield-to-maturity changes as the bond’s maturity changes.



A hypothetical example Maturity of a bond

Yield to maturity

1 2 3 4 5

7% 8% 8.75% 9.25% 9.5%

year year years years years

316

http://actuary88.com •

Term structure of interest is also called the yield curve, which is a 2-D graph showing how the yield to maturity ( Y axis) is a function of the bond’s maturity ( X axis).



Different theories explain why a longer-term bond has a higher or lower yield-to-maturity than does a short-term bond. (No need to learn those theories)



Because a traded bond’s yield of maturity is often used as the market’s fair discount rate (the rate used to discount future cash flows), the discount rate is a function of the length of the investment.

3. Spot rate. •

Spot rate is the return you can lock in at time zero. It’s the return you can get by lending (or depositing) your money at t = 0 and having your money repaid in a future time t1 . Sign contract; Lend money

Time •

0

spot rate

Loan repaid

t1

For example, at t = 0 you bought a two year zero-coupon bond with $100 face amount for a price of $85. In this transaction, you lent $85 at t = 0 . Your money is to be repaid at t = 2 . The return you have locked in at t = 0 can be solved as follows: 85 (1 + r ) = 100 2

r = 8.465%

So at t = 0 you locked in an annual return of 8.465% for two years. r = 8.465% is the 2-year spot rate. •

The word “spot” comes from the phrase “the spot market.” The spot market (or called cash market) is where the seller immediately delivers the product to the buyer on the spot. Example. You pay $10 to a farmer and he immediately gives you 15 tomatoes on the spot. Remember, spot market = immediate delivery. (In the real world, however, many spot transactions are completed within a few hours or a couple of days.)

317

http://actuary88.com •

The phrase “spot rate” is used because at time zero when you buy the investment, the invest vehicle (a bond, a CD, or any other investment opportunity) is immediately delivered to you at time zero.



A spot rate answers this question, “If a one buys a bond now at time zero from a market, what return can he get?”



Spot rate is often denoted as st . This is the return you can get if you invest your money at time zero for t years.

4. Forward rate •

At time zero, you sign an investment contract, which requires you to lend your money at a future time t1 . Your money is to be repaid in another future time t2 where t2 > t1 . The return you earn by lending your money from t1 to t2 is a forward rate. Sign contract

Time

0

Lend money

t1

Loan repaid

forward rate

t2



The word “forward” comes from the forward market. In a forward market, an agreement is made at time zero but the delivery date is at a future time t1 . Example. You pay $10 to a farmer today for him to deliver 15 tomatoes to you in 3 months. You do so perhaps you really use tomatoes and you worry that tomato price may go up in the future. Most likely, people who buy forward products are not directly consumers, but are profit makers. They hope to resell a forward contract at a higher price.



Forward market = future delivery.



A forward rate answers the question, “If a n -year term bond (or another investment vehicle) is delivered at t1 (where t1 > 0 ) to an investor, what return will the investor get for lending money from t1 to t1 + n ? ” ( n is not necessarily an integer)



Forward rate is often denoted as f

j, k

. It’s the interest rate charged

for lending money from j to j + k .

318

http://actuary88.com 5. Relationship between spot rates and forward rates (assuming no arbitrage) s4 s3 s2 s1 = f 0 , 1

Time

0

f1 , 1

1

f2 ,1

2

f3 , 1

3

4

1 + s1 = 1 + f 0 , 1

(1 + s2 ) = (1 + s1 ) (1 + f1 , 1 ) = (1 + f 0 , 1 )(1 + f1 , 1 ) 3 2 (1 + s3 ) = (1 + s2 ) (1 + f 2 , 1 ) = (1 + f0 , 1 )(1 + f1 , 1 )(1 + f 2 , 1 ) 4 3 (1 + s4 ) = (1 + s3 ) (1 + f3 , 1 ) = (1 + f 0 , 1 )(1 + f1 , 1 )(1 + f 2 , 1 )(1 + f3 , 1 ) 2

…… n (1 + sn ) = (1 + sn



1

)

n 1

(1 + f n , ) = (1 + f , )(1 + f , )(1 + f , ) ... (1 + f n , ) 1 1

0 1

1 1

2 1

1 1

Example. s1 = 13.4% , s2 = 15.51% . What’s f 1 , 1 ? You have two options: (1) Invest $1 at time zero. Pull your money out at the end of Year 2. You should reap an annual return of 15.51%. (2) Invest $1 at time zero for one year and pull your money ($1.134) out at the end of Year 1, reaping 13.4% return. Then reinvest your $1.134 during Year 2, reaping a return of f 1 , 1 during Year 2. The above two options should produce identical wealth. If not, loopholes exist for people to exploit.

319

http://actuary88.com Time t

0

1

2

1+s1

3

1+ f 1 , 1

(1 + s2 )

2

(1 + s1 ) (1 + f 1 , 1 ) = (1 + s2 ) f 1, 1

(1 + s2 ) = 1 + s1

2

2

(1 + 15.51% ) 1= 1 + 13.4%

2

1 = 17.66%

Sample problems Problem 1 You are given the following three situations: #1 You earned $1,000,000 in a gambling game at Las Vegas last week. #2 The exchange rate at New York is $2= £1 The exchange rate at London is £2 =$6. #3 Programmers in a software company were working around the clock to finish building a new software package. To boost the morale of his over-worked programmers, the CEO of the company set an “$50 per bug” incentive plan. Under this plan, a programmer was to be awarded $50 for each bug he found in his codes. Explain which situations represent arbitraging opportunities. Solution #1 is NOT an arbitrage. In an arbitrage, one makes money with zero cost; sure profits are made without any risks taken. In gambling, however, a gambler takes lot of risks, yet the profit is not certain. #2 opens to arbitrage. For example, you can change $2 into £1 at New York. Immediately, you change £1 back to $3 at London. You can do these two transactions over the internet and earn $1 profit with zero cost and zero risk. #3 opens to arbitrage. This is a not fake story. A software company actually did something like this. The CEO had a good intention to 320

http://actuary88.com encourage his programmers to quickly discover and fix bugs. However, after the incentive policy was implemented, some programmers purposely added bugs into their codes so they would qualify for more rewards. Soon the CEO found out about the loophole and cancelled the incentive plan.

Problem 2 Consider two bonds A and B in a financial market. A is a one-year bond with 10% annual coupon and $100 par value; A sells for $97. B is a two-year bond with 8% annual coupon and $100 par value bond; B sells for $88. Calculate s1 and s2 , the 1-year spot and 2-year spot rate. Solution s1 is the spot rate for one year, from t = 0 to t = 1 . This is the return for investing money for one year. The cash flows at the end of Year 1 should use s1 as the discount rate. s2 is the spot rate for two years, from t = 0 to t = 2 . This is the return for investing money for two years. The cash flows at the end of Year 2 should use s2 as the discount rate. We need to solve the following two equations:

97 =

110 1 + s1

88 =

8 108 + 1 + s1 (1 + s2 )2

s1 = 13.4%, s2 = 15.51%

321

http://actuary88.com Problem 3 3 months from now (i.e. at t = 0.25 ), you need to borrow $5,000 for six months to fund your six-month long vacation. Worrying that the interest rate may go up a lot in three months, you want to lock in a borrowing rate right now at t = 0 . You are given the following facts: • The 3-month spot rate is 4.5% annual effective. • The 9-month spot rate is 5.7% annual effective. Explain how you can lock in, at t = 0 , a $5,000 loan for six months with a guaranteed interest rate. Calculate the guaranteed borrowing rate. Solution Lock-in strategy: At t = 0 , borrow $5, 000 (1 + 4.5% )

0.25

= 4,945.28 for 9-months @ 5.7% annual

effective. And immediately deposit the borrowed $4,945.28 for 3 months @ 4.5% annual effective. At t =

3 = 0.25 (3 months later), your $4,945.28 will grow into: 12 0.25 4,945.28 (1 + 4.5% ) = $5, 000

You will use this $5,000 to fund your 6-month long vacation. Effectively, you have borrowed $5,000 at t = 0.25 . 9 = 0.75 (9 months later), your vacation is over. Your loan of 12 $4,945.28 is due. You pay off this loan with a following payment at t = 0.75 :

At t =

4,945.28 (1 + 5.7% )

0.75

= $5,196.92

The interest rate you locked from t = 0.25 to t = 0.75 can be solved as follows: 4,945.28 (1 + 4.5% )

(1 + r ) = 4, 945.28 (1 + 5.7% ) 0.5 0.75 0.25 (1 + r ) = (1 + 5.7% ) (1 + 4.5% ) = 1.031044 0.25

0.5

0.75

r = 6.3052%

322

http://actuary88.com Time t (year)

0

0.25

(1 + 4.5% )

0.75

(1 + r )

0.25

(1 + 5.7% )

0.5

0.75

The rate you locked in, r = 6.3052% , is a 6-month forward rate @ t = 0.25 .

Problem 4 3 months from now (i.e. at t = 0.25 ), an insurance company will send you $5,000 cash. You want to invest $5,000 immediately after you receive it and invest it for 6 months. Worrying that the interest rate may go down a lot in three months, you want to lock in an interest rate right now at t = 0. You are given the following facts: • The 3-month spot rate is 4.5% annual effective. • The 9-month spot rate is 5.7% annual effective. Explain how you can lock in, at t = 0 , an investment opportunity where you can lend your future $5,000 for six months with a guaranteed interest rate. Calculate the guaranteed earning rate. Solution This problem is similar to Problem 3. The only difference is that this time we want to lend money and earn a guaranteed interest rate. Lock-in strategy: At t = 0 , borrow $5, 000 (1 + 4.5% )

0.25

= 4,945.28 for 3 months @ 4.5% annual

effective. And immediately deposit the borrowed $4,945.28 for 9-months @ 5.7% annual effective. 3 = 0.25 (3 months later), your borrowed amount of $4,945.28 will 12 grow into:

At t =

4,945.28 (1 + 4.5% )

0.25

= $5, 000

323

http://actuary88.com

At t = 0.25 , you can pay this loan amount using the payment of $5,000 sent to you by the insurance company. As a result, your loan is paid off. At t = 0.75 (9 months later), your original deposit of $4,945.28 will grow into: 4,945.28 (1 + 5.7% )

0.75

= $5,196.92

The interest rate you locked from t = 0.25 to t = 0.75 can be solved as follows: 4,945.28 (1 + 4.5% )

(1 + r )

0.5

0.25

(1 + r )

= (1 + 5.7% )

0.75

0.5

= 4, 945.28 (1 + 5.7% )

(1 + 4.5% )

0.25

0.75

= 1.031044

r = 6.3052%

Time t (year)

0

0.25

(1 + 4.5% )

0.75

(1 + r )

0.25

(1 + 5.7% )

0.5

0.75

The rate you locked in, r = 6.3052% , is a 6-month forward rate @ t = 0.25 .

Problem 5 You are given the following force of interest: ( t ) = 5% + 0.12%t Calculate • s5 , the 5 year spot rate. • s6 , the 6 year spot rate •

f 5 , 1 , the one year forward rate from t = 5 to t = 6

Solution

324

http://actuary88.com

s5 is the annual return you can lock in for investing money at time zero for 5 years.

(1 + s5 )

5 5

= exp

( t ) dt

0

t

t

( x ) dx = ( 5% + 0.12% x ) dx = 5%t + 0.06% t 2

0

0

(1 + s5 )

5 5

= exp

( t ) dt

= exp 5%t + 0.06% t 2

t =5

= e 26.5% = 1.30343098

0

s5 = 5.44296% Similarly,

(1 + s6 )

6 6

= exp

( t ) dt

= exp 5%t + 0.06% t 2

t =6

= e32.16% = 1.37933293

0

s6 = 5.50625% To find f 5 , 1 , we use the no arbitrage principle. Compare the following two options: •

At time zero, lock in a 6 year investment and get s6 = 5.50625% annual return.



At time zero, lock in a 5 year investment and get s5 = 5.44296% annual return. Then reinvest the total money starting from the end of Year 5 and ending at the end of Year 1 (i.e. during Year 6), reaping an annual return of f 5 , 1

These two options should generate the same amount of wealth.

325

http://actuary88.com Time t

0

1

2

3

(1 + s5 )

4

5

5

(1 + f 5 , 1 ) = (1 + s )

6

1+ f 5 , 1

(1 + s6 )

(1 + s5 )

5

6

6

6

(1 + s6 ) f 5, 1 = 5 (1 + s5 )

6

1=

e32.16% 1 = 5.823243% e 26.5%

Problem 6 You are given the following facts about three securities A , B , and C :

Security

Selling price at t=0

Cash flow at t=1

Cash flow at t=2

A B C

$7,570.03 $16,274.27

$5,000 $10,000 $0

$4,000 $9,500 $10,000

Cash flow beyond t=2 $0 $0 $0

Calculate (1) The price of Security C at time zero assuming no arbitrage. (2) If Security C sells for $7,200 at time zero, design an arbitrage strategy and calculate how much profit you can make. (3) If Security C sells for $8,000 at time zero, design an arbitrage strategy and calculate how much profit you can make Solution Calculate the price of Security C at time zero assuming no arbitrage. Let s1 represent the one year spot rate. Let s2 represent the two year spot rate.

326

http://actuary88.com 7,570.03 =

5, 000 4, 000 + 1 + s1 (1 + s2 )2

1 1 = x and = y. 2 1 + s1 (1 + s2 ) 7,570.03 = 5, 000 x + 4, 000 y

16, 274.27 =

10, 000 9,500 + 2 1 + s1 (1 + s2 )

Set

x = 1.1 1 ,

y = 1.15

16, 274.27 = 10, 000 x + 9, 500 y

2

The price of Security C at time zero if no arbitrage: 10, 000 = 10, 000 (1.15 2 ) = 7, 561.44 2 (1 + s2 )

If Security C sells for $7,200 at time zero, design an arbitrage strategy and calculate how much profit you can make. If Security C sells for $7,200 at time zero, then this price is below its market fair price of $7,561.44, creating an arbitrage opportunity. To exploit this opportunity, we first synthetically create a 2-year zerocoupon bond with $10,000 par. This bond is 6 2 ( B-2A ) . 3

Security

Selling price at t=0

Cash flow at t=1

Cash flow at t=2

A

$7,570.03

$5,000

$4,000

B

$16,274.27

$10,000

$9,500

B-2A

$16,274.27-2($7,570.03) = $1,134.21

6

2 3

( B-2A )

6

2 $1,134.21 = $7,561.40 3

$10,000-2($5,000) $9,500-2($4,000) =$0 =$1,500 $0

6

2 $1,500 = $10,000 3

Based on the above table, a 2-year zero-coupon bond with $10,000 par 2 can be created by buying (or selling short) 6 units of Security B’s and 3 2 1 simultaneously short selling (or buying) 6 × 2 = 13 units of Security A’s. 3 3 This synthetically created 2-year zero coupon $10,000 par bond is worth $7,561.40 at time zero.

327

http://actuary88.com Our arbitrage strategy if Security C sells for $7,200 at time zero: •

The arbitrage strategy is always “buy low, sell high.”



At time zero, we spend $7,200 and buy Security C . This is buy low.



Simultaneously at time zero, we short sell the synthetically created security 6 2 ( B-2A ) , which has the same cash flow as Security C 3

but is worthy $7,561.4, $361.4 more than Security C . This is sell high. •

Short selling 6 2 ( B-2A ) means short selling 3

buying •

6

2 A 3

6

2 B 3

and immediately

from the market.

At t = 2 , we get $10,000 from Security C . We’ll use this $10,000 to pay security 6 2 ( B-2A ) ’s cash flow at t = 2 , closing our short 3

position. •

The net result: at time zero, we earn $7,561.40 - $7,200= $361.40 sure profit per transaction above. Of course, if we can make 1,000 such transactions in a day, we’ll make $361,400 in a day.

If Security C sells for $8,000 at time zero, design an arbitrage strategy and calculate how much profit you can make •

At time zero, we sell short Security C . We earn $8,000 cash.



Simultaneously at time zero, we buy the synthetically created security 6 2 ( B-2A ) , which means buying 6 2 3

6



3

B

and short selling

2 ( 2A ) . 3

At t = 2 , we get $10,000 from the synthetically created security 6

2 ( B-2A ) . 3

We’ll use this $10,000 to pay Security C ’s cash flow at

t = 2 and close out our short position.

328

http://actuary88.com •

The net result: at time zero, we earn $8,000- $7,561.40= $438.60 sure profit per transaction above. Of course, if we make 1,000 such transactions in a day, we’ll make $438,600 in a day.

Problem 7 You are given the following spot rates:

s t = 5% +

t% , where t = 1, 2,3 5

Calculate f1 , 1 and f 2 , 1 , the one year forward rates at t = 1 and at t = 2 . Solution

(1 + s1 ) (1 + f 1 , 1 ) = (1 + s2 ) (1 + s2 )

2

2

(1 + f , ) = (1 + s ) 2

1

3

3

Let’s look at the meaning of these two equations. The 1st equation says that if you invest your money year by year, you should have the same wealth at t = 2 as you initially lock in a 2-year investment opportunity. Otherwise, an arbitrage opportunity exits. Time t

0

1

2

1+s1

1+ f 1 , 1

(1 + s2 )

2

Similarly, the 2nd equation compares two investment options.



Option 1. At time zero, investment $1 and lock in a two year 2 investment opportunity and accumulate to (1 + s2 ) at t = 2 . Next, immediately reinvest your wealth of (1 + s2 ) in a one year 2

investment opportunity and earn a forward rate f 2 , 1 during Year 2. Option 1 accumulates a total of (1 + s2 ) (1 + f 2 , 1 ) at t = 3 . 2

329

http://actuary88.com •

Option 2. At time zero, investment $1 in a 3 year investment opportunity, locking in an annual return of s3 . This accumulates a total wealth of (1 + s3 ) at t = 3 . 3



No arbitrage principle requires that the two options generate an identical amount of wealth at t = 3 .

Time t

0

1

(1 + s2 )

2 2

3

1+ f 2 , 1

(1 + s3 )

3

Once you understand the meaning of these two equations, the remaining work is purely algebra.

s t = 5% +

t% 5

1% = 5.2% 5 2% s 2 = 5% + = 5.4% 5 3% s 2 = 5% + = 5.6% 5

s 1 = 5% +

f1 , 1

(1 + s2 ) =

2

1 + s1

(1 + 5.4% ) 1=

2

1 5.60%

1 + 5.2%

(1 + s3 ) f2,1 = 2 (1 + s2 ) 3

(1 + 5.6% ) 1= 2 (1 + 5.4% ) 3

1 6.40%

Problem 8 You are given the following information with respect to a bond: • • • • • •

par amount: $1,000 term to maturity: 3 years annual coupon rate 8% payable annually 1-year continuous spot rate is 5% 1-year forward rate @ t = 1 is 6% 1-year forward rate @ t = 2 is 7%

330

http://actuary88.com

Calculate YTM (yield to maturity) of the bond. Solution Time t

0

1

Cash flow

2

$80

1+s1

3

$80

1+ f 1 , 1

$1,080

1+ f 2 , 1

First, we need to calculate the PV of the bond. PV of the bond is sum of each cash flow discounted at an appropriate spot rate.

PV =

3

CF ( t )

t =1

(1 + st )

t

=

80 80 1, 080 + + 2 1 + s1 (1 + s2 ) (1 + s3 )3

So we need to calculate the 1-year, 2-year, and 3-year spot rates. 1+s1 = e0.05

We are given:

Please note that we are given a continuous spot rate year. We are also given:

= 5% in the 1st

f 1 , 1 = 6% , f 2 , 1 = 7%

Using the relationship between spot rates and forward rates, we have: 2

= (1 + s1 ) (1+ f 1 , 1 ) = e0.05 (1.06 )

3

= (1 + s1 ) (1+ f 1 , 1 )(1+ f 2 , 1 ) = e0.05 (1.06 )(1.07 )

(1 + s2 ) (1 + s3 ) PV =

80 80 1, 080 + 0.05 + 0.05 = 1, 053.66 0.05 e e (1.06 ) e (1.06 )(1.07 )

YTM can be solved in the following equation: 1, 053.66 =

80 80 1, 080 + + 2 1 + y (1 + y ) (1 + y )3

331

http://actuary88.com

To solve this equation, use BA II Plus/BA II Plus Professional. Enter PMT=80, N=3, FV=1,000, PV= - 1,053.66. Let the calculator solve for I/Y. We should get: I/Y=5.99279% So the yield to maturity is 5.99279%. Problem 9 Short term, one-year annual effective interest rates are currently 10%; they are expected to be 9% one year from now, 8% two years from now, 7% three years from now, and 6% four years from now. Calculate • The spot yield of 1-year, 2-year, 3-year, 4-year, and 5-year zero coupon bonds respectively. •

The annual effective yield of a bond redeemed at $100 par value in 5 years and pays 8% coupon annually.

Solution First, let’s draw a diagram:

Time t

0

Cash flow

1+s1

1

2

3

4

5

$8

$8

$8

$8

$108

1+ f 1 , 1

10%

1+ f 2 , 1

9%

(1 + s2 )

8%

1+ f 3 , 1 7%

1+ f 4 , 1 6%

2

(1 + s3 )

3

(1 + s4 )

4

332

http://actuary88.com

(1 + s5 )

5

Let st represent the spot yield over t years. s1 = 10% s2 = 9.499% (1 + s2 ) = 1.1×1.09 , 3 s3 = 8.997% (1 + s3 ) = 1.1×1.09 ×1.08 , 2

(1 + s4 ) (1 + s5 )

4

= 1.1× 1.09 ×1.08 × 1.07 ,

5

= 1.1× 1.09 × 1.08 × 1.07 × 1.06 ,

s4 = 8.494% s5 = 7.991%

Next, let’s calculate the annual effective yield of a bond redeemed at par value of $100 in 5 years and pays 6% coupon annually. The PV of the bond is: 5 t =1

CF ( t )(1 + st ) = t

8 8 8 8 108 + + + + 2 3 4 1 + 10% (1 + 9.499% ) (1 + 8.997% ) (1 + 8.494% ) (1 + 7.991% )5

= 99.43227

Next, we need to solve the equation: 99.43227 = 8 a5 i + 100v5

In BA II Plus TVM, Enter PV= - 99.43227, PMT = 8, FV =100, N=5. Press “CPT” “I/Y.” You should get: I/Y=8.143% So the annual effective yield is 8.143%.

333

http://actuary88.com

Problem 10 (November 2005 FM #6) Consider a yield curved defined by the following equation ik = 0.09 + 0.002k 0.001k 2

here ik is the annual effective rate of return of zero coupon bonds with maturity of k years. Let j be the one-year effective rate during year 5 that is implied by this yield curve. Calculate j . Solution First, let’s draw a diagram:

Time t

0

1

2

1 + i1

3

1+ f 1 , 1

(1 + i2 )

1+ f 2 , 1

4

1+ f 3 , 1

5

1+ f 4 , 1

2

(1 + i3 )

3

(1 + i4 )

4

(1 + i5 )

5

We are asked to find j = f 4 , 1 . Please note that Year 5 is from t=4 to t=5,

(1 + i4 )

4

(1+ f , ) = (1 + i ) 4 1

5

5

334

http://actuary88.com

(1 + i5 ) 4 (1 + i4 )

1 + 0.09 + 0.002 ( 5 ) 0.001( 5 )

5

j = f 4 ,1 =

1=

1 + 0.09 + 0.002 ( 4 ) 0.001( 4 )

2

2

5

4

1=

1.0755 1 = 4.74% 1.0824

Problem 11 The one-year spot interest rate at t = 0 is 6% annual effective. The annual effective yield of a two year bond issued at t = 0 that pays 4% annual coupons and that is redeemed at par value of $100 is 7% annual effective. The issue price at t = 0 of a three-year bond that pays 8% coupons annually is $102 per $100 nominal. Calculate • f1 ,1 , the one-year spot rate at t = 1 •

f 2 ,1 , the one-year forward rate at t = 2 .

Solution Let’s first calculate the PV of the bond issued at t = 0 that pays 4% annual coupons and that is redeemed at par value of $100 is 7% annual effective. PV = 4 a2 7% + 100 (1.07 ) = 94.576 2

Time t

0

1

1 + i1

2

1+ f 1 , 1

3

1+ f 2 , 1

1 + 6%

(1 + i2 )

2

(1 + i3 )

94.576 =

3

4 104 4 104 + = + 2 1 + i1 (1 + i2 ) 1 + 6% (1 + 6% ) (1 + f1 ,1 )

335

http://actuary88.com f1 ,1 = 8.051%

The issue price at t = 0 of a three-year bond that pays 8% coupons annually is $102 per $100 nominal: 102 =

102 =

8 8 8 + 100 + + 2 1 + i1 (1 + i2 ) (1 + i3 )3

8 8 8 + 100 + + 1 + 6% (1 + 6% )(1 + 8.051% ) (1 + 6% )(1 + 8.051% ) (1 + f 2 , 1 )

f 2 , 1 = 7.805%

Problem 12 The n year spot rates at t = 0 are defined as follows: in = 0.05 +

n , where 1, 2,3 1000

Calculate the implied one-year forward rate at t = 1 and t = 2 . Solution We are asked to find f 1 , 1 and f 2 , 1 . Time t

0

1

1 + i1

2

1+ f 1 , 1

(1 + i2 )

3

1+ f 2 , 1

2

(1 + i3 )

3

336

http://actuary88.com

(1 + i2 )

= (1 + i1 ) (1+ f 1 , 1 ) ,

2

i1 = 0.05 +

(1 + i3 )

3

= (1 + i2 ) (1+ f 2 , 1 ) 2

1 2 3 = 0.051 , i2 = 0.05 + = 0.052 , i3 = 0.05 + = 0.053 1000 1000 1000

f 1,1

(1 + i2 ) =

2

1 + i1

(1 + i3 ) f 2 ,1 = 2 (1 + i2 )

1=

1.0522 1.051

1 = 5.3%

1=

1.0533 1.0522

1 = 5.5%

3

Problem 13 The following n -year spot rates are observed at t = 0 : 1-year spot rate is 2% 2-year spot rate is 3% 3-year spot rate is 4% 4-year spot rate is 5% 5-year spot rate is 6% 6-year spot rate is 7% 7-year spot rate is 8% 8-year spot rate is 9% 9-year spot rate is 10% Calculate f 5 , 4 , the 4-year forward rate at t = 5 . Solution

(1 + s5 )

5

(1 + f ) = (1 + s ) 4

5, 4

9

9

s5 = 6% , s9 = 10%

(1 + f ) 5, 4

(1 + s9 ) = (1 + 10% ) = 5 5 (1 + s5 ) (1 + 6% ) 9

4

9

f5 , 4 = 15.21%

337

http://actuary88.com

Problem 14 The spot yield curve on 1/1/2006 is defined as follows: st = 0.04e

0.06 t

+ 0.03

Calculate f 3 , 2 . Solution

(1 + s3 )

3

(1 + f ) = (1 + s ) 2

3, 2

5

5

s3 = 0.04e

0.06 ( 3)

+ 0.03 = 0.0634108

s5 = 0.04e

0.06 ( 5 )

+ 0.03 = 0.059633

(1 + f )

(1 + s5 ) = 3 (1 + s3 )

5

2

3, 2

0.0634108 5 = 0.059633 3

f 3 , 2 = 5.4%

Problem 15 You are given the following forward rates: f 0 ,1 = 5% ,

f1 ,1 = 6% ,

f 2 , 1 = 7%

Calculate the annual effective yield of a 3-year bond that pays 8% annual coupon with face amount of $100. Solution Time t

0

Cash flows

1 + i1

1

2

3

$8

$8

$108

1+ f 1 , 1

(1 + i2 )

1+ f 2 , 1

2

338

http://actuary88.com

(1 + i3 )

3

The price of the bond (i.e. the PV) is: 8 8 8 + 100 8 8 8 + 100 + + = + + = 105.49378 2 3 1 + i1 (1 + i2 ) (1 + i3 ) 1.05 1.05 (1.06 ) 1.05 (1.06 )(1.07 )

Next, we solve the following equation: 105.49378 = 8 a3 i + 100v 3

Using BA II Plus TVM, we should get: i = 5.94675%

Problem 16 (Sample FM #33) You are given the following information with respect to a bond: Par amount: Term to maturity: Annual coupon: Term 1 2 3

$1,000 3 years 6% payable annually Annual spot interest rates 7% 8% 9%

Calculate the value of the bond. Solution Time t

0

1

Cash flows

2

$60

1 + i1

3

$60

1+ f 1 , 1

(1 + i2 )

$1,060

1+ f 2 , 1

2

339

http://actuary88.com

(1 + i3 )

3

The bond is worth its present value. The present value is: 60 60 60 + 100 60 60 60 + 100 + + = + + = 926.03 2 3 2 3 1 + i1 (1 + i2 ) (1 + i3 ) 1 + 7% (1 + 8% ) (1 + 9% )

Problem 17 (Sample FM #34) You are given the following information with respect to a bond: Par amount: $1,000 Term to maturity: 3 years Annual coupon: 6% payable annually Term Annual spot interest rates 1 7% 2 8% 3 9% Calculate the annual effective yield rate for the bond if the bond is sold at a price equal to its value. Solution From Problem 16, we know the present value of the bond is 926.03. 926.03 = 60 a3 i + 1000v3

Using BA II Plus TVM, we get: i = 8.918%

340

http://actuary88.com

Chapter 16 Macaulay duration, modified duration, convexity An asset has the following cash flows: Time t

Cash flow

0

1

CF ( 0 )

P=

n

……

CF (1)

CF ( t ) v =

t =0

CF ( k )

n

CF ( t )

t =0

(1 + r )

t

k

……

n

CF ( n )

t

Asset price = Present value of the future cash flows We want to find out how P , the asset price, is sensitive to the change of the interest rate r , the effective interest per year. We define the following term: Macaulay duration = negative price elasticity relative to (1 + r )

DMAC = Asset Price Elasticity relative to (1 + r ) =

% change in price % change in (1 + r )

Often time, Macaulay duration is simply called duration. If you are asked to calculate the duration, just calculate Macaulay duration. We make the Macaulay duration equal to the negative price elasticity. This way, the Macaulay duration becomes a positive number -- we like positive numbers better. To see why the Macaulay duration is positive, notice that the present value of an asset is inversely related to the interest rate. If r goes up (i.e. % change of r is positive), then we discount cash flows at a higher discount rate, causing the asset price to go down (i.e. % change of the asset price becomes negative). Similarly, if r goes down (i.e. % change of r is negative), then we discount cash flows at a lower discount rate, causing the asset price to go up (i.e. % change of the

341

http://actuary88.com asset price becomes positive). By setting the Macaulay duration equal to the negative price elasticity, we’ll get a positive number. dp dp p p = = dr d (1 + r ) 1+ r 1+ r

DMAC =

P=

n t =0

CF ( t ) v t = n

n

CF ( t )

t =0

(1 + r )

dP d = dr dr

t =0

DMAC =

(1 + r ) n

DMAC =

t =0

(1 + r )

1 dp p dr

t

CF ( t ) (1 + r ) =

1 1+ r

1 dp = p dr

1 p

t

t CF ( t ) v t P

(1 + r ) n

=

t =0 n

n

t CF ( t ) (1 + r ) = t

t =0

1 1+ r

n t =0

1 1+ r

n t =0

t CF ( t ) v t

t CF ( t ) v t

t CF ( t ) v t CF ( t ) v t

t =0

Observation: A zero coupon bond’s duration is simply its maturity. A zero coupon bond has only one cash flow at its maturity n .

DMAC

n CF ( n ) v n = =n CF ( n ) v n

We define the 2nd term: Modified Duration=DMOD = -

DMOD = -

DMOD

1 dP

p dr

=-

1

p

1 dP

p dr

1 1+ r

1 1 == p dr 1 + r p 1 dP

n t =0 n

t =0

t CF ( t ) v t =

t CF ( t ) vt =

1 1 1+ r p

n t =0

t CF ( t ) vt

1 D 1 + r MAC

342

http://actuary88.com Wall Street method of calculating the modified duration: n

DMOD

D 1 = MAC = y y 1+ 1+ m m

t =0 n t =0

t CF ( t ) v t CF ( t ) v t

y is the yield to maturity expressed as a nominal interest rate compounding as often as coupons are paid; m is the # of coupons per y year. Consequently, is the effective interest per coupon period. m

To understand the Wall Street formula for the modified duration, please note that when Wall Street talks about a bond yield, it uses a nominal interest rate compounding as frequently as coupons are paid, not the annual effective interest. For example, for a bond that pays coupons semiannually, if the yield to maturity is 10.25% annual effective, then Wall Street will quote the bond yield as y = i ( 2) . 1+

y 2

2

= 1 + 10.25%

y = 10%

So Wall Street quotes the yield to maturity as 10%. n

Then why does Wall Street use DMOD =

DMAC 1 = y y 1+ 1+ m m

t =1 n t =1

t CF ( t ) v t CF ( t ) v

? t

Please note that here we start off with t = 1 , not t = 0 . This is because a bond’s first coupon does not start off with t = 0 . Let y represent the yield to maturity expressed as the nominal interest rate compounding as frequently as coupons are paid. Assume that coupons are paid m -thly. So essentially, y = i ( m ) and the effective interest m

y rate r = 1 + 1 . Assume the bond’s term to maturity is n . The present m value of the bond is:

343

http://actuary88.com

P=

n

CF ( t )

t =1

(1 + r )

t

=

CF ( t )

n t =1

1 dP 1 d = P dy P dy 1

t =1

DM O D =

y m

mt

CF ( t )

n

1 y 1+ P m

=

1+

1 1 = ( m) P m

mt

1+ n t =1

y m t CF ( t )

mt

y 1+ m

=

1 y 1+ m

t CF ( t )

n

m t +1

t =1

1 P

y m CF ( t )

1+ n t =1

(1 + r )

t

=

1 y 1+ m

DM A C

1 dP 1 = DM A C P dy 1 + y m

So the Wall Street formula and the textbook formula are different:

1 dP P dy (derivative of price relative to nominal yield) Wall Street: DM O DWall Street =

Textbook: DM O D textbook =

1 dP

p dr (derivative of price relative to effective yield) Please note that Bond Worksheet in BA II Plus Professional uses the Wall Street method to calculate the bond price and the modified duration. If you use Bond Worksheet to calculate the bond price and its modified duration, make sure that you use the nominal interest rate. You’ll get a wrong result if you enter the annual effective yield to maturity into Bond Worksheet. In Exam FM, use the textbook definition of the modified duration:

DMOD =

1 D (where r is the annual effective yield to maturity) 1 + r MAC

Don’t use the Wall Street definition.

344

http://actuary88.com For a bond paying coupons annually, these two methods produce the same modified duration. This is because we have m = 1 and y = i (1) = r . We define the 3rd term: Convexity=

1 d 2P

p dr

Convexity = = =

2

=

1 d

n

p dr

t =1

n

1

1

p (1 + r ) 1

t =1

1

(1 + r )

2

1

(1 + r )

= v2

2

2

1

n

p

t =1

t CF ( t ) (1 + r )

t 1

=

1

n

p

t =1

t ( t + 1) CF ( t ) (1 + r )

t 2

t ( t + 1) CF ( t ) v t

n

p

t =1

1

n

p

t =1

t 2 CF ( t ) v t +

1

n

p

t =1

t CF ( t ) v t

t 2 CF ( t ) v t + Duration

t 2 CF ( t ) v t + Duration

How the price of an asset changes if the interest changes by a small amount:

P (r ) =

n t =1

P=

CF ( t ) vt =

n

CF ( t )

t =1

(1 + r )

t

dP 1 d 2P 2 r+ r ) + ... 2 ( dr 2 dr

(Taylor series)

Divide by P : 2 P 1 dP 1 1 d 2P = r+ 2 ( r ) + ... P P dr 2 P dr 2 1 = DMOD r + ( Convexity )( r ) + ... 2 2 1 1 = DMAC r + ( Convexity )( r ) + ... 1+ r 2

If the interest rate change

r is small, we can set

( r)

2

0

345

http://actuary88.com

P P

1 dP 1 r = DMOD r = DMAC r P dr 1+ r

Sample problems Problem 1 Bond face Coupon Term to maturity Yield to maturity

$100 4% semiannual 3 years 5% annual effective.

Use BA II Plus/BA II Plus Professional Cash Flow Worksheet, calculate the duration (i.e. Macaulay duration), modified duration, and convexity of the bond. Solution First, we draw a cash flow diagram. Unit time = 1 year Time t

0

Cash flow

0.5

1

$2

$2

1.5 $2

2 $2

2.5 $2

3 $2 $100

For bonds, often it’s easier if we set the unit time = the payment period: Unit time = 0.5 year Time t

0

Cash flow

1

2

3

4

5

6

$2

$2

$2

$2

$2

$2 $100

3

DMAC =

t =1 3 t =1

t CF ( t ) v t CF ( t ) v

(where t=0.5, 1, 1.5, 2, 2.5, 3) t

Unit time is one year and v =1.05

1

346

http://actuary88.com

=

1 2

6 t =1 6 t =1

t CF ( t ) v t CF ( t ) v

(where t=1, 2, 3, 4, 5, 6) t

Unit time is half a year v =1.05

1 2

We set up the following table (we are using 6 months a one unit time): t

CF ( t )

t CF ( t )

t2

t 2CF ( t )

1 2 3 4 5 6

2 2 2 2 2 102

2 4 6 8 10 612

1 4 9 16 25 36

2 8 18 32 50 3,672

First, we’ll find the bond’s price. We enter CF ( t ) into Cash Flow Worksheet:

t 1 2 3 4 5 6

CF ( t )

Cash flows $2 $2 $2 $2 $2 $102

Interest Rate per coupon period Calculate NPV

NPV =

6 t =1

CF1 CF2 CF3 CF4 CF5 CF6

Cash flow frequency 1 1 1 1 1 1

i = 1.05 1 = 2.4695% 97.41125361

CF ( t ) vt = 97.41125361

347

http://actuary88.com 6

Next, we calculate

t =1

t CF ( t ) v t . We enter the following into Cash Flow

Worksheet:

CF ( t )

t 1 2 3 4 5 6

t CF ( t ) 2 2 2 2 2 102

Cash flows

2 4 6 8 10 612

CF1 CF2 CF3 CF4 CF5 CF6

Cash flow frequency 1 1 1 1 1 1

Interest rate

i = 1.05 1 = 2.4695%

Calculate NPV

NPV=556.1144433

6 t =1 6

DMAC

1 = 2

t =1 6

t CF ( t ) v t = 556.1144433 t CF ( t ) v t

t =1

CF ( t ) vt

=

1 556.1144433 = 2.85446713 2 97.41125361

Next, we’ll calculate the modified duration. Wall Street method:

1 DM A C 1 + effective yield per coupon period 1 2.85446713 = DM A C = = 2.78567468 1.05 1.05

DM O DWall Street =

Textbook method:

DM O DTextbook =

1 DM A C 1 + annual effective yield rate

348

http://actuary88.com =

1 2.85446713 DM A C = = 2.71854012 1.05 1.05

Next, we’ll calculate the convexity using the following formula: Convexity =

3

1

1

p (1 + i )

2

t =1

t ( t + 1) CF ( t ) v t

Unit time is one year

=

3

1

1

p (1 + 5% )

2

t =1

t 2 CF ( t ) 1.05 t +

3 t =1

t CF ( t ) 1.05

t

Unit time is one year

=

6

1

1

p (1 + 5% )

2

t 2

t =1

2

CF ( t ) v + t

6

t CF ( t ) vt t =1 2

Unit time is half a year

v=(1+i )-1 , i = 2.4695%

=

1

1

p (1 + 5% )2

1 4

6 t =1

t 2 CF ( t ) vt +

1 2

6 t =1

t CF ( t ) vt

Unit time is half a year

v=(1+i )-1, i = 2.4695% 6

To calculate

t =1

t 2 CF ( t ) vt , we enter t 2 CF ( t ) (bold numbers below) into

Cash Flow Worksheet: t

CF ( t )

t2

t 2CF ( t )

1 2 3 4 5 6

2 2 2 2 2 102

1 4 9 16 25 36

$2 $8 $18 $32 $50 $3,672

Interest Rate per coupon period Calculate NPV 6 t =1

Cash flows CF1 CF2 CF3 CF4 CF5 CF6

Cash flow frequency 1 1 1 1 1 1

i = 1.05 1 = 2.4695% 3,271.595675

t 2 CF ( t ) vt = 3,271.595675

349

http://actuary88.com We already know that 6 t =1

t CF ( t ) v t = 556.1144433

P=

6 t =1

CF ( t ) v = 97.41125361

Convexity =

1

1

p (1 + 5% )

2

1 4

6 t =1

t 2 CF ( t ) vt +

1 2

6 t =1

t CF ( t ) vt

Unit time is half a year

v=(1+i )-1, i = 2.4695%

=

1

1

97.41125361 (1 + 5% )

2

1 1 ( 3,271.595675) + ( 556.1144433) = 10.2048222 4 2

Alternatively, convexity is:

v

2

1

p

n

t CF ( t ) v + Duration = 2

t =1

t

1

(1 + 5% )

6

1 2

p

t =1

1 2 t CF ( t ) vt + duration 4

Unit time is half a year v =(1+i )-1, i =2.4695%

=

1

(1 + 5% )

2

1 3,271.595675 + 2.85446713 = 10.2048222 4 97.41125361

Though the problem looks complex and intimidating, the solution process is really quick and simple. Let’s summarize the calculation steps: If a bond has Coupons C payable m-thly per year Yield to maturity = r annual effective Term to maturity = n Face amount = F Steps to calculate the bond duration, modified duration, and convexity using BA II Plus/BA II Plus Professional:

350

http://actuary88.com 1 as the unit time. Convert m the annual effective interest rate into the effective rate per coupon period:

Step 1 – Set the per coupon payment period

1

i = (1 + r ) m 1 Step 2 - Set up the cash flow master table: t ( Unit 1 time= ) m

CF ( t )

t CF ( t )

t2

t 2CF ( t )

1 2 3 …

C 2C 3C …

1 4 9 …

C 4C 9C …

mn 1

C C C … C

mn

F+C

( m n 1) C m n(F + C)

( m n 1) 2 (m n)

2

( m n 1) C 2 ( m n) ( F + C ) 2

Step 3- Manually enter the cash flows into Cash Flow Worksheet. Set 1

I = 100i = 100 (1 + r ) m 1 . Calculate the following 3 items:

P=

m n

m n

CF ( t ) v , t

t =1

where v = (1 + r )

t CF ( t ) v ,

m n

t

t =1

t =1

t 2 CF ( t ) vt

1 m

Step 4 – Calculate the duration and convexity using the following formulas:

DMAC =

1mn t CF ( t ) v t m t =1 m n t =1

Convexity =

CF ( t ) vt 1

1

p (1 + r )2

DM O DTextbook =

1mn t CF ( t ) v t m = t =1 P

1 m2

m n t =1

t 2 CF ( t ) vt +

1mn t CF ( t ) vt m t =1

1 DM A C 1+ r

351

http://actuary88.com Alternative method to calculate convexity: Convexity = =

1

(1 + r )

2

1

(1 + r )

m n

1 1

p m2 1 1

2

p m2

t =1

m n t =1

t 2 CF ( t ) vt +

11

m n

pm

t =1

t CF ( t ) vt

t 2 CF ( t ) vt + duration

Problem 2 Bond face Coupon Term to maturity Yield to maturity

$100 4% semiannual 3 years 5% annual effective.

Use BA II Plus Professional Bond Worksheet, calculate the duration (i.e. Macaulay duration) and the modified duration. Solution Please note that Bond Worksheet in BA II Plus can NOT directly calculate the modified duration. BA II Plus Professional does. For this reason, you might want to buy BA II Plus Professional. BA II Plus Professional Bond Worksheet uses the Wall Street convention in quoting a bond by using a nominal yield to maturity. As a result, when Bond Worksheet calculates the price and the modified duration, it uses the nominal yield that compounds as frequently as coupons are paid. We need to find, y , the nominal yield compounding twice a year (coupons are paid twice a year): y 1+ 2

2

= 1 + 5%

y = 4.939%

352

http://actuary88.com Key strokes in BA II Plus Professional: 2nd Bond This activates Bond Worksheet. Enter SDT=1.0100 SDT = settlement date (i.e. purchase date of the bond) This sets SDT=1-01-2000 We arbitrarily set the purchase date of the bond is to 1/1/2000. 1/1/2000 is an easy number to track. Enter CPN=4 Set coupon = 4% of par. Enter RDT=1.0103 RDT = redemption date (or bond’s maturity) This sets RDT=1-01-2003

We set RDT=1-01-2003 (the bond has a 3 year maturity). Redemption value. Because the bond is redeemed at par and the par=100, we set RV=100. Use 360 counting method (i.e. assume a year has 360 days). Don’t use the actual day counting method. 2/Y (i.e. twice a year) Enter 4.939015332, not 4.93901532% (i.e. don’t enter the % sign). We get PRI=97.41125361. This is the bond price. Accrued interest is zero. Don’t worry about this feature. We don’t need it to pass Exam FM. DUR=2.78567468 Remember this is the modified duration under the Wall Street method.

Enter RV=100 Day counting method

Coupon frequency YTD=4.939015332 CPT PRI (compute price of the bond) AI=0 DUR

DMODWall Street = 2.78567468 Next, we will convert the modified duration (Wall Street method) into Macaulay duration. 1

DMODWall Street = 1+

YLD m

( m)

DMAC

353

http://actuary88.com

Wall Street

DMAC = DMOD

= 2.78567468 1 +

YLD ( m) 1+ m 4.93901532% = 2.85446713 2

Finally, we’ll find the modified duration during the textbook definition: 1 DM A C 1 + annual effective yield 1 2.85446713 = DM A C = = 2.71854012 1.05 1.05

DM O DTextbook =

Please note that Bond Worksheet can not calculate the convexity. To calculate convexity, we have to use Cash Flow Worksheet or use a formula-driven approach. Problem 3 Bond face Coupon Term to maturity Yield to maturity Redemption

$100 4% semiannual 3 years 5% annual effective. $110

Use BA II Plus Professional Bond Worksheet, calculate the duration (i.e. Macaulay duration) and the modified duration. Solution Compared with Problem 2, Problem 3 has a bond not deemed at par. When calculating the duration of a bond not deemed at par using BA II Plus Professional Bond Worksheet, we need to convert such a bond to a bond redeemed at par. Such as conversion is needed because BA II Plus Professional Bond Worksheet can NOT calculate the duration of a bond not redeemed at par. If we don’t convert a non-par bond to a par bond, BA II Plus will give a wrong result. Let’ see.

354

http://actuary88.com Key strokes in BA II Plus Professional Bond Worksheet: 2nd Bond This activates Bond Worksheet. Enter SDT=1.0100 We arbitrarily set the purchase This sets SDT=1-01-2000 date of the bond is to 1/1/2000. Enter CPN=4 Set coupon = 4% of par. Enter RDT=1.0103 RDT = redemption date This sets RDT=1-01-2003

We set RDT=1-01-2003 (the bond has a 3 year maturity). Redemption value. Use 360 counting method 2/Y (i.e. twice a year) Don’t enter 4.93901532% We get PRI=106.0496293. This is the bond price. This price is correct. Accrued interest is zero. DUR=2.78567468

Enter RV=110 Day counting method Coupon frequency YLD=4.939015332 CPT PRI (compute price of the bond) AI=0 DUR

Notice anything strange here? Even though the bond in Problem 3 is different from the bond in Problem 2, BA II Plus gives us the same modified duration (Wall Street version of modified duration). Something must be wrong. If we use this modified duration (Wall Street version), then the Macaulay duration under the textbook definition is:

DMODWall Street =

1

YLD( 1+ m

Wall Street

DMAC = DMOD

= 2.78567468 1 +

m)

DMAC

YLD ( m) 1+ m 4.93901532% = 2.85446713 2

We know this figure is wrong. Next, let’s calculate the bond’s real duration using Cash Flow Worksheet.

355

http://actuary88.com Cash flow diagram: Unit time = 0.5 year Time t

0

Cash flow

1

2

3

4

5

6

$2

$2

$2

$2

$2

$2 $110

The effective interest rate per coupon period is i = 1.05 1 = 2.4695% . We’ll use the generic procedure described in Problem 2 to find the duration. First, we come up with following cash flow table (we are using 6 months a one unit time): t

CF ( t )

t CF ( t )

1 2 3 4 5 6

2 2 2 2 2 112

2 4 6 8 10 672

Using Cash Flow Worksheet, we find: 6 t =1

CF ( t ) vt = 106.04962959 , 6

DMAC =

1 2

t =1 6

t CF ( t ) v t

t =1

CF ( t ) vt

=

6 t =1

t CF ( t ) v t = 607.94469918

1 607.94469918 = 2.866321653 2 106.04962959

So the correct duration is 2.866321653 . The modified duration is:

DM O DTextbook =

1 2.866321653 DM A C = = 2.72983 1 + annual effective yield 1.05

We see that BA II Plus Professional calculates a bond’s duration assuming a bond is always deemed at par, no matter what redemption value is. For example, in this problem you can enter RV=0 or any other non-negative number and you’ll still get DUR=2.78567468. You can check this for yourself. We can overcome this issue by converting a non-par bond into a par bond. In this problem, the conversion goes like this: 356

http://actuary88.com

original coupon rate × par 4% × 100 = = 3.6366364% redemption value 110

new coupon rate =

This will give us the right duration. Revised key strokes in BA II Plus Professional Bond Worksheet: 2nd Bond Enter SDT=1.0100 This sets SDT=1-01-2000 Enter CPN=3.63636364 Enter RDT=1.0103

This activates Bond Worksheet. We arbitrarily set the purchase date of the bond is to 1/1/2000. Revised coupon rate RDT = redemption date

This sets RDT=1-01-2003

We set RDT=1-01-2003 (the bond has a 3 year maturity). Redemption value. You can even enter RV=0. This won’t affect the result. Use 360 counting method 2/Y (i.e. twice a year) Don’t enter 4.93901532% If you compute the bond’s price, you’ll get a garbage number because we convert the original bond. So ignore PRI. Accrued interest is zero. DUR=2.79721351 (This is Wall Street’s version of the modified duration)

Enter RV=110 Day counting method Coupon frequency YLD=4.939015332 PRI

AI=0 DUR

Next, we’ll convert the Wall Street’s modified duration into Macaulay duration:

DMODWall Street =

1

YLD( 1+ m

Wall Street

DMAC = DMOD

= 2.79721351 1 +

m)

DMAC

YLD ( m) 1+ m 4.93901532% = 2.79721351 1.05 = 2.86636165 2

357

http://actuary88.com This number matches the duration calculated using Cash Flow Worksheet. Key point to remember: If you ever use BA II Plus Professional Bond Worksheet to calculate a bond’s duration, whether the bond is redeemed at par or not, always calculate the new coupon rate new coupon rate =

original coupon rate × par redemption value

If the bond is redeemed at par, the new coupon rate = original coupon rate. Enter this new coupon rate into Bond Worksheet. Next, convert the Wall Street modified duration into Macaulay duration. Problem 4 Bond face Coupon Term to maturity Yield to maturity Redemption Value

$100 4% semiannual 3 years 5% annual effective. $95

Calculate the bond’s duration using BA II Plus Professional Bond Worksheet. Solution new coupon rate =

original coupon rate × par 4% × 100 = = 4.21052632% redemption value 95

358

http://actuary88.com 2nd Bond Enter SDT=1.0100 This sets SDT=1-01-2000 Enter CPN=4.21052632 Enter RDT=1.0103

This activates Bond Worksheet. We arbitrarily set the purchase date of the bond is to 1/1/2000. Revised coupon rate RDT = redemption date

This sets RDT=1-01-2003

We set RDT=1-01-2003 (the bond has a 3 year maturity).

Enter RV= any non-negative number (such as zero or 0.1) Day counting method Coupon frequency YLD=4.939015332 PRI

Use 360 counting method 2/Y (i.e. twice a year) Don’t enter 4.93901532% If you compute the bond’s price, you’ll get a garbage number because we convert the original bond. So ignore PRI. Accrued interest is zero. DUR=2.77908513 (This is Wall Street’s version of the modified duration)

AI=0 DUR

Next, we’ll convert the Wall Street’s modified duration into Macaulay duration:

DMODWall Street =

1

YLD( 1+ m

Wall Street

DMAC = DMOD

= 2.77908513 1 +

m)

DMAC

YLD ( m) 1+ m 4.93901532% = 2.77908513 1.05 = 2.84771485 2

Problem 5 Bond face Coupon Term to maturity Yield to maturity

$100 4% semiannual 3 years 5% annual effective.

359

http://actuary88.com Use the formula driven approach, calculate the duration, the modified duration, and convexity. Solution Unit time = 0.5 year Time t

0

Cash flow

1

2

3

4

5

6

$2

$2

$2

$2

$2

$2 $100

The effective interest rate per coupon period is i = 1.05 1 = 2.4695% .

DMAC =

1 2

6 t =1 6 t =1

6 t =1 6 t =1

t CF ( t ) v t CF ( t ) v t

CF ( t ) vt = 2a6 + 100v 6 @ i = 2.4695% t CF ( t ) v t = 2 ( Ia )6 + 6 (100 ) v 6 @ i = 2.4695%

DMAC =

1 2

6 t =1 6 t =1

=

t CF ( t ) v t CF ( t ) v t

6 1 2 ( Ia )6 + 6 (100 ) v = 2a6 + 100v 6 2

1 556.1144433 = 2.85446713 2 97.41125361

DM O DTextbook =

=

1 DM A C 1 + annual effective yield

1 2.85446713 DM A C = = 2.71854012 1.05 1.05

However, convexity is nasty to calculate.

Convexity =

1

1

p (1 + 5% )

2

1 4

6 t =1

t 2 CF ( t ) vt +

1 2

6 t =1

t CF ( t ) vt

360

http://actuary88.com The tricky part is to evaluate 6 t =1

t 2 CF ( t ) vt = 2

6 t =1

t 2 vt + 62 (100 ) v 6

Though Kellison gives us a formula for evaluating

n

t 2 vt (his formula

t =1

9.23), such a formula is unwieldy and not worth memorizing – so do NOT memorize it. Because here we have n = 6 (not too big), we simply calculate 6

t 2 vt directly without using any formulas.

t =1

6

t CF ( t ) v = 1 ( 2 ) 1.05 2

t =1

t

2

+ 42 ( 2 ) 1.05 6 t =1

1 2

4 2

+ 2 ( 2 ) 1.05 2

+ 52 ( 2 ) 1.05

5 2

2 2

+ 3 ( 2 ) 1.05 2

+ 62 (102 ) 1.05

3 2

6 2

t 2 CF ( t ) vt = 3,271.595675

We already know that 6 t =1

t CF ( t ) v t = 556.1144433

P=

6 t =1

CF ( t ) vt = 97.41125361

Convexity = =

1

1

p (1 + 5% )

2

1

1 4

6 t =1

t 2 CF ( t ) vt +

1

97.41125361 (1 + 5% )

2

1 2

6 t =1

t CF ( t ) vt

1 1 ( 3,271.595675 ) + ( 556.1144433) 4 2

= 10.2048222

361

http://actuary88.com Alternatively,

Convexity =

1

(1 + r )

2

1

n

p

t =1

t 2 CF ( t ) v t + Duration

Unit time is one year

=

1

(1 + r )

2

1

6

p

t =1

1 2 t CF ( t ) vt + Duration 4

Unit time = half a year

=

1

(1 + 5% )

1 ( 3,271.595675) + 2.85446713 = 10.2048222 97.41125361 4 1

2

I think it’s unlikely that SOA will ask you to calculate the convexity of a coupon bond with a long maturity; the calculation is too intensive. However, SOA can ask you to calculate the convexity of a simple bond such as • •

a zero bond 1-year or 2-year bond, with coupons payable annually or semiannually

Make sure you know how to calculate. In addition, you need to be able to quickly calculate the duration of a regular bond. Duration is always easy to calculate. Problem 6 Bond face Coupon Term to maturity Yield to maturity Redemption

$100 6% semiannual 4 years 8% annual effective. $105

Calculate the bond’s duration using the following method: • BA II Plus/BA II Plus Professional Cash Flow Worksheet • BA II Plus Professional Bond Worksheet • Formula-driven approach I’ll let you solve the problem. The correct answer is: 3.61512007

362

http://actuary88.com Problem 7 Bond face Coupon Term to maturity Yield to maturity Redemption

$100 6% semiannual 4 years 8% annual effective. $92

Calculate the bond’s duration using the following method: • BA II Plus/BA II Plus Professional Cash Flow Worksheet • BA II Plus Professional Bond Worksheet • Formula-driven approach I’ll let you solve the problem. The correct answer is: 3.573273749 Problem 8 Bond face Coupon Term to maturity Yield to maturity Duration Convexity Bond price

$100 4% semiannual 3 years 5% annual effective. 2.854 10.205 97.41

Calculate the bond updated price if the yield to maturity changes to 6% annual effective. Solution We’ll calculate the new price twice. First time, we’ll use the duration and convexity. The second time, we’ll directly calculate the price without using the duration and convexity. We’ll compare the two results. Find the new price using the duration and convexity:

P P

2 2 1 dP 1 1 d 2P 1 1 r+ DMAC r + ( Convexity )( r ) 2 ( r) = P dr 2 P dr 1+ r 2

We have:

r = 5% ,

r = 1% , P = 97.41 , DMAC = 2.854 , Convexity = 10.205

363

http://actuary88.com

P P

2 1 1 DMAC r + ( Convexity )( r ) 1+ r 2

P P

2 1 1 ( 2.854 )1% + (10.205)(1% ) = 2.667% 1 + 5% 2

P ( 2.667% )

P

P' = P + P

P (1 2.667% ) = 97.41(1 2.667% ) = 94.81

If we ignore the convexity, we’ll have:

P P

1 1 DMAC r = ( 2.854 )1% = 2.718% 1+ r 1 + 5% P ' = P + P P (1 2.718% ) = 97.41(1 2.718% ) = 94.76

Next, we directly calculate the new price: P ' = 2a6 + 100v 6

@ i = 1 + 6% 1 = 2.9563%

P ' = 2a6 + 100v 6 = 94.81 We see that for a small change of the yield, the duration and convexity are good at predicting the change of the bond price. Problem 9 (SOA May 2003 Course 6 #5 simplified) You are given the following with respect to a five-year bond: • annual coupons of ( 2 + t ) % are payable at the end of each year • par value of $1,000 • yield-to-maturity ( y ) of 5.5% Calculate the Macaulay duration. Solution 5

DMAC =

t =1 5 t =1

t CF ( t ) v t CF ( t ) v t

364

http://actuary88.com Method #1 – Use Cash Flow Worksheet: Cash flow table: t

CF ( t )

t CF ( t )

1 2 3 4 5

(2+1)%(1,000)=30 (2+2)%(1,000)=40 (2+3)%(1,000)=50 (2+4)%(1,000)=60 (2+5)%(1,000)+1,000=1,070

30 80 150 240 5,350

First, we’ll find the bond’s price. We enter CF ( t ) into Cash Flow Worksheet:

t

CF ( t )

Cash flows

1 2 3 4 5

$30 $40 $50 $60 $1,070

CF1 CF2 CF3 CF4 CF5

i = 5.5% 974.0815620

Interest Rate per coupon period Calculate NPV 5 t =1

To calculate

Cash flow frequency 1 1 1 1 1

CF ( t ) v t =974.0815620 5 t =1

t CF ( t ) vt , we enter t CF ( t ) (bold numbers below) into

Cash Flow Worksheet:

t

CF ( t )

t CF ( t )

1 2 3 4 5

(2+1)%(1,000)=30 (2+2)%(1,000)=40 (2+3)%(1,000)=50 (2+4)%(1,000)=60 (2+5)%(1,000)+1,000=1,070

30 80 150 240 5,350

Interest Rate per coupon period Calculate NPV

Cash flows CF1 CF2 CF3 CF4 CF5

Cash flow frequency 1 1 1 1 1

i = 5.5% 4,515.255073

365

http://actuary88.com 5 t =1 5

DMAC =

t =1 5 t =1

t CF ( t ) vt = 4,515.255073 t CF ( t ) v t CF ( t ) v t

=

4,515.255073 = 4.63539733 974.0815620

Method #2 – directly calculate 5

DMAC =

t =1 5 t =1

5 t =1

5 t =1

t CF ( t ) v t CF ( t ) v t

t CF ( t ) v t =

CF ( t ) vt =

1( 30 ) 2 ( 40 ) 3 ( 50 ) 4 ( 60 ) 5 (1070 ) + + + + =4,515.255073 1.055 1.0552 1.0553 1.0554 1.0555

5

DMAC =

30 40 50 60 1070 + + + + =974.0815620 2 3 4 1.055 1.055 1.055 1.055 1.0555

t =1 5 t =1

t CF ( t ) v t CF ( t ) v t

=

4,515.255073 = 4.63539733 974.0815620

Please note that for this problem we can NOT use the bond worksheet in BA II Plus Professional to calculate the duration; we don’t have a standard bond here. Problem 10 What’s the duration of a 5 year zero coupon bond? [A] 4 years [B] 5 years [C] 6 years [D] 7 years [E] 8 years

366

http://actuary88.com Solution The duration of a zero coupon bond is always equal to its maturity. The correct answer is [B] Problem 11 What’s the modified duration of a 20 year zero coupon bond, if the yield to maturity is 10% annual effective? Solution DM O DTextbook =

=

1 DM A C 1 + annual effective yield 1 20 DM A C = = 18.18 (years) 1.1 1.1

Since the coupons (which happen to be zero) are paid annually, the nominal yield to maturity compounding annually is the same as the annual effective yield. Consequently, the modified duration under the Wall Street method is the same as the one under the textbook method. DM O DWall Street =

=

1 DM A C 1 + annual nominal yield

1 20 DM A C = = 18.18 (years) 1.1 1.1

Problem 12 8 years ago, Mark bought a 15 zero coupon bond. Today, while the bond still has 7 years to maturity, Mark sold it to John. X = the duration of Mark’s bond. Y = the duration of John’s bond.

Calculate X Y . Solution The duration of a zero coupon bond is always it maturity. So X = 15 years and Y = 7 years. X Y = 8 .

367

http://actuary88.com Problem 13 Cash flows

perpetual annuity immediate

Interest rate

5% annual effective

Calculate the duration of the cash flows. Solution The key formula is

DMAC =

P=

1 r

dp p = d (1 + r ) 1+ r

(1 + r )

1 dp p dr

(This is the price of a perpetual annuity immediate)

dP d 1 = = dr dr r

DMAC =

(1 + r ) 1 r

1 r2

1 dp = p dr

DMAC = 1 + = 1 +

(1 + r )

r

1 r2

=

1+ r 1 = 1+ r r

1 = 21 5%

Problem 14 Calculate the duration and the convexity of the liability, given: • The liability has a continuous payment stream of $5,000 per year over the next ten year • The interest rate is 6% annual effective

368

http://actuary88.com Solution For a continuous payment stream, n

Duration =

n

t v t CF ( t ) dt

0

n

v t CF ( t ) dt

=

t v t CF ( t ) dt

0

P

0

Convexity =

n

1

1

p (1 + r )2

t ( t + 1) CF ( t ) v t dr =

0

1

1

p (1 + r ) 2

n

n

t 2 CF ( t ) v t dr + t CF ( t ) v t dr

0

0

If CF ( t ) = c where c is constant for any t , then n

c t v t dt Duration =

=

0

n

c v t dt

( Ia ) an

n

=

( Ia )

10

a10

0

( Ia )10 = a10

=

i

a10

10v10

a10 =

6% ( 7.36008705 ) = 7.57874546 ln1.06

In the above, a10 is calculated using BA II Plus/BA II Plus Professional TVM.

( Ia )

10

=

a10 10v10

Duration =

( Ia )

10

a10

=

=

7.57874546 10 (1.06 ln1.06

10

) = 34.23434140

34.23434140 = 4.51715150 7.57874546

Next, we’ll calculate the convexity.

369

http://actuary88.com

Convexity =

1 1 2 n (1 + r ) CF (t ) v t dr

n

n

t 2 CF ( t ) v t dr + t CF ( t ) v t dr

0

0

0

Because the cash flow is constant, we have: n

n

Convexity =

1 2 (1 + r )

n

t v dr + t v dr t

2

t

0

=

0 n

v t dr

1 2 (1 + r )

t 2 v t dr + ( Ia )

0

n

an

0

n

n

t v dt = t e 2

t

t

2

0

1

dt =

0

n t

dt 2 = 2 te

0

2

t de 0

n

e

n

t

t

=

1

n

n

e

t

dt

2

2

t e

t

0

dt = 2 ( Ia )n ,

0

t2 e

t

n 0

= n2e

n

= n 2v n

0

n

t v dt = 2

t

0

10

t v dt = 2

t

2 ( Ia )n

n2v n

2 ( Ia )10

102 v n

0

n

2 ( 34.23434140 ) 1021.06 = ln1.06

10

= 216.7400337

t 2 v t dr + ( Ia )

n 1 0 2 an (1 + r ) 1 216.7400337 + 34.23434140 = = 29.47272758 1.062 7.57874546

Convexity =

Alternatively, Convexity = =

1

1

(1 + r ) 1 1.06

2

2

p

n

t 2 CF ( t ) v t dr + Duration

0

216.7400337 + 4.51715150 = 29.47272758 7.57874546

370

http://actuary88.com Problem 15 (#6 May 2005 FM) John purchased three bonds to form a portfolio as follows: •

Bond A has semi-annual coupons at 4%, a duration of 21.46 years, and was purchased for 980.



Bond B is a 15-year bond with a duration of 12.35 years and was purchased for 1015.



Bond C has a duration of 16.67 years and was purchased for 1000.

Calculate the duration of the portfolio at the time of purchase. (A) (B) (C) (D) (E)

16.62 16.67 16.72 16.77 16.82

years years years years years

Solution As a general rule, the duration (or convexity) of a portfolio is the weighted average duration (or convexity) of the assets, with weight being the present value of each asset. n

porfolio duration =

k =1

PVk × Duration k n k =1

n

porfolio convexity =

k =1

PVk

PVk × Convexityk n k =1

PVk

Let’s prove this. To make our proof simple, let’s assume that a portfolio consists of two assets, A and B . The proof is the same if we have more than two assets.

371

http://actuary88.com The duration of the portfolio consisting of two assets is: 1 d PV PV di

(1 + i )

D=

PV = PVA + PVB

1 d D = (1 + i ) ( PVA + PVB ) = PVA + PVB di

(1 + i ) PVA =

But

(1 + i ) D=

d d PVA + PVB di (1 + i ) di PVA + PVB

1 d PVA + (1 + i ) PVB PVA di PVA + PVB

1 d PVA = DA , PVA di

(1 + i )

1 d PVB PVB di

1 d PVB = DB PVB di

PVA DA + PVB DB PVA PVB = DA + DB PVA + PVB PVA + PVB PVA + PVB

Similarly, we can show that the convexity of the portfolio is the weighted average convexity with weights being the present value of each asset. C=

1 d2 PV PV di 2

PV = PVA + PVB C=

2

1 d ( PVA + PVB ) = PVA + PVB di 2

PVA =

But

1 d2 PVA + PVB PVA di 2 PVA + PVB

1 d2 PVA = C A , PVA di 2

d2 d2 PV PVB + A di 2 di 2 PVA + PVB

1 d2 PVB PVB di 2

1 d2 PVB = CB PVB di 2

372

http://actuary88.com

C=

PVAC A + PVB CB PVA PVB = CA + CB PVA + PVB PVA + PVB PVA + PVB

Come back to the problem. Asset #1 #2 #3

PV 980 1,015 1,000

Duration 21.46 12.35 16.67

The weighted average duration is:

980 ( 21.46 ) + 1,015 (12.35 ) + 1,000 (16.67 ) = 16.7733 980 + 1,015 + 1,000 So the answer is [D] Problem #16 A portfolio consists of the following three assets: Asset #1 #2 #3

PV 1,000 1,500 2,000

Convexity 20 32 15

Calculate the portfolio’s convexity. Solution The weighted average convexity is:

1, 000 ( 20 ) + 1,500 ( 32 ) + 2,000 (15 ) = 21.78 1, 000 + 1,500 + 2,000 So the portfolio’s convexity is 21.78. Problem #17 (SOA May 2000 EA-1 #8) = 0.07

373

http://actuary88.com The modified duration of a 20-year bond with 7% annual coupons with a maturity and par value of $1,000 is y . Calculate y . Solution e = 1+ i ,

DMOD =

i=e

1 = e0.07 1 = 7.25% (annual effective interest rate)

1 D 1 + i MAC

Let’s first calculate DMAC . time t (year) CF ( t )

20

DMAC =

t =1 20 t =1

DMOD =

0

t CF ( t ) v t CF ( t ) v

=

1

2

3



20

$70

$70

$70

$70

$1,070

70 ( Ia ) 20 i + ( 20 )1, 000v 20

t

1 D = 11.22 e 1 + i MAC

70 a20 i + 1, 000v 20

0.07

=

70 ( 85.67388 ) + 4,931.93928 973.9385

= 11.22

= 10.46

Problem #18 (SOA May 2001 EA-1 #13) Purchase date of a perpetuity:

1/1/2001

Date of the 1st payment:

12/31/2001

Frequency of payments:

Annual

Amount of each payment:

$1

Interest rate:

6% per year, compounded annually

Calculate the absolute value of the difference between the modified duration of the perpetuity and the present value of the perpetuity. Solution

374

http://actuary88.com The present value of perpetuity immediate is P =

1 i

The Macaulay duration of perpetuity immediate is:

(1 + i )

D MAC =

1 dP = P di

(1 + i ) i

d 1 = di i

(1 + i ) i

1 1+ i = 2 i i

The modified duration is: D MOD =

D MAC 1 1+ i 1 = × = 1+ i 1+ i i i

P D MOD =

1 1 =0 i i

375

http://actuary88.com

Chapter 17

Immunization

The need for immunization • Insurance companies often have to pay prescheduled payments in the future. •

For example, some insurance companies offer CD like investment products called GIC (guaranteed investment contracts). In GIC, investor deposits cash to the GIC account, which earns a guaranteed interest rate for a specific period of time. For example, an investor deposited $10,000 today. The GIC offers a guaranteed interest rate of 8% for 5 years. Then at the end of Year 5, the insurance company must pay the investor $10,000(1+8%)5=$14,693.28.



In this GIC, the insurance company promises to pay the investor 8% annual effective. So the insurance company needs to earn at least 8% annual effective to break even.



How can the insurance company invest its collected deposit of $10,000 wisely so it can earn at least 8% for 5 years?



At first glance, we might suggest that the insurance company buys, at time zero, a 5-year bond with 8% annual coupon and $10,000 face amount. Assume the current market interest rate is 8%. Then this bond costs exactly $10,000 at t = 0 . And it generates exactly $14,693.28 at t = 5 .



Let’s calculate the total cash flow at t = 5 if we bought the bond at t = 0 . Assume the market interest rat is 8%. Then we can reinvest each coupon of $800 at 8%.



The total cash flow at t = 5 : 800 s5 8% + 10, 000 = 14, 693.28 This amount exactly offsets our payment of $14,693.28. Time t

Cash flow



0

1

$800

2

$800

3

4

$800

$800

5

$10,800

After more analysis, however, we realize that buying this bond may not work. What if that immediately after we bought the bond, the

376

http://actuary88.com interest drops to 7% and stays at 7%? If this happens, our total accumulated cash flow at t = 5 is: 800 s5 7% + 10, 000 = 14, 600.59 < 14, 693.28 •

The accumulated value is lower because now we have to reinvest the coupons at 7% (instead of 8%).



A general question arises. In many occasions, an insurance company has to pay its prescheduled payments in the future. However, the returns the insurance company gets from its investment are volatile, depending on the current market interest rate. If the interest rates are too volatile, the insurance company won’t have sufficient investment income to pay its fixed liabilities.



How can the insurance company invest its collected premiums in such a way that no matter how the interest rate changes, it always has enough money to pay its prescheduled liabilities in the future?



This led many actuaries to search for an investment strategy that will produce a guaranteed level of cash flows no matter what happens to the interest rate. One strategy is immunization.

Basic ideas behind immunization • First, we don’t worry that the interest rate may change. At time zero, we simply hold assets that exactly offset liabilities. We make this happen by forcing PV of Assets = PV of Liabilities. This ensures that we have enough assets to pay our liabilities if the interest rate does not change. •

Next, we’ll consider the possibility that the interest rate may change. We can’t stop the interest rate from going up or down, but we can make our assets and liabilities equally sensitive to the change of the interest rate. If assets and liabilities can increase and decrease by roughly the same amount for a given change in the interest rate, then the market value of the asset and the market value of the liability will offset each other. As a result, our assets will be enough to pay our liabilities.



To make our assets and liabilities equally sensitive to the change of the interest rate, we first force our assets and liabilities to have an equal duration. Duration is the 1st derivative of the interest rate. As a result, we want assets and liabilities to match duration.



Next, we force our assets and liabilities have an equal second derivative (convexity) as to the interest rate. To be safe, we force 377

http://actuary88.com our assets to have a slightly bigger second derivative than our liabilities does. This ensures that for a given change in the interest rate, our assets change by a slightly bigger amount than our liabilities. •

We can follow this line of thinking and force assets and liabilities match 3rd or 4th derivatives to the interest rate. However, matching assets and liabilities in their 3rd or 4th derivatives is more difficult and more expensive than matching the 1st and 2nd derivatives.

Sample problems and solutions Problem 1 You have a leaky roof. Every time it rains, water drips down from the ceiling. In the past every time you decided to have the roof fixed, something came up; you used up all the money you had and had no money left to repair the roof. Finally, you had enough. You vowed to repair the leaky roof. These are the facts: • According to the weather forecast, there will not be any rainfall in the next five years in your town. However, there will be a big rain 5 years and 10 days from today. •

You decide to have the roof fixed 5 years from now (i.e. at t = 5 ).



The cost of having the roof fixed at t = 5 is $10,000.



Your only investment opportunity is in your local bank, which offers only two products -- saving accounts and CD (certified deposit). Besides your local bank, there are no other investment opportunities for you.



A saving account in your local bank earns an interest rate adjustable once every 6 months. Every 6 months, the board of directors of your local bank sets an interest rate according to their whims. For example, on January 1 last year, the board of directors felt the economy was good and decided that all saving accounts should earn a 10% annual effective interest rate for the next 6 months. However, on July 1, the board of directors felt that 10% was too high and declared that all saving accounts should earn a 1% annual effective interest rate per year for the next 6 months.

378

http://actuary88.com •

CD offered by your local bank has a 5 year term and earns a guaranteed annual effective interest rate of 3%.

Not to repeat your past failures, you decide to immunize your cost of repairing the roof at t = 5 . Explain what immunization means in this case. Design an immunization strategy. Solution Immunization in this case means that you invest money somewhere at t = 0 to generate exactly $10,000 at t = 5 , no matter how the interest rate changes during the next five years. If you can have exactly $10,000 at t = 5 no matter how low the interest can be, you have secured your future payment of $10,000 at t = 5 . You have only 2 investment options – investing in a saving account or in a 5 year CD. Investing in a saving account will not work. The interest rate you earn in a saving account is unpredictable for the next 5 years. Unless you deposit $10,000 at t = 0 , there’s no way to guarantee that you will have $10,000 at t = 5 . You can invest your money in the 5 year CD. Because the 5 year CD offers a guaranteed interest rate of 3%, your initial deposit at t = 0 should be: 10, 000 (1 + 3% ) = 8, 626.09 5

If you invest 8,626.09 in the 5 year CD, you are guaranteed to have $10,000 at t = 5 . You have immunized your payment of $10,000 at t = 5 against any adverse interest rate changes. In other words, you have locked in a guaranteed interest rate of 3% no matter how low the market interest rate can be. Comment one: If a saving account in your local bank offers a guaranteed interest rate, then you can invest in a saving account and immunize your future payment at t = 5 . For example, if the saving account offers a guaranteed a 2% annual effective interest rate, then your initial deposit at t = 0 should 5 be: 10, 000 (1 + 2% ) = 9, 057.31 If you invest $9,057.31 in a saving account today, you are 100% certain to get $10,000 at t = 5 . You have immunized your payment of $10,000 at

379

http://actuary88.com t = 5 . However, this immunization strategy is less favorable than if you invest in a 5 year CD.

Comment Two: Immunization typically means investing money in bonds, not in a saving account or CD, because the interest rate generated by a saving account or CD is too low. In this problem, we use a saving account and CD as simple examples to illustrate the essence of immunization. Essence of immunization When the interest is volatile, we need to find ways to lock in a guaranteed return and accumulate just enough money to exactly pay our future liability. Problem 2 You have a leaky roof. Every time it rains, water drips down from the ceiling. In the past every time you decided to have the roof fixed, something came up; you used up all the money you had and had no money left to repair the roof. Finally, you had enough. You vowed to repair the leaky roof. These are the facts: • According to the weather forecast, there will not be any rainfall in the next five years in your town. However, there will be a big rain 5 years and 10 days from today. •

You decide to have the roof fixed 5 years from now (i.e. at t = 5 ).



The cost of having the roof fixed at t = 5 is $10,000.



The bond market offers a 5 year zero-coupon bond with $10,000 face amount yielding 6% annual effective.

Not to repeat your past failures, you decide to immunize your cost of repairing the roof at t = 5 . Design an immunization strategy. Solution Your goal is to invest money somewhere today to accumulate exactly $10,000 at t = 5 . If you buy this 5 year zero bond with $10,000 par value today, you are guaranteed to have $10,000 at t = 5 . The market price of the bond is: 10, 000 (1 + 6% ) = 7, 472.58 5

380

http://actuary88.com To immunize your future payment of $10,000 at t = 5 , you should buy this 5 year zero bond at $7,472.58 today. By doing so, you have locked in a 6% interest rate, no matter how low the market interest can be. Problem 3 You have a leaky roof. Every time it rains, water drips down from the ceiling. In the past every time you decided to have the roof fixed, something came up; you used up all the money you had and had no money left to repair the roof. Finally, you had enough. You vowed to repair the leaky roof. These are the facts: • According to the weather forecast, there will not be any rainfall in the next five years in your town. However, there will be a big rain 5 years and 10 days from today. •

You decide to have the roof fixed 5 years from now (i.e. at t = 5 ).



The cost of having the roof fixed at t = 0 is $10,000. Due to inflation and the shortage of roof repair skills, the cost of labor and materials increases by 8% per year.



To fund your repair cost, at t = 0 you bought a 5 year $10,000 par value bond that offers annual coupons of 8%. The current market interest rate is also 8%.

Analyze whether your bond will generate sufficient fund to pay the repair cost t = 5 under the following scenarios: (1) the market interest rate stays at 8% forever. (2) immediately after you have bought the bond, the market interest rate drops to 7.5% and stays at 7.5%. (3) immediately after you have bought the bond, the market interest rate rises to 8.5% and stays at 8.5%. Solution Scenario 1 – the market interest rate stays at 8%. Let’s first calculate your repair cost at t = 5 . If you repair your roof now, you pay $10,000. If you repair it at t = 5 , you’ll pay 5 10, 000 (1 + 8% ) = 14, 693.28 Next, let’s see how much money your bond can accumulate at t = 5 . The bond pays you 5 annual coupons of $800 each. You can reinvent these 381

http://actuary88.com coupons at the market rate of 8% and accumulate to 800 s5 8% at t = 5 . In addition, you’ll get a payment of $10,000 at t = 5 . Your total money at t = 5 is: 800 s5 8% + 10, 000 = 14, 693.28 Your bond will accumulate just enough money to pay your repair cost. Scenario 2 – immediately after you bought the bond, the market interest rate drops to 7.5% and stays at 7.5%. You still get 5 annual coupons each worth $800, but this time you can reinvest them only at 7.5%. Your accumulated value at t = 5 is: 800 s5 7.5% + 10, 000 = 14, 646.71 Your repair cost at t = 5 is still $14,693.28. Your shortfall: $14,693.28 - $14,647.71 = $45.57 Why this time don’t you have enough money to pay your repair cost at t = 5 ? Because when the interest rate drops to 7.5%, you can no longer reinvest your coupons at 8%. However, to come up with $14,693.28, you must be able to reinvest your coupons at least at 8%. Scenario 3 – the market interest rate rises to 8.5% immediately after you bought the bond and stays at 8.5%. This time, you can reinvest your coupons at 8.5%. Your accumulated value at t = 5 is: 800 s5 8.5% + 10, 000 = 14, 740.30 You’ll have more than enough to pay the repair cost at t = 5 . Your surplus at t = 5 is $14,740.30 – $14,693.28 = $47.02 Moral of this problem: When you first bought your asset (5 year 8% annual coupon with par $10,000), at the then market interest rate of 8%, your asset will generate the exact amount of money to pay your repair cost at t = 5 . However, if the interest rate goes up or down after you bought your asset, your asset will accumulate more or less money than your payment at t = 5 . While you are happy if you end up with more than you need at t = 5 , you’ll be sad if you incur a loss at t = 5 .

382

http://actuary88.com Now imagine a bank or an insurance company that must pay $10,000,000,000 five years from now. If the company does not diligently manage the volatility of the interest rate, it may loss millions of dollars. This is why immunization is important. Problem 4 5 You want to invest your money now to have 10, 000 (1 + 8% ) = 14, 693.28 at t = 5 to fix your leaky roof. Originally, you were thinking of buying a 5 year bond with $10,000 par and 8% annual coupons. Then you did some math (like in Problem 3). You realized that if the interest rate drops immediately after you bought the bond, you won’t be able to have $14,693.28 at t = 5 .

Then an immunization wizard advises you to buy a 6 year bond with $10,000 par and 8% annual coupons. He says that if you buy this bond, even if the interest rate rises or drops a bit, you will always be able to accumulate $14,693.28 at t = 5 . Test the validity of the wizard’s advice under the following scenarios: • Immediately after you have bought the bond, the market interest rate drops to 7.5% and stays at 7.5%. •

Immediately after you have bought the bond, the market interest rate rises to 8.5% and stays at 8.5%.

Explain your findings. Solution Let’s calculate the accumulated value at t = 5 under the two scenarios: Scenario 1 - Immediately after you have bought the bond, the market interest rate drops to 7.5% and stays at 7.5%. Your money at t = 5 comes from two sources. First, you can reinvest your annual coupons at 7.5% and let them accumulate to t = 5 . Second, you can sell your bond at t = 5 . At t = 5 , your bond still has 1 year term left with an incoming cash flow of $10,800 at t = 6 . You can sell this cash flow at 7.5% interest rate. Your total money at t = 5 is: 800 s5 7.5% +

10,800 = 14, 693.22 14, 693. 28 1.075

383

http://actuary88.com

Scenario 2 - Immediately after you have bought the bond, the market interest rate drops to 8.5% and stays at 8.5%. Your total money at t = 5 is: 800 s5 8.5% +

10,800 = 14, 694.22 14, 693.28 1.085

Let’s analyze why you can always accumulate 10, 000 (1 + 8% ) = 14, 693.28 at 5

t = 5 no matter the interest rate rises or falls. You bought a 6 year bond with $10,000 par and 8% annual coupons

The accumulated value at t = 5 by reinvesting coupons at the market interest rate.

The sales price of the bond’s remaining cash flow (a zero coupon bond with cash flow of $10,800 one year from now)

Total

Scenario #1

800 s5 8% = 4, 693.28

$10,800/1.08=$10,000

$4,693.28+$10,000 = $14,693.28

The market interest rate stays at 8% (this should exactly accumulate the needed amount) Scenario #2 Immediately after you bought the bond, the market interest rate dropped to 7.5% and stayed at 7.5%

Scenario #3 Immediately after you bought the bond, the market interest rate rose to 8.5% and stayed at 8.5%

We sell the remaining zero bond at par.

800 s5 7.5% = 4, 646.71

$10,800/1.075=$10,046.51

$4,646.71+10,046.51 = $14,693.22 .

Compare Scenario #2 and #1 Compare Scenario #2 and #1 Decrease = 4,693.28-4,646.71 = 46.57 800 s5 8.5% = 4, 740.30

Compare Scenario #3 and #1 Increase = 4,740.30-4,693.28 = 47.02

Increase: =$10,0046.51 -$10,000 = $46.51 We sell the remaining zero bond at a premium of $46.51. $10,800/1.085=$9,953.92

Loss due to investing coupons at a lower rate is almost exactly offset by the gain of discounting the bond’s remaining cash flow at a lower rate. $4,740.30+9,953.92 = $14,694.22.

Compare Scenario #3 and #1 Decrease: =$10,000 - $9,953.92 = $46.08 We sell the remaining zero bond at a discount of $47.02

Gain due to investing coupons at a higher rate almost exactly offsets the loss of discounting the bond’s remaining cash flow at a higher interest rate.

384

http://actuary88.com Moral of this problem: When you buy a bond and sell it before its maturity, you have two sources of incomes: the accumulated value of coupons reinvested at the market interest rate and the sales price of the bond’s remaining cash flows discounted at the market interest rate. These are two opposing forces. If the market interest rate goes down and stays low immediately after you buy a bond, you reinvest your coupons at a lower interest rate and incur a loss. At the same time, however, you can sell the bond’s remaining cash flows discounted at a lower interest rate and have a gain. If the market interest rate goes up and stays high immediately after you buy a bond, you can reinvest your coupons at a higher interest rate and have a gain. At the same time, however, you sell the bond’s remaining cash flows discounted at a higher interest rate and incur a loss. If you can find the right bond and hold it for the right amount of time, you can make these two opposing forces exactly offset each other, accumulating a guaranteed amount of money no matter the interest rate goes up or down. This is how immunization works. The key is to find the right bond and hold it for the right period of time. How? This is our next problem. Problem 5 State the 3 requirements of immunization. Verify that the 3 requirements are satisfied in the following immunization arrangement: Asset

6 year bond with $10,000 par and 8% annual coupons

Liability

10, 000 (1 + 8% ) = 14, 693.28 at t = 5 .

Interest rate

8%

5

Solution The 3 requirements of immunization: • PV Assets = PV Liabilities • Duration of Assets = Duration of Liabilities • Convexity of Asset > Convexity of Liabilities

385

http://actuary88.com If these 3 conditions are met, then the assets will generate sufficient cash flows to pay the future known liabilities, even though the interest rate might uniformly increase or uniformly decrease by a small amount. Next, let’s verify that the 3 immunization requirements are met. (1) PV Assets = PV of 6 year bond with $10,000 par and 8% annual coupons discounted at 8%. Without doing any calculations (with or without a calculator), you should immediately know that PV of a 6 year bond with $10,000 par and 8% annual coupons discounted at 8% is $10,000. As a general rule, for any annual coupon bond, if the discount rate is equal to the annual coupon rate, the PV of the bond is the par value. PV Liabilities = $10,000. PV Assets = PV Liabilities (2) Next, we check whether Duration of Assets = Duration of Liabilities. We’ll use the general formula: n

Duration =

t =1 n

t v t CF ( t )

t =1

v CF ( t )

, where CF ( t ) stands for a cash flow at t .

t

Asset Time t

CF ( t )

1 2 3 4 5

800 800 800 800 10,800

Duration of Assets: 6 t =1 6

t v t CF ( t )

t =1

v t CF ( t )

=

(1) 800v + ( 2 ) 800v 2 + ( 3) 800v3 + ( 4 ) 800v 4 + ( 5)10,800v5 800v + 800v 2 + 800v3 + 800v 4 + 10,800v5 49,927 = 4.9927 5 10,000

386

http://actuary88.com Duration of liability (only one cash flow of 10, 000 (1 + 8% ) = 14, 693.28 at 5

t = 5:

Liability: Time t

CF ( t )

5

10, 000 (1 + 8% )

Duration =

t =5

5

t v t CF ( t )

t =5

( 5 )10, 000 (1 + 8% ) (1 + 8% ) = 5 5 10, 000 (1 + 8% ) (1 + 8% ) v t CF ( t ) 5

5

=5

Generally, the duration of a single cash flow (such as a zero-coupon bond) at time t is t . Duration of Assets = Duration of Liabilities. (3) Let check whether Convexity of Assets > Convexity of Liabilities n

Convexity = v 2

1

n

p

t =1

t 2 CF ( t ) v t + Duration = v 2

t =1 n t =1

t 2 CF ( t ) v CF ( t )

+ Duration

t

Because Duration Asset = Duration Liability (we already checked this), to make Convexity Asset > Convexity Liability , we just need to

n t =1 n t =1

t 2 AssetCF ( t ) v t AssetCF ( t )

m

>

t =1 m t =1

t 2 LiabiltyCF ( t ) v t LiabilityCF ( t )

In the above equation, n is the term to maturity of the asset and m is the term to maturity of the liability. In other words, if assets and liabilities already have an equal duration, then by making

387

http://actuary88.com

n t =1 n t =1

t 2 AssetCF ( t ) v t AssetCF ( t )

m t =1 m

>

t =1

t 2 LiabiltyCF ( t ) v t LiabilityCF ( t )

We’ll surely have

Convexity Asset > Convexity Liability n t =1 n t =1

t 2CF ( t ) v CF ( t )

is called the Macaulay convexity.

t

Let’s summarize the above discussion. Original conditions for immunization PV asset = PV of liability Duration of asset = Duration of liability Convexity of asset > Convexity of liability Revised conditions for immunization PV asset = PV of liability Duration of asset = Duration of liability Macaulay convexity of asset > Macaulay convexity of liability So in the future, we will use the revised conditions to check whether the immunization stands true; it’s easier to calculate the Macaulay convexity than to calculate the convexity. Let’s check: For assets: 6

Macaulay Convexity Asset =

t 2 v t CF ( t )

t =1 6

t =1

v t CF ( t )

(1 ) 800v + ( 2 ) 800v + ( 3 ) 800v + ( 4 ) 800v + ( 5 )10,800v = 2

2

2

2

3

2

4

2

5

800v + 800v 2 + 800v3 + 800v 4 + 10,800v 5

277,229.8141 27.72 10, 000 388

http://actuary88.com

For liability: Macaulay Convexity Liability =

t 2 v t CF ( t ) v t CF ( t )

= 52 = 25

(because liability has only one cash flow at t=5)

Macaulay Convexity Asset > Macaulay Convexity Liability Convexity Asset > Convexity Liability So Convexity of Assets > Convexity of Liabilities. As a shortcut, to check whether the convexity of assets exceeds the convexity of liabilities, you often don’t need to calculate the convexity of assets and the convexity of liabilities. You can simply draw the cash flows of assets and the cash flows of liabilities. If you can visually see that the cash flows of assets are more spread out than the cash flows of liabilities, then the convexity of assets will exceeds the convexity of liabilities.

389

http://actuary88.com

Cash flow amount

0

1

2

3

4

5

Time t

Asset cash flows Cash flow amount

0

1

2

3

4

5

Time t

Liability cash flows You can see that asset cash flows are more spread out than liabilities cash flows. As a result, the convexity of assets is greater than the convexity of the liabilities. Why so? The answer lies in our reinterpretation of the meaning of duration and convexity.

390

http://actuary88.com n

Duration =

t =1 n

t v t CF ( t )

t =1

=

v CF ( t )

t t =1

t

v t CF ( t ) n t =1

n

Macaulay Convexity =

n

v CF ( t )

t 2 v t CF ( t )

t =1 n

t =1

= E (t )

t

v t CF ( t )

n

=

t2 t =1

v t CF ( t ) n t =1

v t CF ( t )

= E (t 2 )

Spread out = Varance ( t ) = E ( t 2 ) E 2 ( t )

In other words, duration is the weighted average time of the cash flows, with weights being the present value of each cash flow. Macaulay convexity is the weighted average time squared of the cash flows, with weights being the present value of each cash flow. How cash flows spread out can be measured by variance of the time of the cash flows. Then we can see that if the duration of asset is equal to the duration of liability and the asset cash flows are more spread out (i.e. having a bigger variance) than the liabilities cash flows, the convexity of the asset is greater than the convexity of the liability. Mathematically, this is: Liabililty If E Asset ( t ) = E (t ) ,

If Varance Asset ( t ) > Varance Liability ( t ) Then E Asset ( t 2 ) > E Liabililty ( t 2 ) Because E ( t 2 ) = E 2 ( t ) + Varance ( t ) Then Convexity Asset > Convexity Liability

391

http://actuary88.com Problem 6 Explain why if the following 3 conditions are met: • PV Assets = PV Liabilities • Duration of Assets = Duration of Liabilities • Convexity of Asset > Convexity of Liabilities then even if the interest rate uniformly increases or uniformly decreases by a small amount, the present value of the asset will be greater than the present value of the liability (i.e. you have immunized against a small change of the interest rate). Solution One easy way to understand this is to use Taylor series. Assume the current interest rate is r . Immediately after you purchase an asset (such as a bond), the interest rate changes by r . In other words, the new interest rate is r + r immediately after you purchase the asset.

P (r ) =

n t =1

P

P P

CF ( t ) v t =

n

CF ( t )

t =1

(1 + r )

dP 1 d 2P 2 r+ r) 2 ( dr 2 dr

t

(Taylor series)

2 2 1 dP 1 1 d 2P 1 1 r+ DMAC r + ( Convexity )( r ) 2 ( r) = P dr 2 P dr 1+ r 2

2 P Asset 1 1 ( Asset Duration ) r + ( Asset Convexity )( r ) Asset 1+ r 2 P Liability 2 P 1 1 ( Liability Duration ) r + ( Liability Convexity )( r ) Liability 2 1+ r P

if the 3 conditions of immunization are satisfied, we have:

P Asset = P Liab Asset Duration = Liability Duration Asset Convexity > Liability Convexity

P Asset P Liability > P Asset P Liability P Asset > P Liability

392

http://actuary88.com

' PAsset = P Asset + P Asset

' > PLiability = P Liability + P Liability

So if the interest rate changes, the PV of the asset is greater than the PV of the liability. Problem 7 Besides satisfying the 3 conditions • PV Assets =PV Liabilities • Duration of Assets = Duration of Liabilities • Convexity of Asset > Convexity of Liabilities immunization implicitly assumes that the following standards are met: • • • •

both the timing and the dollar amount of each liability cash flows must be 100% known in advance the interest rate change must be small the interest rate change takes place immediately after the asset is purchased the interest rate change must be uniform (i.e. the same increase is applied to any time t, resulting a constant discount rate i0 + i ).

Explain why immunization implicitly assumes so. Solution If either the timing or the dollar amount of a liability cash flow is not 100% certain at t=0, then we don’t know when we need to pay our bill. Hence, we don’t have a clear goal to start immunization. For example, if you don’t know when you are going to fix your roof (timing unknown), or if you don’t know how much it will cost you to fix the roof at t=5 (amount unknown), you really don’t know how much money you need to invest now. Immunization can’t be used. All the other implicit assumptions must be met for the Taylor series to stand. For example, if i is not small (for example, if i =500%), or if i is not a constant, or if i doesn’t take place immediately after t = 0 , the Taylor series will not stand.

393

http://actuary88.com

Chapter 18

Cash flow matching

Why to match liability cash flows? • An alternative strategy to immunization •

Like immunization, cash flow matching shields us from adverse interest rate changes so we have enough money to pay bills.



If assets have exactly the same cash flows as do the liabilities, no matter how the interest rate changes in the future, we are guaranteed to have just enough cash to pay our liabilities.



If assets’ cash flows perfectly match liability cash flows, we are 100% safe against any interest rate changes. In contrast, immunization shields us against only a small parallel interest rate shift.



However, perfectly matching liability cash flows is (a) impossible -- bonds exceeding 30 years maturity are hard to find; (b) or prohibitively expensive

Matching procedures (working backwards, starting from the final liability cash flow to the earliest liability cash flow) 1. Match the final liability cash flow by buying the following bond •

The bond matures at the same time when the final liability cash flow is due.



The bond’s final coupon plus the redemption value is equal to the final liability cash flow.

2. Remove the matching asset’s cash flows (periodic coupons plus a final redemption value) from the liability cash flows. Throw away our first matching asset. It has done its share and won’t be needed any more. Now the # of available assets for us to match the remaining liability cash flows is reduced by one. 3. After the 1st matching asset’s cash flows are removed, the final liability cash flow becomes zero; the next-to-final liability cash flow pops up and becomes the final liability cash flow. Then, we apply Step 1 and Step 2 to this new final liability cash flow. 4. Repeat Step 3 until the earliest liability cash flow is matched.

394

http://actuary88.com Sample problems and solutions Problem 1 (Sample FM #51, #52, and #53) The following information applies to questions 51 through 53. Joe must pay liabilities of 1,000 due 6 months from now and another 1,000 due one year from now. There are two available assets: • A 6-month bond with face amount of 1,000, 8% nominal annual coupon rate convertible semiannually, and a 6% nominal annual yield rate convertible semiannually; •

A 1-year bond with face amount of 1,000, a 5% nominal annual coupon rate convertible semiannually, and a 7% nominal annual yield rate convertible semiannually.

Q #51 - How much of each bond should Joe purchase in order to exactly (absolutely) match the liabilities? Q #52 - What’s Joe’s total cost of purchasing the bonds required to exactly (absolutely) match the liabilities? Q #53 - What’s the annual effective yield rate for investing the bonds required to exactly (absolutely) match the liabilities? Solution Q #51 – buy bonds to match the liability cash flows. As always, we draw cash flow diagrams for our liability and two bonds. Liability Time t

0

Cash flow

0.5

1

$1,000

$1,000

0.5

1

Bond #1 Time t

0

Cash flow

$40 $1,000

PV = 1, 040 1 +

6% 2

1

= 1, 009.7087

2 YTM = i ( ) = 6%

395

http://actuary88.com Bond #2 Time t

0

Cash flow

PV = 25a2 + 1, 000v 2

0.5

1

$25

$25 $1,000

@ 3.5%

2 YTM = i ( ) = 7%

PV = 981.0031

Let’s combine the three diagrams into a table: Selling price t=0 Liability Bond #1 Bond #2

$1,009.7087 $981.0031

Cash flow at t=0.5 $1,000 $1,040 $25

Cash flow at t=1 $1,000 $0 $1,025

Procedure to match liability cash flows: Step 1 – Match the final liability cash flow. The final liability cash flow is $1,000 at t = 1 . Because Bond #1 doesn’t have any cash flows at t = 1 , we can’t use it to match the final cash flow. So we are left with Bond #2, which produces $1,025 at t = 1 . Assume we buy X units of Bond #2. To match the final liability cash flow, we need to have: 1, 000 X (1,025 ) = 1, 000 X= = 0.97561 1, 025 Selling price t=0 Liability Bond #1

X

(Bond #2)

$1,009.7087 1, 000 981.0031 1, 025 = 957.0762

Cash flow at t=0.5 $1,000 $1,040 1, 000 25 1, 025 = 24.3902

Cash flow at t=1 $1,000 $0 1, 000 1, 025 1, 025 = 1, 000

396

http://actuary88.com Step 2 – Remove the cash flows of the matched asset from the liability cash flows. Throw away Bond #2. Now we have only Bond #1 left. Selling price t=0 Liability – X(Bond 2) Bond #1

$1,009.7087

Cash flow at Cash flow at t=0.5 t=1 $1,000-24.3902 $1,000-$1,000 =975.6098 =0 $1,040 $0

Now, the liability cash flow at t=0.5 becomes the final liability cash flow. Step 3 –Match the current final cash flow of $975.6098 at t=0.5. Assume we buy Y units of Bond #1. Then to match the final liability cash flow, we need to have:

Y (1,040 ) = 975.6098

Y = 0.9381 Selling price t=0

Cash flow at t=0.5

$1,009.7087(0.9381) =947.2077

$975.6098-975.6098 =$0 $1,040(0.9381) =975.6098

Liability – X(Bond 2) -Y(Bond 1) Y(Bond #1)

After matching the liability cash flow at t=0.5, we need to throw away Bond #1. Now we have no assets left. Fortunately, we have no liability cash flows left either. All of the liability cash flows are matched. To match our liability cash flows, we need to buy X = 0.9756 units of Bond #2 and Y = 0.9381 units of bond #1; we assume that we can buy a fractional bond. Next, we’ll find the cost of the matching assets. This should be easy.

X ( PV Bond #2 ) + Y ( PV Bond #1) = 957.0762 + 947.2077=1,904.2839

397

http://actuary88.com Finally, we’ll calculate annual effective yield for investing X units of Bond #2 and Y units of Bond #1. We’ll draw a cash flow diagram:

Matching Asset = X ( Bond #2 ) + Y ( Bond #1) Time t

0

0.5

Matching asset’s cash flows

1

$1,000

$1,000

PV of the matching asset = 1,904.2839

We need to solve the following equation: 1,904.2839 = 1, 000a2

i = 3.33269524%

i

Next, we’ll convert i = 3.33269524% (interest rate per 6-month) into an annual effective rate:

(1 + i )

2

1 = (1 + 3.33269524% )

2

1 = 6.77645906%

Problem 2 (SOA Course 6, #9, May 2001) You are given the following information: Projected liability cash flows Year 1 43

Year 2 123

Year 3 214

Year 4 25

Year 5 275

Available assets for investment: • 2-year bond with annual coupon of 5% • 3-year bond with annual coupon of 8% • 5-year bond with annual coupon of 10% Face amount of the bond: Current market yield curve:

100 7% for all durations

Calculate the initial cost to cash-flow match the projected liability cash flows utilizing the assets listed above. Solution

398

http://actuary88.com We can use the standard backward matching method to solve this problem. We first match the liability cash flow at Year 5. Then we move on to Year 4, 3, 2, and 1. This is the method used in the SOA official solution. Please download the SOA official solution. Make sure you can recreate the solution. However, we’ll use a quicker method to solve this problem. Notice that the three bonds have an identical yield of 7% at all durations. As a result, we can discount all the cash flows of our constructed matching asset (which is a mixture of bond #1,#2, and #3) at 7%. The constructed matching asset will have the same cash flows as does the liability. So we really don’t need to know how to construct the matching asset. No matter how we mix the three bonds, the resulting matching asset will surely have the following cash flows: Year 1 43

Year 2 123

Year 3 214

Year 4 25

Year 5 275

As a result, the cost of the matching asset is simply the above cash flows discounted at 7%: 43 (1.07

1

) + 123 (1.07 ) + 214 (1.07 ) + 25 (1.07 ) + 275 (1.07 ) = 537.4512 2

3

4

5

We don’t need to manually calculate the above result. We can simply enter the cash flows and the interest rate into BA II Plus/BA II Plus Professional Cash Flow Worksheet. The calculator will generate the result for us. Please note that this method works only when all the assets have the same yield to maturity. Otherwise, we have to use the standard backward matching method. Problem 3 (FM #10, Nov 2005) A company must pay liabilities of 1000 and 2000 at the end of years 1 and 2, respectively. The only investments available to the company are the following two zero-coupon bonds: Maturity (years) 1 2

Effective annual yield 10% 12%

Par 1000 1000

Determine the cost to the company today to match its liabilities exactly.

399

http://actuary88.com

Solution SOA made this problem easy because there aren’t any coupons. To match the liability, you just need to buy a 1-year zero bond with face amount of $1,000 and buy two 2-year zero bonds with face amount of $1,000 each. Your total cost of the matching assets is: 1, 000 2, 000 + 1.1 1.12 2

2,503

Key point to remember Cash flow matching problems are not hard, but it’s very easy for candidates to make silly mistakes. To eliminate errors, use a systematic approach.

400

http://actuary88.com

Value of this PDF study manual 1. Don’t pay the shipping fee (can cost $5 to $10 for U.S. shipping and over $30 for international shipping). Big saving for Canadian candidates and other international exam takers. 2. Don’t wait a week for the manual to arrive. You download the study manual instantly from the web and begin studying right away. 3. Load the PDF in your laptop. Study as you go. Or if you prefer a printed copy, you can print the manual yourself. 4. Use the study manual as flash cards. Click on bookmarks to choose a chapter and quiz yourself. 5. Search any topic by keywords. From the Adobe Acrobat reader toolbar, click Edit ->Search or Edit ->Find. Then type in a key word.

401

http://actuary88.com

About the author Yufeng Guo was born in central China. After receiving his Bachelor’s degree in physics at Zhengzhou University, he attended Beijing Law School and received his Masters of law. He was an attorney and law school lecturer in China before immigrating to the United States. He received his Masters of accounting at Indiana University. He has pursued a life actuarial career and passed exams 1, 2, 3, 4, 5, 6, and 7 in rapid succession after discovering a successful study strategy. Mr. Guo’s exam records are as follows: Fall 2002 Passed Course 1 Spring 2003 Passed Courses 2, 3 Fall 2003 Passed Course 4 Spring 2004 Passed Course 6 Fall 2004 Passed Course 5 Spring 2005 Passed Course 7 Mr. Guo currently teaches an online course for Exam P, FM, MLC, and MFE. For more information, visit http://actuary88.com. If you have any comments or suggestions, you can contact Mr. Guo at [email protected].

402

Deeper Understanding Exam FM Part II: Derivatives Markets Yufeng Guo

Derivatives Markets Chapter 0

c

Yufeng Guo

www.actuary88.com

FM: Derivatives Markets

c

Yufeng Guo

2

Contents 0 Introduction 0.1 Recommended study method . . . . . . . . . . . . . . . . . . . . . . . . . . . 0.2 Types of questions to be tested . . . . . . . . . . . . . . . . . . . . . . . . . . 0.3 How to use this study manual . . . . . . . . . . . . . . . . . . . . . . . . . . . 1 Introduction to derivatives 1.1 What is a derivative? . . . . . . . . . . . . 1.1.1 Definition of derivatives . . . . . . 1.1.2 Major types of derivatives . . . . . 1.1.3 Basic vocabulary . . . . . . . . . . 1.1.4 Uses of Derivatives . . . . . . . . . 1.1.5 Perspectives on Derivatives . . . . 1.1.6 Financial Engineering and Security 1.2 The role of financial markets . . . . . . . 1.2.1 Financial Markets and Averages . 1.2.2 Risk-sharing . . . . . . . . . . . . 1.3 Derivatives in practice . . . . . . . . . . . 1.3.1 Growth in derivatives trading . . . 1.3.2 How are derivatives used? . . . . . 1.4 Buying and short-selling financial assets . 1.4.1 The lease rate of an asset . . . . . 1.4.2 Risk and scarcity in short-selling . 1.5 Chapter summary . . . . . . . . . . . . . 1.6 Review questions . . . . . . . . . . . . . .

5 5 5 6

. . . . . . . . . . . . . . . . . .

. . . . . . . . . . . . . . . . . .

. . . . . . . . . . . . . . . . . .

. . . . . . . . . . . . . . . . . .

. . . . . . . . . . . . . . . . . .

. . . . . . . . . . . . . . . . . .

. . . . . . . . . . . . . . . . . .

. . . . . . . . . . . . . . . . . .

. . . . . . . . . . . . . . . . . .

. . . . . . . . . . . . . . . . . .

. . . . . . . . . . . . . . . . . .

. . . . . . . . . . . . . . . . . .

. . . . . . . . . . . . . . . . . .

7 7 8 8 9 10 10 11 11 11 11 12 12 12 12 12 13 13 13

2 An Introduction to Forwards and Options 2.1 Forward contracts . . . . . . . . . . . . . . . . . . . . . 2.1.1 Definition . . . . . . . . . . . . . . . . . . . . . . 2.1.2 The payoff on a forward contract . . . . . . . . . 2.1.3 Graphing the payoff on a forward contract . . . . 2.1.4 Comparing a forward and outright purchase . . . 2.1.5 Zero-coupon bonds in payoff and profit diagrams 2.1.6 Cash settlement vs. delivery . . . . . . . . . . . . 2.1.7 Credit risk . . . . . . . . . . . . . . . . . . . . . 2.2 Call options . . . . . . . . . . . . . . . . . . . . . . . . . 2.2.1 Option terminology . . . . . . . . . . . . . . . .

. . . . . . . . . .

. . . . . . . . . .

. . . . . . . . . .

. . . . . . . . . .

. . . . . . . . . .

. . . . . . . . . .

. . . . . . . . . .

. . . . . . . . . .

. . . . . . . . . .

. . . . . . . . . .

. . . . . . . . . .

. . . . . . . . . .

15 15 15 15 16 16 17 17 17 17 17

3

. . . . . . . . . . . . . . . . . . . . . . . . . . . . . . Design . . . . . . . . . . . . . . . . . . . . . . . . . . . . . . . . . . . . . . . . . . . . . . . . . . . . . . . .

. . . . . . . . . . . . . . . . . .

. . . . . . . . . . . . . . . . . .

Derivatives Markets Chapter 0

2.3

2.4 2.5 2.6

c

Yufeng Guo

www.actuary88.com

2.2.2 Payoff and profit for a purchased call option 2.2.3 Payoff and profit for a written call option . Put options . . . . . . . . . . . . . . . . . . . . . . 2.3.1 Payoff and profit for a purchased put option 2.3.2 Payoff and profit for a written put option . Summary of forward and option positions . . . . . Options are insurance . . . . . . . . . . . . . . . . Example: equity-linked CD . . . . . . . . . . . . .

3 Insurance, Collars, and Other Strategies 3.1 Basic insurance strategies . . . . . . . . . 3.1.1 Insuring a long position: floors . . 3.1.2 Insuring a short position: caps . . 3.1.3 Selling insurance . . . . . . . . . . 3.2 Synthetic forwards . . . . . . . . . . . . . 3.2.1 Put-call parity . . . . . . . . . . . 3.3 Spreads and collars . . . . . . . . . . . . . 3.3.1 Bull and bear spreads . . . . . . . 3.3.2 Box spreads . . . . . . . . . . . . . 3.3.3 Ratio spreads . . . . . . . . . . . . 3.3.4 Collars . . . . . . . . . . . . . . . . 3.4 Speculating on volatility . . . . . . . . . . 3.4.1 Straddle . . . . . . . . . . . . . . . 3.4.2 Butterfly spread . . . . . . . . . . 3.4.3 Asymmetric butterfly spreads . . . 3.5 Example: another equity-linked note . . .

. . . . . . . .

. . . . . . . .

. . . . . . . .

. . . . . . . .

. . . . . . . .

. . . . . . . .

. . . . . . . .

. . . . . . . .

. . . . . . . .

. . . . . . . .

. . . . . . . .

. . . . . . . .

. . . . . . . .

. . . . . . . .

. . . . . . . .

18 18 18 18 18 18 19 19

. . . . . . . . . . . . . . . .

. . . . . . . . . . . . . . . .

. . . . . . . . . . . . . . . .

. . . . . . . . . . . . . . . .

. . . . . . . . . . . . . . . .

. . . . . . . . . . . . . . . .

21 21 21 22 22 23 23 24 24 25 25 25 26 26 26 26 27

4 Introduction to risk management 4.1 Basic risk management: the producer’s perspective . . . . . . . . . . 4.1.1 Hedging with a forward contract . . . . . . . . . . . . . . . . 4.1.2 Insurance: guaranteeing a minimum price with a put option . 4.1.3 Insuring by selling a call . . . . . . . . . . . . . . . . . . . . . 4.1.4 Adjusting the amount of insurance . . . . . . . . . . . . . . . 4.2 Basic risk management: the buyer’s perspective . . . . . . . . . . . . 4.2.1 Hedging with a forward contract . . . . . . . . . . . . . . . . 4.2.2 Insurance: guaranteeing a maximum price with a call option 4.3 Why do firms manage risk? . . . . . . . . . . . . . . . . . . . . . . . 4.3.1 An example where hedging adds value . . . . . . . . . . . . . 4.3.2 Reasons to hedge . . . . . . . . . . . . . . . . . . . . . . . . . 4.3.3 Reasons not to hedge . . . . . . . . . . . . . . . . . . . . . . 4.3.4 Empirical evidence on hedging . . . . . . . . . . . . . . . . . 4.4 Golddiggers revisited . . . . . . . . . . . . . . . . . . . . . . . . . . .

. . . . . . . . . . . . . .

. . . . . . . . . . . . . .

. . . . . . . . . . . . . .

. . . . . . . . . . . . . .

. . . . . . . . . . . . . .

29 29 29 29 29 30 30 30 30 30 30 31 31 32 32

FM: Derivatives Markets

. . . . . . . . . . . . . . . .

c

Yufeng Guo

. . . . . . . . . . . . . . . .

. . . . . . . . . . . . . . . .

. . . . . . . . . . . . . . . .

. . . . . . . . . . . . . . . .

. . . . . . . . . . . . . . . .

. . . . . . . . . . . . . . . .

. . . . . . . . . . . . . . . .

. . . . . . . . . . . . . . . .

. . . . . . . . . . . . . . . .

. . . . . . . . . . . . . . . .

. . . . . . . . . . . . . . . .

. . . . . . . . . . . . . . . .

. . . . . . . . . . . . . . . .

4

Derivatives Markets Chapter -1

c

Yufeng Guo

www.actuary88.com

5 Financial forwards and futures 5.1 Alternative ways to buy a stock . . . . . . . . . . . . . . . . . . . . . 5.2 Tailing . . . . . . . . . . . . . . . . . . . . . . . . . . . . . . . . . . . 5.3 Pricing a forward contract . . . . . . . . . . . . . . . . . . . . . . . . 5.4 Forward premium . . . . . . . . . . . . . . . . . . . . . . . . . . . . . 5.5 Creating a synthetic forward contract . . . . . . . . . . . . . . . . . 5.5.1 Synthetic forwards in market-making and arbitrage . . . . . . 5.5.2 No-arbitrage bounds with transaction costs . . . . . . . . . . 5.5.3 Quasi-arbitrage . . . . . . . . . . . . . . . . . . . . . . . . . . 5.5.4 Does the forward price predict the future spot price? . . . . . 5.6 Futures contracts . . . . . . . . . . . . . . . . . . . . . . . . . . . . . 5.6.1 Role of the clearing house . . . . . . . . . . . . . . . . . . . . 5.6.2 S&P 500 futures contract . . . . . . . . . . . . . . . . . . . . 5.6.3 Difference between a forward contract and a futures contract 5.6.4 Margins and markings to market . . . . . . . . . . . . . . . . 5.6.5 Comparing futures and forward prices . . . . . . . . . . . . . 5.6.6 Arbitrage in practice: S&P 500 index arbitrage . . . . . . . . 5.6.7 Appendix 5.B: Equating forwards and futures . . . . . . . . . 8 Swaps 8.1 An example of a commodity swap . . . . . . . . . . 8.1.1 Physical versus financial settlement . . . . . . 8.1.2 Pricing swaps . . . . . . . . . . . . . . . . . . 8.2 Interest rate swap . . . . . . . . . . . . . . . . . . . 8.2.1 Key features of an interest rate swap . . . . . 8.2.2 Example of a plain vanilla interest rate swap 8.2.3 Motivations for an interest swap . . . . . . . 8.2.4 How to price an interest rate swap . . . . . . 8.2.5 The swap curve . . . . . . . . . . . . . . . . . 8.2.6 Swap’s implicit loan balance . . . . . . . . . . 8.2.7 Deferred swaps . . . . . . . . . . . . . . . . . 8.2.8 Why swap interest rates? . . . . . . . . . . . 8.2.9 Amortizing and accrediting swaps . . . . . .

FM: Derivatives Markets

c

Yufeng Guo

. . . . . . . . . . . . .

. . . . . . . . . . . . .

. . . . . . . . . . . . .

. . . . . . . . . . . . .

. . . . . . . . . . . . .

. . . . . . . . . . . . .

. . . . . . . . . . . . .

. . . . . . . . . . . . .

. . . . . . . . . . . . .

. . . . . . . . . . . . . . . . .

. . . . . . . . . . . . .

. . . . . . . . . . . . . . . . .

. . . . . . . . . . . . .

. . . . . . . . . . . . . . . . .

. . . . . . . . . . . . .

. . . . . . . . . . . . . . . . .

. . . . . . . . . . . . .

. . . . . . . . . . . . . . . . .

33 33 33 34 35 36 37 38 39 40 40 40 40 41 41 44 45 46

. . . . . . . . . . . . .

53 53 54 55 56 56 57 58 63 69 70 70 70 71

5

Derivatives Markets Chapter -1

c

Yufeng Guo

www.actuary88.com

FM: Derivatives Markets

c

Yufeng Guo

6

Chapter 0

Introduction 0.1

Recommended study method

The following are my recommendations on how to prepare for Derivatives Markets: 1. Download the FM syllabus from the SOA website. Know precisely what chapters of Derivatives Markets are the required readings for Exam FM. 2. Buy Derivatives Markets (2nd Edition). Derivatives Markets is expensive, but you’d better buy it. No study guides can thoroughly explain every detail of the textbook. Or if it does, it’ll be more expensive than the textbook. Besides, if you buy Derivatives Markets, you can use it for Exam MF, MFE, and C. 3. Read all of the required readings of Derivatives Markets. Work through all of the examples in Derivatives Markets. 4. Work through all of the problems in Derivatives Markets at the end of the required chapter. Use the solution manual to check your answers. 5. Download load the sample problems for Derivatives Markets from the SOA website. Work through these problems.

0.2

Types of questions to be tested

SOA can write two types of questions on Derivatives Markets: 1. Numerical calculations. For example, SOA can give you some facts about an interest rate swap and asks you to calculate the fixed swap rate R. Many students find that solving a numerical question is easier because they can use a formula to calculate the answer. 2. Essay-type multiple choice questions. This is an example: “Which of the following statements is true about the interest rate swap (or Eurodollars futures contracts)? ” This is another example: “All the followings are differences between a forward contract and a futures contract EXCEPT...” Essay-type questions are generally harder than numerical calculations because you don’t have a formula to produce the answer. To answer an 7

Derivatives Markets Chapter 0

c

Yufeng Guo

www.actuary88.com

essay type multiple choice question correctly, you’ll probably need to memorize lot of facts. When reading Derivatives Markets, be prepared to learn formulas and memorize facts. Memorizing facts is no fun, but you’ll probably have to do it to answer essay-type questions. Many people find that reading the textbook multiple times and using flash cards help them memorize facts. You can try these two methods.

0.3

How to use this study manual

This manual is written under the following philosophy: “If the textbook explains it well, I’ll skip it or explain it briefly; if the textbook doesn’t explain it well, I’ll try to explain it well.” It’s best that you read this manual together with the textbook. This manual is not a replacement of Derivatives Markets.

FM: Derivatives Markets

c

Yufeng Guo

8

Chapter 1

Introduction to derivatives 1.1

What is a derivative?

Today, a U.S. company (called Mr. US) signed a purchase contract with a manufacturer in Britain (called Mr. UK). According to the contract, Mr. US will buy an expensive machine from Mr. UK. The price of the machine is 1,000,000 pounds. Mr. UK will deliver the machine to Mr. US in exactly 6 months from today. Upon receiving the machine, Mr. US will pay Mr. UK exactly 1,000,000 pounds. Mr. US faces the currency risk. The exchange rate between US dollars and British pounds fluctuates. If 6 months from today the value of British pounds goes up and the value of American dollars goes down, Mr. US will need to spend more dollars to convert to 1 million British pound, incurring a loss. For example, the exchange rate today is 1 British Pound=1.898 US Dollars. If Mr. US pays Mr. UK today, Mr. US will spend $1,898,000 to buy 1 million pounds. If 6 months from today the exchange rate is 1 British Pound=2 US Dollars, Mr. US will need to spend $2,000,000 to buy 1 million pounds. How can Mr. US reduce its currency risk? • One simple approach is for Mr. US to buy 1 million pound today at the price of 1,898,000 US dollars. This eliminates the currency risk, but it requires Mr. US to spend $1,898,000 today. • Another approach is for Mr. US to pre-order 1 million pounds at a fixed exchange rate from a third party. All Mr. US has to do is to find a third party willing to sell, in 6 months, 1 million pounds at a fixed exchange rate. When the pre-order contract is signed, no money exchanges hands. Mr. US is not required to take out any capital. After 6 months has passed, Mr. US will buy 1 millon British pound from the third party at the pre-agreed exchange rate (say 1 British pound=1.9 US dollars), regardless of the market exchange rate between pounds and US dollars 6 months from today. The pre-order contract here is called the forward contract. A forward contract is an agreement today to transact in the future. Under the currency forward contract, Mr. US (buyer) agrees to buy a specific amount of goods or services (1 million British pound) from the seller (the third party) at a pre-agreed price (1 British pound=1.9 US dollars) in a specific future date (6 months from today). And the seller (the third party) agrees to deliver 9

Derivatives Markets Chapter 1

c

Yufeng Guo

www.actuary88.com

the specific amount of goods or services (1 million pounds) at a pre-agreed price (1 British pound=1.9 US dollars) in a specific future date (6 months from today). When the forward is signed, no money or goods exchanges hands between Mr. US and Mr. UK. When the specified future date (6 months from today) arrives, Mr. US and Mr. UK each fulfill their promises. Then the forward contract terminates. The currency forward contract is just one example of financial derivatives.

1.1.1

Definition of derivatives

Derivatives are financial contracts designed to create pure price exposure to an underlying commodity, asset, rate, index, or event. In general they do not involve the exchange or transfer of principal or title. Rather, their purpose is to capture, in the form of price changes, some underlying price change or event. 1 You can think of derivatives as children and the underlying assets as parents. Parents give birth to children; the underlying gives birth to derivatives. Two key characteristics of a derivative: 1. A derivative is a contract between two parties. One party is a seller and the other buyer. 2. A derivative is a conditional asset whose value depends on something else (called the underlying asset or the underlying for short). In the currency forward contract between Mr. US and Mr. UK, the forward contract by itself doesn’t have any value. Its value depends on the currency exchange rate; the currency exchange rate is the underlying asset. If, 6 months from today, the British pound becomes more valuable thus more US dollars are required to buy one British pound, Mr. US has a gain in the currency forward and Mr. UK has a loss. If on the other hand the British pound becomes less valuable (thus fewer US dollars are required to buy one British pound), Mr. US has a loss and Mr. UK has a gain.

1.1.2

Major types of derivatives

There are 2 major types of derivatives: • Option type derivatives. An option is a contract between a buyer (i.e. the owner of the option) and seller (one who sold the option) that gives the option owner the right, not the obligation, to buy or sell an asset at a pre-determined price in a specified future date. Two major types of options: – Call. A call option gives the option owner the right to buy an asset for a specified price (the exercise price or strike price) on or before a specified expiration date. – Put. A put option gives the option owner the right to sell an asset for a specified price (the exercise price or strike price) on or before a specified expiration date. 1

Definition by http://www.financialpolicy.org/dscprimer.htm

FM: Derivatives Markets

c

Yufeng Guo

10

Derivatives Markets Chapter 1

c

Yufeng Guo

www.actuary88.com

• Forward type derivatives. In forward-type derivatives, buyers and sellers agree to do business in a specified future date at a pre-determined price. The major use of forward type derivatives is pricing-locking. Forward type derivatives include: – Forward. A forward contract is a commitment to buy/sell in a future date. The difference between a forward and an option is that an option represents the right, not obligation, to buy or sell in the future (hence the word option) but a forward contract represents the obligation to buy or sell in the future. – Futures. A futures contract is also a commitment between a buyer and a seller to transact in a future date at a price agreed upon today. One key difference between a forward contract and a futures contract is that a futures contract is a standardized contract traded over the exchange whereas a forward contract is a privately negotiated contract traded over the counter. – Swap. In a swap, two parties agree to exchange cash flows in the future. One common swap is an interest rate swap, where one party pays floating (ie. variable) interest rate and receives fixed interest rate and the other party pays a fixed interest rate and receives a floating interest rate.2

1.1.3

Basic vocabulary

The following terms are used throughout the textbook. They look odd to newcomers. However, after a while, these terms will become your second nature. 1. Non-arbitrage. Financial derivatives are generally priced using the non-arbitrage principle. The non-arbitrage principle means that “there’s no free lunch.” If two things have identical cash flows, they must sell at the same price. Otherwise, anyone can become an instant millionaire by ”buy low, sell high.” 2. Long and short. Long means to own or buy something. Short means to sell something. If you are long in a stock, then you have bought a stock or you own a stock. If you are short on a stock, then you have sold a stock. Like credits and debits in accounting, long and short were invented by professionals probably to scare novice. 3. Short selling. If you borrow something from someone else and sell you what you have borrowed, you are selling short. 4. Spot price, futures price, and forward price. For virtually every commodity, there are two prices: spot price and futures price (or forward price). Spot price is the price you pay to get something immediately (spot=immediate delivery). Futures price or forward price is the price you pay in the future date in order to get something in the same future date (futures and forward mean future delivery). Most of us pay the spot price (like shopping in a department store). Only a small portion of people pay forward prices or futures prices. 5. Spot market and futures (or forward) market. The market associated with the spot price is the spot market. The market associated with the futures (or forward) 2

For more information, visit http://www.answers.com/topic/interest-rate-swap

FM: Derivatives Markets

c

Yufeng Guo

11

Derivatives Markets Chapter 1

c

Yufeng Guo

www.actuary88.com

price is the futures (forward) market. Most of us trade in the spot market. The spot market is the largest. The forward market is the smallest. The futures market is in between. 6. In the money, at the money, out of the money. Suppose you own a call or put option. • If exercising your call or put option brings you some profit, then you are in the money or the option is in the money. • If exercising your call or put option brings you a loss, then you are out of the money or the option is out of the money. • If after the option is exercised your wealth stays unchanged, then you are at the money or your option is at the money. For example, you own a call option on one IBM stock. The call option allows you to buy one share of IBM stock for $80. • If IBM stocks sell for $85 per share, then by exercising your call option you’ll make $5 profit. Then you are in the money. • If IBM stocks sell for $75 per share, then exercising the call option brings you $5 loss. Then you are out of the money. • If IBM stocks sell for $80 per share, then you gain zero profit if exercising the call option. You are at the money.

1.1.4

Uses of Derivatives

The textbook lists four major uses of derivatives: • Risk management. For example, companies can use forwards and futures to lock in fixed prices of raw materials and exchange rates and protect themselves against fluctuations of raw materials and foreign currency exchange rates. • Speculation. You can use derivatives to earn some profits (or incur losses). • Reducing transaction costs. For example, by entering an interest rate swap, a firm can reduce its borrowing cost. • Regulatory arbitrage. For example, you can use derivatives to produce temporary losses to lower your tax. These are common sense uses of derivatives and should be pretty easy to remember.

1.1.5

Perspectives on Derivatives

• End-user perspective. • Market-maker perspective. • Economic observer Note: There isn’t much meat in this section. Feel free to skip it. FM: Derivatives Markets

c

Yufeng Guo

12

Derivatives Markets Chapter 1

1.1.6

c

Yufeng Guo

www.actuary88.com

Financial Engineering and Security Design

First, let’s think about the term financial engineering. Engineering means to build. Mechanical engineering is about building mechanical devices (engines for example). Electrical engineering is about building electrical devices. Similarly, financial engineering is about building financial devices (ie. financial products). Let’s look at an example of a new financial product called “index linked CD (certified deposit).” While a conventional CD guarantees a fixed interest rate, a market index linked CD offers a variable interest rate linked to performance of a market index (such as S&P 500). If the market index goes up, the CD owner earns a higher interest rate. If the market index goes down, however, the CD owner is guaranteed a minimal interest rate. However, there’s a difference between financial engineering and mechanical engineering. While a new model of cars can be built better than the old model, newly built financial products are not necessarily better than an older product. A newly built product might give an investor a higher returns but it also exposes him to higher risks.3 Next, the textbook says that financial engineering has four implications: 1. The fact that new financial products with desired payoffs can be built using basic ingredients (such as CD’s, stocks, bonds, calls, puts, forwards, futures, swaps) makes it feasible for banks and other financial institutions to produce new financial products and hedge the risks in the newly built financial products. 2. Sellers can build a special product for a special customer. 3. Sellers can refine their process for building new financial products (since building a new financial product is no longer a mysterious process any more). 4. A company can evade tax or circumvent regulations by building a new financial product that behaves the same ways as a security that’s taxed or regulated (so the company can say it doesn’t directly own the security and thus is not subject to the tax code or regulations otherwise applicable).

1.2

The role of financial markets

1.2.1

Financial Markets and Averages

Many financial risks are split and parceled out to others. Risk sharing is an important function of financial markets.

1.2.2

Risk-sharing

Risk is an inevitable; individuals and companies all face risks. Naturally, we want to set up risk-sharing mechanism where the lucky shares with the unlucky (similar to life insurance where the healthy help pay the death benefits of the unhealthy). Some risks are diversifiable; others are non-diversifiable. Risk-sharing benefits everyone. 3

For drawbacks of the market index linked CD, visit http://www.usatoday.com/money/perfi/columnist/ block/2006-05-01-cds_x.htm

FM: Derivatives Markets

c

Yufeng Guo

13

Derivatives Markets Chapter 1

1.3

c

Yufeng Guo

www.actuary88.com

Derivatives in practice

The use and types of derivatives have grown a lot in the past 30 years. Derivatives based on government statistics, industrial production, retail sales, and consumer price indexes were invented.

1.3.1

Growth in derivatives trading

When the price of an asset fluctuates a lot, derivatives on this asset are often invented to help manage the price risk. Millions of contracts are traded annually at the Chicago Board of Trade (CBT), Chicago Mercantile Exchange (CME), and the New York Mercantile Exchange (NYMEX), the three largest exchanges in the U.S.. The use of futures contracts has grown significantly over the last 30 years.

1.3.2

How are derivatives used?

Little is known about how companies use derivatives to manage risks.

1.4

Buying and short-selling financial assets

When calculating the cost of buying an asset, remember commission and the bid-ask spread. How to memorize bid-ask spread Bid price=dealer’s Buying price=what you get if you sell your security to a dealer aSk price=dealer’s Selling price=what you pay if you buy a security from a dealer Short Selling. Short selling XYZ stocks means that you first borrow XYZ stocks and then selling them. 3 reasons to short sell: • To speculate. • To borrow money. • To hedge a risk.

1.4.1

The lease rate of an asset

If you borrow an asset from the asset owner, you may have to pay a fee to the owner. This fee is called the least rate of an asset. If you short sell a stock and the stock pays dividend before you return the stock to the lender, you need to pay the dividend to the lender. FM: Derivatives Markets

c

Yufeng Guo

14

Derivatives Markets Chapter 1

1.4.2

c

Yufeng Guo

www.actuary88.com

Risk and scarcity in short-selling

• Credit risk is the risk that the short-seller won’t return the borrowed asset. To address the credit risk, the lender keeps as collateral the proceeds generated from the short sale. • Since the lender keeps the proceeds from the short sale, the lender should give the proceeds back to the short seller after the short seller returns the borrowed asset. In addition, the lender should pay the short seller interest for temporarily holding on the proceeds. The amount of interest on collateral depends on supply and demand. If lot of people want to short sell an item, the lender may pay a small interest. The interest rate paid on collateral is called the repo rate in bond market and the short rebate in the stock market. The difference between this rate and the market interest rate is another cost of borrowing.

1.5

Chapter summary

Derivatives are conditional assets whose values depend on something else. The three major derivatives are forwards (or futures), options, and swaps.

1.6

Review questions

Problem 1 You are the owner of an airline company. Fuel cost is a significant portion of your annual operating expense. You heard the news that the rising fuel cost brought several airlines to bankruptcy. Outline ways you can manage you fuel cost using financial derivatives. Solution One way to lock in fuel cost is for you to enter an energy swap with a fuel supplier. In this swap, you pay fixed cost each year to the supplier over a number of years. In return, the supplier gives you certain amount of fuel each year while the swap is effective. Now your fuel cost is fixed during the duration of the energy swap. A swap generally doesn’t require any initial cash outlay. You can also buy a certain number of call options on fuel price. If fuel price goes up, then you can exercise your call option and buy fuel at the predetermined strike price. However, unlike an energy swap, buying call options requires you to pay premiums upfront. You can also enter a futures or forward contract to order certain amount of fuel to be delivered to you at specified future dates with prices determined today. Problem 2 Why is it reasonable for us to assume that financial markets are free of arbitrage? Solution FM: Derivatives Markets

c

Yufeng Guo

15

Derivatives Markets Chapter 1

c

Yufeng Guo

www.actuary88.com

There are well-informed buyers and sellers in the market. If there are arbitrage opportunities, intelligent buyers and sellers will rush to the opportunities and soon the opportunities will vanish. ♦

FM: Derivatives Markets

c

Yufeng Guo

16

Chapter 2

An Introduction to Forwards and Options 2.1 2.1.1

Forward contracts Definition

Buyer of a forward contract • Has an obligation to buy an asset (the underlying) • At a specific future date (maturity/expiration date) • In an amount (contract size) • At a priced agreed upon today (the forward price) Seller of a forward contract • Has an obligation to deliver an asset (the underlying) • At a specific future date (maturity/expiration date) • In an amount (contract size) • At a priced agreed upon today (the forward price) A prepaid forward contract is a forward contract except that the payment is made at t = 0 when the contract is signed.

2.1.2

The payoff on a forward contract

If you buy something, you have a long position (long=having more of something). If you sell something, you have a short position (short=having less of something). In a forward contract, the buyer’s payoff=A − B, where 17

Derivatives Markets Chapter 2

c

Yufeng Guo

www.actuary88.com

• A=what the buyer has to pay to get the asset at T assuming he doesn’t have a forward contract • B= what the buyer has to pay to get the asset at T if he has a forward contract Clearly, A=the spot price at T (without a forward contract, the buyer has to buy the asset from the market at T ); B=the forward price at T . So the buyer’s payoff at T =Spot price at T - forward price at T Since the buyer of a forward contract has a long position in the forward contract, so payoff to long forward at T =Spot price at T - forward price at T The seller’s payoff=C-D 1. C=What the seller has to sell at T because he has a forward contract 2. D=What the seller can sell at T if he doesn’t have a forward contract Clearly, C=forward price at T ; D=Spot price at T Since the seller in a forward contract has a short position, payoff to short forward at T =Forward price at T - Spot price at T payoff to long forward + payoff to short forward=0 A forward is a zero sum game. If one party gains, the other party must lose.

2.1.3

Graphing the payoff on a forward contract

If you transfer the data from Table 2.1 to a graph (putting S&R Index in 6 months as X and Payoff as Y), you’ll get Figure 2.2. Don’t make it overly-complex. Just draw dots and connect the dots. To draw the payoff diagram for the long forward position, draw the following points: (900, −120), (950, −70), (1000, −20), (1020, 0), (1050, 30), (1100, 80). Then connect these dots. To draw the payoff diagram for the short forward position, draw the following points: (900, 120), (950, 70), (1000, 20), (1020, 0), (1050, −30), (1100, −80). Then connect these dots.

2.1.4

Comparing a forward and outright purchase

What’s the difference between buying an asset at t = 0 as opposed to entering into a forward contract and getting the same thing at T ? If you buy an asset outright at t = 0: • What’s good: You get what you need right away. You don’t need to enter into a forward contract. It takes time to find a seller who wants to sign a forward contract with you. You may have to hire an attorney to review the complex clauses of a forward contract. Finally, buying an asset outright eliminates the risk that the seller might not deliver the asset at T . FM: Derivatives Markets

c

Yufeng Guo

18

Derivatives Markets Chapter 2

c

Yufeng Guo

www.actuary88.com

• What’s bad: Outright purchase ties up your capital. In addition, if you don’t need the asset right away, you have to store the asset. Storing the asset may cost you money. If you get the asset at T through a forward contract • What’s good: Just in time delivery saves you storage cost. In addition, you don’t have to pay anything at t = 0. • What’s bad: You need to take the time to find a willing seller who has what you need and is willing to enter a forward contract with you. In addition, the seller in a forward contract may break his promise (credit risk). Make sure you can reproduce Figure 2.3. Make sure you memorize the conclusion of this section. Since it costs zero to store an index (index is not a physical asset such as wheat or corn), assuming there’s zero credit risk (assuming both parties in a forward contract keep their promises), then a forward contract and a cash index are equivalent investments, differing only in the timing of the cash flows. Neither form of investing has an advantage over the other. Make sure you understand the difference between payoff and profit; between a payoff diagram and a profit diagram.

2.1.5

Zero-coupon bonds in payoff and profit diagrams

The textbook says that “buying the physical index is like entering into the forward contract and simultaneously investing $1000 in a zero-coupon bond.” Please note that “investing $1000 in a zero-coupon bond” simply means that you put $1000 in a savings account. This way, at the expiration date T , your initial $1000 will grow into 1000 × 1.02 = 1200, which is exactly what you need to pay the seller to buy the index.

2.1.6

Cash settlement vs. delivery

The idea is simple. If you owe me an orange (worth $2) and I owe you an apple (worth $5), one way to settle out debts is that you give me an orange and I give you an apple. This is called the physical delivery. A simpler approach is that I give you $3. This is called the cash settlement.

2.1.7

Credit risk

This is the risk that one party in a contract breaks his promise and doesn’t do what he is supposed to do according to the contract.

2.2 2.2.1

Call options Option terminology

Make sure you understand the following terms: strike price, exercise, expiration, exercise style (American and European). The textbook explains well. Just follow the textbook. FM: Derivatives Markets

c

Yufeng Guo

19

Derivatives Markets Chapter 2

2.2.2

c

Yufeng Guo

www.actuary88.com

Payoff and profit for a purchased call option

Just memorize the key formulas: • Purchased call payoff=max(0, ST − K) • Purchased call profit=Purchased call payoff - FV of Option Premium • Purchased call profit=max(0, ST − K)- FV of Option Premium where ST is the stock price at expiration date T and K is the strike price.

2.2.3

Payoff and profit for a written call option

• Written call payoff + Purchased call payoff =0 (zero sum game) • Written call payoff =−max(0, ST − K) • Written call profit + Purchased call profit =0 (zero sum game) • Written call profit =FV of Option Premium −max(0, ST − K)

2.3 2.3.1

Put options Payoff and profit for a purchased put option

• Purchased put payoff=max(0, K − ST ) • Purchased put profit=Purchased put payoff - FV of Option Premium • Purchased put profit=max(0, K − ST ) - FV of Option Premium

2.3.2

Payoff and profit for a written put option

• Written put payoff + Purchased put payoff =0 (zero sum game) • Written put payoff = −max(0, K − ST ) • Written put profit + Purchased put profit =0 (zero sum game) • Written put profit = FV of Option Premium −max(0, K − ST )

2.4

Summary of forward and option positions

Make sure you understand Table 2.4. Make sure you are comfortable with the following terms: 1. Long Positions (a) Long forward. (b) Purchased call. (c) Written call. FM: Derivatives Markets

c

Yufeng Guo

20

Derivatives Markets Chapter 2

c

Yufeng Guo

www.actuary88.com

2. (a) Short forward. (b) Written call. (c) Purchased call.

2.5

Options are insurance

This is the basic idea. First, options are insurance. This shouldn’t surprise you. A call option is an insurance policy against the risk that the price of a stock may go up in the future. Similarly, a put option is an insurance policy against the risk that the price of a stock may go down in the future. Next, a homeowner insurance policy is a put option. This is a simple idea too. If you have a put option on a stock, you are guaranteed to sell your stock at a fixed price no matter how low the stock price has become. Similarly, if you have a homeowner insurance policy, no matter how low your house has become, you are guaranteed to sell your house to the insurance company at a fixed price (i.e. the insurance company will give you a fixed amount of money regardless of how low your house has become). Make sure you can recreate Figure 2.13. To recreate Figure 2.13, just draw a few critical points and then connect these points. There are 2 critical points: (House price, Profit)=(0, 160, 000), (200, 000, −15, 000). This is how to get the first critical point (0, 160, 000). If the house is blown away by big wind, the house price is zero. Then the insurer pays the full replacement value of the house $200,000 subject to a deductible of $25,000. So the insurance company will pay you 200, 000 − 25, 000 = 175, 000. However, you pay premium $15,000 at t = 0. If we ignore the lost interest of your premium (had you put your premium in a savings account, you would have earned some interest), then your profit is 175, 000 − 15, 000 = 160, 000. This is how to get the second critical point (200, 000, −15, 000). If after you bought the insurance policy, nothing bad happens to your house, then the payoff of your insurance is zero. You lost your premium $15,000; you pay $15,000 for nothing. Your loss is $15,000. This gives us the critical point (200, 000, −15, 000). Your can verify for yourself that if nothing bad happens to your house, even if the value of your house goes up, you’ll always lose $15,000 from the insurance policy. Now you should get Figure 2.13.

2.6

Example: equity-linked CD

SOA can easily test Formula (2.11). Memorize it. ST VT = V0 × 1 + k × max 0, −1 S0 





• VT is the CD value at time T • V0 is the CD value at time 0 • ST is the index value at time T • S0 is the index value at time 0 FM: Derivatives Markets

c

Yufeng Guo

21

Derivatives Markets Chapter 2

c

Yufeng Guo

www.actuary88.com

• k is the participation rate Make sure you can recreate Figure 2.14 and Table 2.5. ♦

FM: Derivatives Markets

c

Yufeng Guo

22

Chapter 3

Insurance, Collars, and Other Strategies Unfortunately, this is a messy chapter with lot of new phrases such as bull spread and collars. Just follow the text and understand these new phrases. The most important topic in this chapter is the put-call parity. Most likely SOA will test this topic. Make sure you understand the put-call parity. I’ll highlight the key points in this chapter.

3.1 3.1.1

Basic insurance strategies Insuring a long position: floors

A put option gives you a price floor. If you have a put option on an asset, then you don’t need to worry that the asset price drops below the strike price. If indeed the asset’s price drops below the strike price, you can exercise the put and sell the asset at the strike price. Make sure you can recreate Table 3.1 and Figure 3.1 The 6-month interest rate is 2%. A put with 1000-strike price with 6-month to expiration sells for $74.201. A call with 1000-strike price with 6-month to expiration sells for $93.809. Payoff at expiration T of Put + Stock is max(ST , K). If you have a put and a stock, then the value of Put+ Stock at the expiration date T is the greater of the strike price K and the stock price ST . If, at T , ST < K, then you can exercise your put and sell your stock for K. If ST ≥ K, then you just let your put expire worthless and your payoff is ST . So the payoff of Put + Stock at expiration=max(ST , K). When recreating Table 3.1, please note that Profit = Payoff - (Cost + Interest) To recreate Figure 3.1 (a), (b), (c), and (d), just plug in the data from Table 3.1. For example, to recreate Figure 3.1(d), enter the following points of (S&R index, Profit): (900, −95.68), (950, −95.68), (1000, −95.68), (1050, −45.68), (1100, 4.32), (1150, 54.32), and (1200, 104.32) Connect these points and you should get Figure 3.1(d). Make sure you can recreate Figure 3.2. Figure 3.2 is continuation of Figure 2.13. This is how to draw the profit diagram for the uninsured house. If the house is washed away by flood (ie. the price of the house becomes zero), then you loose $200,000. If the house appreciates and 23

Derivatives Markets Chapter 3

c

Yufeng Guo

www.actuary88.com

the new price is $250,000, then you gain 250, 000 − 200, 000 = 50, 000. So you have two data points: (Price of the house, Profit)=(0, −200, 000), (250, 000, 50, 000). Connecting these two points, you’ll get the profit line for the uninsured house. This is how to draw the diagram for the insured house. The profit is −40, 000 when the house price is from 0 to 175,000. If the damage to your house is equal to or greater than the deductible $25,000 (ie. the house price is 200, 000 − 25, 000 = 175, 000 or less), you’ll lose both the deductible and the premium. So your total loss is 25, 000 + 15, 000 = 40, 000. This is a straight line from (0, −40, 000) to (175, 000, −40, 000). If after you bought the insurance, nothing bad happens to your house and the price of your house goes up to say $250,000, then you gain $50,000 in the value of your house but you lose your premium. So your profit is 50, 000 − 15, 000 = 35, 000. This gives us the point (250, 000, 35, 000). If you connect (175, 000, −40, 000), (200, 000, −15, 000), and (250, 000, 35, 000), you’ll get the profit diagram when the house price is 175,000 or greater. The conclusion drawn from Figure 3.2 is that an insured house has a profit diagram that looks like a call option.

3.1.2

Insuring a short position: caps

A call option gives you a price ceiling (or cap). If you have a call option on an asset, then you don’t need to worry that the asset price may go up. If indeed the price of the asset goes above the strike price, you can exercise the call and buy the asset at the strike price. Make sure you can reproduce Table 3.2 and Figure 3.3.

3.1.3

Selling insurance

If you sell a call on a stock, you’d better have a stock on hand. This way, if the stock price goes up and the call is exercised, you already have a stock to deliver to the owner of the call. If you buy a stock and simultaneously sell a call option, you are selling a covered call. A covered call limits your loss in case the stock price goes up (because you already have a stock on hand). If you sell a call option but don’t have a stock on hand, you are selling a naked call. Writing a naked call is a big risk if the stock price goes up, in which case the call writer has to buy a stock from the market. Covered put. If you sell a put, you have two ways to cover it. One is to short sell a stock: when you sell a put, you simultaneously short sell a stock. If the stock happens to be worthless and the buyer of the put sells the worthless stock to you at the guaranteed price (i.e. strike price), you turn around and give the worthless stock to the broker who lent you a stock for short selling. Another way to cover your put is to set aside the present value of the strike price at t = 0. If the owner of the put exercises the put, you already have the strike price in your pocket to pay the owner of the put. If you don’t cover your put and the stock happens to be worthless, then you may suddenly find that you don’t have the money to pay the strike price. FM: Derivatives Markets

c

Yufeng Guo

24

Derivatives Markets Chapter 3

3.2

c

Yufeng Guo

www.actuary88.com

Synthetic forwards

Buying a call and selling a put on the same underlying with each option having the same strike price and time to expiration produces a synthetic forward. Difference between a synthetic long forward and an actual forward: 1. The actual forward contract has zero premium, while a synthetic forward requires a net option premium. 2. With the forward contract, we pay the forward price; with the synthetic forward, we pay the strike price.

3.2.1

Put-call parity

The net cost of buying the index using options must equal the net cost of buying the index using a forward contract. Call(K, T ) − P ut(K, T ) = P V (F0,T − K) To prove the parity equation, let’s rewrite it as Call(K, T ) + PV(K) = Put(K, T ) + PV(F0,T ). Proof. Assume that at t = 0, you hold two portfolios A and B. Portfolio A consists of two assets: (1) a call option with strike price K and maturity T , and (2) the present value of the strike price PV(K). Please note that at time T , PV(K) will become K and you’ll have K in your pocket. Portfolio B consists of two assets: (1) a put option with strike price K and maturity T , and (2) the present value of the forward price PV(F0,T ). Please note that at time T , PV(F0,T ) will accumulate to (F0,T ), which is the price you pay to buy one stock from the seller in the forward contract. Let’s consider 3 situations. 1. If at T , ST > K. For Portfolio A, you exercise the call option and buy one share of the stock at price K; Portfolio A is worth ST . For Portfolio B, you let your put option expire worthless. Portfolio B is also worth ST . 2. If at T , ST < K. For Portfolio A, you let your call option expire worthless. Portfolio A is worth K. For Portfolio B, you exercise the put option, sell your stock, and get K. So Portfolio B is also worth K. 3. If at T , ST = K. Both the call option and the put option expire worthless. Portfolio A and B are both worth ST = K. Under any situation, Portfolio A and B have equal values. Hence Call(K, T ) + P V (K) = P ut(K, T ) + P V (F0,T ) =⇒ Call(K, T ) − P ut(K, T ) = P V (F0,T − K) FM: Derivatives Markets

c

Yufeng Guo

25

Derivatives Markets Chapter 3

3.3

Spreads and collars

3.3.1

Bull and bear spreads

c

Yufeng Guo

www.actuary88.com

Suppose you expect that a stock price will go up. To make some money, you can do one of the following three things today • buy a share (or a forward contract) • buy a call • sell a put

Buying a stock is expensive and has the risk that the stock price may drop in the future. Buying a call requires paying upfront premium. Selling a put receives money upfront but has a large downside risk that the stock price may go down. Instead of the above 3 strategies, you can use a bull spread. Bull spread. Two ways to build a bull spread. • Buy one call at a lower strike price K1 and sell another call at a higher strike price K2 (> K1 ), with two calls on the same stock and having the same maturity. This strategy costs you a some money upfront. You get some premiums for selling another call at a higher strike price K2 (> K1 ), but by doing so you give up some upside potentials. If the stock price exceeds K2 , your selling price is capped at K2 . • Buy one put with a lower strike price K1 and sell another put with a higher strike price K2 (> K1 ), with two puts on the same stock and having the same maturity. This strategy gives you positive cash flows upfront but has zero or negative payoff. No matter you use calls or puts, in a bull spread, you always buy low and sell high. The word ”spread” means buying and selling the same type of option. The word ”bull” means that you can possibly make money if the stock price goes up. Bear spread. Two ways to build a bear spread. • Buy one call at a higher strike price and sell another call at a lower strike price, with two calls on the same stock and having the same maturity. (This strategy gives you positive cash flows upfront) • Buy one put with a higher strike price and sell another put with a lower strike price, with two puts on the same stock and having the same maturity. (This strategy costs you some money upfront) The word ”bear” means that you can possibly make money if the stock price goes down. No matter you use calls or puts, in a bear spread, you always buy high and sell low. FM: Derivatives Markets

c

Yufeng Guo

26

Derivatives Markets Chapter 3

3.3.2

c

Yufeng Guo

www.actuary88.com

Box spreads

A box spread is accomplished by using options to create a synthetic long forward at one price and a synthetic short forward at another price. A box spread is a means of borrowing or lending money: it has no stock price risk. Consider the textbook example. You can do the following two transactions at the same time: 1. Buy a $40-strike call and sell a $40-strike put. 2. Sell a $45-strike call and buy a $45-strike put. If you buy a $40-strike call and sell a $40-strike put, you are really buying a stock for $40. If you sell a $45-strike call and buy a $45-strike put, you are really selling a stock for $45. By doing these two things, you buy a stock for $40 and sell it for $45. This produces a guaranteed profit of $5. Of course, there’s no free lunch. The $5 profit to be earned at t = 0.25 (T = 90 days) must be discounted by the risk free interest rate. So the box spread must sell at t = 0 for 5 × 1.0833−0.25 = 4.9 As the textbook points out, $4.9 is exactly the net premium of the 4 options in the box spread.

3.3.3

Ratio spreads

Ratio spread. An options strategy in which an investor simultaneously holds an unequal number of long and short positions. A commonly used ratio is two short options for every option purchased. For example, a ratio spread can be achieved by buying one call option with a strike price of $45 and selling two call options with a strike price of $50. This allows the investor to capture a gain on a small upward move in the underlying stock’s price. However, any move past the higher strike price ($50) of the sold options will cause this position to lose value. Theoretically, an extremely large increase in the underlying stock’s price can cause an unlimited loss to the investor due to the extra short call.

3.3.4

Collars

Collars. A spread involves either a call or a put but not both. However, in a collar, you use both a call and a put. A collar is to buy a put with one strike price K1 and sell a call with a higher strike price K2 > K1 , with both options having the same underlying asset and having the same expiration date. The collar width is K2 − K1 . Say you have lot of stocks on Google. Say Google stocks are currently selling $200 per share. You are worrying that Google stocks may go down. You can put a lower bound on Google stock price by buying a put with strike price $180. This way, if Google’s stock price goes to $100, you can sell your Google stocks for $180. FM: Derivatives Markets

c

Yufeng Guo

27

Derivatives Markets Chapter 3

c

Yufeng Guo

www.actuary88.com

However, buying a put option requires money up front. To raise money to buy a put, you can sell a call option with strike price equal to $220. If the money you get from selling a call offsets the cost of buying a put, you have a zero-cost collar. Selling a call limits the stock’s upside potential. If Google’s stock price goes above $220, you won’t realize the gain because your stock will be called away at $220.

3.4

Speculating on volatility

Here the investor thinks that the stock price may change by a substantial amount but is unsure about the direction of the change (i.e. not sure whether the price will go up or down). The investor’s goal is to make some money whether the stock price will go up or down as long as it is within a range. The investor is speculating on the stock’s volatility.

3.4.1

Straddle

Straddle. In a straddle, you buy both a call and a put on the same stock with the same strike price and the same expiration date. Suppose that a court will issue its ruling on a company. The ruling may make or break the company so the company’s stock price may double or be cut in half. Unsure of the outcome, you can buy both a call and a put. If the stock price goes up, you exercise the call; if the stock price goes down, you exercise the put. The disadvantage of a straddle is the high premium cost. The investor has to pay both the call premium and the put premium. Strangle. A strangle is similar to a straddle except that now you buy an out-of-money call and an out-of-money put. A strangle is cheaper than a straddle. Written straddle v.s. purchased straddle. If you think the stock volatility is higher than the market’s assessment (i.e. you think that the option is underpriced), you buy a straddle. If you think that the stock volatility is lower than expected (i.e. you think the option is over priced), you write (i.e. sell) a straddle.

3.4.2

Butterfly spread

A butterfly spread insures against large losses on a straddle.

3.4.3

1

Asymmetric butterfly spreads

Pay attention to the following formulas λ=

K3 − K2 K3 − K1

K2 = λK1 + (1 − λ)K3 1

Refer http://www.bigtrends.com/showQuestion.do?q_id=1735

FM: Derivatives Markets

c

Yufeng Guo

28

Derivatives Markets Chapter 3

3.5

c

Yufeng Guo

www.actuary88.com

Example: another equity-linked note

The main point of this example is that debt can be designed like an option. Marshall & Ilsley Corp paid its debt by making annual coupons 6.5% before maturity and by paying shares of its stocks at maturity. The number of shares paid by Marshall & Ilsley Corp at the maturity of the debt was calculated using the formula in Table 3.7. According to Exam FM syllabus, candidates need to know definitions of key terms of financial economics at an introductory level. This section seems to be beyond the introductory level. So don’t spend lot of time on convertible bond. ♦

FM: Derivatives Markets

c

Yufeng Guo

29

Derivatives Markets Chapter 3

c

Yufeng Guo

www.actuary88.com

FM: Derivatives Markets

c

Yufeng Guo

30

Chapter 4

Introduction to risk management Firms convert inputs into goods and services. A firm is profitable if the cost of what it produces exceeds the cost of the inputs. A firm that actively uses derivatives and other techniques to alter its risk and protect its profitability is engaging in risk management.

4.1

Basic risk management: the producer’s perspective

This is a case study on Golddiggers. Golddiggers has to pay its fixed cost. As long as the gold price is higher than the variable cost, Golddiggers should continue its production. Golddiggers sells gold and has an inherent long position in gold. It needs to manage the risk that the price of gold may go down. To lock in price of gold, Golddiggers can sell forwards, buy puts, and buy collars.

4.1.1

Hedging with a forward contract

A producer can use a short forward contract to lock in a price for his output. Golddiggers can enter into a short forward contract, agreeing to sell gold at a price of $420/oz. in 1 year. Make sure you can reproduce Table 4.2 and Figure 4.1.

4.1.2

Insurance: guaranteeing a minimum price with a put option

A downside of locking-in gold price with a short forward contract is that gold price may go up. If gold price goes up, a firm can’t profit from the rise of the price because it has locked in fixed price during the life of the forward contract. To gain from the rise of the product price and still have a floor on the price, a producer can buy a put. Make sure you understand Table 4.3 and Figure 4.2. Sometimes a short forward gives a producer more profit; other times buying a put yield more profit. No hedging strategy always outperforms all other strategies.

4.1.3

Insuring by selling a call

Golddiggers can sell a call. Selling a call earns premiums. However, if the gold price goes up beyond the call strike price, Golddiggers has to sell gold at the strike price. 31

Derivatives Markets Chapter 4

c

Yufeng Guo

www.actuary88.com

Make sure you understand Figure 4.4.

4.1.4

Adjusting the amount of insurance

Buying a put is like buying insurance. Insurance is expensive. There are at least 2 ways to reduce insurance premium: • Reduce the insured amount by lowering the strike price of the put. This permits some additional losses. • Sell some of the gain and put a cap on the potential gain.

4.2

Basic risk management: the buyer’s perspective

A buyer faces price risk on an input and has an inherent short position in the commodity. If the price of the input goes up, the buyer’s profit goes down.

4.2.1

Hedging with a forward contract

A long forward contract lets a buyer lock in a price for his input. For example, Auric can lock in gold price at $420 per ounce. Make sure you understand Table 4.4 and Figure 4.6.

4.2.2

Insurance: guaranteeing a maximum price with a call option

Auric might want to put a cap on gold price but pay the market price if the price of gold falls. Auric can buy a call option. Make sure you understand Table 4.5.

4.3

Why do firms manage risk?

In a world with fairly priced derivatives, no transaction costs, and no other market imperfections such as taxes, derivatives don’t increase the value of cash flow; they change the distribution of cash flows. Using derivatives, Golddiggers shifts dollars from high gold prices states to low gold price states. Hedging is beneficial for a firm when an extra dollar of income received in times of high profits is worth less than an extra dollar of income received in times of low profits.

4.3.1

An example where hedging adds value

Make sure you can reproduce the calculation that the expected profit before tax is $0.1. Make sure you know why the expected profit after tax is -$0.14. On the after tax basis, there’s 50% chance that the profit is $0.72 and 50% chance that the profit is -$1. So the expected after-tax profit is: 0.5 × (0.72 − 1) = −0.14 Make sure you can reproduce Figure 4.8. Point A and B are based on Table 4.6. Point C is based on the fact that if the unit price is $10.1, then the after-tax profit is $0.06. The FM: Derivatives Markets

c

Yufeng Guo

32

Derivatives Markets Chapter 4

c

Yufeng Guo

www.actuary88.com

product cost is $10. So the price $10.1 gives us $0.1 profit before tax, which is $0.06 after tax. Point D is the expected after-tax profit of -$0.14 if the unit price is either $9 or $11.2 with equal probability. It’s calculated as follows: 0.5 × (0.72 − 1) = −0.14 Line ACB is concave. A concave is an upside down U shape. When profits are concave, the expected value of profits is increased by reducing uncertainty.

4.3.2

Reasons to hedge

Concave profit patterns can arise from: • Tax. If you can fully deduct your loss as it occurs, then you might not need to hedge every loss; you can just incur loss and deduct your loss from your gain to reduce your taxable income. However, if you can’t fully deduct your loss or if you can deduct this year loss the next year, then losses are no good. You might want to use derivatives to hedge against potential losses. • Bankruptcy and distress costs. A large loss can threaten the survival of a firm. A firm may be unable to meet its fixed obligations (such as debt payments and wages). Customers may be less willing to purchase goods of a firm in distress. Hedging allows a firm to reduce the probability of bankruptcy or financial distress. • Costly external financing. If you pay losses, you have less money for money-making projects. You might have to borrow money to fund your projects. Raising funds externally can be costly. There are explicit costs (such as bank and underwriting fees) and implicit costs due to asymmetric information. So you might want to use derivatives to hedge against losses. Costly external financing can lead a firm to forego investment projects it would have taken had cash been available to use for financing. Hedging can safeguard cash reserves and reduce the need of raising funds externally. • Increase debt capacity. The amount that a firm can borrow is its debt capacity. When raising funds, a firm may prefer debt to equity because interest expense is taxdeductible. However, lenders may be unwilling to lend to a firm that has a high level of debt; high-debt companies have higher probability of bankruptcy. Hedging allows a firm to credibly reduce the riskiness of its cash flows, and thus increase its debt capacity. • Managerial risk aversion. Managers have incentives to reduce uncertainty through hedging. • Nonfinancial risk management. If you start a new business, you may need to decide on where to set up the plan and choose between leasing and buying equipment. You need to think through various risks associated with your decision and find ways to manage your risk. Risk management is not a simple matter of hedging or not hedging using financial derivatives, but rather a series of decisions that start when the business is first conceived.

4.3.3

Reasons not to hedge

Reasons why firms may elect not to hedge: FM: Derivatives Markets

c

Yufeng Guo

33

Derivatives Markets Chapter 4

c

Yufeng Guo

www.actuary88.com

• High transaction cost in using derivatives (commissions and the bid-ask spread) • A firm must assess costs and benefits of a given strategy; this requires costly expertise. (You need to hire experts to do hedging for you. Experts aren’t cheap!) • The firm must set up control procedures to monitor transactions and prevent unauthorized trading. • If you use derivatives to hedge, be prepared for headaches in doing accounting and filing tax returns. Accounting and tax for derivatives are complex. • A firm can face collateral requirements if their derivatives lose money.

4.3.4

Empirical evidence on hedging

• About half of nonfinancial firms report using derivatives • Big firms are more likely to use derivatives than small firms. • Among firms that do use derivatives, less than 25% of perceived risk is hedged • Firms are more likely to hedge short term risks than long term risks. • Firms with more investment opportunities are more likely to hedge. • Firms that use derivatives have a higher market value and more leverages.

4.4

Golddiggers revisited

This section discusses additional hedging strategies for Golddiggers. The textbook has clear explanations. Follow the textbok. ♦

FM: Derivatives Markets

c

Yufeng Guo

34

Chapter 5

Financial forwards and futures 5.1

Alternative ways to buy a stock

Make sure you understand Table 5.1 from the textbook. • Outright purchase. This is like buying things from Wal-Mart. After paying the cashier at the checkout, you immediately own the items bought. • Fully leveraged purchase. This is like buying a car with a car loan. You go to a bank and borrow the full price of the car. Then you pay the car dealer and bring the car home right away. • Prepaid forward contract. This is pre-ordering an item. You pay the full price at t = 0. The product is delivered to you at time T > 0. • Forward contract. This is another way of pre-ordering an item. You don’t pay anything at t = 0. The product is delivered to you at time T > 0. You’ll pay the full price at T .

5.2

Tailing

Next, let’s learn a phrase called tailing. Tailing answers the following questions: My goal is to possess one share of stock at time T . How many shares of stock do I need to buy at t = 0 so I’ll have one share of stock at time T ? If the stock doesn’t pay dividend, you’ll need to buy one share of stock at t = 0 in order to have one share at time T . However, if the stock pays dividend at a continuous rate of δ, you can reinvest your dividend continuously and keep buying additional fractional shares of stocks. This way, you need to buy only a fractional share of stock at t = 0 to have one share of stock at time T . The textbook shows you that if you buy one share at t = 0, after dividends are continuously reinvested, your initial one share will become eδT shares at time T . Consequently, you’ll need to buy e−δT share of stock at t = 0 to have one share of stock at T . 35

Derivatives Markets Chapter 5

c

Yufeng Guo

www.actuary88.com

t =0 1 share

Time T e T shares

t =0 e T share

Time T 1 share

Tailing Diagram: e−δT share at t = 0 becomes one share at time T . δ is the continuously compounded dividend rate.

5.3

Pricing a forward contract

Price of a prepaid forward contract is: P = S - PV of future dividends (if dividends are discrete) • F0,T 0 P = S e−δT (if dividends are continuous) • F0,T 0

Price of a forward contract is: • F0,T = S0 ert - FV of dividends at time T (if dividends are discrete) • F0,T = S0 e(r−δ)T (if dividends are continuous) The textbook derives these formulas using 3 approaches: by analogy, by discounting cash flows, and by the no-arbitrage principle. The easiest way to derive and memorize these formulas, however, is to use the cost-of-carry concept. Cost of carry: The forward price F0,T must equal the spot price at t = 0 plus the cost of carrying the spot price asset from t = 0 to T for delivery. Suppose you and I sign a forward contract at t = 0. This contract requires me to deliver a stock at time T to you and requires you to pay me F0,T at time T . The essence of this forward contract is that I need to sell a stock to you at time T for a preset price F0,T . Let’s calculate how much it costs me to have a stock ready for delivery at time T . If the stock pays continuous dividend at the rate of δ, to have one share of stock at time T , I just need to buy e−δT share at t = 0. Then the cost of having a stock ready for delivery at T is: Spot price at time zero + Cost-of-carry during [0, T ] Spot price at time zero = Cost of buying e−δT share of stock at time zero=S0 e−δT FM: Derivatives Markets

c

Yufeng Guo

36

Derivatives Markets Chapter 5

c

Yufeng Guo

www.actuary88.com

Cost-of-carry during [0, T ]=Foregone interest of tying my capital during [0, T ] The foregone interest of tying my capital during [0, T ] is S0 e−δT (erT − 1), where r is the continuously compounded risk-free interest rate. Had I put S0 e−δT in a bank account (instead of buying stocks) at t = 0, my fund balance at T would be S0 e−δT erT and I would have earned S0 e−δT erT − S0 e−δT = S0 e−δT (erT − 1) interest during [0, T ]. So the cost of having a stock ready for delivery at T is: S0 e−δT + S0 e−δT (erT − 1) = S0 e−δT erT = S0 e(r−δ)T The forward price is: F0,T = S0 e−δT erT = S0 e(r−δ)T If the stock doesn’t pay any dividend, to find the forward price, just set δ = 0: F0,T = S0 erT What if the stock pays discrete dividends? Easy. To have a stock ready for delivery at T , I can buy one share of stock at time zero and pay S0 . The foregone interest of tying up my capital S0 during [0, T ] is S0 erT − S0 . The dividend I receive will reduce my cost of carry. Then the forward price is: S0 + S0 erT − S0 - FV of all the dividends received=S0 erT - FV of all the dividends received Next, let’s apply the cost-of-carry concept to a prepaid forward contract. With a prepaid forward contract, the buyer pays me right away and I don’t tie up my capital any more; my foregone interest is zero. The PV of the dividends I will receive during [0, T ] will reduce of the cost of carrying the asset from time zero to T . Now we have: P =Spot price + Cost of carry =S - PV of future dividends (If dividends are discrete) F0,T 0

However, if the dividend is continuous, to have one stock ready for delivery at T , I just need to buy e−δT share of stock at time zero. So the prepaid forward price is: P = S e−δT F0,T 0 Now you shouldn’t have trouble memorizing the pricing formulas for a prepaid forward and a typical (post-paid) forward contract.

5.4

Forward premium

The forward premium is the ratio of the forward price to the spot price: Forward Premium=

F0,T S0

The annualized forward premium (AFP) as:  is defined  F0,T 1 AF P ×T S0 e = F0,T =⇒ AF P = T ln S0

FM: Derivatives Markets

c

Yufeng Guo

37

Derivatives Markets Chapter 5

5.5

c

Yufeng Guo

www.actuary88.com

Creating a synthetic forward contract

The seller in a forward contract faces the risk that the security’s spot price at time T (that is ST ) may exceed the forward price F0,T by a large amount. For example, you and I sign a forward contract at t = 0. Under the contract, I need to give you one share of ABC stock for $20 at time T . If one share of ABC stock will sell for $100 at T , I will lose $80. (If I sell one share of ABC stock in the market at T , I can earn $100. However, I have to sell one share of ABC stock to you at for $20. So I lose $80 and you gain $80.) At the expiration date T : • The payoff earned by the buyer in the forward contract is ST − F0,T • The payoff earned by the seller in the forward contract is F0,T − ST • The payoff earned by the seller + the payoff earned by the buyer=0 To hedge the risk that the spot price ST may rise, the seller in a forward contract can buy e−δT share of stock at t = 0 by paying a fixed cost of S0 e−δT and hold it to T for delivery. This way, he’ll already have one share of the stock and doesn’t have to worry how high ST may become. (If he doesn’t have one share of stock ready at T for delivery and ST happens to be very high, the seller is forced to buy the stock at in the open market and will suffer a huge loss). To avoid tying up his own money, the seller can go to a bank and borrow S0 e−δT . When the delivery time T arrives, the seller just needs to pay back the principal and the interest to the bank. The sum of the principal and the interest due at T is S0 e−δT erT . This strategy costs the seller nothing to set up but gives the seller ST − S0 e−δT erT at time T . Please note that ST − S0 e−δT erT is exactly the net payment at T by the seller to the buyer in a fairly priced forward contract. A fairly priced forward contract on one share of stock to be delivered at T is priced at F0,T = S0 e−δT erT . Then at time T , the buyer needs to pay the seller F0,T = S0 e−δT erT ; the seller needs to deliver a stock worth ST . Instead of the buyer writing the seller a check in the amount of F0,T = S0 e−δT erT and the seller delivering a stock worth ST to the buyer, the forward contract can be settled by having the seller send the buyer a check in the amount of ST − S0 e−δT erT . These transactions are summarized in Table 5.3. The textbook gives you the formula “Forward = Stock - Zero-coupon bond.” To understand what this means, notice that borrowing money from a bank is the same as issuing a zero coupon bond; lending money to someone is just like buying a zero-coupon bond. Say you borrow $100 from a bank at time zero. You plan to pay off your loan at time one. The interest rate is 8%. Your total loan payment (interest plus principal) to the bank at time one is 100 × 1.08 = 108. Conceptually, this is the same as you issuing (ie. selling) a 1-year zero coupon bond with $100 par value and 8% annual coupon to the bank. At time zero, you collect $100 from a bank. At time one, you pay the bank face amount plus coupon in the total amount of 100 + 8=108. So conceptually, borrowing from a bank is selling a zero coupon bond to the bank. Lending your money to bank (ie. putting money in savings account) is the same as buying a zero coupon bond. FM: Derivatives Markets

c

Yufeng Guo

38

Derivatives Markets Chapter 5

c

Yufeng Guo

www.actuary88.com

“Forward = Stock - Zero-coupon bond” means that borrowing money and buying some stock can synthetically create a forward contract. As explained earlier, if at time zero you borrow S0 e−δT and buy e−δT share of stock, then at time T you’ll have ST − S0 e−δT erT , which is exactly the payoff earned by the buyer in the forward contract. To help memorize the equation “Forward = Stock - Zero-coupon bond”, notice that negative zero-coupon bond means borrowing money (ie issuing a zero coupon bond). Make sure you understand Table 5.4 and Table 5.5. Table 5.4 has two transaction. First, you enter a forward contract to buy one share of stock at T (i.e. having a long position in a forward contract). To make sure you indeed can pay the forward price F0,T = S0 e−δT erT at time T , at time zero you deposit the present value of this forward price F0,T e−rt = S0 e−δT erT e−rt = S0 e−δT to earn risk free interest rate r. Then at time T , you’ll have exactly F0,T = S0 e−δT erT , which you give to the seller and receive one share of stock. So your initial cost at time zero is S0 e−δT . Your wealth at time T is ST (ie. one share of stock). The formula “Stock=Forward + Zero-coupon bond” means that buying a forward contract and a zero coupon bond is the same as owning a stock. Please note a positive sign before zero-coupon bond means that you are lending more (ie buying a zero-coupon bond). Table 5.5 shows how to synthetically create a savings account. At time zero, you sell one forward contract and simultaneously buy eδT share of stock. Then at T , you have exactly one share of stock, which you deliver to the buyer and get F0,T . So your initial cost at time zero is S0 eδT ; your wealth at time T is F0,T . This is like you putting S0 eδT at time zero into a savings account and getting F0,T at time T . The formula “Zero-coupon bond = Stock - Forward” means that buying a stock and selling is the same as lending more (ie. buying a zero-coupon bond).

5.5.1

Synthetic forwards in market-making and arbitrage

The seller in a forward contract can hedge his risk. The seller’s biggest risk is this: If at time T the stock price is ST = ∞ and he doesn’t already have one share of stock ready for delivery at T , then he’s forced to buy a share of stock from the open market at time T and pay ST = ∞; he’ll be bankrupt. To hedge this risk, the seller needs to have one share of stock ready ahead of time. The seller’s hedging transactions are listed at Table 5.6. The buyer in a forward contract can also hedge his risk. The buyer’s biggest risk is this: if at time T the stock price is ST = 0, he has to pay F0,T and buy a worthless stock. To hedge this risk, the buyer should somehow get rid of this worthless stock immediately after receiving it. This can be achieved by shorting selling e−δT share of stock at time zero (then at time T one share of stock needs to be returned to the broker who facilitates the short selling). These hedging transactions are listed in Table 5.7. How to arbitrage if the forward price is too high or too low. If the forward price is too high, you can use cash-and-carry to arbitrage: you buy stocks at time zero and carry it forward to T for delivery. Cash-and-carry transactions are listed in Table 5.6. If, on the other hand, the forward price is too low, you use reverse cash-and-carry: at time zero, you short sell stocks and simultaneously buy a forward. Reverse cash-and-carry FM: Derivatives Markets

c

Yufeng Guo

39

c

Yufeng Guo

Derivatives Markets Chapter 5

www.actuary88.com

transactions are listed in Table 5.7.

5.5.2

No-arbitrage bounds with transaction costs

How to derive the no-arbitrage upper bond. If the forward price F0,T is too high, this is how to make some money: • (1) Buy low: At t = 0 pay ask price and buy one stock. Incur transaction cost k. Total cash outgo: S0a + k. • (2) Sell high: At t = 0 sell a forward contract. Incur transaction cost k. Total cash outgo: k. • (3) At t = 0, borrow S0a + 2k to pay for (1) and (2) • (4) At time T , deliver the stock to the buyer. Get F0,T . • (5) At time T , pay (S0a + 2k)er

bT

Your initial cash outgo after (1) through (5) is zero. Your payoff at time T is F0,T − (S0a + 2k)er

bT

Arbitrage is possible if: F0,T − (S0a + 2k)er

bT

>0

⇒ F0,T > F + = (S0a + 2k)er

bT

How to derive the no-arbitrage lower bond. If the forward price F0,T is too low, this is how to make some money: • (1) Buy low: At t = 0 enter a forward contract to buy one stock. Incur transaction cost k. Total cash outgo: k. • (2) Sell high: At t = 0 sell a stock short and get bid price S0b . Incur transaction cost k. Total cash inflow: S0b − k. • (3) After (1) and (2), the net cash inflow is S0b − 2k. At time zero, lend S0b − 2k at the lending rate rl (i.e. depositing S0b − 2k into a savings account). At time T , your l account should be (S0b − 2k)er T . • (4) At time T , pay F0,T and buy one stock from the seller in the forward contract. Return the stock to the broker who facilitated the short sale. l

• (5) At time T , take out (S0b − 2k)er T from the savings account. Your initial cash outgo after (1) through (5) is zero. Your payoff at time T is l

(S0b − 2k)er T − F0,T FM: Derivatives Markets

c

Yufeng Guo

40

c

Yufeng Guo

Derivatives Markets Chapter 5

www.actuary88.com

Arbitrage is possible if: l

(S0b − 2k)er T − F0,T > 0 l

⇒ F0T < F − = (S0b − 2k)er T To avoid arbitrage, we need to have: l

bT

l

bT

(S0b − 2k)er T = F − ≤ F0,T ≤ F + = (S0a + 2k)er How to memorize (S0b − 2k)er T = F − ≤ F0,T ≤ F + = (S0a + 2k)er

Please note that S0b < S0a due to the bid-ask spread. In addition, rl < rb ; when you put money in a savings account (i.e. lending money to a bank), you get a lower interest rate; when you borrow money from a bank, you pay a higher interest rate. Your lower bound should be small; your higher bound should be big. So it makes sense to have l b (S0b − 2k)er T = F − ≤ F0,T ≤ F + = (S0a + 2k)er T Please note that there may be complex structure for the transaction cost where the above non-arbitrage formulas can’t be blindly used. Under a complex transaction cost structure, you’ll want to derive the non-arbitrage bound using the approach presented above. Please refer to my solution to the textbook problem 5.15 on how to handle complex transaction cost.

5.5.3

Quasi-arbitrage

In a full arbitrage, you either borrow money from a bank to buy a stock (if it’s priced low relative to a forward) or short sell a stock (if it is priced high relative to a forward). If you don’t need to borrow money or short sell a stock yet you can still earn free money, you are doing a quasi-arbitrage. Quasi-arbitrage Example 1 (textbook example) The corporation borrows at 8.5% and lends at 7.5%. Knowing the formula “Zero-coupon bond = Stock - Forward,” it can build (synthetically create) a zero coupon bond using stocks and forward. In this zero coupon bond, the corporation lends money at time zero and gets principal and interest paid at time T . If it earns more than 7.5% (such as 8%) in this synthetically created zero-coupon bond, the corporate can stop lending at 7.5% and keep building lots of zero-coupon bonds to earn an 8% interest rate. This is a quasi-arbitrage. Here the corporation doesn’t need to borrow money from a bank to make free money (because it already has borrowed money at 8.5% and it’s now trying to pay off its debt). If the corporation doesn’t already have a 8.5% debt to begin with, it will be silly for the corporation to borrow at 8.5%, buy a bunch of stocks, and sell bunch of forwards so it can use the formula “Zero-coupon bond = Stock - Forward” to earn an 8% interest rate. Quasi-arbitrage Example 2 Suppose the forward price on a stock is too low and you already own a stock. To arbitrage, you can sell your stock (instead of selling a stock short) and buy a forward contract. You FM: Derivatives Markets

c

Yufeng Guo

41

Derivatives Markets Chapter 5

c

Yufeng Guo

www.actuary88.com

can deposit the sales proceeds in a savings account and earn interest. Then on the expiration date T of the forward contract, you pay a low forward price F0,T and buy a stock. The net effect is that you temporarily give up your ownership of a stock during [0, T ] but regain the ownership of a stock at T . Such a transaction will give you some free money if F0,T is too low.

5.5.4

Does the forward price predict the future spot price?

Though the author says that the forward price F0,T doesn’t reflect the future spot price ST , this is a controversial topic. Experts disagree on this issue. Some argue that F0,T reflects ST . For example, if you think ST will be low, you don’t want to pay a high forward price F0,T . However, since SOA chose Derivatives Markets as the textbook, you might want to accept the view that F0,T doesn’t reflect the expected future spot price ST at least when you are taking the exam.

5.6 5.6.1

Futures contracts Role of the clearing house

The clearinghouse guarantees the performance of every buyer and every seller of a futures contract. It stands as a buyer to every seller and a seller to every buyer. The buyer and the seller don’t have to worry that the counterparty may default. Instead of having the seller and the buyer in a forward contract deal with each other directly, the clearinghouse sits between them serving as a counterparty to each of them. To the buyer, the clearinghouse is the seller; the clearinghouse is obligated to deliver the asset to the buyer at the agreed upon price. To the seller, the clearinghouse is the buyer; the clearinghouse is obligated to buy the asset at the expiration date with the agreed upon price.

Money

Money Clearing House

Buyer Asset

Seller Asset

With a clearinghouse as the counterparty, the buyer/seller in a futures contract can easily liquidate his position prior to the expiration date. If the buyer or seller wants to back out from a futures contract before the contract matures, he can do a reverse trade and close out his position.

5.6.2

S&P 500 futures contract

The most important concept for the purpose of passing the exam is the value of one S&P 500 futures contract. The value of one S&P 500 futures contract is calculated by multiplying the FM: Derivatives Markets

c

Yufeng Guo

42

Derivatives Markets Chapter 5

c

Yufeng Guo

www.actuary88.com

futures price by 250: One S&P 500 futures contract value = 250× futures price For example, if the S&P 500 futures price is 1000, then one S&P 500 futures contract is worth 250, 000 = 250, 000.

5.6.3

Difference between a forward contract and a futures contract

This is a very important topic. The textbook has a nice comparison of forwards and futures. Make sure you memorize the differences. • Forward contracts are settled at expiration; futures contracts are settled daily through marking to market. • Forward contracts are less liquid; futures contracts are more liquid. • Forward contracts are customized to fit the special needs of the buyer and the seller (hence less liquid); futures contracts are standardized (hence more liquid). • Forward contracts have credit risk (any party may break its promises); futures contracts have minimal credit risks. • Futures markets have daily price limits while forward contracts don’t. Please note that these are general difference. Some forward contracts (such as in foreign exchanges) are standardized and liquid; some futures contracts give the buyer and the seller greater flexibility in contractual obligations. Final point. Forward contracts are typically simpler than futures contracts. Forwards are settled at the expiration date; futures contracts are settled daily and have margins requirement. As a result, forward contracts require less management control and are good for unsophisticated and infrequent users.

5.6.4

Margins and markings to market

Open interest. Open interest is the total number of outstanding contracts held by market participants at the end of each day. For each seller of a futures contract there is a buyer. A seller and a buyer together are counted as only one contract. (The clearinghouse is excluded in the calculation of the open interest since its net position is zero). To determine the total open interest for any given market we need only to know the totals from one side or the other, buyers or sellers, not the sum of both. If two parties have just signed a futures contract, the open interest will increase by one contract. If the buyer and the seller have closed out a futures contract, the open interest will decrease by one contract. If one party passes off his position to a new party (one old buyer sells to one new buyer), then the open interest will not change. Initial margin, Maintenance margin, Margin call. In the futures market, margin is the initial “good faith” deposit made into an account in order to enter into a futures contract. When you open a futures contract, the futures exchange will ask you to deposit a minimum amount of money (cash or cash equivalent such as T-bills) into your account. This original FM: Derivatives Markets

c

Yufeng Guo

43

c

Yufeng Guo

Derivatives Markets Chapter 5

www.actuary88.com

deposit is called the initial margin. At initial execution of a trade, both the seller and the buyer must establish a margin account. When your position is closed out, you will be refunded the initial margin plus or minus any gains or losses that occur over the span of the futures contract. The initial margin is the minimum deposit one needs to put in the margin account before he can enter into a new futures contract. The maintenance margin is the lowest amount an account can reach before needing to be replenished. If the margin account drops to a certain level due to daily losses, the buyer or the seller will get a margin call asking him to deposit more money into the account to bring the margin back up. Marking to market. On any date when futures contracts are traded, futures prices may go up or down. Instead of waiting until the maturity date for the buyer and the seller to realize all gains and losses, the clearinghouse requires that the buyer and the seller recognize their gain and loss at the end of each day. The best way to explain marking to market is through an example. Example 1. Assume the current futures price for gold for delivery 6 days form today is $495 per ounce. Contract size is 100 ounces. The initial margin is 5% of the contract value. The maintenance margin is $2,000. You are given the following prices of gold futures. Day 0 (Today) 1 2 3 4 5 6 (Delivery Date)

Futures Price $495 $496 $487 $493 $480 $490 $485

Calculate the daily account value of the buyer’s margin account. Determine when the buyer gets the margin call. Solution Day 0 1 2 3 4 5 6

Futures price $495 $496 $487 $493 $480 $490 $485

price change

Gain/loss

$1 -$9 $6 -$13 $10 -$5 Total

$100 -$900 $600 -$1,300 $1,000 -$500 -$1,000

Margin Bal $2,475 $2,575 $1,675 $2,600 $1,300 $3,000 $2,500

Margin call $0 $325 $0 $700 $0 $0

Explanations: FM: Derivatives Markets

c

Yufeng Guo

44

c

Yufeng Guo

Derivatives Markets Chapter 5

www.actuary88.com

• The futures prices $495, $496, $487, $493, $480, $490, and $485 all have Day 6 as the delivery time. The Day 6 futures price $485 is the spot price on Day 6. • The initial margin is 2475 = 100×495×5%. Both the buyer and the seller must deposit at least this amount to their respective margin account to initiate a futures contract. • 2, 575 = 2, 475 + 100. Each day the clearinghouse updates the delivery price using the current day’s futures price. So on Day 1, the clearinghouse (who acts as the seller to the buyer) assumes that it will delivery 100 ounces of gold to the buyer at the delivery price of $496 per ounce. The buyer gain $1 per ounce (it locked in the $495 delivery price, while the current delivery price is $496). $100 gains are immediately added to the buyer’s margin account. • 1, 675 = 2, 575 − 900. The buyer loses $9 per ounce. Yesterday, the clearinghouse could delivery gold at the price of $496 per ounce. However, today the clearinghouse can deliver it at a cheaper price of $487 per ounce. The total loss $900 was immediately deducted from the buyer’s margin account. • 325 = 2, 000 − 1, 675. The margin balance $1,675 is below the maintenance margin of $2,000. So the buyer must deposit $325 to bring his margin account to the maintenance margin level. • 2, 600 = 1, 675 + 325 + 600. The buyer gains $6 per ounce.

The seller’s margin account: Day 0 1 2 3 4 5 6

Futures price $495 $496 $487 $493 $480 $490 $485

price change

Gain/loss

$1 -$9 $6 -$13 $10 -$5 Total

-$100 $900 -$600 $1,300 -$1,000 $500 $1,000

Margin Bal $2,475 $2,375 $3,275 $2,675 $3,975 $2,975 $3,475

Margin call $0 $0 $0 $0 $0 $0

Example 2 Reproduce Table 5.8 in Derivatives Markets. Solution Facts: • The notional value of one contract = 250 × 1, 100 • The number of contracts is 8 • Initial margin = 10% • Margin account earns 6% continuously compounding FM: Derivatives Markets

c

Yufeng Guo

45

c

Yufeng Guo

Derivatives Markets Chapter 5 week 0 1 2 3 4 5 6 7 8 9 10

multiplier 2,000.00 2,000.00 2,000.00 2,000.00 2,000.00 2,000.00 2,000.00 2,000.00 2,000.00 2,000.00 2,000.00

Futures Price $1,100.00 $1,027.99 $1,037.88 $1,073.23 $1,048.78 $1,090.32 $1,106.94 $1,110.98 $1,024.74 $1,007.30 $1,011.65

price change $0.00 -$72.01 $9.89 $35.35 -$24.45 $41.54 $16.62 $4.04 -$86.24 -$17.44 $4.35

www.actuary88.com

interest earned

Loss

$253.99 $88.01 $110.95 $192.70 $136.47 $232.54 $271.19 $280.83 $82.02 $41.85

-$144,020 $19,780 $70,700 -$48,900 $83,080 $33,240 $8,080 -$172,480 -$34,880 $8,700

Margin balance $220,000.00 $76,233.99 $96,102.01 $166,912.96 $118,205.66 $201,422.13 $234,894.67 $243,245.86 $71,046.69 $36,248.72 $44,990.57

Explanations • The margin account balance at the end of Week 0 is 220, 000 = 8 × 250 × 1, 100 × 10% 6%

• The interest earned in Week 1 is 253.99 = 220, 000(e 52 − 1) • The loss incurred in Week 1 is −144, 020 = 8 × 250 × (−72.01) • The margin account balance at the end of Week 1 is 76, 233.99 = 220, 000 + 253.99 − 144, 020 6%

• The interest earned in Week 2 is 88.01 = 76, 233.99(e 52 − 1)

5.6.5

Comparing futures and forward prices

At expiration T , forward price and futures price are both equal to the spot price ST . At T − 1 (one day before expiration), a futures contracts will be marked to market the very next day T and a forward contract will also be marked to market at the very next day T (a forward contract is marked to market only at T ). As a result, one day prior to expiration, a forward contracts and a futures contracts have identical cash flows and should have the same price. The price of a futures contract two or more days before expiration is more complex. However, the following general conclusions can be drawn: • If interest rates are positively correlated with futures prices (i.e. interest rates go up if futures prices go up), then the investor holding a long position (i.e. the buyer) will prefer a futures contract to a forward contract. When futures prices go up, the buyer realizes his gain through daily marking-to-market. This gain earns a higher interest rate. On the other hand, if futures prices decline, interest rates also decline. The buyer suffer losses through marking to market but losses are incurred when the opportunity cost is lower (because interest rates are lower). On average, a long futures contract will outperform a long forward contract. FM: Derivatives Markets

c

Yufeng Guo

46

Derivatives Markets Chapter 5

c

Yufeng Guo

www.actuary88.com

• If interest rates are negatively correlated with futures prices (i.e. interest rates go down if futures prices go up), then the investor holding a long position (i.e. the buyer) will prefer a forward contract to a futures contract. When futures prices go up, the buyer’s gain through marking to market is reinvested at a lower interest rate. When future prices go down, the buyer’s losses are incurred at a higher interest rate. On average, a long futures contract will perform worse than a long forward contract.

5.6.6

Arbitrage in practice: S&P 500 index arbitrage

Multiple theoretically correct prices may exist for futures contracts. Reasons include: • Future dividends are uncertain. • Transaction costs are not zero and hence a range of fair prices may exist. • Assumptions (such as margin requirement, credit risk, daily settlement, clearing house guarantee, etc) in futures pricing may be different, producing difference prices for the same futures contract. • Arbitrageurs usually buy a subsect of 500-stock index. As such, futures contracts and the offsetting position in stock may not move together.

FM: Derivatives Markets

c

Yufeng Guo

47

Derivatives Markets Chapter 5

5.6.7

c

Yufeng Guo

www.actuary88.com

Appendix 5.B: Equating forwards and futures

The most important thing about Appendix 5.B is Table 5.13. Before taking Exam FM, make sure can reproduce Table 5.13. To understand Table 5.13, let’s look at three different positions. Position 1 You enter into 8 long forward contracts on S&P 500 Index at time zero. The delivery date is 10 weeks from today. The forward price is 1100. Your initial margin for 8 long forwards is 217,727.21. The margin account earns a continuously compounded annual interest rate 6%. What’s your profit from your long forward position? Additional information is: Week 0 1 2 3 4 5 6 7 8 9 10

Forward price $1,100.00 $1,027.99 $1,037.88 $1,073.23 $1,048.78 $1,090.32 $1,106.94 $1,110.98 $1,024.74 $1,007.30 $1,011.65

In the above Table, $1,100.00 is the forward price you’ll pay at Week 10. $1,027.99 is the new forward price one week later (i.e if you enter into a long S&P 500 forward at Week 1 to receive the index at Week 10, you’ll pay $1,027.99 at Week 10 to receive the index). Similarly,$1,037.88 is the forward price at Week 2 to receive the index at Week 10. So on and so forth. The final price $1,011.65 is the spot price at Week 10. Since a forward contract is only marked to market at the expiration date, the forward prices at Week 1, 2, 3, ..., and 9 are not needed for calculating the profit. Only the forward price at Week 1 and the forward price at Week 10 matter. Please note one S&P 500 forward contract is worth 250 times the forward price. Since you long 8 contracts, your the profit is 10

217, 727.21e0.06× 52 + 250 × 8 × (1, 011.65 − 1, 100) = 43, 554.997 = 43, 554.00 Your profit is: 10

43, 554.00 − 217, 727.21e0.06× 52 = −176, 700 Alternative calculation of the profit: 250 × 8 × (1, 011.65 − 1, 100) = −176, 700

FM: Derivatives Markets

c

Yufeng Guo

48

c

Yufeng Guo

Derivatives Markets Chapter 5

www.actuary88.com

Position 2 You enter into 8 long futures contracts on S&P 500 Index at time zero. The delivery date is 10 weeks from today. The futures price is 1100. Your initial margin for 8 long futures is 217,727.21. The margin account earns a continuously compounded annual interest rate 6%. Assuming weekly marking to market. What’s your profit from your long futures position? Additional information is: Week Futures price 0 $1,100.00 1 $1,027.99 2 $1,037.88 3 $1,073.23 4 $1,048.78 5 $1,090.32 6 $1,106.94 7 $1,110.98 8 $1,024.74 9 $1,007.30 10 $1,011.65 The solution is Week 0 1 2 3 4 5 6 7 8 9 10

Multiplier 2,000 2,000 2,000 2,000 2,000 2,000 2,000 2,000 2,000 2,000 2,000

Futures Price $1,100.00 $1,027.99 $1,037.88 $1,073.23 $1,048.78 $1,090.32 $1,106.94 $1,110.98 $1,024.74 $1,007.30 $1,011.65

Price Change

Margin Balance $217,727.21 $73,958.58 $93,823.96 $164,632.29 $115,922.36 $199,136.19 $232,606.09 $240,954.64 $68,752.83 $33,952.20 $42,691.40

-$72.01 $9.89 $35.35 -$24.45 $41.54 $16.62 $4.04 -$86.24 -$17.44 $4.35

Explanation of the above table. 250 × 8 = 2, 000 1 0.06× 52

217, 727.21e

+ 2, 000 × (−72.01) = 73, 958.58 1

73, 958.58e0.06× 52 + 2, 000 × 9.89 = 93, 823.96 Your profit is: 10

42, 691.40 − 217, 727.21e0.06× 52 = −177, 562.60 If you compare Position 1 with Position 2, you’ll see that your profit on 8 long futures contracts is lower than your profit on 8 long forward contracts. This is due to futures marking to market. FM: Derivatives Markets

c

Yufeng Guo

49

Derivatives Markets Chapter 5

c

Yufeng Guo

www.actuary88.com

Position 3 Your enter into long futures contracts S&P 500 Index at time zero. Your goal is for Position 3 to produce exactly the same profit as in Position 1. The futures prices are as follows: Week Futures price 0 $1,100.00 1 $1,027.99 2 $1,037.88 3 $1,073.23 4 $1,048.78 5 $1,090.32 6 $1,106.94 7 $1,110.98 8 $1,024.74 9 $1,007.30 10 $1,011.65 How can you make Position 3 and Position 1 have the same profit? Solution

FM: Derivatives Markets

c

Yufeng Guo

50

c

Yufeng Guo

Derivatives Markets Chapter 5 Week 0 1 2 3 4 5 6 7 8 9 10

Multiplier 1,979.34 1,981.62 1,983.91 1,986.20 1,988.49 1,990.79 1,993.09 1,995.39 1,997.69 2,000.00

Futures Price $1,100.00 $1,027.99 $1,037.88 $1,073.23 $1,048.78 $1,090.32 $1,106.94 $1,110.98 $1,024.74 $1,007.30 $1,011.65

Price Change -$72.01 $9.89 $35.35 -$24.45 $41.54 $16.62 $4.04 -$86.24 -$17.44 $4.35

www.actuary88.com Margin Balance $217,727.21 $75,446.43 $95,131.79 $165,372.88 $117,001.18 $199,738.33 $233,055.87 $241,377.01 $69,573.26 $34,813.80 $43,554.00

Explanation. Since marking to market magnifies gains and losses, you’ll need to hold fewer than 8 long futures contracts at t = 0. To determine how many contracts to hold at t = 0, notice that if Week 1 futures price is one dollar more than Week 0 futures price, you’ll realize one dollar gain. This one dollar will be deposited into your margin account at the end of Week 1 and accumulate with interest to the end of Week 10 (i.e. earning 9-month interest). 9 To undo the effect of marking to market, you should hold 8e−0.06× 52 = 7.91735295 contracts. The contract size is 250 × 7.91735295 = 1, 979.34, which is less than 2,000. What if the Week 1 futures price is one dollar less than Week 0 futures price? In this case, you’ll realize one dollar loss. This one dollar will be deducted from your margin account at the end of Week 1, reducing the future value of your margin account. Once again, you 9 need to hold 8e−0.06× 52 = 7.91735295 contracts. 8 Similarly, you need to hold 8e−0.06× 52 = 1, 981.62 contracts at the end of Week 1 (every dollar gain recognized at the end of Week 2 will accumulate 8 months interest to Week 10). 7 And you need to hold 8e−0.06× 52 = 7.93564486 at the end of Week 3. So on and so forth. After marking to market at the end of Week 9, the futures contract becomes a forward contract (since both contracts will be marked to market at the final week). At the end of Week 9, you should hold exactly 8 futures contracts. Explanations of other calculations. 217, 727.21e

0.06 52

75, 446.43e

+ 1, 979.34 × (−72.01) = 75, 446.43

0.06 52

+ 1, 981.62 × (9.89) = 95, 131.79

You ending margin account value at the end of Week 10 is 34, 813.80e

0.06 52

+ 2, 000 × (4.35) = 43, 554.00

Your profit is: 10

43, 554.00 − 217, 727.21e0.06× 52 = −176, 700.00 Now you see that Position 1 and Position 3 have the same ending margin account and the same profit.

FM: Derivatives Markets

c

Yufeng Guo

51

c

Yufeng Guo

Derivatives Markets Chapter 5

www.actuary88.com

Practice problem You enter into 8 long futures contracts on S&P 500 Index at time zero. The delivery date is 10 weeks from today. The initial margin for the 8 contracts is 300,000. The margin account earns a continuously compounded annual interest rate 6%. Calculate your profit from the long position under the following 3 scenarios: 1. Marking to market is done only at the expiration date. 2. Marking to market is done weekly and you always hold 8 contracts. 3. Marking to market is done weekly. You re-balance your holdings weekly to produce the same profit as Scenario 1. Futures prices are: Week 0 1 2 3 4 5 6 7 8 9 10

Futures Price $1,000 $1,020 $1,015 $1,026 $1,110 $1,065 $1,105 $1,085 $1,093 $1,008 $1,004

Solution Scenario 1 The ending balance of the margin account is 10

300, 000e0.06× 52 + 8 × 250 × (1, 004 − 1, 000) = 311, 481.59 The profit is: 10

311, 481.59 − 300, 000e0.06× 52 The profit is: 8 × 250 × (1, 004 − 1, 000) = 8, 000

FM: Derivatives Markets

c

Yufeng Guo

52

Derivatives Markets Chapter 5

c

Yufeng Guo

www.actuary88.com

Scenario 2 Week 0 1 2 3 4 5 6 7 8 9 10

Multiplier 2,000 2,000 2,000 2,000 2,000 2,000 2,000 2,000 2,000 2,000

Futures Price $1,000 $1,020 $1,015 $1,026 $1,110 $1,065 $1,105 $1,085 $1,093 $1,008 $1,004

Price Change

Margin Balance $300,000.00 $340,346.35 $330,739.29 $353,121.13 $521,528.81 $432,130.92 $512,629.82 $473,221.66 $489,768.00 $320,333.44 $312,703.27

$20 -$5 $11 $84 -$45 $40 -$20 $8 -$85 -$4

The ending balance at the end of Week 10 is $312,703.27. The profit is: 312, 703.27 − 10 300, 000e0.06× 52 = 9, 221.69 Sample calculations: 1

340, 346.35 = 300, 000e0.06× 52 + 2000 × 20 1

330, 739.29 = 340, 346.35e0.06× 52 + 2000 × (−5)

FM: Derivatives Markets

c

Yufeng Guo

53

c

Yufeng Guo

Derivatives Markets Chapter 5

www.actuary88.com

Scenario 3 Week 0 1 2 3 4 5 6 7 8 9 10

Multiplier 1,979.34 1,981.62 1,983.91 1,986.20 1,988.49 1,990.79 1,993.09 1,995.39 1,997.69 2,000.00

Futures Price $1,000 $1,020 $1,015 $1,026 $1,110 $1,065 $1,105 $1,085 $1,093 $1,008 $1,004

Price Change $20 -$5 $11 $84 -$45 $40 -$20 $8 -$85 -$4

Margin Balance $300,000.00 $339,933.12 $330,417.46 $352,621.95 $519,870.00 $430,987.93 $511,117.13 $471,845.44 $488,353.32 $319,113.17 $311,481.59 10

The ending balance at the end of Week 10 is 311,481.59. The profit is 311, 481.59−300, 000e0.06× 52 = 8, 000 Sample calculation: 9 1, 979.34 = 8 × 250e−0.06× 52 8

1, 981.62 = 8 × 250e−0.06× 52 1

1, 997.69 = 8 × 250e−0.06× 52 0

2, 000 = 8 × 250e−0.06× 52 1

339, 933.12 = 300, 000e0.06× 52 + 1, 979.34 × 20 1

330, 417.46 = 339, 933.12e0.06× 52 + 1, 981.62 × (−5) ♦

FM: Derivatives Markets

c

Yufeng Guo

54

Chapter 8

Swaps Chapter 8 of Derivative Markets is a difficult but important chapter. The most important concept is pricing of swaps. SOA can easily come up questions on this topic. Make sure you know how to calculate the price of a swap.

8.1

An example of a commodity swap

First, let’s understand what a swap is. Let’s not bother memorizing the textbook definition of a swap but focus on the essence of a swap. A swap is multiple forward contracts combined into one big contract. For example, you are an owner of an airline company. Your two biggest operating expenses will be labor and fuel. Labor cost can be pretty much controlled by adding or reducing staff. However, the fuel cost is subject to a wild fluctuation. From time to time, we all complain about how expensive gas is, but wait till you own multiple airplanes and see how quickly your gas bill goes to the roof. One way to control the fuel cost is to use a series of forward contracts. For example, at t = 0, you sign up 3 separate forward contracts with the same supplier to lock in the fuel cost for the next 3 years. The first forward contract requires the supplier to deliver 100 barrels of oil at t = 1 (end of Year 1) at $20 per barrel; the second contract requires the supplier to deliver 100 barrels of oil at t = 2 at $25 per barrel; and the third contract requires the supplier to deliver 100 barrels of oil at $30 per barrel. Accordingly, you’ll make 3 separate payments to the supplier. Your first payment is 100 × 20 = 2, 000 at t = 1; your second payment is 100 × 25 = 2, 500 at t = 2; and your third payment is 100 × 30 = 3, 000 at t = 3. As a busy CEO, you quickly realize that signing up 3 separate contracts and keeping track of 3 separate payments is a bit of work. ”Why don’t I sign one big contract and make 3 level payments to the supplier?” you ask. Indeed, combing 3 separate forward contracts into one and making level payments is a good idea. In the combined contract, the supplier is required to deliver 100 barrels of oil at $20 per barrel at t = 1, 100 barrels of oil at $25 per barrel at t = 2, and 100 barrels of oil at $30 per barrel at t = 3. Accordingly, you make payments of X at = 1, 2, 3 respectively. Next, let’s calculate the level payment X. 55

c

Yufeng Guo

Derivatives Markets Chapter 8

www.actuary88.com

Your payments BEFORE the 3 forward contracts are combined: t Payment

0

1 $2,000

2 $2,500

3 $3,000

Your payments AFTER the 3 forward contracts are combined: t Payment

0 X

1 X

2 X

3 X

Assume that the interest rate is 10% per year. The present value of your total payments should be the same whether you pay 3 forward contracts separately or you pay level payments in a combined contract. 2000v + 2500v 2 + 3000v 3 = X(v + v 2 + v 3 ) v = 1.1−1 ⇒ X = 2468.28 What if the interest rate is not level? Let’s say that the interest rate is 10% per year during t ∈ [0, 1], 12% per year during t ∈ [0, 2], and 14% per year during t ∈ [0, 3]. No problem. The present value of your total payments should be the same whether you pay 3 forward contracts separately or you pay level payments in a combined contract. 2000 2500 3000 + + =X 2 1.1 1.12 1.143



1 1 1 + + 2 1.1 1.12 1.143



⇒ X = 24650.84 The combined contract is called a swap. A swap is a series of exchanges between two parties that takes place in multiple dates in the future. Typically, one party of the swap pays level cash flows and receives variable cash flows (or receives something of variable values); the other party receives level cash flows and pays variable cash flows (or delivers something of variable values). In our example, you (owner of an airline) pay the supplier X at t = 1, 2, 3 respectively; the supplier gives you 100 barrels at t = 1, 2, 3 respectively.

8.1.1

Physical versus financial settlement

Next, let’s talk about how a swap is settled. Use our example and assume the interest rate is 10% for all years. The swap can be settled if you pay the supplier $2,468.28 at t = 1, 2, 3 respectively and the supplier has you come to its warehouse to pick up 100 barrels of oil at t = 1, 2, 3 respectively. This is physical settlement. However, the swap can be settled in a simpler way called financial settlement. You still write a check of $2,468.28 at t = 1, 2, 3 respectively. However, instead of giving you 100 barrels of oil at t = 1, 2, 3, the supplier simply gives you the market price of 100 barrels at FM: Derivatives Markets

c

Yufeng Guo

56

Derivatives Markets Chapter 8

c

Yufeng Guo

www.actuary88.com

t = 1, 2, 3. You just go to the market and buy 100 barrels using the money the supplier gives you. For example, if the market price is $20 per barrel at t = 1, $25 per barrel at t = 2, and $30 per barrel at t = 3, the supplier gives you a check of $2,000 at t = 1, another check of $2,500 at t = 2, and the third check of $3,000 at t = 3. You can go to the market place and purchase 100 barrels of oil at t = 1, 2, 3 yourself. The settlement can be simplified further. Instead of you writing the supplier a check of $2,468.28 at t = 1, 2, 3 respectively and the supplier writing you a check of $2,000 at t = 1, $2,500 at t = 2, and $3,000 at t = 3, you and the supplier can focus on the net payment. The swap can be settled this way: • You write the supplier a check of 2, 468.28 − 2, 000 = 468.28 at t = 1 • The supplier writes you a check of 2, 500 − 2, 468.28 = 31.72 at t = 2 • The supplier writes you a check of 3, 000 − 2, 468.28 = 531.72 at t = 3

Typically in a swap the two parties don’t write checks to each other back and forth. At each settlement date, the net payment is made from the party who owes more obligation to the other party who owes less.

8.1.2

Pricing swaps

Now let’s think about how we calculated the price of the level payment from the fixed-payer to the floating-payer. The equation used for solving the fixed level payments is:

PV of fixed payments = PV of floating payments

This equation says that when the swap contract is signed at t = 0, two parties promise to exchange equal value of cash flows; nobody wins and nobody loses. The market value of the swap is zero. The swap is a fair game. Remember this equation. Whenever you need to calculate the fixed payment, use this equation. Though the market value of a swap is zero at t = 0, as time passes, the market value of the swap is generally not zero. Reasons include: • The supply and demand may change. For example, one day after the swap contract is signed, a war breaks out in a major oil-producing country. Suddenly, there’s a shortage of oil worldwide. The oil price goes up. If this happens, the supplier’s cost of delivering 100 barrels of oil to you at t = 1, 2, 3 will go up. However, your payment to the supplier at t = 1, 2, 3 is fixed at X, which was based the old market condition. You get a good deal from the swap. If, on the other hand, one day after the swap contract is signed, there’s an over-abundance of oil supply in the market and the oil price plummets. No matter how low the oil price turns out to be at t = 1, 2, 3, you still have to pay the supplier the pre-set price X at t = 1, 2, 3. You will suffer a loss. FM: Derivatives Markets

c

Yufeng Guo

57

c

Yufeng Guo

Derivatives Markets Chapter 8

www.actuary88.com

• The interest rate may change. The fixed payment X is calculated according to the expected yield curve at t = 0. If, after the swap contract is signed, the actual yield curve is different from the yield curve expected at t = 0, the present value of the fixed payments will be different from the present value of the floating payments. • Even if the supply/demand and interest rate won’t change, the value of the swap is zero only before the first swap payment X is made. Once the first payment X is made, the present value of the fix-payer’s cash flows for the remaining duration of the swap is no longer equal to the present value of the float-payer’s cash flows for the remaining duration of the swap. For example, if the interest rate is 10%, you need to pay the level amount of $2,468.28 at t = 1, 2, 3. If you pay as you go (instead of paying level payments $2,468.28 at t = 1, 2, 3), you will pay $2,000 at t = 1, $2,500 at t = 2, and $3,000 at t = 3. Compared with the level payment $2,468.28 at t = 1, 2, 3, you overpay $468.28 at t = 1, underpay 2, 500 − 2, 468.28 = 31.72 at t = 2, and underpay 3, 000 − 2, 468.28 = 531.72 at t = 3. The present value of your overpayment and under-payments should be zero: 468.28v − 31.72v 2 − 531.72v 3 = 0 where v = 1.1−1 If the two parties want to back out from the swap after t = 1, the floating-payer needs to refund the fixed-payer $468.28. If the two parties want to back out from the swap after t = 2, the fixed-payer needs to needs to pay the floating-payer 531.72v = 483.38.

8.2

Interest rate swap

The most common interest rate swap is “fixed-for-floating” swap, commonly referred to as a “plain vanilla swap.”

8.2.1

Key features of an interest rate swap

• The notional principal is fixed at t = 0. • The notional principal is never exchanged. It’s a scaling factor to calculate the interest rate payments. • One party agrees to pay a fixed interest rate applied to the notional principal regularly during the life of the swap. The life of a swap is called the swap term or tenor. • The other party agrees to pay a floating (ie. variable) interest rate applied to the notional principal regularly during the life of a swap. The floating rate is typically based on a benchmark rate such as LIBOR. • The floating rate is “set in advance, paid in arrears.” The floating rate is determined at the beginning of a settlement period but is paid at the end of the settlement period. FM: Derivatives Markets

c

Yufeng Guo

58

Derivatives Markets Chapter 8

8.2.2

c

Yufeng Guo

www.actuary88.com

Example of a plain vanilla interest rate swap

Consider a hypothetical 5-year swap initiated on 3/1/2006 between Microsoft and Intel. The details of the swap are as follows: • Swap initiation date: March 1, 2006 • Swap tenor: 3 years • Settlement dates: September 1 and March 1 • Notional principal: $100 million • Fixed rate payer: Microsoft pays 6.04% per year • Floating rate payer: Intel pays 6-month LIBOR Cash flows received by Microsoft (in one million dollars): Time 0 0.5 1 1.5 2 2.5 3

Dates 3/1/2006 9/1/2006 3/1/2007 9/1/2007 3/1/2008 9/1/2008 3/1/2009

6-month LIBOR0 = 5.80% LIBOR0.5 =? LIBOR1 =? LIBOR1.5 =? LIBOR2 =? LIBOR2.5 =?

Intel to Microsoft

Microsoft to Intel

Net $ to Microsoft

$2.9 ? ? ? ? ?

$3.02 $3.02 $3.02 $3.02 $3.02 $3.02

($0.12)

Explanations: • 2.9 = 5.8% × 180 360 × 100. This is the 6-month interest on $100 million principal with LIBOR rate. The 6-month LIBOR rates are simple annual rates. Here we assume 1 year=360 days and 6 months =180 days. In reality, the two parties in the swap can determine the day counting method for calculating the interest payment. For example, they can use ActualDays to calculate the interest payment. 365 • 3.02 = 6.04% × 180 360 × 100. This is the 6-month interest on $100 million principal with 6.04% rate. The 6% rate is a simple annual interest rate. • When the swap contract is signed on 3/1/2006, only the LIBOR for the next 6 months is known. The LIBOR rates beyond 9/1/2006 are unknown. • LIBOR is set in advance and paid in arrears. For example, on 3/1/2006, the LIBOR rate 5.8% is used for calculating Intel’s payment to Microsoft on 9/1/2006. On 9/1/2006, the next 6-month LIBOR rate is used for calculating the interest payment on 3/1/2007. • LIBOR rates beyond 3/1/2009 are irrelevant to the swap. The floating interest payment on the final settlement date 3/1/2009 is based on LIBOR from 9/1/2008 to 3/1/2009. FM: Derivatives Markets

c

Yufeng Guo

59

Derivatives Markets Chapter 8

c

Yufeng Guo

www.actuary88.com

Suppose the LIBROR rates are known. Then the cash flow diagrams are as follows: Time 0 0.5 1 1.5 2 2.5 3

8.2.3

Dates 3/1/2006 9/1/2006 3/1/2007 9/1/2007 3/1/2008 9/1/2008 3/1/2009

6-month 5.80% 5.90% 6.00% 6.10% 6.20% 6.30%

Intel to Microsoft

Microsoft to Intel

Net $ to Microsoft

$2.90 $2.95 $3.00 $3.05 $3.10 $3.15

($3.02) ($3.02) ($3.02) ($3.02) ($3.02) ($3.02)

($0.12) ($0.07) ($0.02) $0.03 $0.08 $0.13

Motivations for an interest swap

Example 1 Interest rate swap changes floating liabilities to fixed liabilities or vice versa. Consider a bank that receives deposits from small investors like you and me and lends out money in mortgage loans. Here the bank has a mismatch of interest rates. On the one hand, it receives fixed interest rate on mortgages; most new homeowners avoid variable rate loans and want to lock in a fixed interest on their mortgages for the next 15 or 30 years. On the other hand, the bank pays floating interest rate to individual depositors. We want our banks to gives us interest rate that reflects the market interest rate. If the market interest is high but we get a low interest rate from a bank, we can deposit our money elsewhere and earn a higher interest rate. Suppose the bank pays deposits at a floating rate of LIBOR - 1% but receives mortgage payments at fixed 6%. If LIBOR=7%, the bank will break even. If LIBOR> 7%, the bank will lose money. To hedge the risk that LIBOR may go up, the bank can enter a swap and “pay fixed, and get float.” In the swap, the bank pays fixed 6% and receives LIBOR. The counterparty receives fixed 6% and pays LIBOR. Before the swap, the bank faces the risk that LIBOR may go up and exceed 7%. After the swap, the bank receives 6% mortgage payments from new homeowners and passes the payments to the counterparty. The counterparty receives 6% fixed and sends LIBOR to the bank. The bank pays its depositors LIBOR - 1%, earning 1% profit no matter how high LIBOR may be.

FM: Derivatives Markets

c

Yufeng Guo

60

c

Yufeng Guo

Derivatives Markets Chapter 8

www.actuary88.com

Depositors

LIBOR – 1% Fixed 6% Counter Party

Bank LIBOR

6%

New Homeowners

Similarly, a firm can use interest rate swaps to change its floating assets to fixed assets or vice versa. Consider an firm which holds a floating rate note (FRN). An FRN is a bond except that the coupon rate is a variable rate (such as LIBOR + 1%) instead of a fixed rate (such as 8%). The firm originally bought an FRN instead of a fixed coupon bond because it forecast that the interest rate would rise. Suppose two years later after purchasing the FRN, the firm forecast that the interest rate would fall. The firm can use an interest rate swap to change its floating interest rate payment received from the FRN issuer to a set of fixed cash flows. Example 2 Interest rate swap reduces a firm’s borrowing cost (comparative advantage) First, a few words on comparative advantage. The idea behind comparative advantage is “Do what you the very best, no even your second best.” A classic example is that the best attorney in town happens to be the best typist in down. Since the attorney earns more income by doing litigation rather than typing, he should work exclusively on litigation and give up typing. He can hire a secretary to type for him. Suppose there are two companies. One is a well known company, which can borrow money at lower rate. The other is a new startup and doesn’t have a track record. It has to borrow money at a higher rate. Their borrowing rates are as follows: FM: Derivatives Markets

c

Yufeng Guo

61

Derivatives Markets Chapter 8

c

Yufeng Guo

www.actuary88.com

Company Fixed borrowing rate Floating borrowing rate Mr. Established 10% LIBOR+0.3% Mr. StartUp 11.20% LIBOR+1% Mr. StartUp minus Mr. Established 4fixed = 1.2% 4float = 0.7% Comparative advantage 4fixed -4 float=0.5% Mr. Established has absolute advantage in borrowing fixed and floating rate debt. Mr. StartUp, on the other hand, has comparative advantage in borrowing floating rate; it pays only 0.7% more than Mr. Established in borrowing floating rate debt yet pays 1.2% more than Mr. Established in borrowing at a fixed rate. Suppose Mr. Established wants to borrow some money at LIBOR and Mr. StartUp wants to borrow the same amount of money at a fixed rate. Total borrowing cost BEFORE using an interest rate swap: Mr. Established wants to borrow at a floating rate. So it goes out, finds a lender, and borrows $1 million at a floating rate of LIBOR+0.3% per year. Mr. StartUp wants to borrow at a fixed rate. So it goes out, finds a lender, and borrows at 11.2%. Total borrowing cost: LIBOR + 0.3% + 11.2% = LIBOR + 11.5% Total borrowing cost AFTER using an interest rate swap: Mr. Established goes out, finds a lender, and borrow $1 million at a fixed rate of 10%. Mr. StartUp goes out, finds a lender, and borrows $1 million at a floating rate of LIBOR+1%. Then Mr. Established and Mr. StartUp enter an interest rate swap where Mr. Established pays fixed and gets float and Mr. StartUp pays float and gets fixed. Total borrowing cost: 10% + LIBOR + 1% = LIBOR + 11% By entering a swap, Mr. Established and Mr. StartUp achieve 0.5% saving. Let’s assume that Mr. Established and Mr. StartUp share the 0.5% saving equally (i.e each gains 0.25% saving). Mr. Established’s cash flows are as follows: • Go to a lender and borrow $1M at 10%. Pay 10% interest rate to this lender. So the annual payment to the lender is 1 × 10% = 0.1M • Pay Mr. StartUp LIBOR on $1M principal. For example, if LIBRO happens to be 9% in Year 1, Mr. Established will pay Mr. StartUp 9% × 1M=0.09M at the end of Year 1. • Receive 9.95% interest payment on $1M principal (i.e. receive $0.0995M) annual payment from Mr. StartUp. Mr. StartUP’s cash flows are as follows: • Go to a lender and borrow $1 M at LIBOR+1%. • Pay Mr. Established 9.95% annual interest on $1 M principal (ie. Pay $0.0995M) • Receive LIBOR interest payment on $1M principal from Mr. Established. FM: Derivatives Markets

c

Yufeng Guo

62

c

Yufeng Guo

Derivatives Markets Chapter 8 DIRECT BORROWING Mr. Established wants to borrow $1M and pay floating interest rate

Pay LIBOR+0.3% on $1M principal annually

$1M

www.actuary88.com

Mr. StartUp wants to borrow $1M and pay fixed interest rate

Pay 11.2% on $1M principal annually

$1M

Mr. StartUp’s Lender

Mr. Establisher’s Lender

The net result of direct borrowing: • Mr. Established gets $1M loan and pays LIBOR+0.3%. • Mr. StartUp gets $1M loan and pays 11.2%. • Total borrowing cost: LIBOR+0.3% + 11.2% = LIBOR+11.5%

INTEREST SWAP WITHOUT A SWAP DEALER Assume Mr. StartUp directly negotiate a swap without a swap dealer. FM: Derivatives Markets

c

Yufeng Guo

Established and Mr.

63

Derivatives Markets Chapter 8

Mr. Established wants to borrow $1M and pay floating interest rate

c

Yufeng Guo

www.actuary88.com

pay LIBOR interest on Mr. StartUp $1M principal annually wants to borrow $1M and pay fixed interest rate pay 9.95% interest on $1M principal annually

Pay 10% interest on $1M principal annually

$1M

Pay LIBOR + 1% on $1M principal annually

Mr. Establisher’s Lender

$1M

Mr. StartUp’s Lender

Result after the swap:

• Mr. Established gets $1M loan. Its annual interest rate for loan repayment on $1M principal is 10% − 9.95% + LIBOR = LIBOR + 0.05%. Mr. Established borrows $1M and pays LIBOR + 0.05%. This is 0.25% less direct borrowing rate of LIBOR +0.3% without a swap.

• Mr. StartUp gets $1M loan. Its annual interest rate for loan repayment on $1M principal is LIBOR+1% + 9.95% - LIBOR =10.95%. This is 0.25% less than direct borrowing rate of 11.2%.

• Total borrowing cost: LIBOR+0.05% + 10.95% = LIBOR+11%

INTEREST SWAP WITH A DEALER Suppose that the dealer receives 9.97% from Mr. StartUp but passes on only 9.93% to Mr. Established (swap bid/ask spread). The dealer takes away 9.97% - 9.93%= 0.04% as a compensation for setting up the swap. FM: Derivatives Markets

c

Yufeng Guo

64

Derivatives Markets Chapter 8

Mr. Established wants to borrow $1M and pay floating interest rate

Pay 10% interest on $1M principal annually

c

Yufeng Guo

9.93%

Swap Dealer

www.actuary88.com

Mr. StartUp 9.97% wants to borrow $1M and pay fixed interest LIBOR rate

LIBOR

Pay LIBOR + 1% on $1M principal annually

$1M

Mr. Establisher’s Lender

$1M

Mr. StartUp’s Lender

The net result after the swap using a swap dealer: • Mr. Established gets $1M loan. Its annual interest rate for loan repayment on $1M principal is 10% - 9.93% + LIBOR = LIBOR + 0.07%. Mr. Established borrows $1M and pays LIBOR+0.07%. This is 0.23% less than the direct borrowing rate of LIBOR +0.3% without a swap. • Mr. StartUp gets $1M loan. Its annual interest rate for loan repayment on $1M principal is LIBOR+1% + 9.97% - LIBOR =10.97%. This is 0.23% less than the direct borrowing rate of 11.2%. • Total borrowing cost: LIBOR+0.07% + 10.97% = LIBOR+11.04%

8.2.4

How to price an interest rate swap

Let’s walk through the example on Derivatives Markets page 255 “Pricing and the swap counterparty.” This example relies on Table 7.1. Make sure you understand Table 7.1. Excerpt of Table 7.1 from Derivatives Markets Yrs to maturity 1 2 3

Zero-coupon bond yield 6.00% 6.50% 7.00%

FM: Derivatives Markets

Zero-coupon bond price P0 (0, 1) = 0.943396 P0 (0, 2) = 0.881659 P0 (0, 3) = 0.816298

c

Yufeng Guo

1-Yr implied forward rate r(0, 1) = 0.06 r(1, 2) = 0.0700236 r(2, 3) = 0.0800705 65

c

Yufeng Guo

Derivatives Markets Chapter 8

www.actuary88.com

Here the zero-coupon bond prices with maturity of 1 year, 2 years, and 3 years are bond selling prices in the market. They are the known values. The remaining values are based on bond price. Yrs to maturity 1 2 3

Zero-coupon bond yield ? ? ?

Zero-coupon bond price P0 (0, 1) = 0.943396 P0 (0, 2) = 0.881659 P0 (0, 3) = 0.816298

1-Yr implied forward rate r(0, 1) =? r(1, 2) =? r(2, 3) =?

Zero-coupon bond yields are calculated as follows. Consider a 1-year zero coupon bond. For each dollar to be received at t = 1, the buyer has to pay 0.943396 at t = 0. So the bond yield is calculated as follows: 0.943396(1 + i) = 1 i ≈ 6% Consider a 2-year zero coupon bond. For each dollar to be received at t = 2, the buyer has to pay 0.881659 at t = 0. So the bond yield is calculated as follows: 0.881659(1 + i)2 = 1 i ≈ 6.5% Consider a 3-year zero coupon bond. For each dollar to be received at t = 3, the buyer has to pay 0.816298 at t = 0. So the bond yield is calculated as follows: 0.816298(1 + i)3 = 1 i ≈ 7% The 1-year implied forward rates are calculated as follows.

Time t

0

1

r ( 0,1)

2

r (1, 2 )

3

r ( 2,3)

6%

6.5% per Yr 7% per Yr

FM: Derivatives Markets

c

Yufeng Guo

66

c

Yufeng Guo

Derivatives Markets Chapter 8

www.actuary88.com

At t = 0, the 1-year implied forward rate during t ∈ [0, 1] is obviously 6%. So r(0, 1) = 6% At t = 1, the 1-year implied forward rate during t ∈ [1, 2] is r(1, 2). (1 + 6%)[1 + r(1, 2)] = (1 + 6.5%)2 r(1, 2) = 7.00236% At t = 2, the 1-year implied forward rate during t ∈ [2, 3] is r(2, 3). (1 + 6.5%)2 [1 + r(2, 3)] = (1 + 7%)3 r(2, 3) = 8.00705%

Time t

0

1

2

3

r ( 0,1)

r (1, 2 )

r ( 2,3)

6%

7.0024%

8.0071%

6.5% per Yr 7% per Yr

Next, let’s determine the price of the swap (ie. finding the constant rate R by which the swap dealer gets paid by the counterparty). Remember the swap dealer gets fixed and pays float. And the counterparty (the investor) gets float and pays fixed. The annual payment by the swap dealer to the counterparty is: R× Notional Principal The annual payment by counterparty to the swap dealer is: LIRBOR× Notional Principal As explained before, when two parties enter an interest rate swap, only the LIBOR rate during the 1st settlement period is known. The LIBOR rates beyond the first settlement is not known. Then how can we determine the floating payments if we know only the LIBOR rate for the first settlement period? When pricing a swap, we can create various LIBOR interest rates using forward interest rate agreements. For the non-arbitrage principal to hold, the LIBOR rates must be close to the 1-year forward interest rate extracted from the forward interest rate agreements. FM: Derivatives Markets

c

Yufeng Guo

67

c

Yufeng Guo

Derivatives Markets Chapter 8

Time t

0

1

www.actuary88.com

2

3

r ( 0,1)

r (1, 2 )

r ( 2,3)

6%

7.0024%

8.0071%

6.5% per Yr 7% per Yr

Payment to the swap dealer

R

R

Pay by the dealer to the counterparty

6%

R

7.0024%

8.0071%

Assuming the notional principal is $1.

Set up the equation: PV of fixed payments = PV of floating payments. Set the notional principal to $1. We have: 

R

1 1 1 + + 2 1.06 1.065 1.073 R=

R=

R=



=

6% 7.0024% 8.0071% + + 1.06 1.0652 1.073

6% 7.0024% 8.0071% 1.06 + 1.0652 + 1.073 1 1 1 1.06 + 1.0652 + 1.073

0.943396 × 6% + 0.881659 × 7.0024% + 0.816298 × 8.0071% 0.943396 + 0.881659 + 0.816298

P0 (0, 1)r(0, 1) + P0 (0, 2)r(1, 2) + P0 (0, 3)r(2, 3) = P0 (0, 1) + P0 (0, 2) + P0 (0, 3)

Pn=3 i=1

P0 (0, ti )r(ti−1 , ti ) = 6.9543% i=1 P0 (0, ti )

Pn=3

The general formula is: Pn

R=

i=1 P0 (0, ti )r(ti−1 , ti ) Pn i=1 P0 (0, ti )

Here n is the total number of settlements (ie. the number of times when cash flows change hand between two parties in a swap); ti represents the time the i-th settlement takes place. P0 (0, ti ) is the present value of $1 at ti discounted to t = 0. It is also the price of a zero-coupon bond maturing on the date ti . r(ti−1 , ti ) is the implied forward rate during t ∈ [ti−1 , ti ]. Next, let’s use a shortcut to calculate the present value of the floating payments. The floating payments in an interest rate swap are very similar to the cash flows in a floating rate FM: Derivatives Markets

c

Yufeng Guo

68

c

Yufeng Guo

Derivatives Markets Chapter 8

www.actuary88.com

bond. In a floating rate bond, the coupon rates are floating (instead of being fixed like in a typical bond). The floating coupon rates are set in advance and paid in arrears. Consider a floating rate bond. Its annual coupon rates for Year 1, Year 2, and Year 3 are 6%, 7.0024%, and 8.0071% respectively. Suppose the face amount of this floating rate bond is $1. Then the bond holder will get $0.06 at t = 1, $0.070024 at t = 2, and $1.080071 at t = 3. Time t

0

1

2

3

r ( 0,1)

r (1, 2 )

r ( 2,3)

6%

7.0024%

8.0071%

6.5% per Yr 7% per Yr

Cash flow received by the bondholder

$6% =$0.06

$7.0024% =$0.070024

$8.0071%+$1 =$1.080071

Assuming the face amount of the floating rate note is $1.

What’s the present value of this floating rate bond? Surprisingly, the present value is the face amount $1. To see why, notice that if you discount $1.080071 from t = 3 to t = 2, you’ll get $1: 1.080071 =1 1 + 8.0071% This $1 is combined with the second floating payment $0.070024, becoming $1.070024. If we discount this cash flow from t = 2 to t = 1, once again we get $1: 1.070024 =1 1 + 7.0024% This $1 is combined with the 1st floating payment $0.06, becoming $1.06. If we discount this cash flow from t = 1 to t = 0, once again we get $1: 1.06 =1 1 + 6% Here is another way to see why the present value of the floating note is its face amount. The present value of our floating note is: 6% 7.0024% 1 + 8.0071% + + 2 1 + 6% (1 + 6.5%) (1 + 7%)3 Notice that (1 + 7%)3 = (1 + 6%) × (1 + 7.0024%) × (1 + 8.0071%) FM: Derivatives Markets

c

Yufeng Guo

69

c

Yufeng Guo

Derivatives Markets Chapter 8

www.actuary88.com

(1 + 6.5%)2 = (1 + 6%) × (1 + 7.0024%) Then you can verify for yourself that 6% 1 + 8.0071% 7.0024% + =1 + 1 + 6% (1 + 6.5%)2 (1 + 7%)3 Key point to remember: The present value of a floating rate bond is its face amount. Next, we’re going to quickly calculate the present value of the floating payments using the present value formula for a floating rate bond. At the final settlement time t = 3, suppose the floating-payer (the swap dealer in this case) gives the fixed-payer $1 and immediately gets $1 back from the fixed payer. The cash flow diagram is: Time t

0

1

2

3

r ( 0,1)

r (1, 2 )

r ( 2,3)

6%

7.0024%

8.0071%

6.5% per Yr 7% per Yr

Payment to the swap dealer

R

R

R

Pay by the dealer $6% $7.0024% $(1+8.0071%) - $1 to the counterparty The equation to solve for R is: PV fixed payments=PV floating payments. Assuming the notional principal   is $1. 1 1 1 6% 7.0024% 1 + 8.0071% 1 R + + = + + − 2 3 2 3 1.06 1.065 1.07 1.06 1.065 1.07 1.073

The present value of a floating rate bond is its face amount: 7.0024% 1 + 8.0071% 6% + + =1 1.06 1.0652 1.073 This gives us: 1 1 1 + + R 2 1.06 1.065 1.073 

R=

1 1.06

1 1 − 1.07 3 1 + 1.065 + 2

1 1.073

=



=1−

1 1.073

1 − P0 (0, 3) 1 − P0 (0, 3) = Pn=3 P0 (0, 1) + P0 (0, 2) + P0 (0, 3) i=1 P0 (0, ti )

The general formula is: FM: Derivatives Markets

c

Yufeng Guo

70

Derivatives Markets Chapter 8

c

Yufeng Guo

1 − P0 (0, tn ) R = Pn = i=1 P0 (0, tn )

www.actuary88.com

Pn

i=1 P0 (0, ti )r(ti−1 , ti ) Pn i=1 P0 (0, ti )

Make sure you understand the symbols in this formula. If you find it hard to memorize this formula, don’t worry. You can just use the following procedure to calculate the fixed rate R in an interest rate swap: 1. Identify the LIBOR yield curve. 2. Identify fixed and floating payments. 3. Set up the equation that PV of fixed payments =PV of floating payments

8.2.5

The swap curve

To understand this section, you’ll need to read Section 5.7. The most important part of Section 5.7 is Equation 5.19. Though Section 5.7 is excluded from the syllabus, I recommend that you read it any way to help you understand the swap curve. In addition, you’ll need to know what’s Eurodollars. Here is some background on Eurodollars. Eurodollars is not the joint European currency, the Euro. Eurodollars are simply US dollars deposited in commercial banks outside the United States, and thus are not under the jurisdiction of the Federal Reserve. As a result, such deposits are subject to much less regulation than similar deposits within the United States, allowing for higher margins. Historically, such deposits were held mostly by European banks and financial institutions, and hence were called “Eurodollars”. Such deposits are now available in many countries worldwide. However, they continue to be called as “Eurodollars” regardless of the location. Eurodollar interest rate is the interest rate earned on Eurodollars deposited by one bank with another bank. Three-month Eurodollar futures contracts are futures contracts on threemonth Eurodollar interest rate. Make sure you know how to read the quote of the Eurodollar futures contract. If F is a Eurodollars futures quote, then (100 − F )% is the Eurodollars futures interest rate for a 3-month period. This formula is explained in Derivatives Markets Section 5.7, which says that the Eurodollars futures price at expiration of the contract is: 100− Annualized 3-month LIBOR Now you see that if the annualized 3-month LIBOR rate goes up, the quote F goes down; if the annualized 3-month LIBOR rate goes down, the quote F goes up. For example, a Eurodollars futures price 95.53 corresponds to (100 − 95.53)% = 4.47% annualized three-month interest rate or 4.47% = 1.1175% interest rate for a 3-month period 4 on $1,000,000 loan. The interest due on $1,000,000 loan during the 3-month borrowing period is 1.1175% × $1, 000, 000 = $11, 175. Now you shouldn’t have trouble understanding Equation 5.19: r91 = (100 − F ) × FM: Derivatives Markets

1 1 91 × × 100 4 90

c

Yufeng Guo

71

c

Yufeng Guo

Derivatives Markets Chapter 8

www.actuary88.com

This formula can be rewritten as: r91 =

(100 − F )% 91 × 4 90

)% Here (100−F is the un-annualized or the actual interest rate per 3-month period. The 4 91 factor 90 scales the 90-day actual interest rate into a 91-day actual interest rate. Now you should be able to follow the textbook and understand Table 8.4. For example, the 0.0037% interest rate is calculated as follows:

(100 − F )% 91 (100 − 98.555)% 91 × = × = 0.37% 4 90 4 90 One not obvious point to know is that the 3-month LIBOR rate starts when the Eurodollars futures contract expires. In other words, you enter into a Eurodollars futures contract to lock in the next 3-month LIBOR rate. This is why in Table 8.4 the maturity date is ti (Column 1 in Table 5.4) and the implied quarterly rate is r(ti , ri+1 ). The swap spread represents the credit risk in the swap relative to the corresponding riskfree Treasury yield. It is the price tag on the risk that one of the parties to the swap will fail to make a payment. r91 =

8.2.6

Swap’s implicit loan balance

At its inception, an interest rate swap has zero value to both parties. However, as time passes, the market value of the swap may no longer be zero.

8.2.7

Deferred swaps

The equation for solving the fixed rate R in a deferred swap is still PV of fixed payments = PV of floating payments. The formula is: P0 (0, tk−1 ) − P0 (0, tn ) Pn R= = i=k P0 (0, ti )

8.2.8

PT

i=k

P0 (0, ti )r(ti−1 , ti ) i=1 P0 (0, ti )

Pn

Why swap interest rates?

The main idea of this section is that by using an interest rate swap a firm can lower its borrowing cost. Firms like to borrow short-term loans; borrowing a loan for a short period of time is generally less risky and cheaper (i.e paying lower interest rate) than borrowing the same amount of money for a longer period. However, it’s hard for firms to borrow lot of money on the short-term basis. Lenders still worry that the borrower won’t pay back the loan, even if the loan is a short term loan. In an interest rate swap, the notional principal doesn’t change hands and there’s no risk that any party to the swap may default the principal. As a result, an interest rate swap has lower credit risk than a short term loan. By entering an interest rate swap, a firm can borrow at the short term interest rate. FM: Derivatives Markets

c

Yufeng Guo

72

c

Yufeng Guo

Derivatives Markets Chapter 8

8.2.9

www.actuary88.com

Amortizing and accrediting swaps

If the notional principal in an interest rate swap decreases over time, the swap is called an amortizing swap. If the notional principal increases over time, the interest rate swap is called an accrediting swap. When the notional principal is not a constant, then the equation for solving the fixed rate R becomes: Pn

R=

i=1 Qti P0 (0, ti )r(ti−1 , ti ) Pn i=1 Qti P0 (0, ti )

where Qti is notional principal at time ti . ♦

FM: Derivatives Markets

c

Yufeng Guo

73

Solution to Derivatives Markets: for Exam FM Yufeng Guo

www.actuary88.com

c Yufeng Guo

ii

Contents Introduction

vii

1 Introduction to derivatives

1

2

7

Introduction to forwards and options

3 Insurance, collars, and other strategies

29

4 Introduction to risk management

79

5 Financial forwards and futures

129

8 Swaps

141

iii

CONTENTS

www.actuary88.com

CONTENTS

c Yufeng Guo

iv

Preface This is Guo’s solution to Derivatives Markets (2nd edition ISBN 0-321-28030X) for Exam FM. Unlike the official solution manual published by AddisonWesley, this solution manual provides solutions to both the even-numbered and odd-numbered problems for the chapters that are on the Exam FM syllabus. Problems that are out of the scope of the FM syllabus are excluded. Please report any errors to [email protected]. This book is the exclusive property of Yufeng Guo. Redistribution of this book in any form is prohibited.

v

PREFACE

www.actuary88.com

PREFACE

c Yufeng Guo

vi

Introduction Recommendations on using this solution manual: 1. Obviously, you’ll need to buy Derivatives Markets (2nd edition) to see the problems. 2. Make sure you download the textbook errata from http://www.kellogg. northwestern.edu/faculty/mcdonald/htm/typos2e_01.html

vii

CHAPTER 0. INTRODUCTION

www.actuary88.com

c Yufeng Guo

viii

Chapter 1

Introduction to derivatives Problem 1.1. Derivatives on weather are not as farfetched as it might appear. Visit http: //www.cme.com/trading/ and you’ll …nd more than a dozen weather derivatives currently traded at CME such as "CME U.S. Monthly Weather Heating Degree Day Futures" and "CME U.S. Monthly Weather Cooling Degree Day Futures." a. Soft drink sales greatly depend on weather. Generally, warm weather boosts soft drink sales and cold weather reduces sales. A soft drink producer can use weather futures contracts to reduce the revenue swing caused by weather and smooth its earnings. Shareholders of a company generally want the earnings to be steady. They don’t want the management to use weather as an excuse for poor earnings or wild ‡uctuations of earnings. b. The recreational skiing industry greatly dependents on weather. A ski resort can lose money due to warm temperatures, bitterly cold temperatures, no snow, too little snow, or too much snow. A resort can use weather derivatives to reduce its revenue risk. c. Extremely hot or cold weather will result in greater demand for electricity. An electric utility company faces the risk that it may have to buy electricity at a higher spot price. d. Fewer people will visit an amusement park under extreme weather conditions. An amusement park can use weather derivatives to manage its revenue risk. How can we buy or sell weather? No one can accurately predict weather. No one can deliver weather. For people to trade on weather derivatives, weather indexes need to be invented and agreed upon. Once we have weather indexes, we can link the payo¤ of a weather derivative to a weather index. For more information on weather derivatives, visit: http://hometown.aol.com/gml1000/wrms.htm http://www.investopedia.com 1

CHAPTER 1. INTRODUCTION TO DERIVATIVES Problem 1.2. Anyone (such as speculators and investors) who wants to earn a pro…t can enter weather futures. If you can better predict a weather index than does the market maker, you can enter weather futures and make a pro…t. Of course, it’s hard to predict a weather index and hence loss may occur. Two companies with opposite risks may enter weather futures as counter parties. For example, a soft drink company and a ski-resort operator have opposite hedging needs and can enter a futures contract. The soft drink company can have a positive payo¤ if the weather is too cold and a negative payo¤ if warm. This way, when the weather is too cold, the soft drink company can use the gain from the weather futures to o¤set its loss in sales. Since the soft drink company makes good money when the weather is warm, it doesn’t mind a negative payo¤ when the weather is cold. On the other hand, the ski resort can have a negative payo¤ if the weather is too cold and a positive payo¤ if too warm. The ski resort can use the gain from the futures to o¤set its loss in sales.

Problem 1.3. a. 100 41:05 + 20 = 4125 b. 100 40:95 20 = 4075 c. For each stock, you buy at $41:05 and sell it an instant later for $40:95. The total loss due to the ask-bid spread: 100 (41:05 40:95) = 10. In addition, you pay $20 twice. Your total transaction cost is 100 (41:05 40:95) + 2 (20) = 50 Problem 1.4. a. 100 41:05 + 100 41:05 0:003 = 4117: 315 b. 100 40:95 100 40:95 0:003 = 4082: 715 c. For each stock, you buy at $41:05 and sell it an instant later for $40:95. The total loss due to the ask-bid spread: 100 (41:05 40:95) = 10. In addition, your pay commission 100 41:05 0:003 + 100 40:95 0:003 = 24: 6. Your total transaction cost is 10 + 24: 6 = 34: 6 Problem 1.5. The market maker buys a security for $100 and sells it for $100:12. If the market maker buys 100 securities and immediately sells them, his pro…t is 100 (100:12 100) = 12 Problem 1.6. www.actuary88.com

c Yufeng Guo

2

CHAPTER 1. INTRODUCTION TO DERIVATIVES Your sales proceeds: 300 (30:19) 300 (30:19) (0:005) = 9011: 715 Your cost of buying 300 shares from the market to close your short position is: 300 (29:87) + 300 (29:87) (0:005) = 9005: 805 Your pro…t: 9011: 715 9005: 805 = 5: 91 Problem 1.7. a. Consider the bid-ask spread but ignore commission and interest. Your sales proceeds: 400 (25:12) = 10048 Your cost of buying back: 400 (23:06) = 9224 Your pro…t: 10048 9224 = 824 b. If the bid-ask spread and 0.3% commission Your sales proceeds: 400 (25:12) 400 (25:12) (0:003) = 10017: 856 Your cost of buying back: 400 (23:06) + 400 (23:06) (0:003) = 9251: 672 Your pro…t: 10017: 856 9251: 672 = 766: 184 Pro…t drops by: 824 766: 184 = 57: 816 c. Your sales proceeds stay in your margin account, serving as a collateral. Since you earn zero interest on the collateral, your lost interest is If ignore commission: 10048 (0:03) = 301: 44 If consider commission: 10017: 856 (0:03) = 300: 54 Problem 1.8. By signing the agreement, you allow your broker to act as a bank, who lends your stocks to someone else and possibly earns interest on the lent stocks. Short sellers typically leave the short sale proceeds on deposit with the broker, along with additional capital called a haircut. The short sale proceeds and the haircut serve as a collateral. The short seller earns interest on this collateral. This interest is called the short rebate in the stock market. The rebate rate is often equal to the prevailing market interest rate. However, if a stock is scarce, the broker will pay far less than the prevailing interest rate, in which case the broker earns the di¤erence between the short rate and the prevailing interest rate. This arrangement makes short selling easy. Also short selling can be used to hedge …nancial risks, which is good for the economy. By the way, you are not hurt in any way by allowing your broker to lend your shares to short sellers. Problem 1.9. According to http://www.investorwords.com, the ex-dividend date was created to allow all pending transactions to be completed before the record date. If an investor does not own the stock before the ex-dividend date, he or she will www.actuary88.com

c Yufeng Guo

3

CHAPTER 1. INTRODUCTION TO DERIVATIVES be ineligible for the dividend payout. Further, for all pending transactions that have not been completed by the ex-dividend date, the exchanges automatically reduce the price of the stock by the amount of the dividend. This is done because a dividend payout automatically reduces the value of the company (it comes from the company’s cash reserves), and the investor would have to absorb that reduction in value (because neither the buyer nor the seller are eligible for the dividend). If you borrow stock to make a short sale, you’ll need to pay the lender the dividend distributed while you maintain your short position. According to the IRS, you can deduct these payments on your tax return only if you hold the short sale open for a minimum period (such as 46 days) and you itemize your deductions. In a perfect market, if a stock pays $5 dividend, after the ex-dividend date, the stock price will be reduced by $5. Then you could buy back stocks from the market at a reduced price to close your short position. So you don’t need to worry whether the dividend is $3 or $5. However, in the real world, a big increase in the dividend is a sign that a company is doing better than expected. If a company pays a $5 dividend instead of the expected $3 dividend, the company’s stock price may go up after the announcement that more dividend will be paid. If the stock price goes up, you have to buy back stocks at a higher price to close your short position. So an unexpected increase of the dividend may hurt you. In addition, if a higher dividend is distributed, you need to pay the lender the dividend while you maintain your short position. This requires you to have more capital on hand. In the real world, as a short seller, you need to watch out for unexpected increases of dividend payout. Problem 1.10. http://www.investopedia.com/articles/01/082201.asp o¤ers a good explanation of short interest: Short Interest Short interest is the total number of shares of a particular stock that have been sold short by investors but have not yet been covered or closed out. This can be expressed as a number or as a percentage. When expressed as a percentage, short interest is the number of shorted shares divided by the number of shares outstanding. For example, a stock with 1.5 million shares sold short and 10 million shares outstanding has a short interest of 15% (1.5 million/10 million = 15%). Most stock exchanges track the short interest in each stock and issue reports at month’s end. These reports are great because by showing what short sellers are doing, they allow investors to gauge overall market sentiment surrounding a particular stock. Or alternatively, most exchanges provide an online tool to calculate short interest for a particular security. www.actuary88.com

c Yufeng Guo

4

CHAPTER 1. INTRODUCTION TO DERIVATIVES Reading Short Interest A large increase or decrease in a stock’s short interest from the previous month can be a very telling indicator of investor sentiment. Let’s say that Microsoft’s (MSFT) short interest increased by 10% in one month. This means that there was a 10% increase in the amount of people who believe the stock will decrease. Such a signi…cant shift provides good cause for us to …nd out more. We would need to check the current research and any recent news reports to see what is happening with the company and why more investors are selling its stock. A high short-interest stock should be approached for buying with extreme caution but not necessarily avoided at all costs. Short sellers (like all investors) aren’t perfect and have been known to be wrong from time to time. In fact, many contrarian investors use short interest as a tool to determine the direction of the market. The rationale is that if everyone is selling, then the stock is already at its low and can only move up. Thus, contrarians feel that a high short-interest ratio (which we will discuss below) is bullish - because eventually there will be signi…cant upward pressure on the stock’s price as short sellers cover their short positions (i.e. buy back the stocks they borrowed to return to the lender). The more likely that investors can speculate on the stock, the higher the demand for the stock and the higher the short interest. A broker can short sell more than his existing inventory. For example, if a broker has 500 shares of IBM stocks, he can short sell 600 shares of IBM stocks as long as he knows where to …nd the additional 100 shares of IBM stocks. If all the brokers simultaneously lend out more than what they have in their stock inventories, then the number of stocks sold short might exceed the total number of the stocks outstanding. NASDAQ short interest is available by issue for a rolling twelve months and is based on a mid-month settlement date. For more information, visit http://www.nasdaqtrader.com/asp/short_interest.asp. Problem 1.11. You go to a bank. The bank uses its customers’deposits and lends you an asset worth $100. Then 90 days later you buy back the asset at $102 from the open market (i.e. you come up with $102 from whatever sources) and return $102 to the bank. Now your short position is closed. Problem 1.12. We need to borrow an asset called money from a bank (the asset owner) to pay for a new house. The asset owner faces credit risk (the risk that we may not be able to repay the loan). To protect itself, the bank needs collateral. The house is collateral. If we don’t pay back our loan, the bank can foreclose the house. www.actuary88.com

c Yufeng Guo

5

CHAPTER 1. INTRODUCTION TO DERIVATIVES To protect against the credit risk, the bank requires a haircut (i.e. requires that the collateral is greater than the loan). Typically, a bank lends only 80% of the purchase price of the house, requiring the borrower to pay a 20% down payment.

www.actuary88.com

c Yufeng Guo

6

Chapter 2

Introduction to forwards and options Problem 2.1. Long a stock=Own a stock (or buy a stock). If you own a stock, your payo¤ at any time is the stock’s market price because you can sell it any time at the market price. Let S represent the stock price at T = 1. Your payo¤ at T = 1 is S. Your pro…t at T = 1 is: Payo¤ - FV(initial investment)= S 50 (1:1) = S 55. You can see that the pro…t is zero when the stock price S = 55. Alternatively, set S 55 = 0 ! S = 55.

7

CHAPTER 2. Payo¤=S

INTRODUCTION TO FORWARDS AND OPTIONS

Pro…t=S

55

80

60

Payoff

40

20

0 10

20

30

40

50

60

70

80

Stock Price

-20

-40

Profit

Payo¤ and pro…t: Long one stock

www.actuary88.com

c Yufeng Guo

8

CHAPTER 2.

INTRODUCTION TO FORWARDS AND OPTIONS

Problem 2.2. Short a stock=Short sell a stock. If you short sell a stock, your payo¤ at any time after the short sale is the negative of the stock’s market price. This is because to close your short position you’ll need to buy the stock back at the market price and return it to the broker. Your payo¤ at T = 1 is S. Your pro…t at T = 1 is: Payo¤ - FV(initial investment)= S + 50 (1:1) = 55 S You can see that the pro…t is zero when the stock price S = 55. Alternatively, set 55 S = 0 ! S = 55.

Payoff or Profit

50

Profit

40 30 20 10 0 10

20

30

-10

40

50

60

Stock Price

-20 -30

Payoff

-40 -50 -60

Payo¤ and pro…t: Short one stock

www.actuary88.com

c Yufeng Guo

9

CHAPTER 2.

INTRODUCTION TO FORWARDS AND OPTIONS

Problem 2.3. The opposite of a purchased call is written call (or sold call). The opposite of a purchased put is written put (or sold put). The main idea of this problem is: The opposite of a purchased call 6= a purchased put The opposite of a purchased put 6= a purchased call Problem 2.4. a. Long forward means being a buyer in a forward contract. Payo¤ of a buyer in a forward at T is P ayof f = ST F = ST 50 ST P ayof f = ST 50 40 10 45 5 50 0 55 5 60 10

b. Payo¤ of a long call (i.e. owning a call) at expiration T is: P ayof f = max (0; ST K) = max (0; ST 50) ST 40 45 50 55 60

P ayof f = max (0; ST 0 0 0 5 10

50)

c. A call option is a privilege. You exercise a call and buy the stock only if your payo¤ is positive. In contrast, a forward is an obligation. You need to buy the stock even if your payo¤ is negative. A privilege is better than an obligation. Consequently, a long call is more expensive than a long forward on the same underlying stock with the same time to expiration.

www.actuary88.com

c Yufeng Guo

10

CHAPTER 2.

INTRODUCTION TO FORWARDS AND OPTIONS

Problem 2.5. a. Short forward = Enter into a forward as a seller Payo¤ of a seller in a forward at T is P ayof f = F ST = 50 ST ST P ayof f = 50 ST 40 10 45 5 50 0 55 5 60 10

b. Payo¤ of a long put (i.e. owning a put) at expiration T is: P ayof f = max (0; K ST ) = max (0; 50 ST ) ST 40 45 50 55 60

P ayof f = max (0; 50 10 5 0 0 0

ST )

c. A put option is a privilege. You exercise a put and sell the stock only if your payo¤ is positive. In contrast, a forward is an obligation. You need to sell the stock even if your payo¤ is negative. A privilege is better than an obligation. Consequently, a long put is more expensive than a short forward on the same underlying stock with the same time to expiration.

www.actuary88.com

c Yufeng Guo

11

CHAPTER 2.

INTRODUCTION TO FORWARDS AND OPTIONS

Problem 2.6. 91 (1 + r) = 100 ! r = 0:0989 The e¤ective annual interest rate is 9.89%. If you buy the bond at t = 0, your payo¤ at t = 1 is 100 Your pro…t at t = 1 is 100 91 (1 + 0:0989) = 0 regardless of the stock price at t = 1. If you buy a bond, you just earn the risk-free interest rate. Beyond this, your pro…t is zero.

Payoff

101.0

100.5

100.0

99.5

99.0 0

10

20

30

40

50

60

Stock Price

Payo¤: Long a bond

www.actuary88.com

c Yufeng Guo

12

CHAPTER 2.

Profit

INTRODUCTION TO FORWARDS AND OPTIONS

1.0 0.8 0.6 0.4 0.2 0.0 10

20

30

40

50

60

-0.2

70

80

90

100

Stock Price

-0.4 -0.6 -0.8 -1.0

Pro…t of longing a bond is zero.

www.actuary88.com

c Yufeng Guo

13

CHAPTER 2.

INTRODUCTION TO FORWARDS AND OPTIONS

If you sell the bond at t = 0, your payo¤ at t = 1 is 100 (you need to pay the bond holder 100). Your pro…t at t = 1 is 91 (1 + 0:0989) 100 = 0 regardless of the stock price at t = 1 If you sell a bond, you just earn the risk-free interest rate. Beyond this, your pro…t is zero.

0

Payoff

10

20

30

40

Stock Price 50

60

-99.0

-99.5

-100.0

-100.5

-101.0

Payo¤: Shorting a bond

www.actuary88.com

c Yufeng Guo

14

CHAPTER 2.

Profit

INTRODUCTION TO FORWARDS AND OPTIONS

1.0 0.8 0.6 0.4 0.2 0.0 10

20

30

40

50

60

-0.2

70

80

90

100

Stock Price

-0.4 -0.6 -0.8 -1.0

Pro…t of shorting a bond is zero.

www.actuary88.com

c Yufeng Guo

15

CHAPTER 2.

INTRODUCTION TO FORWARDS AND OPTIONS

Problem 2.7. a. It costs nothing for one to enter a forward contract. Hence the payo¤ of a forward is equal to the pro…t. Suppose we long a forward (i.e. we are the buyer in the forward). Our payo¤ and pro…t at expiration is: ST F = ST 55

Payoff (Profit)

40 30 20 10 0 10

20

30

40

50

60

-10

70

80

90

100

Stock Price

-20 -30 -40 -50

Payo¤ (and pro…t) of a long forward

www.actuary88.com

c Yufeng Guo

16

CHAPTER 2.

INTRODUCTION TO FORWARDS AND OPTIONS

Suppose we short a forward (i.e. we are the seller in the forward), our payo¤ and pro…t at expiration is: F ST = 55 ST

Payoff (Profit)

50 40 30 20 10 0 10

20

30

40

50

60

-10

70

80

90

100

Stock Price

-20 -30 -40

Payo¤ (and pro…t) of a short forward

www.actuary88.com

c Yufeng Guo

17

CHAPTER 2.

INTRODUCTION TO FORWARDS AND OPTIONS

b. If the stock doesn’t pay dividend, buying a stock outright at t = 0 and getting a stock at T = 1 through a forward are identical. There’s no bene…t to owning a stock early. c. Suppose the stock pays dividend before the forward expiration date T = 1. Please note that if you own a stock prior to the dividend date, you will receive the dividend. In contrast, if you are a buyer in a forward contract, at T = 1, you’ll get a stock but you won’t receive any dividend. If the stock is expected to pay dividend, then the stock price is expected to drop after the dividend is paid. The forward price agreed upon at t = 0 already considers that a dividend is paid during (0; T ); the dividend will reduce the forward rate. There’s no advantage to buying a stock outright over buying a stock through a forward. Otherwise, there will be arbitrage opportunities. If the stock is not expected to pay dividend but actually pays dividend (a surprise dividend), then the forward price F agreed upon at t = 0 was set without knowing the surprise dividend. So F is the forward price on a non-dividend paying stock. Since dividend reduces the value of a stock, F is higher than the forward price on an otherwise identical but dividendpaying stock. If you own a stock at t = 0, you’ll receive the windfall dividend. If you buy a stock through a forward, you’ll pay F , which is higher than the forward price on an otherwise identical but dividendpaying stock. Hence owning a stock outright is more bene…cial than buying a stock through a forward. Problem 2.8. r =risk free interest rate Under the no-arbitrage principle, you get the same pro…t whether you buy a stock outright or through a forward. Pro…t at T = 1 if you buy a stock at t = 0 is: ST 50 (1 + r) Pro…t at T = 1 if you buy a stock through a forward: ST 53 ! ST

50 (1 + r) = ST

www.actuary88.com

53

50 (1 + r) = 53

c Yufeng Guo

r = 0:06

18

CHAPTER 2.

INTRODUCTION TO FORWARDS AND OPTIONS

Problem 2.9. a. Price of an index forward contract expiring in one year is: F Index = 1000 (1:1) = 1100 To see why: If the seller borrows 1000 at t = 0, buys an index, and holds it for one year, then he’ll have one stock to deliver at T = 1. The seller’s cost is 1000 (1:1) = 1100. To avoid arbitrage, the forward price must be 1100. Pro…t at T = 1 of owning a forward on an index: ST F Index = ST 1100

ST

If you buy an index at t = 0, your pro…t at T = 1 is ST 1100

1000 (1:1) =

So you get the same pro…t whether you buy the index outright or buy the index through a forward. This should make sense. If owning a stock outright and buying it through a forward have di¤erent pro…ts, arbitrage opportunities exist. b. The forward price 1200 is greater than the fair forward price 1100. No rational person will want to enter such an unfair forward contract. Thus the seller needs to pay the buyer an up-front premium to incite the buyer. The 1200 1100 buyer in the forward needs to receive = 90: 91 at t = 0 to make 1:1 the forward contract fair. Of course, the buyer needs to pay the forward price 1200 at T = 1. c. Now the forward price 1000 is lower than the fair forward price 1100. You can imagine thousands of bargain hunters are waiting in line to enter this forward contract. If you want to enter the forward contract, you have to pay the 1100 1000 = 90: 91 at t = 0. In addition, seller a premium in the amount of 1:1 you’ll need to pay the forward price 1000 at T = 1: Problem 2.10. a. Pro…t=max (0; ST 1000) 95:68 Set pro…t to zero: max (0; ST 1000) 95:68 = 0 ! ST 1000 95:68 = 0 ST = 1000 + 95:68 = 1095: 68 b. The pro…t of a long forward (i.e. being a buyer in a forward): ST ST 1020 = max (0; ST 1000) 95:68 If ST > 1000, there’s no solution If ST 1000: ST 1020 = 0 95:68 ! ST = 1020 95:68 = 924: 32 www.actuary88.com

c Yufeng Guo

1020

19

CHAPTER 2.

INTRODUCTION TO FORWARDS AND OPTIONS

Problem 2.11. a. Pro…t of a long (i.e. owning) put is max (0; 1000

ST )

75:68

max (0; 1000 ST ) 75:68 = 0 1000 ST 75:68 = 0 ST = 924: 32 b. Pro…t of a short forward (i.e. being a seller in a forward) is 1020 max (0; 1000 ST ) 75:68 = 1020 ST If 1000 ST 1000

S

75:68 = 1020

If 1000 ST 75:68 = 1020

www.actuary88.com

ST

S

ST

no solution

ST = 1020 + 75:68 = 1095: 68

c Yufeng Guo

20

CHAPTER 2.

INTRODUCTION TO FORWARDS AND OPTIONS

Problem 2.12. Table 2.4 is: Position Long forward (buyer in forward) Short forward (seller in forward) Long call (own a call) Short call (sell a call) Long put (own a put) Short put (sell a put)

Maximum Loss -Forward price Unlimited -FV (premium) Unlimited -FV(premium) PV(premium)-Strike Price

Maximum Gain Unlimited Forward Price Unlimited FV(premium) Strike Price - FV(premium) FV(premium)

If you are a buyer in a forward, the worst that can happen to you is ST = 0 (i.e. stock price at T is zero). If this happens, you still have to pay the forward price F at T to buy the stock which is worth zero. You’ll lose F . Your best case is ST = 1, where you have an unlimited gain. If you are a seller in a forward, the worst case is that ST = 1; you’ll incur unlimited loss. Your best case is that ST = 0, in which case you sell a worthless asset for the forward price F . If you buy a call, your worst case is ST < K, where K is the strike price. If this happens, you just let the call expire worthless. You’ll lose the future value of your premium (if you didn’t buy the call and deposit your money in a bank account, you could earn the future value of your deposit). Your best case is that ST = 1, where you’ll have an unlimited gain. If you sell a call, your worst case is ST = 1, in which case you’ll incur an unlimited loss. Your best case is ST < K, in which case the call expires worthless; the call holder wastes his premium and your pro…t is the future value of the premium you received from the buyer. If you buy a put, your worst case is that ST K, in which case you’ll let your put expire worthless and you’ll lose the future value of the put premium. Your best case is ST = 0, in which case you sell a worthless stock for K. Your pro…t is K F V (premium). If you sell a put, your worst case is ST = 0, in which case the put holder sells you a worthless stock for K; your pro…t is F V (premium) K. Your best case is ST K, where the written put expires worthless and your pro…t is F V (premium).

www.actuary88.com

c Yufeng Guo

21

CHAPTER 2.

INTRODUCTION TO FORWARDS AND OPTIONS

Problem 2.13.

Let S represent the stock price at the option expiration date. I’ll draw a separate diagram for the payo¤ and a separate diagram for the pro…t. a. Suppose you long a call (i.e. buy a call). 0 if S 35 if Your pro…t at expiration= Payo¤ - FV (premium) = max (0; S 35) 9:12 (1:08) = max (0; S 35) 9: 849 6 0 if S < 35 9: 849 6 if = 9: 849 6 = S 35 if S 35 S 44: 849 6 if

(i) Payo¤ at expiration is max (0; S

35) =

S < 35 S 35

S < 35 35 S

60 50

Payoff

40 30

Profit

20 10 0 10

20

30

40

50

60

70

80

90

100

Stock Price

-10

Payo¤ and Pro…t: Long a 35 strike call

www.actuary88.com

c Yufeng Guo

22

CHAPTER 2.

INTRODUCTION TO FORWARDS AND OPTIONS

0 if S 40 if Your pro…t at expiration= Payo¤ - FV (premium) = max (0; S 40) 6:22 (1:08) = max (0; S 40) 6: 717 6 0 if S < 40 6: 717 6 if = 6: 717 6 = S 40 if S 40 S 46: 717 6 if

(ii) Payo¤ at expiration is max (0; S

40) =

S < 40 S 40

S < 40 S 40

60

50

Payoff

40

30

Profit

20

10

0 10

20

30

40

50

60

70

80

90

100

Stock price at expiration

Payo¤ and Pro…t: Long a 40 strike call

www.actuary88.com

c Yufeng Guo

23

CHAPTER 2.

INTRODUCTION TO FORWARDS AND OPTIONS

0 if S 45 if Your pro…t at expiration= Payo¤ - FV (premium) = max (0; S 45) 4:08 (1:08) = max (0; S 45) 4: 406 4 0 if S < 45 4: 406 4 if = 4: 406 4 = S 45 if S 45 S 49: 406 4 if

(iii) Payo¤ at expiration is max (0; S

45) =

S < 45 S 45

S < 45 S 45

50

Payoff

40

30

Profit

20

10

0 10

20

30

40

50

60

70

80

90

100

Stock price at expiration

Payo¤ and Pro…t: Long a 45 strike call

b. The payo¤ of a long call is max (0; S K). As K increases, the payo¤ gets worse and the option becomes less valuable. Everything else equal, the higher the strike price, the lower the call premium.

www.actuary88.com

c Yufeng Guo

24

CHAPTER 2.

INTRODUCTION TO FORWARDS AND OPTIONS

Problem 2.14. Suppose we own a put (i.e. long put). S if S 35 0 if S > 35 Your pro…t at expiration = Payo¤ - FV (premium) = max (0; 35 S) 1:53 (1:08) = max (0; 35 S) 1: 652 4 35 S if S 35 33: 347 6 S if S 35 = 1: 652 4 = 0 if S > 35 1: 652 4 if S > 35

a. Payo¤ at expiration is max (0; 35

35

S) =

30

20

10

0 10

20

30

40

50

60

70

80

90

100

Stock price at expiration

Payo¤ and pro…t: Long a 35 strike put The blue line is the payo¤. The black line is the pro…t.

www.actuary88.com

c Yufeng Guo

25

CHAPTER 2.

INTRODUCTION TO FORWARDS AND OPTIONS

S if S 40 0 if S > 40 Your pro…t at expiration = Payo¤ - FV (premium) = max (0; 40 S) 3:26 (1:08) = max (0; 40 S) 3: 520 8 40 S if S 40 36: 479 2 S if S 40 = 3: 520 8 = 0 if S > 40 3: 520 8 if S > 40

b. Payo¤ at expiration is max (0; 40

40

S) =

40

30

20

10

0 10

20

30

40

50

60

70

80

90

100

Stock price at expiration

Payo¤ and pro…t: Long a 40 strike put

The blue line is the payo¤. The black line is the pro…t.

www.actuary88.com

c Yufeng Guo

26

CHAPTER 2.

INTRODUCTION TO FORWARDS AND OPTIONS

S if S 45 0 if S > 45 Your pro…t at expiration = Payo¤ - FV (premium) = max (0; 45 S) 5:75 (1:08) = max (0; 45 S) 6: 21 45 S if S 45 38: 79 S if S 45 = 6: 21 = 0 if S > 45 6: 21 if S > 45

c. Payo¤ at expiration is max (0; 45

45

S) =

40

30

20

10

0 10

20

30

40

50

60

70

80

90

100

Stock price at expiration

Payo¤ and pro…t: Long a 45 strike put The blue line is the payo¤. The black line is the pro…t. As the strike price increases, the payo¤ of a put goes up and the more valuable a put is. Everything else equal, the higher the strike price, the more expensive a put is.

www.actuary88.com

c Yufeng Guo

27

CHAPTER 2.

INTRODUCTION TO FORWARDS AND OPTIONS

Problem 2.15. If you borrow money from a bank to buy a $1000 S&R index, your borrowing cost is known at the time of borrowing. Suppose the annual e¤ective risk free interest rate is r. If you borrow $1000 at t = 0, then at T you just pay the bank T 1000 (1 + r) . You can sleep well knowing that your borrowing cost is …xed in advance. In contrast, if you short-sell n number of IBM stocks and use the short sale proceeds to buy a $1000 S&R index, you own the brokerage …rm n number of IBM stocks. If you want to close your short position at time T , you need to buy n stocks at T . The cost of n stocks at T is nST , where ST is the price of IBM stocks per share at T . Since ST is not known in advance, if you use short selling to …nance your purchase of a $1000 S&R index, your borrowing cost nST cannot be known in advance. This brings additional risk to your position. As such, you can’t determine your pro…t. Problem 2.16. Skip this problem. SOA is unlikely to ask you to design a spreadsheet on the exam.

www.actuary88.com

c Yufeng Guo

28

Chapter 3

Insurance, collars, and other strategies Problem 3.1. The put premium is 74:201. At t = 0, you spend 1000 to buy an S&R index spend 74:201 to buy a 1000-strike put borrow 980:39 take out (1000 + 74:201)

980:39 = 93: 811 out of your own pocket.

So your total borrowing is 980:39 + 93: 811 = 1074: 20. The future value is 1074: 20 (1:02) = 1095: 68 S&R index 900 950 1000 1050 1100 1150 1200

S&R Put 100 50 0 0 0 0 0

Payo¤ 1000 1000 1000 1050 1100 1150 1200

-(Cost+Interest) 1095: 68 1095: 68 1095: 68 1095: 68 1095: 68 1095: 68 1095: 68

29

Pro…t 1000 1000 1000 1050 1100 1150 1200

1095: 68 = 95: 68 1095: 68 = 95: 68 1095: 68 = 95: 68 1095: 68 = 45: 68 1095: 68 = 4: 32 1095: 68 = 54: 32 1095: 68 = 104: 32

CHAPTER 3. INSURANCE, COLLARS, AND OTHER STRATEGIES Payo¤. The payo¤ of owning an index is S, where S is the price of the index at the put expiration The payo¤ of owning a put is max (0; 1000 S) at expiration. Total payo¤: 1000 S if S 1000 1000 if S 1000 S+max (0; 1000 S) = S+ = 0 if S > 1000 S if S > 1000 Pro…t is: 1000 if S 1000 95: 68 if S 1000 1095: 68 = S if S > 1000 S 1095: 68 if S > 1000

Payoff (Profit)

2000

Payoff 1500

1000

Profit 500

0 200

400

600

800 1000 1200 1400 1600 1800 2000

Stock price at expiration

Payo¤ and Pro…t: index + put

www.actuary88.com

c Yufeng Guo

30

CHAPTER 3. INSURANCE, COLLARS, AND OTHER STRATEGIES Problem 3.2. At t = 0 you short sell one S&R index, receiving $1000 sell a 1000-strike put, receiving $74:201 deposit 1000 + 74:201 = 1074: 201 in a savings account. This grows into 1074: 201 (1:02) = 1095: 68 at T = 1 The payo¤ of the index sold short is S The payo¤ of a sold put: max (0; 1000

S)

The total payo¤ at expiration is: S max (0; 1000

S) =

1000 0

S

The pro…t at expiration is: 1000 if S 1000 + 1095: 68 = S if S > 1000 S&R index 900 950 1000 1050 1100 1150 1200

S&R Put 100 50 0 0 0 0 0

www.actuary88.com

Payo¤ 1000 1000 1000 1050 1100 1150 1200

S

if if

S 1000 = S > 1000

95: 68 1095: 68

-(Cost+Interest) 1095: 68 1095: 68 1095: 68 1095: 68 1095: 68 1095: 68 1095: 68

c Yufeng Guo

S

if if

1000 if S if

S 1000 S > 1000

S 1000 S > 1000

Pro…t 1000 + 1095: 68 = 95: 68 1000 + 1095: 68 = 95: 68 1000 + 1095: 68 = 95: 68 1050 + 1095: 68 = 45: 68 1100 + 1095: 68 = 4: 32 1150 + 1095: 68 = 54: 32 1200 + 1095: 68 = 104: 32

31

CHAPTER 3. INSURANCE, COLLARS, AND OTHER STRATEGIES

Payo¤=

Pro…t=

1000 if S if

1000 if S if

200

400

S 1000 S > 1000

S 1000 + 1095: 68 S > 1000

600

800

1000

1200

1400

Index Price

1600

1800

2000

0

Profit -500

-1000

-1500

Payoff -2000

short index +sell put

You can verify that the pro…t diagram above matches the textbook Figure 3.5 (d).

www.actuary88.com

c Yufeng Guo

32

CHAPTER 3. INSURANCE, COLLARS, AND OTHER STRATEGIES Problem 3.3. Option 1: Buy S&R index for 1000 and buy a 950-strike put Option 2: Invest 931:37 in a zero-coupon bond and buy a 950-strike call. Verify that Option 1 and 2 have the same payo¤ and the same pro…t. Option 1: If you own an index, your payo¤ at any time is the spot price of the index S. The payo¤ of owning a 950-strike put is max (0; 950 S). Your total payo¤ at the put expiration is S+max (0; 950

S) = S+

950 S 0

if if

S 950 = S > 950

950 if S if

S 950 S > 950

To calculate the pro…t, we need to know the initial investment. At t = 0, we spend 1000 to buy an index and 51:777 to buy the 950-strike put. The total investment is 1000 + 51:777 = 1051: 777. The future value of the investment is 1051: 777 (1:02) = 1072: 81 So the pro…t is: 950 if S 950 S if S > 950

www.actuary88.com

1072: 81 =

S

122: 81 if 1072: 81 if

c Yufeng Guo

S 950 S > 950

33

CHAPTER 3. INSURANCE, COLLARS, AND OTHER STRATEGIES

Payo¤=

950 if S if

S 950 S > 950

950 if S if

Pro…t=

S 950 S > 950

1072: 81

2000

Payoff

1500

1000

500

Profit

0 200

400

600

800

1000

1200

1400

1600

1800

2000

Index

index + put

www.actuary88.com

c Yufeng Guo

34

CHAPTER 3. INSURANCE, COLLARS, AND OTHER STRATEGIES Option 2: Payo¤ of the zero-coupon bond at T = 0:5 year is: 931:37 (1:02) = 950 Payo¤ of owning a 950-strike call: max (0; S 950) Total payo¤: 950+max (0; S

950) = 950+

S

0 if 950 if

S 950 = S > 950

950 if S if

S 950 S > 950

To calculate the pro…t, we need to know the initial investment. We spend 931:37 to buy a bond and 120:405 to buy a 950-strike call. The future value of the investment is (931:37 + 120:405) 1:02 = 1072: 81. The pro…t is: 950 if S if

S 950 S > 950

1072: 81 =

S

122: 81 if 1072: 81 if

S 950 S > 950

Option 1 and 2 have the same payo¤ and the same pro…t. But why? It’s because the put-call parity: C (K; T ) + P V (K) = P (K; T ) + S0 Option 1 consists of buying S&R index and a 950-strike put Option 2 consists of investing P V (K) = 950 1:02 1 = 931: 37 and buying a 950-strike call. Due to the put-call parity, Option 1 and 2 have the same payo¤ and the same pro…t.

www.actuary88.com

c Yufeng Guo

35

CHAPTER 3. INSURANCE, COLLARS, AND OTHER STRATEGIES Problem 3.4. Option 1: Short sell S&R index for 1000 and buy a 950-strike call Option 2: Borrow 931:37 and buy a 950-strike put Verify that Option 1 and 2 have the same payo¤ and the same pro…t. Option 1: At t = 0:5, your payo¤ from the short sale of an index is S, where S is the index price at T = 0:5. At T = 0:5, your payo¤ from owning a 0 if S < 950 call is max (0; S 950) = . S 950 if S 950 Your total payo¤ is 0 if S+ S 950 if

S < 950 = S 950

S if 950 if

S < 950 S 950

Please note that when calculating the payo¤, we’ll ignore the sales price of the index $1; 000 and the call purchase price 120: 405. These two numbers a¤ect your pro…t, but they don’t a¤ect your payo¤. Your payo¤ is the same no matter whether you sold your index for $1 or $1000, and no matter whether you buy the 950-strike call for $10 or $120: 41. Next, let’s …nd the pro…t at T = 0:5. At t = 0, you sell an index for 1000. Of 1000 you get, you spend 120: 405 t 120: 41 to buy a 950-strike call. You have 1000 120: 41 = 879: 59 left. This will grow into 879: 59 1:02 = 897: 18 at T = 0:5 At T = 0:5, your pro…t is 897: 18 plus the payo¤:

Pro…t =897: 18 +

www.actuary88.com

S if 950 if

S < 950 = S 950

c Yufeng Guo

897: 18 S 52: 82

if if

S < 950 S 950

36

CHAPTER 3. INSURANCE, COLLARS, AND OTHER STRATEGIES

Payo¤=

S if 950 if

S < 950 S 950

Pro…t =897: 18+

S if 950 if

S < 950 S 950

800

Profit

600 400 200 0 200

400

600

800

1000

1200

1400

1600

-200

1800

2000

Index Price

-400 -600

Payoff

-800

Payo¤ and Pro…t: Short index + Long call

www.actuary88.com

c Yufeng Guo

37

CHAPTER 3. INSURANCE, COLLARS, AND OTHER STRATEGIES Option 2 payo¤. At t = 0:5, you need to pay the lender 931:37 1:02 = 950, so, a payo¤ of 950 (you’ll write the lender a check of 950). At T = 0:5, the pay950 S if S < 950 o¤ from buying a 950-strike put is max (0; 950 S) = . 0 if S 950 Your total payo¤ at T = 0:5 is: 950 +

950 0

S

if if

S < 950 = S 950

S if 950 if

S < 950 . S 950

This is the same as the payo¤ in Option 1. Option 2 pro…t. There are two ways to calculate the pro…t. Method 1. The total pro…t is the sum of pro…t earned from borrowing 931:37 and the pro…t earned by buying a 950-strike put. The pro…t from borrowing 931:37 is zero; you borrow 931:37 at t = 0. This grows into 931:37 1:02 = 950 at T = 0:5 in your savings account. Then at T = 0:5, you take out 950 from your savings account and pay the lender. Now your savings account is zero. So the pro…t earned from borrowing 931:37 is zero. Next, let’s calculate the pro…t from buying the put. The put premium is $51.78. So your pro…t earned from buying the put option is 51:78 1:02 + max (0; 950 S) = 52: 82 + max (0; 950 S) =

52: 82 +

950 0

S

if if

S < 950 = S 950

897: 18 S 52: 82

if if

S < 950 S 950

S if S < 950 . We just 950 if S 950 need to deduct the future value of the initial investment. At t = 0, you receive 931:37 from the lender and pay 51:78 to buy the put. So your total cash is 931:37 51:78 = 879: 59, which grows into 879: 59 1:02 = 897: 18 at t = 0:5. Hence, your pro…t is: Method 2. We already know the payo¤ is

S if 950 if

S < 950 + 897: 18 = S 950

897: 18 S 52: 82

if if

S < 950 S 950

No matter whether you use Method 1 or Method 2, the Option 2 pro…t is the same as the Option 1 pro…t. You might wonder why Option 1 and Option 2 have the same payo¤ and the same pro…t. The parity formula is Call (K; T )

P ut (K; T ) = P V (F0;T

K) = P V (F0;T )

P V (K)

Rearranging this equation, we get: www.actuary88.com

c Yufeng Guo

38

CHAPTER 3. INSURANCE, COLLARS, AND OTHER STRATEGIES Call (K; T ) + P V (K) = P ut (K; T ) + P V (F0;T ) Since P V (F0;T ) = S0 , now we have: P V (K) = P ut (K; T ) + Call (K; T ) + | {z } | {z } | {z } own a call

own PV of strike price

own a put

own one index

The above equation can also be read as; P V (K) = P ut (K; T ) + Call (K; T ) + | {z } | {z } {z } | buy a call

invest PV of strike price

buy a put

Rearranging the above formula, we get: Call (K; T ) + S = P ut (K; T ) + |{z}0 | | {z } {z }

S0 |{z}

S0 |{z}

buy one index

The above equation can also be read as:

|

P V (K) {z }

Call (K; T ) + {z } |

|

P V (K) {z }

own a call

buy a call

sell one index

S |{z}0

sell one index

own a put

= P ut (K; T ) + | {z } buy a put

b orrow PV of strike price

b orrow PV of strike price

According to the parity equation, Option 1 and Option 2 are identical portfolios and should have the same payo¤ and the same pro…t. In this problem, Option 1 consists of shorting an S&R index and buying a 950-strike call. Option 2 consists of borrowing P V (K) = 950 1:02 1 = 931: 37 and buying a 950-strike put. As a result, Option 1 and 2 have the same payo¤ and the same pro…t.

www.actuary88.com

c Yufeng Guo

39

CHAPTER 3. INSURANCE, COLLARS, AND OTHER STRATEGIES Problem 3.5. Option 1: Short sell index for 1000 and buy 1050-strike call Option 2: Borrow 1029.41 and buy a 1050-strike put. Verify that Option 1 and 2 have the same payo¤ and the same pro…t. Option 1: Payo¤: 0 if S < 1050 S if S+max (0; S 1050) = S+ = S 1050 if S 1050 1050 if Pro…t: Your receive 1000 from the short sale and spend 71.802 to buy the 1050-strike call. The future value is: (1000 71:802) 1:02 = 946: 76 So the pro…t is: S if S < 1050 + 946: 76 = 1050 if S 1050

www.actuary88.com

946: 76 S 103: 24

c Yufeng Guo

if if

S < 1050 S 1050

40

S < 1050 S 1050

CHAPTER 3. INSURANCE, COLLARS, AND OTHER STRATEGIES

Payo¤=

S if 1050 if

S < 1050 S 1050

Pro…t=

S if 1050 if

S < 1050 + 946: 76 S 1050

800

Profit

600 400 200 0 200

400

600

800

1000

1200

1400

1600

-200

1800

2000

Index Price

-400 -600 -800

Payoff

-1000

Payo¤ and Pro…t: Short index + Long call

www.actuary88.com

c Yufeng Guo

41

CHAPTER 3. INSURANCE, COLLARS, AND OTHER STRATEGIES Option 2: Payo¤: If you borrow 1029:41, you’ll need to pay 1029:41 (1:02) = 1050 at T = 0:5 So the payo¤ of borrowing 1029:41 is 1050. Payo¤ of the purchased put is max (0; 1050 Total payo¤ is: 1050 = 1050 + 0

S

if if

S < 1050 = S 1050

S)

S if 1050 if

S < 1050 S 1050

Initially, you receive 1029:41 from a bank and spend 101:214 to buy a 950strike put. So your net receipt at t = 0 is 1029:41 101:214 = 928: 196. Its future value is 928: 196 (1:02) = 946: 76. Your pro…t is: S if 1050 if

S < 1050 + 946: 76 = S 1050

946: 76 S 103: 24

if if

S < 1050 S 1050

Option 1 and 2 have the same payo¤ and the same pro…t. This is because the put-call parity: S = P ut (K; T ) + P V (K) Call (K; T ) + |{z}0 {z } | {z } | {z } | buy a call

sell one index

www.actuary88.com

buy a put

b orrow PV of strike price

c Yufeng Guo

42

CHAPTER 3. INSURANCE, COLLARS, AND OTHER STRATEGIES Problem 3.6. (a) buy an index for 1000 (b) buy a 950-strike call, sell a 950-strike put, and lend 931:37 Verify that (a) and (b) have the same payo¤ and the same pro…t. (a)’s payo¤ is S. Pro…t is S

1000 (1:02) = S

1020

(b)’s payo¤: buy a 950-strike call sell a 950-strike put lend 931.37 Total Total payo¤: max (0; S 950) =

=

0 S

S

if 950 if

Payo¤ max (0; S 950) max (0; 950 S) 931:37 (1:02) = 950

max (0; 950 S < 950 S 950

(950 S) + 950 if 950 + 950 if

Initial receipt 120:405 51:777 931:37 120:405 + 51:777

931:37 =

S) + 950 950 0

S

if if

S < 950 + 950 S 950

S < 950 =S S 950

Total pro…t: S 1000 (1:02) = S 1020 (a) and (b) have the same payo¤ and the same pro…t. Why? Call (K; T ) + {z } | buy a call

!

S0 |{z}

S |{z}0

sell one index

=

buy one index

www.actuary88.com

= P ut (K; T ) + | {z } buy a put

|

P V (K) {z }

b orrow PV of strike price

Call (K; T ) + P ut (K; T ) + | {z } | {z } buy a call

sell a put

c Yufeng Guo

P V (K) | {z }

lend PV of strike price

43

1000

CHAPTER 3. INSURANCE, COLLARS, AND OTHER STRATEGIES (a) and (b) have the following common payo¤ and pro…t.

Payo¤=S

Pro…t=S

1020

2000

Payoff 1000

0 200

400

600

800

1000

Profit

1200

1400

1600

1800

2000

Index Price

-1000

www.actuary88.com

c Yufeng Guo

44

CHAPTER 3. INSURANCE, COLLARS, AND OTHER STRATEGIES Problem 3.7. (a) short index for 1000 (b) sell 1050-strike call, buy a 1050-strike put, and borrow 1029.41 Verify that (a) and (b) have the same payo¤ and the same pro…t. (a) Payo¤ is (b) Payo¤: max (0; 1050 =

=

1050 0

S. Pro…t is

S + 1000 (1:02) = 1020

S)

max (0; S

1050)

if if

S < 1050 S 1050

0 S

S

(1050 S) 1050 if (S 1050) 1050 if

S

1029:41 (1:02) if 1050 if

S < 1050 = S 1050

S < 1050 S 1050 S S

if if

1050

S < 1050 = S 1050

S

We need to calculate the initial investment of (b). At t = 0, we Receive 71:802 from selling a 1050-strike call Pay 101:214 to buy a 1050-strike put Receive 1029:41 from a lender Our net receipt is 71:802 101:214 + 1029:41 = 1000. The future value is 1000 (1:02) = 1020 So the pro…t at T = 0:5 is

S + 1020 = 1020

S:

You see that (a) and (b) have the same payo¤ and the same pro…t. Why? From the put-call parity, we have: S P V (K) = P ut (K; T ) + Call (K; T ) + |{z}0 {z } | {z } | {z } | buy a call

!

S |{z}0

sell one index

=

sell one index

www.actuary88.com

|

buy a put

b orrow PV of strike price

Call (K; T ) + P ut (K; T ) + {z } | {z }

sell a call

buy a put

c Yufeng Guo

|

P V (K) {z }

b orrow PV of strike price

45

CHAPTER 3. INSURANCE, COLLARS, AND OTHER STRATEGIES Payo¤=

S.

Pro…t= 1020

S

1000

Profit 0 200

-1000

400

600

800

1000

1200

1400

1600

1800

2000

Index Price

Payoff

-2000

www.actuary88.com

c Yufeng Guo

46

CHAPTER 3. INSURANCE, COLLARS, AND OTHER STRATEGIES Problem 3.8. Put-call parity:

Call (K; T ) + | {z } buy a call

P V (K) | {z }

invest PV of strike price

109:2 + P V (K) = 60:18 + 1000 P V (K) = 60:18 + 1000 109:2 = 950: 98 P V (K) =

K 1:02

= P ut (K; T )+ | {z } buy a put

S0 |{z}

buy one index

K = 950: 98 (1:02) = 970

Problem 3.9. Buy a call (put) a lower strike + Sell an otherwise identical call (put) with a higher strike=Bull call (put) spread Option 1: buy 950-strike call and sell 1000-strike call Option 2: buy 950-strike put and sell 1000-strike put. Verify that option 1 and 2 have the same pro…t. Option 1: Payo¤=max (0; S 950) max (0; S 1000) 0 if S < 950 0 if S < 1000 = S 950 if S 950 S 1000 if S 1000 8 < 0 S = : S

if 950 if 950 if

8 < 0 S 950 = : S 950 Initial cost:

S < 950 1000 > S S 1000

950

if if 1000) if

(S

8 < 0 0 : S

S < 950 1000 > S S 1000

if if 1000 if

S < 950 1000 > S S 1000

8 if < 0 950 = S 950 if : 50 if

950

S < 950 1000 > S S 1000

950

Spend 120:405 to buy 950-strike call Sell 1000-strike call receiving 93:809 Total initial investment: 120:405 93:809 = 26: 596 The future value is 26: 596 (1:02) = 27: 127 92 Pro…t 8 is: if < 0 S 950 if = : 50 if =

8 <

S < 950 1000 > S S 1000

27:13 if S 977: 13 if : 22: 87 if

www.actuary88.com

950

S < 950 1000 > S S 1000

27:13 =

8 <

27:13 if S 950 27:13 if : 50 27:13 if

S < 950 1000 > S S 1000

950

c Yufeng Guo

47

950

CHAPTER 3. INSURANCE, COLLARS, AND OTHER STRATEGIES 8 if < 0 S 950 if Payo¤= : 50 if 8 if < 0 S 950 if Pro…t= : 50 if

S < 950 1000 > S S 1000

950

S < 950 1000 > S S 1000

950

27:13

50

Payoff

40 30 20

Profit

10 0 200

400

600

800

1000

1200

1400

1600

1800

2000

Index Price

-10 -20

www.actuary88.com

c Yufeng Guo

48

CHAPTER 3. INSURANCE, COLLARS, AND OTHER STRATEGIES Option 2: buy 950-strike put and sell 1000-strike put. The payo¤: max (0; 950 S) max (0; 1000 S) 950 S if S < 950 1000 S if S < 1000 = 0 if S 950 0 if S 1000 8 < 950 0 = : 0

S

if if if

S < 950 1000 > S S 1000

8 < (950 S) (1000 (1000 S) = : 0

950

S) if if

8 < 1000 1000 : 0

S < 950 1000 > S S 1000

S S

if if if

950 =

Initial cost:

8 <

S < 950 1000 > S S 1000

S : 0

50

950

if 1000 if if

S < 950 1000 > S S 1000

Buy 950-strike put. Pay 51:777 Sell 1000-strike put. Receive 74:201 Net receipt: 74:201 51:777 = 22: 424 Future value: 22: 424 (1:02) = 22: 872 48 The 8 pro…t is: if S < 950 < 50 S 1000 if 1000 > S 950 + 22: 87 : 0 if S 1000 8 <

50 + 22: 87 if S 1000 + 22: 87 if = : 22: 87 if

www.actuary88.com

S < 950 1000 > S S 1000

950 =

c Yufeng Guo

8 <

27: 13 if S 977: 13 if : 22: 87 if

S < 950 1000 > S S 1000

49

950

950

CHAPTER 3. INSURANCE, COLLARS, AND OTHER STRATEGIES 8 if < 50 Payo¤= S 1000 if : 0 if 8 if < 50 Pro…t= S 1000 if : 0 if

S < 950 1000 > S S 1000

950

S < 950 1000 > S S 1000

950 + 22: 87

Profit

20

10

0 200

400

600

800

1000

1200

1400

1600

1800

2000

Index Price

-10

-20

-30

-40

Payoff

-50

www.actuary88.com

c Yufeng Guo

50

CHAPTER 3. INSURANCE, COLLARS, AND OTHER STRATEGIES The payo¤ of the …rst option is $50 greater than the payo¤ of the second option. However, at t = 0, we pay 26: 596 to set up option 1; we pay 22: 424 (i.e. we receive 22: 424) to set up option 2. It costs us 26: 596 ( 22: 424) = 49: 02 more initially to set up option 1 than option 2. The future value of this initial set up cost is 49: 02 (1:02) = 50. As a result, option 1 and 2 have the same pro…t at T = 0:5. This should make sense in a world of no arbitrage. Consider two portfolios A and B. If for any stock price P ayof f (A) = P ayof f (B) + c, then InitialCost (A) = InitialCost (B) + P V (c) to avoid arbitrage. P rof it (A) = P ayof f (A) F V [InitialCost (A)] = P ayof f (B) + c F V [InitialCost (B)] c = P ayof f (B) F V [InitialCost (B)] = P rof it (B) Similarly, if InitialCost (A) = InitialCost (B) + P V (c) ! P ayof f (A) = P ayof f (B) + c ! P rof it (A) = P rof it (B) Finally, let’s see why option 1 is always $50 higher than option 2 in terms of the payo¤ and the initial set up cost. The put-call parity is: P V (K) = P ut (K; T ) + S0 Call (K; T ) + |{z} | {z } | {z } {z } | buy a call

invest PV of strike price

buy a put

buy one index

The timing of the put-call parity is at t = 0. The above equation means Call (K; T ) + P V (K) | {z } | {z }

Cost of buying a call at t=0

Cost of investing PV of strike price at t=0

=

+

P ut (K; T ) | {z }

Cost of buying a put at t=0

S0 |{z}

Cost of buying an index at t=0

If we are interested in the payo¤ at expiration date T , then the put-call parity is: K = P ut (K; T ) + S Call (K; T ) + |{z} |{z} | {z } | {z } Payo¤ a call at T

strike price at T

Index price at T

Payo¤ of a put at T

Now we set up the initial cost parity for two strike prices K1 < K2 Call (K1 ; T ) | {z }

cost of buying a call

Call (K2 ; T ) {z } |

cost of buying a call

2

+ P V (K1 ) | {z }

invest PV of K1

+ P V (K2 ) | {z }

cost of buying a call

P ut (K1 ; T ) + | {z }

cost of buying a put

=

invest PV of K2

6 ! 4 Call (K1 ; T ) | {z }

www.actuary88.com

=

cost of buying one index at t=0

P ut (K2 ; T ) + | {z }

cost of buying a put

3

S0 |{z} S0 |{z}

cost of buying one index at t=0

7 Call (K2 ; T ) 5 + [P V (K1 ) | {z }

P V (K2 )]

cost of buying a call

c Yufeng Guo

51

CHAPTER 3. INSURANCE, COLLARS, AND OTHER STRATEGIES 2

3

6 = 4 P ut (K1 ; T ) | {z }

7 P ut (K2 ; T ) 5 | {z }

cost of buying a put

cost of buying a put

2

3

6 ! 4 Call (K1 ; T ) | {z } | 2

cost of buying a call

cost of buying a put

} 3

7 P ut (K2 ; T ) 5 + [P V (K2 ) | {z }

cost of buying a put

{z

Put spread

2

6 ! 4 Call (K1 ; T ) | {z } | 2

+

cost of buying a call

{z

cost of buying a put

+ {z

P V (K1 )]

}

3

|

7 Call (K2 ; T ) 5 {z }

|

7 P ut (K2 ; T ) 5 + [P V (K2 ) {z }

cost of selling a call

Call spread

6 = 4 P ut (K1 ; T ) | {z } |

cost of buying a call

Call spread

6 = 4 P ut (K1 ; T ) | {z } |

{z

7 Call (K2 ; T ) 5 | {z }

} 3

cost of selling a put

Put spread

P V (K1 )]

}

So the initial cost of setting up a call bull spread always exceeds the initial set up cost of a bull put spread by a uniform amount P V (K2 ) P V (K1 ). In this problem, P V (K2 ) P V (K1 ) = (1000 950) 1:02 1 = 49: 02 Set up the payo¤ parity at T : Call (K1 ; T ) + K1 = |{z} | {z } strike price at T

Payo¤ a call at T

Call (K2 ; T ) + | {z }

6 !4 |

=

strike price at T

Payo¤ a call at T

2

K1 |{z}

Call (K1 ; T ) {z } |

Payo¤ a long call at T

{z

P ut (K1 ; T ) | {z }

+

P ut (K2 ; T ) | {z }

+

Payo¤ of a put at T

Payo¤ of a put at T

Call (K2 ; T ) {z } |

Payo¤ a long call at T

Call bull payo¤ at T

www.actuary88.com

c Yufeng Guo

3

S |{z}

Index price at T

S |{z}

Index price at T

7 5 }

52

CHAPTER 3. INSURANCE, COLLARS, AND OTHER STRATEGIES 2

6 =4 |

P ut (K2 ; T ) | {z }

Payo¤ of a long put at T

6 !4 6 =4 |

P ut (K1 ; T ) | {z }

Payo¤ of a long put at T

Put spread payo¤ at T

2

| 2

{z

3

Call (K1 ; T ) | {z }

Payo¤ a long call at T

+ {z

|

3

7 Call (K2 ; T ) 5 {z }

Payo¤ short call at T

Call spread payo¤ at T

P ut (K2 ; T ) | {z }

Payo¤ of a long put at T

+ {z

7 5 + K2

|

}

P ut (K1 ; T ) {z }

Payo¤ of a short put at T

Put spread payo¤ at T

K1

}

3

7 5 + K2

K1

}

In this problem, K2 K1 = 1000 950 = 50 So the payo¤ of a call bull spread at T = 0:5 always exceeds the payo¤ of a put bull spread by a uniform amount 50.

www.actuary88.com

c Yufeng Guo

53

CHAPTER 3. INSURANCE, COLLARS, AND OTHER STRATEGIES Problem 3.10. Buy a call (put) a higher strike + Sell an otherwise identical call (put) with lower strike=Bear call (put) spread. In this problem, K1 = 1050, K2 = 950 (a bear spread) 1 P V2(K2 ) P V (K1 ) = (950 1050) 1:02 3 = ( 100) 1:02 6 ! 4 Call (K1 ; T ) | {z } | 2

cost of buying a call

{z

cost of buying a put

| 2

7 Call (K2 ; T ) 5 {z }

|

7 P ut (K2 ; T ) 5 + [P V (K2 ) {z }

+ {z

|

7 P ut (K2 ; T ) 5 {z }

cost of buying a put

cost of selling a put

Put spread

2

6 !4 | 2

6 =4

Call (K1 ; T ) | {z }

Payo¤ a long call at T

+ {z

6 =4

|

P ut (K2 ; T ) | {z }

} 3 }

Payo¤ short call at T

+ {z

Payo¤ of a long put at T

+

{z

P V (K1 )]

3 }

|

P ut (K1 ; T ) {z }

|

P ut (K1 ; T ) {z }

Payo¤ of a short put at T

Put spread payo¤ at T

P ut (K2 ; T ) | {z }

98: 04

98: 04

7 Call (K2 ; T ) 5 {z }

Call spread payo¤ at T

Payo¤ of a long put at T

| 2

} 3

cost of selling a put

+

{z

=

cost of selling a call

Put spread

6 = 4 P ut (K1 ; T ) | {z }

|

|

Call spread

6 = 4 P ut (K1 ; T ) {z } |

|

+

1

Payo¤ of a short put at T

Put spread payo¤ at T

3

7 5 + K2 } 3 7 5

K1

100

}

For any index price at expiration, the payo¤ of the call bear spread is always 100 less than the payo¤ of the put bear spread. Consequently, as we have seen, to avoid arbitrage, the initial set-up cost of the call bear spread is less than the initial set-up cost of the put bear spread by the amount of present value of the 100. The call bear spread and the put bear spread have the same pro…t at expiration.

www.actuary88.com

c Yufeng Guo

54

CHAPTER 3. INSURANCE, COLLARS, AND OTHER STRATEGIES Next, let’s draw the payo¤ and pro…t diagram for each spread. Payo¤ of the call bear spread: Payo¤=max (0; S 1050) max (0; S

Buy 1050-strike call = Sell 950-strike call Total 8 < 0 950 S = : 100

if if if

S < 950 0 0 0

950 0 950 950

950) S < 1050 S S

1050 S S 1050 950 S 100

S < 950 950 S < 1050 S 1050

The initial set-up cost of the call bear spread: Buy 1050-strike call. Pay 71:802 Sell 950-strike call. Receive 120:405 Net receipt: 120:405 71:802 = 48: 603 Future value: 48: 603 (1:02) = 49: 575 06 So the pro…t of the call bear spread at expiration is 8 8 if S < 950 < 0 < 49: 58 950 S if 950 S < 1050 +49: 58 = 999: 58 S = : : 100 if S 1050 50: 42

www.actuary88.com

c Yufeng Guo

if if if

S < 950 950 S < 1050 S 1050

55

CHAPTER 3. INSURANCE, COLLARS, AND OTHER STRATEGIES 8 < 0 950 S Payo¤= : 100 8 < 0 950 S Pro…t= : 100

if if if

S < 950 950 S < 1050 S 1050

if if if

S < 950 950 S < 1050 + 49: 58 S 1050

40 20 0 200

400

600

800

1000

1200

1400

1600

1800

2000

Index Price

-20

Profit

-40 -60 -80

Payoff -100

Payo¤ and Pro…t: Call bear spread

www.actuary88.com

c Yufeng Guo

56

CHAPTER 3. INSURANCE, COLLARS, AND OTHER STRATEGIES Payo¤ of the put bear spread: Payo¤=max (0; 1050 S) max (0; 950

Buy 1050-strike put = Sell 950-strike put Total 8 < 100 1050 = : 0

Payoff

S

if if if

S < 950 1050 S S 950 100

S)

950 S < 1050 1050 S 0 1050 S

1050 0 0 0

S

S < 950 950 S < 1050 S 1050

100 90 80 70 60 50 40 30 20 10 0 0

200

400

600

800

1000

1200

1400

1600

1800

2000

Index Price

Payo¤ of the put bear spread

www.actuary88.com

c Yufeng Guo

57

CHAPTER 3. INSURANCE, COLLARS, AND OTHER STRATEGIES The initial set-up cost: Buy 1050-strike put. Pay 101:214 Sell 950-strike put. Receive 51:777 Net cost: 101:214 51:777 = 49: 437 Future value: 49: 437 (1:02) = 50: 42 The 8 pro…t at expiration is: if S < 950 < 100 1050 S if 950 S < 1050 = : 0 if S 1050

8 < 49: 58 999: 58 S 50: 42 = : 50: 42

if if if

S < 950 950 S < 1050 S 1050

We see that the call bear spread and the put bear spread have the same pro…t.

www.actuary88.com

c Yufeng Guo

58

CHAPTER 3. INSURANCE, COLLARS, AND OTHER STRATEGIES Problem 3.11. Buy S&R index Buy 950-strike put Sell 1050-strike call Total FV (initial cost)

Initial Cost 1000 51:777 71:802 1000 + 51:777 71:802 = 979: 975 979: 975 (1:02) = 999: 574 5

Payo¤ S max (0; 950 max (0; S

S) 1050)

The net option premium is: 51:777 71:802 = 20: 025. So we receive 20: 025 if we enter this collar. Payo¤ S < 950 950 S < 1050 1050 S Buy S&R index S S S Buy 950-strike put 950 S 0 0 Sell 1050-strike call 0 0 1050 S Total 950 S 1050 The 8 payo¤ at expiration is: < 950 if S < 950 S if 950 S < 1050 : 1050 if S 1050 The pro…t at 8 < 950 if S if : 1050 if =

expiration is: S < 950 950 S < 1050 S 1050

8 <

49: 57 if S 999: 57 if : 50: 43 if

www.actuary88.com

8 < 950 999: 57 if S 999: 57 if 999: 57 = : 1050 999: 57 if

S < 950 950 S < 1050 S 1050

S < 950 950 S < 1050 S 1050

c Yufeng Guo

59

CHAPTER 3. INSURANCE, COLLARS, AND OTHER STRATEGIES

Pro…t=

8 <

49: 57 if S 999: 57 if : 50: 43 if

Profit

S < 950 950 S < 1050 S 1050

50 40 30 20 10 0 200

400

600

800

1000

-10

1200

1400

1600

1800

2000

Index Price

-20 -30 -40 -50

Pro…t: long index, long 950-strike put, short 1050-strike call

The net option premium is 20: 025. So we receive 20: 025 if we enter this collar. To construct a zero-cost collar and keep 950-strike put, we need to increase the strike price of the call such that the call premium is equal to the put premium of 51:777.

www.actuary88.com

c Yufeng Guo

60

CHAPTER 3. INSURANCE, COLLARS, AND OTHER STRATEGIES Problem 3.12. Buy S&R index Buy 950-strike put Sell 1107-strike call Total FV (initial cost)

Initial Cost 1000 51:777 51:873 1000 + 51:777 51:873 = 999: 904 999: 904 (1:02) = 1019: 902 08

Payo¤ S max (0; 950 max (0; S

S) 1050)

The net option premium is: 51:777 51:873 = 0:096 . So we receive 0:096 if we enter this collar. This is very close to a zero-cost collar, where the net premium is zero. Payo¤ Buy S&R index Buy 950-strike put Sell 1050-strike call Total

S < 950 S 950 S 0 950

The 8 pro…t is: < 950 if S < 950 S if 950 S < 1107 : 1107 if S 1107

www.actuary88.com

950 S 0 0 S

S < 1107

1019: 90 =

c Yufeng Guo

8 <

1107 S 0 1107 1107

S

S

69: 9 if S 1019: 90 if : 87: 1 if

S < 950 950 S < 1107 S 1107

61

CHAPTER 3. INSURANCE, COLLARS, AND OTHER STRATEGIES

Pro…t=

8 <

69: 9 if S 1019: 90 if : 87: 1 if

Profit

S < 950 950 S < 1107 S 1107

80 60 40 20 0 200

400

600

800

1000

1200

1400

1600

1800

2000

Index Price

-20 -40 -60

Pro…t: long index, long 950-strike put, short 1107-strike call

www.actuary88.com

c Yufeng Guo

62

CHAPTER 3. INSURANCE, COLLARS, AND OTHER STRATEGIES Problem 3.13. a. 1050-strike S&R straddle Straddle = buy a call and put with the same strike price and time to expiration. Buy1050-strike call Buy 1050-strike put Total FV (initial cost)

Initial Cost 71:802 101:214 71:802 + 101:214 = 173: 016 173: 016 (1:02) = 176: 476 32

Payo¤ max (0; S 1050) max (0; 1050 S)

Payo¤ Buy1050-strike call Buy 1050-strike put Total The pro…t is: 1050 S if S 1050 if

Profit

S < 1050 0 1050 S 1050 S

S < 1050 S 1050

S S 0 S

1050 1050 1050

176: 48 =

873: 52 S S 1226: 48

if if

S < 1050 S 1050

900 800 700 600 500 400 300 200 100 0 200

400

600

800

1000 1200 1400 1600 1800 2000 2200

-100

Index Price

Pro…t: long 1050-strike call and long 1050-strike put

www.actuary88.com

c Yufeng Guo

63

CHAPTER 3. INSURANCE, COLLARS, AND OTHER STRATEGIES b. written 950-strike S&R straddle Initial revenue short 950-strike call 120:405 short 950-strike put 51:777 Total 120:405 + 51:777 = 172: 182 FV (initial cost) 172: 182 (1:02) = 175: 625 64

Payo¤ max (0; S 950) max (0; 950 S)

Payo¤ sell 950-strike call sell 950-strike put Total The pro…t is: S 950 if 950 S if

Profit

S < 950 0 S 950 S 950

S 950 950 S 0 950 S

S < 950 + 175: 66 = S 950

S 774: 34 1125: 66 S

if if

S < 950 S 950

100 0 200

400

600

800

1000 1200 1400 1600 1800 2000 2200

-100

Index Price

-200 -300 -400 -500 -600 -700 -800 -900 -1000

Pro…t: short 950-strike call and short 950-strike put

www.actuary88.com

c Yufeng Guo

64

CHAPTER 3. INSURANCE, COLLARS, AND OTHER STRATEGIES c. simultaneous purchase of 1050-straddle and sale of 950-straddle Pro…t = Pro…t of purchase of 1050-straddle + Pro…t of Sale of 950-straddle Pro…t 873: 52 S if S < 1050 S 774: 34 if S < 950 + S 1226: 48 if S 1050 1125: 66 S if S 950 8 if < 873: 52 S 873: 52 S if : S 1226: 48 if

8 S < 950 < S 774: 34 950 S < 1050 + 1125: 66 S : S 1050 1125: 66 S

8 if < 873: 52 S + (S 774: 34) 873: 52 S + (1125: 66 S) if = : S 1226: 48 + (1125: 66 S) if

8 < 99: 18 1999: 18 2S = : 100: 82

Profit

if if if

if if if

S < 950 950 S < 1050 S 1050

S < 950 950 S < 1050 S 1050

S < 950 950 S < 1050 S 1050

100 80 60 40 20 0 950

1000

1050

-20

1100

1150

1200

Index Price

-40 -60 -80 -100

Pro…t: long 1050-strike straddle and short 950 straddle

www.actuary88.com

c Yufeng Guo

65

CHAPTER 3. INSURANCE, COLLARS, AND OTHER STRATEGIES Problem 3.14. The put-call parity is: P V (K) Call (K; T ) + | {z } | {z } buy a call

invest PV of strike price

! Call (K1 ; T ) + | {z } buy a call

P V (K1 ) | {z }

= P ut (K; T ) + | {z } buy a put

invest PV of strike price

! Call (K2 ; T ) + | {z } buy a call

P V (K2 ) | {z }

invest PV of strike price

2

6 ! 4Call (K1 ; T ) | {z } buy a call

2

3

buy a put

buy one index

= P ut (K2 ; T ) + | {z } buy a put

S0 |{z}

buy one index

7 Call (K2 ; T )5+P V (K1 | {z } buy a call

3

2

S0 |{z}

3

6 K2 ) = 4P ut (K1 ; T ) | {z }

7 P ut (K2 ; T )5 | {z }

buy a put

buy a put

2

buy a put

3 2

sell a put

3

K1 )

3

K1 )

6 7 6 7 ! 4Call (K1 ; T ) + Call (K2 ; T )5 4P ut (K1 ; T ) + P ut (K2 ; T )5 = P V (K2 | | {z } | {z } {z } | {z } buy a call

sell a call

buy a put

sell a put

3 2

2

6 7 6 7 ! 4Call (K1 ; T ) + Call (K2 ; T )5+4 P ut (K1 ; T ) + P ut (K2 ; T )5 = P V (K2 | | {z } | {z } {z } | {z } buy a call

sell a call

sell a put

buy a put

The initial cost is P V (K2 K1 ) at t = 0. The payo¤ at expiration T = 0:5 is K2 K1 . The transaction is equivalent to investing P V (K2 K1 ) in a savings account at t = 0 and receiving K2 K1 at T , regardless of the S&R price at expiration. So the transaction doesn’t have any S&R price risk. We just earn the risk free interest rate over the 6- month period. In = 1000 2 this problem, K1 = 950 and3K2 2 3 6 7 6 7 4Call (K1 ; T ) + Call (K2 ; T )5 + 4 P ut (K1 ; T ) + P ut (K2 ; T )5 | {z } | {z } | {z } | {z } buy a call

= P V (1000

sell a call

sell a put 1

950) = P V (50) = 50 1:02

buy a put

= 49: 02

So the total initial cost is 49:02. The payo¤ is 49:02 (1:02) = 50. The pro…t is 0. We earn a 2% risk-free interest rate over the 6-month period. www.actuary88.com

c Yufeng Guo

3

6 7 K2 ) = 4P ut (K1 ; T ) + P ut (K2 ; T )5 | {z } | {z }

sell a call

2

buy one index

= P ut (K1 ; T ) + | {z }

6 7 ! 4Call (K1 ; T ) + Call (K2 ; T )5+P V (K1 | {z } | {z } buy a call

S0 |{z}

66

CHAPTER 3. INSURANCE, COLLARS, AND OTHER STRATEGIES Problem 3.15. a. Buy a 950-strike call and sell two 1050-strike calls Initial cost buy a 950-strike call 120:405 sell two 1050-strike calls 2 (71:802) = 143: 604 Total 120:405 143: 604 = 23: 199 FV (initial cost) 23: 199 (1:02) = 23: 662 98

Payo¤ max (0; S 950) 2 max (0; S 1050)

Payo¤ buy a 950-strike call sell two1050-strike calls Total The pro…t is: 8 if < 0 S 950 if : 1150 S if

8 < 23: 66 S 926: 34 = : 1173: 66 S

S < 950 0 0 0

950 S < 1050 S 950 0 S 950

S S

1050 950 2 (S 1050) S 950 2 (S

8 if S < 950 < 23: 66 S 950 + 23: 66 if 950 S < 1050 +23: 66 = : 1150 S + 23: 66 if S 1050 if if if

1050) = 1150

S < 950 950 S < 1050 S 1050

S < 950 950 S < 1050 S 1050

Profit 120 100 80 60 40 20 0 850

900

950

1000

1050

1100

1150

1200

Index Price

-20

long 950-strike call and short two1050-strike calls

www.actuary88.com

c Yufeng Guo

67

S

CHAPTER 3. INSURANCE, COLLARS, AND OTHER STRATEGIES b. Buy two 950-strike calls and sell three 1050-strike calls Initial cost Payo¤ buy two 950-strike calls 2 (120:405) = 240: 81 2 max (0; S 950) sell three 1050-strike calls 3 (71:802) = 215: 406 3 max (0; S 1050) Total 240: 81 215: 406 = 25: 404 FV (initial cost) 25: 404 (1:02) = 25: 912 08 Payo¤ buy two 950-strike calls sell three 1050-strike calls Total Pro…t: 8 if < 0 2 (S 950) if : 1250 S if =

S < 950 950 S < 1050 S 1050

8 <

25: 91 if 2S 1925: 91 if : 1224: 09 S if

Profit

S < 950 0 0 0

950 2 (S 0 2 (S

S < 1050 950) 950)

25: 91 =

S < 950 950 S < 1050 S 1050

S 1050 2 (S 950) 3 (S 1050) 2 (S 950) 3 (S

8 <

25: 91 if 2 (S 950) 25: 91 if : 1250 S 25: 91 if

160 140 120 100 80 60 40 20 0 950

1000

1050

-20

1100

1150

1200

1250

Index Price

long two 950-strike calls and short three1050-strike calls

www.actuary88.com

c Yufeng Guo

68

1050) = 1250

S < 950 950 S < 1050 S 1050

S

CHAPTER 3. INSURANCE, COLLARS, AND OTHER STRATEGIES c. Buy n 950-strike calls and short m 1050-strike calls such that the initial premium is zero. 71:802 n = = 0:596 3 120:405n = 71:802m ! m 120:405 Problem 3.16. A spread consists of buying one option at one strike price and selling an otherwise identical option with a di¤erent strike price. A bull spread consists of buying one option at one strike and selling an otherwise identical option but at a higher strike price. A bear spread consists of buying one option at one strike and selling an otherwise identical option but at a lower strike price. A bull spread and a bear spread will never have zero premium because the two options don’t have the same premium. A butter‡y spread might have a zero net premium. Problem 3.17. According to http://www.daytradeteam.com/dtt/butterfly-options-trading. asp , a butter‡y spread combines a bull and a bear spread. It uses three strike prices. The lower two strike prices are used in the bull spread, and the higher strike price in the bear spread. Both puts and calls can be used. A very large pro…t is made if the stock is at or very near the middle strike price on expiration day. When you enter a butter‡y spread, you are entering 3 options orders at once. If the stock remains or moves into a de…ned range, you pro…t, and if the stock moves out of the desired range, you lose. The closer the stock is to the middle strike price on expiration day, the larger your pro…t. For the strike price K1 < K2 < K3 K2 = K1 + (1 ) K3 So for each K2 -strike option sold, there needs to be units of K1 -strike options bought and (1 ) units of K3 -strike options bought. In this problem, K1 = 950, K2 = 1020, K3 = 1050 1020 = 950 + (1 ) 1050 ! = 0:3 For every ten 1020-strike calls written, there needs to be three 950-strike calls purchased and seven 1050-strike calls purchased (so we buy three 950 1020 bull spreads and seven 1020 1050 bear spreads).

sell ten 1020-strike calls three 950-strike calls seven 1050-strike calls Total FV (initial cost) www.actuary88.com

Initial cost 10 ( 84:47) = 844: 7 3 (120:405) = 361: 215 7 (71:802) = 502: 614 844: 7 + 361: 215 + 502: 614 = 19: 129 19: 129 (1:02) = 19: 511 58 c Yufeng Guo

Payo¤ 10 max (0; S 1020) 3 max (0; S 950) 7 max (0; S 1050)

69

CHAPTER 3. INSURANCE, COLLARS, AND OTHER STRATEGIES Payo¤ 950-strike calls 1020-strike calls 1050-strike calls Total 3 (S 950) 10 (S 3 (S 950) 10 (S

S < 950 950 S < 1020 0 3 (S 950) 0 0 0 0 0 3 (S 950) 1020) = 7350 7S 1020) + 7 (S 1050) = 0

8 0 if > > < 3 (S 950) if The payo¤ = 7350 7S if > > : 0 if

A key point to payo¤ is zero if S The 8 pro…t is: 0 > > < 3 (S 950) 7350 7S > > : 0 8 > > <

1020 S < 1050 3 (S 950) 10 (S 1020) 0 7350 7S

1050 S 3 (S 950) 10 (S 1020) 7 (S 1050) 0

S < 950 950 S < 1020 1020 S < 1050 1050 S

remember is that for a butter‡y spread K1 < K2 < K3 , K1 or S K3 . 8 19: 51 if S < 950 > > < 3 (S 950) 19: 51 if 950 S < 1020 19: 51 = 7350 7S 19: 51 if 1020 S < 1050 > > : 19: 51 if 1050 S

19: 51 3S 2869: 51 = 7330: 49 7S > > : 19: 51

www.actuary88.com

if if if if

the

if if if if

S < 950 950 S < 1020 1020 S < 1050 1050 S

c Yufeng Guo

70

S < 950 950 S < 1020 1020 S < 1050 1050 S

CHAPTER 3. INSURANCE, COLLARS, AND OTHER STRATEGIES 8 0 if > > < 3 (S 950) if Payo¤ = 7350 7S if > > : 0 if

8 0 if > > < 3 (S 950) if Pro…t= 7350 7S if > > : 0 if

S < 950 950 S < 1020 1020 S < 1050 1050 S

S < 950 950 S < 1020 1020 S < 1050 1050 S

19: 51

200 180 160 140 120 100 80 60 40 20 0 850

900

950

1000

-20

1050

1100

1150

1200

Stock Price

Butter‡y spread K1 = 950, K2 = 1020, K3 = 1050

The black line is the pro…t line. The blue line is the payo¤ line.

www.actuary88.com

c Yufeng Guo

71

CHAPTER 3. INSURANCE, COLLARS, AND OTHER STRATEGIES Problem 3.18. The option price table is: Strike Call premium 35 6:13 40 2:78 45 0:97

Put premium 0:44 1:99 5:08

Time to expiration is T = 91=365 0:25 The annual e¤ective rate is 8:33% p The quarterly e¤ective rate is 4 1:0833 1 = 2 02% a. Buy 35–strike call, sell two 40-strike calls, and buy 45-strike call. Let’s reproduce the textbook Figure 3.14.

buy a 35-strike call sell two 40-strike calls buy a 45-strike call Total FV (initial cost)

Initial cost 6:13 2 ( 2:78) = 5: 56 0:97 6:13 5: 56 + 0:97 = 1: 54 1: 54 (1:0202) = 1: 571 108

Payo¤ S < 35 35 S < 40 35-strike call 0 S 35 40-strike calls 0 0 45-strike call 0 0 Total 0 S 35 S 35 2 (S 40) = 45 S S 35 2 (S 40) + S 45 = 0 The 8 payo¤ is: if > > 0 < S 35 if 45 S if > > : 0 if

The 8 pro…t 0 > > < S 35 45 S > > : 0

is: if if if if

40 S < 45 S 35 2 (S 40) 0 45 S

45 S S 35 2 (S 40) S 45 0

S < 35 35 S < 40 40 S < 45 45 S

S < 35 35 S < 40 40 S < 45 45 S

www.actuary88.com

8 > > <

1: 57 S 36: 57 1: 57 = 43: 43 S > > : 1: 57

c Yufeng Guo

if if if if

S < 35 35 S < 40 40 S < 45 45 S

72

CHAPTER 3. INSURANCE, COLLARS, AND OTHER STRATEGIES 8 0 if > > < S 35 if Payo¤= 45 S if > > : 0 if

8 0 if > > < S 35 if Pro…t= 45 S if > > : 0 if

S < 35 35 S < 40 40 S < 45 45 S

S < 35 35 S < 40 40 S < 45 45 S

1: 57

5

4

Black line is Payoff

3

Blue line is Profit

2

1

0 25

30

35

40

45

50

55

60

Stock Price

-1

Butter‡y spread K1 = 35, K2 = 40, K3 = 45

www.actuary88.com

c Yufeng Guo

73

CHAPTER 3. INSURANCE, COLLARS, AND OTHER STRATEGIES b. Buy a 35–strike put, sell two 40-strike puts, and buy a 45-strike put. Let’s reproduce the textbook Figure 3.14. Initial cost buy a 35-strike put 0:44 sell two 40-strike puts 2 ( 1:99) = 3: 98 buy a 45-strike put 5:08 Total 0:44 3: 98 + 5:08 = 1: 54 FV (initial cost) 1: 54 (1:0202) = 1: 571 108 Payo¤ 35-strike put 40-strike puts 45-strike put Total 35 S 2 (40

S < 35 35 S 2 (40 45 S 0

S)

35 S < 40 0 2 (40 S) 45 S S 35

40 0 0 45 45

S < 45

S S

45 0 0 0 0

S

2 (40 S) + 45 S = 0 S) + 45 S = S 35

8 0 if > > < S 35 if The payo¤ = 45 S if > > : 0 if 8 0 if > > < S 35 if The pro…t = 45 S if > > : 0 if

www.actuary88.com

S < 35 35 S < 40 40 S < 45 45 S S < 35 35 S < 40 40 S < 45 45 S

c Yufeng Guo

8 > > <

1: 57 S 36: 57 1: 57 = 43: 43 S > > : 1: 57

if if if if

S < 35 35 S < 40 40 S < 45 45 S

74

CHAPTER 3. INSURANCE, COLLARS, AND OTHER STRATEGIES 8 0 if > > < S 35 if Payo¤ = 45 S if > > : 0 if

8 0 if > > < S 35 if Pro…t = 45 S if > > : 0 if

S < 35 35 S < 40 40 S < 45 45 S

S < 35 35 S < 40 40 S < 45 45 S

1: 57

5

4

3

2

1

0 25

30

35

40

45

50

55

60

Stock Price

-1

Butter‡y spread K1 = 35, K2 = 40, K3 = 45

The black line is the payo¤; the blue line is the pro…t.

www.actuary88.com

c Yufeng Guo

75

CHAPTER 3. INSURANCE, COLLARS, AND OTHER STRATEGIES c. Buy one stock, buy a 35 put, sell two 40 calls, and buy a 45 call. The put-call parity is: P V (K) = P ut (K; T ) + S0 Call (K; T ) + |{z} | {z } | {z } {z } | buy a call

invest PV of strike price

buy a put

buy one sto ck

!Buy stock + buy 35 put=buy 35 call + PV(35)

Buy one stock, buy a 35 put, sell two 40 calls, and buy a 45 call is the same as: buy a 35 call , sell two 40 calls, and buy a 45 call, and deposit PV(35) in a savings account. We already know from Part a. that "buy a 35 call , sell two 40 calls, and buy a 45 call" reproduces the textbook pro…t diagram Figure 3.14. Depositing PV(35) = 35 1:0202 1 = 34: 31 won’t change the pro…t because any deposit in a savings account has zero pro…t. Hence the pro…t diagram of "Buy one stock, buy a 35 put, sell two 40 calls, and buy a 45 call" is Figure 3.14.

www.actuary88.com

c Yufeng Guo

76

CHAPTER 3. INSURANCE, COLLARS, AND OTHER STRATEGIES Problem 3.19. a. The parity is: Call (K; T ) P ut (K; T ) = P V (F0;T K) We are told that Call (K; T ) P ut (K; T ) = 0 ! P V (F0;T K) = 0 P V (F0;T ) = P V (K) Since P V (F0;T ) = S0 ! P V (K) = S0 b. Buying a call and selling an otherwise identical put creates a synthetic long forward. c. We buy the call at the ask price and sell the put at the bid price. So we have to pay the dealer a little more than the fair price of the call when we buy the call from the dealer; we’ll get less than the fair price of put when we sell a put to the dealer. To ensure that the call premium equals the put premium given there’s a bid-ask spread, we need to make the call less valuable and the put more valuable. To make the call less valuable and the put more valuable, we can increase the strike price. In other words, if there’s no bid-ask spread, then K = F0;T . If there’s bid-ask spread, K > F0;T . d. A synthetic short stock position means "buy put and sell call." To have zero net premium after the bid-ask spread, we need to make the call more valuable and the put less valuable. To achieve this, we can decrease the strike price. In other words, if there’s no bid-ask spread, then K = F0;T . If there’s bid-ask spread, K < F0;T . e. Transaction fees is not really a wash because there’s a bid-ask spread. We pay more if we buy an option and we get less if we sell an option. Problem 3.20. This problem is about building a spreadsheet. You won’t be asked to build a spreadsheet in the exam. Skip this problem.

www.actuary88.com

c Yufeng Guo

77

CHAPTER 3. INSURANCE, COLLARS, AND OTHER STRATEGIES

www.actuary88.com

c Yufeng Guo

78

Chapter 4

Introduction to risk management Problem 4.1. Let S= the price of copper per pound at T = 1 P BH =Pro…t per pound of copper at T = 1 before hedging P AH =Pro…t per pound of copper at T = 1 after hedging For each pound of copper produced, XYZ incurs $0.5 …xed cost and $0.4 variable cost. P BH = S (0:5 + 0:4) = S 0:9 XYZ sells a forward. The pro…t of the forward at T = 1 is: F0;T S = 1 S ! P AH = (S 0:9) + (F0;T S) = F0;T 0:9 We are told that F0;T = 1 ! P AH = 1 0:9 = 0:1

79

CHAPTER 4. INTRODUCTION TO RISK MANAGEMENT P BH = S

Profit

0:9

P AH = 0:1

ing g ed eh r fo Be

0.3

0.2

After hedging 0.1

0.0 0.7

0.8

0.9

1.0

1.1

1.2

Copper price

-0.1

-0.2

-0.3

Pro…t: before hedging and after hedging

www.actuary88.com

c Yufeng Guo

80

CHAPTER 4. INTRODUCTION TO RISK MANAGEMENT Problem 4.2. P AH = F0;T 0:9 If F0;T = 0:8 ! P AH = 0:8 0:9 = 0:1 If XYZ shuts down its production, its pro…t at T = 1 is 0:5 (it still has to pay the …xed cost) If XYZ continues its production, its after hedging pro…t at T = 1 is 0:1 0:1 > 0:5 !XYZ should continue its production If F0;T = 0:45 ! P AH = 0:45 0:9 = 0:45 If XYZ shuts down its production, its pro…t at T = 1 is 0:5 (it still has to pay the …xed cost) If XYZ continues its production, its after hedging pro…t at T = 1 is 0:45 0:45 > 0:5 !XYZ should continue its production Problem 4.3. The pro…t of a long K-strike put at T = 1: K S max (0; K S) F V (P remium) = 0 ! P AH = P BH + =S

=

K S

K 0

0:9 +

K 0

if if

S
View more...

Comments

Copyright ©2017 KUPDF Inc.
SUPPORT KUPDF